Cardiac Electrophysiology
Andrea Natale • Amin Al-Ahmad Paul J. Wang • John P. DiMarco (Editors)
Cardiac Electrophysiology Clinical Case Review
Editors Andrea Natale Texas Cardiac Arrhythmia Institute St. David’s Medical Center 1015 East 32nd Street Suite 516, Austin, TX 78705 USA Amin Al-Ahmad School of Medicine Stanford University 300 Pasteur Drive H-2146, Stanford, CA 94305 USA
Paul J. Wang School of Medicine Stanford University 300 Pasteur Drive H-2146, Stanford, CA 94305 USA John P. DiMarco Cardiovascular Division University of Virginia Health System 1215 Lee Street Charlottesville, VA 22908 USA
ISBN 978-1-84996-389-3 e-ISBN 978-1-84996-390-9 DOI 10.1007/978-1-84996-390-9 Springer London Dordrecht Heidelberg New York British Library Cataloguing in Publication Data A catalogue record for this book is available from the British Library Library of Congress Control Number: 2010937972 © Springer-Verlag London Limited 2011 Apart from any fair dealing for the purposes of research or private study, or criticism or review, as permitted under the Copyright, Designs and Patents Act 1988, this publication may only be reproduced, stored or transmitted, in any form or by any means, with the prior permission in writing of the publishers, or in the case of reprographic reproduction in accordance with the terms of licenses issued by the Copyright Licensing Agency. Enquiries concerning reproduction outside those terms should be sent to the publishers. The use of registered names, trademarks, etc., in this publication does not imply, even in the absence of a specific statement, that such names are exempt from the relevant laws and regulations and therefore free for general use. Product liability: The publisher can give no guarantee for information about drug dosage and application thereof contained in this book. In every individual case the respective user must check its accuracy by consulting other pharmaceutical literature. Cover design: eStudioCalamar, Figueres/Berlin Printed on acid-free paper Springer is part of Springer Science+Business Media (www.springer.com)
To our wives and families, for the constant support and patience.
Preface
As the field of cardiac electrophysiology continues to evolve and advance, we continue to be challenged to educate new generations of cardiac electrophysiologists with the basics as well as the latest advances in the field. While there are many outstanding books that provide an in-depth review of topics in electrophysiology, there are few case-based books that comprehensively cover clinical electrophysiology, devices, and ablation. Case reviews offer a simple, yet effective way in teaching important concepts, offering insight into both the basic pathophysiology of a problem as well as the clinical reasoning that leads to a solution. In “Cardiac Electrophysiology: Clinical Case Review” we sought to put together the most comprehensive case-based review of electrophysiology that would appeal to all students of the field whether they are fellows, allied professionals, or practicing electrophysiologists. In “Cardiac Electrophysiology: Clinical Case Review” we are fortunate to have many true experts in the field contributing cases that they have encountered and summarizing the important learning objectives in a succinct way. The book covers clinical electrophysiology, device troubleshooting and analysis, as well as intracardiac electrogram analysis and ablation. It is our sincere hope that the readers of “Cardiac Electrophysiology: Clinical Case Review” will find the cases useful as a review of electrophysiology or in their day-to-day interactions with patients. Stanford CA, USA Charlottesville VA, USA Cleveland OH, USA
Amin Al-Ahmad Paul J. Wang John P. DiMarco
vii
Contents
Section 1 Ablation Case 1 . . . . . . . . . . . . . . . . . . . . . . . . . . . . . . . . . . . . . . . . . . . . . . . . . . . . . . . . . . . . . . . Amin Al-Ahmad
3
Case 2 . . . . . . . . . . . . . . . . . . . . . . . . . . . . . . . . . . . . . . . . . . . . . . . . . . . . . . . . . . . . . . . Michel Haissaguerre
7
Case 3 . . . . . . . . . . . . . . . . . . . . . . . . . . . . . . . . . . . . . . . . . . . . . . . . . . . . . . . . . . . . . . . Amin Al-Ahmad
11
Case 4 . . . . . . . . . . . . . . . . . . . . . . . . . . . . . . . . . . . . . . . . . . . . . . . . . . . . . . . . . . . . . . . Michel Haissaguerre
15
Case 5 . . . . . . . . . . . . . . . . . . . . . . . . . . . . . . . . . . . . . . . . . . . . . . . . . . . . . . . . . . . . . . . Anurag Gupta and Amin Al-Ahmad
19
Case 6 . . . . . . . . . . . . . . . . . . . . . . . . . . . . . . . . . . . . . . . . . . . . . . . . . . . . . . . . . . . . . . . Michel Haissaguerre
23
Case 7 . . . . . . . . . . . . . . . . . . . . . . . . . . . . . . . . . . . . . . . . . . . . . . . . . . . . . . . . . . . . . . . Michel Haissaguerre
29
Case 8 . . . . . . . . . . . . . . . . . . . . . . . . . . . . . . . . . . . . . . . . . . . . . . . . . . . . . . . . . . . . . . . Michel Haissaguerre
35
Case 9 . . . . . . . . . . . . . . . . . . . . . . . . . . . . . . . . . . . . . . . . . . . . . . . . . . . . . . . . . . . . . . . Alberto Diaz, Dimpi Patel, William R. Lewis, and Andrea Natale
39
Case 10 . . . . . . . . . . . . . . . . . . . . . . . . . . . . . . . . . . . . . . . . . . . . . . . . . . . . . . . . . . . . . . Antonio Rossillo, Sakis Themistoclakis, Aldo Bonso, Andrea Corrado, and Antonio Raviele
41
Case 11 . . . . . . . . . . . . . . . . . . . . . . . . . . . . . . . . . . . . . . . . . . . . . . . . . . . . . . . . . . . . . . Yash Y. Lokhandwala, Anoop K. Gupta, and Ranjan K. Thakur
47
ix
x
Contents
Case 12 . . . . . . . . . . . . . . . . . . . . . . . . . . . . . . . . . . . . . . . . . . . . . . . . . . . . . . . . . . . . . . Eric Buch, Shiro Nakahara, Marmar Vaseghi, Noel G. Boyle, and Kalyanam Shivkumar
51
Case 13 . . . . . . . . . . . . . . . . . . . . . . . . . . . . . . . . . . . . . . . . . . . . . . . . . . . . . . . . . . . . . . Bradley P. Knight
53
Case 14 . . . . . . . . . . . . . . . . . . . . . . . . . . . . . . . . . . . . . . . . . . . . . . . . . . . . . . . . . . . . . . M. Eyman Mortada, Jasbir S. Sra, and Masood Akhtar
57
Case 15 . . . . . . . . . . . . . . . . . . . . . . . . . . . . . . . . . . . . . . . . . . . . . . . . . . . . . . . . . . . . . . Eric Buch, Shiro Nakahara, Marmar Vaseghi, Noel G. Boyle, and Kalyanam Shivkumar
63
Case 16 . . . . . . . . . . . . . . . . . . . . . . . . . . . . . . . . . . . . . . . . . . . . . . . . . . . . . . . . . . . . . . Antonio Rossillo, Sakis Themistoclakis, Aldo Bonso, Andrea Corrado, and Antonio Raviele
65
Case 17 . . . . . . . . . . . . . . . . . . . . . . . . . . . . . . . . . . . . . . . . . . . . . . . . . . . . . . . . . . . . . . Bradley P. Knight
69
Case 18 . . . . . . . . . . . . . . . . . . . . . . . . . . . . . . . . . . . . . . . . . . . . . . . . . . . . . . . . . . . . . . Antonio Rossillo, Sakis Themistoclakis, Aldo Bonso, Andrea Corrado, and Antonio Raviele
71
Case 19 . . . . . . . . . . . . . . . . . . . . . . . . . . . . . . . . . . . . . . . . . . . . . . . . . . . . . . . . . . . . . . Yash Y. Lokhandwala, Anoop K. Gupta, and Ranjan K. Thakur
81
Case 20 . . . . . . . . . . . . . . . . . . . . . . . . . . . . . . . . . . . . . . . . . . . . . . . . . . . . . . . . . . . . . . Marmar Vaseghi, Noel G. Boyle, and Kalyanam Shivkumar
83
Case 21 . . . . . . . . . . . . . . . . . . . . . . . . . . . . . . . . . . . . . . . . . . . . . . . . . . . . . . . . . . . . . . Bradley P. Knight
85
Case 22 . . . . . . . . . . . . . . . . . . . . . . . . . . . . . . . . . . . . . . . . . . . . . . . . . . . . . . . . . . . . . . Antonio Rossillo, Sakis Themistoclakis, Aldo Bonso, Andrea Corrado, and Antonio Raviele
87
Case 23 . . . . . . . . . . . . . . . . . . . . . . . . . . . . . . . . . . . . . . . . . . . . . . . . . . . . . . . . . . . . . . Yash Y. Lokhandwala, Anoop K. Gupta, and Ranjan K. Thakur
95
Case 24 . . . . . . . . . . . . . . . . . . . . . . . . . . . . . . . . . . . . . . . . . . . . . . . . . . . . . . . . . . . . . . 101 Marmar Vaseghi, Noel G. Boyle, and Kalyanam Shivkumar Case 25 . . . . . . . . . . . . . . . . . . . . . . . . . . . . . . . . . . . . . . . . . . . . . . . . . . . . . . . . . . . . . . 103 Bradley P. Knight Case 26 . . . . . . . . . . . . . . . . . . . . . . . . . . . . . . . . . . . . . . . . . . . . . . . . . . . . . . . . . . . . . . 107 Richard H. Hongo and Andrea Natale
Contents
xi
Case 27 . . . . . . . . . . . . . . . . . . . . . . . . . . . . . . . . . . . . . . . . . . . . . . . . . . . . . . . . . . . . . . 111 M. Eyman Mortada, Jasbir S. Sra, and Masood Akhtar Case 28 . . . . . . . . . . . . . . . . . . . . . . . . . . . . . . . . . . . . . . . . . . . . . . . . . . . . . . . . . . . . . . 115 Yash Y. Lokhandwala, Anoop K. Gupta, and Ranjan K. Thakur Case 29 . . . . . . . . . . . . . . . . . . . . . . . . . . . . . . . . . . . . . . . . . . . . . . . . . . . . . . . . . . . . . . 123 Eric Buch, Shiro Nakahara, Marmar Vaseghi, Noel G. Boyle, and Kalyanam Shivkumar Case 30 . . . . . . . . . . . . . . . . . . . . . . . . . . . . . . . . . . . . . . . . . . . . . . . . . . . . . . . . . . . . . . 125 Luis C. Sáenz and Miguel A. Vacca Case 31 . . . . . . . . . . . . . . . . . . . . . . . . . . . . . . . . . . . . . . . . . . . . . . . . . . . . . . . . . . . . . . 133 Bradley P. Knight Case 32 . . . . . . . . . . . . . . . . . . . . . . . . . . . . . . . . . . . . . . . . . . . . . . . . . . . . . . . . . . . . . . 135 Luigi Di Biase, Rodney P. Horton, and Andrea Natale Case 33 . . . . . . . . . . . . . . . . . . . . . . . . . . . . . . . . . . . . . . . . . . . . . . . . . . . . . . . . . . . . . . 139 M. Eyman Mortada, Jasbir S. Sra, and Masood Akhtar Case 34 . . . . . . . . . . . . . . . . . . . . . . . . . . . . . . . . . . . . . . . . . . . . . . . . . . . . . . . . . . . . . . 143 Antonio Rossillo, Sakis Themistoclakis, Aldo Bonso, Andrea Corrado, and Antonio Raviele Case 35 . . . . . . . . . . . . . . . . . . . . . . . . . . . . . . . . . . . . . . . . . . . . . . . . . . . . . . . . . . . . . . 147 Yash Y. Lokhandwala, Anoop K. Gupta, and Ranjan K. Thakur Case 36 . . . . . . . . . . . . . . . . . . . . . . . . . . . . . . . . . . . . . . . . . . . . . . . . . . . . . . . . . . . . . . 151 Eric Buch, Shiro Nakahara, Marmar Vaseghi, Noel G. Boyle, and Kalyanam Shivkumar Case 37 . . . . . . . . . . . . . . . . . . . . . . . . . . . . . . . . . . . . . . . . . . . . . . . . . . . . . . . . . . . . . . 153 Bradley P. Knight Case 38 . . . . . . . . . . . . . . . . . . . . . . . . . . . . . . . . . . . . . . . . . . . . . . . . . . . . . . . . . . . . . . 157 Luigi Di Biase, Rodney P. Horton, and Andrea Natale Case 39 . . . . . . . . . . . . . . . . . . . . . . . . . . . . . . . . . . . . . . . . . . . . . . . . . . . . . . . . . . . . . . 159 M. Eyman Mortada, Jasbir S. Sra, and Masood Akhtar Case 40 . . . . . . . . . . . . . . . . . . . . . . . . . . . . . . . . . . . . . . . . . . . . . . . . . . . . . . . . . . . . . . 163 Antonio Rossillo, Sakis Themistoclakis, Aldo Bonso, Andrea Corrado, and Antonio Raviele Case 41 . . . . . . . . . . . . . . . . . . . . . . . . . . . . . . . . . . . . . . . . . . . . . . . . . . . . . . . . . . . . . . 181 Yash Y. Lokhandwala, Anoop K. Gupta, and Ranjan K. Thakur
xii
Case 42 . . . . . . . . . . . . . . . . . . . . . . . . . . . . . . . . . . . . . . . . . . . . . . . . . . . . . . . . . . . . . . 185 Eric Buch, Shiro Nakahara, Marmar Vaseghi, Noel G. Boyle, and Kalyanam Shivkumar Case 43 . . . . . . . . . . . . . . . . . . . . . . . . . . . . . . . . . . . . . . . . . . . . . . . . . . . . . . . . . . . . . . 189 Bradley P. Knight Case 44 . . . . . . . . . . . . . . . . . . . . . . . . . . . . . . . . . . . . . . . . . . . . . . . . . . . . . . . . . . . . . . 191 Antonio Rossillo, Sakis Themistoclakis, Aldo Bonso, Andrea Corrado, and Antonio Raviele Case 45 . . . . . . . . . . . . . . . . . . . . . . . . . . . . . . . . . . . . . . . . . . . . . . . . . . . . . . . . . . . . . . 195 Yash Y. Lokhandwala, Anoop K. Gupta, and Ranjan K. Thakur Case 46 . . . . . . . . . . . . . . . . . . . . . . . . . . . . . . . . . . . . . . . . . . . . . . . . . . . . . . . . . . . . . . 199 Eric Buch, Shiro Nakahara, Marmar Vaseghi, Noel G. Boyle, and Kalyanam Shivkumar Case 47 . . . . . . . . . . . . . . . . . . . . . . . . . . . . . . . . . . . . . . . . . . . . . . . . . . . . . . . . . . . . . . 201 Bradley P. Knight Case 48 . . . . . . . . . . . . . . . . . . . . . . . . . . . . . . . . . . . . . . . . . . . . . . . . . . . . . . . . . . . . . . 203 Antonio Rossillo, Sakis Themistoclakis, Aldo Bonso, Andrea Corrado, and Antonio Raviele Case 49 . . . . . . . . . . . . . . . . . . . . . . . . . . . . . . . . . . . . . . . . . . . . . . . . . . . . . . . . . . . . . . 217 Eric Buch, Shiro Nakahara, Marmar Vaseghi, Noel G. Boyle, and Kalyanam Shivkumar Case 50 . . . . . . . . . . . . . . . . . . . . . . . . . . . . . . . . . . . . . . . . . . . . . . . . . . . . . . . . . . . . . . 221 Bradley P. Knight Case 51 . . . . . . . . . . . . . . . . . . . . . . . . . . . . . . . . . . . . . . . . . . . . . . . . . . . . . . . . . . . . . . 223 Yash Y. Lokhandwala, Anoop K. Gupta, and Ranjan K. Thakur Case 52 . . . . . . . . . . . . . . . . . . . . . . . . . . . . . . . . . . . . . . . . . . . . . . . . . . . . . . . . . . . . . . 229 Eric Buch, Shiro Nakahara, Marmar Vaseghi, Noel G. Boyle, and Kalyanam Shivkumar Case 53 . . . . . . . . . . . . . . . . . . . . . . . . . . . . . . . . . . . . . . . . . . . . . . . . . . . . . . . . . . . . . . 233 Bradley P. Knight Case 54 . . . . . . . . . . . . . . . . . . . . . . . . . . . . . . . . . . . . . . . . . . . . . . . . . . . . . . . . . . . . . . 235 Yash Y. Lokhandwala, Anoop K. Gupta, and Ranjan K. Thakur Case 55 . . . . . . . . . . . . . . . . . . . . . . . . . . . . . . . . . . . . . . . . . . . . . . . . . . . . . . . . . . . . . . 241 Eric Buch, Shiro Nakahara, Marmar Vaseghi, Noel G. Boyle, and Kalyanam Shivkumar Case 56 . . . . . . . . . . . . . . . . . . . . . . . . . . . . . . . . . . . . . . . . . . . . . . . . . . . . . . . . . . . . . . 245 Bradley P. Knight
Contents
Contents
xiii
Case 57 . . . . . . . . . . . . . . . . . . . . . . . . . . . . . . . . . . . . . . . . . . . . . . . . . . . . . . . . . . . . . . 247 Yash Y. Lokhandwala, Anoop K. Gupta, and Ranjan K. Thakur Case 58 . . . . . . . . . . . . . . . . . . . . . . . . . . . . . . . . . . . . . . . . . . . . . . . . . . . . . . . . . . . . . . 253 Roopinder Sandhu, Dimpi Patel, William R. Lewis, and Andrea Natale Case 59 . . . . . . . . . . . . . . . . . . . . . . . . . . . . . . . . . . . . . . . . . . . . . . . . . . . . . . . . . . . . . . 263 M. Eyman Mortada, Jasbir S. Sra, and Masood Akhtar Case 60 . . . . . . . . . . . . . . . . . . . . . . . . . . . . . . . . . . . . . . . . . . . . . . . . . . . . . . . . . . . . . . 267 Matthew D. Hutchinson Case 61 . . . . . . . . . . . . . . . . . . . . . . . . . . . . . . . . . . . . . . . . . . . . . . . . . . . . . . . . . . . . . . 271 Ronald Lo, Henry H. Hsia, and Amin Al-Ahmad Case 62 . . . . . . . . . . . . . . . . . . . . . . . . . . . . . . . . . . . . . . . . . . . . . . . . . . . . . . . . . . . . . . 275 Richard H. Hongo and Andrea Natale Case 63 . . . . . . . . . . . . . . . . . . . . . . . . . . . . . . . . . . . . . . . . . . . . . . . . . . . . . . . . . . . . . . 279 David J. Callans Case 64 . . . . . . . . . . . . . . . . . . . . . . . . . . . . . . . . . . . . . . . . . . . . . . . . . . . . . . . . . . . . . . 283 J. David Burkhardt, Dimpi Patel, and Andrea Natale Case 65 . . . . . . . . . . . . . . . . . . . . . . . . . . . . . . . . . . . . . . . . . . . . . . . . . . . . . . . . . . . . . . 287 Matthew D. Hutchinson Case 66 . . . . . . . . . . . . . . . . . . . . . . . . . . . . . . . . . . . . . . . . . . . . . . . . . . . . . . . . . . . . . . 291 Matthew D. Hutchinson Case 67 . . . . . . . . . . . . . . . . . . . . . . . . . . . . . . . . . . . . . . . . . . . . . . . . . . . . . . . . . . . . . . 295 Matthew D. Hutchinson Case 68 . . . . . . . . . . . . . . . . . . . . . . . . . . . . . . . . . . . . . . . . . . . . . . . . . . . . . . . . . . . . . . 299 Matthew D. Hutchinson Case 69 . . . . . . . . . . . . . . . . . . . . . . . . . . . . . . . . . . . . . . . . . . . . . . . . . . . . . . . . . . . . . . 303 Matthew D. Hutchinson Case 70 . . . . . . . . . . . . . . . . . . . . . . . . . . . . . . . . . . . . . . . . . . . . . . . . . . . . . . . . . . . . . . 307 Matthew D. Hutchinson Case 71 . . . . . . . . . . . . . . . . . . . . . . . . . . . . . . . . . . . . . . . . . . . . . . . . . . . . . . . . . . . . . . 311 Matthew D. Hutchinson Section 2 Devices Case 72 . . . . . . . . . . . . . . . . . . . . . . . . . . . . . . . . . . . . . . . . . . . . . . . . . . . . . . . . . . . . . . 317 Amin Al-Ahmad and Paul J. Wang
xiv
Case 73 . . . . . . . . . . . . . . . . . . . . . . . . . . . . . . . . . . . . . . . . . . . . . . . . . . . . . . . . . . . . . . 319 Amin Al-Ahmad and Paul J. Wang Case 74 . . . . . . . . . . . . . . . . . . . . . . . . . . . . . . . . . . . . . . . . . . . . . . . . . . . . . . . . . . . . . . 321 Kenneth A. Ellenbogen Case 75 . . . . . . . . . . . . . . . . . . . . . . . . . . . . . . . . . . . . . . . . . . . . . . . . . . . . . . . . . . . . . . 325 Nora Goldschlager Case 76 . . . . . . . . . . . . . . . . . . . . . . . . . . . . . . . . . . . . . . . . . . . . . . . . . . . . . . . . . . . . . . 327 Fred M. Kusumoto, Jennifer Crain, and Nora Goldschlager Case 77 . . . . . . . . . . . . . . . . . . . . . . . . . . . . . . . . . . . . . . . . . . . . . . . . . . . . . . . . . . . . . . 331 Gregory M. Marcus and Nora Goldschlager Case 78 . . . . . . . . . . . . . . . . . . . . . . . . . . . . . . . . . . . . . . . . . . . . . . . . . . . . . . . . . . . . . . 335 Amin Al-Ahmad and Paul J. Wang Case 79 . . . . . . . . . . . . . . . . . . . . . . . . . . . . . . . . . . . . . . . . . . . . . . . . . . . . . . . . . . . . . . 337 Kenneth A. Ellenbogen and Rod Bolanos Case 80 . . . . . . . . . . . . . . . . . . . . . . . . . . . . . . . . . . . . . . . . . . . . . . . . . . . . . . . . . . . . . . 341 Nora Goldschlager Case 81 . . . . . . . . . . . . . . . . . . . . . . . . . . . . . . . . . . . . . . . . . . . . . . . . . . . . . . . . . . . . . . 343 Fred M. Kusumoto, Jennifer Crain, and Nora Goldschlager Case 82 . . . . . . . . . . . . . . . . . . . . . . . . . . . . . . . . . . . . . . . . . . . . . . . . . . . . . . . . . . . . . . 345 Amin Al-Ahmad and Paul J. Wang Case 83 . . . . . . . . . . . . . . . . . . . . . . . . . . . . . . . . . . . . . . . . . . . . . . . . . . . . . . . . . . . . . . 347 Kenneth A. Ellenbogen Case 84 . . . . . . . . . . . . . . . . . . . . . . . . . . . . . . . . . . . . . . . . . . . . . . . . . . . . . . . . . . . . . . 349 Nora Goldschlager Case 85 . . . . . . . . . . . . . . . . . . . . . . . . . . . . . . . . . . . . . . . . . . . . . . . . . . . . . . . . . . . . . . 351 Anurag Gupta and Amin Al-Ahmad Case 86 . . . . . . . . . . . . . . . . . . . . . . . . . . . . . . . . . . . . . . . . . . . . . . . . . . . . . . . . . . . . . . 353 Fred M. Kusumoto, Jennifer Crain, and Nora Goldschlager Case 87 . . . . . . . . . . . . . . . . . . . . . . . . . . . . . . . . . . . . . . . . . . . . . . . . . . . . . . . . . . . . . . 357 Kenneth A. Ellenbogen Case 88 . . . . . . . . . . . . . . . . . . . . . . . . . . . . . . . . . . . . . . . . . . . . . . . . . . . . . . . . . . . . . . 361 Nora Goldschlager Case 89 . . . . . . . . . . . . . . . . . . . . . . . . . . . . . . . . . . . . . . . . . . . . . . . . . . . . . . . . . . . . . . 363 Kenneth A. Ellenbogen, Rod Bolanos, and Mark A. Wood
Contents
Contents
xv
Case 90 . . . . . . . . . . . . . . . . . . . . . . . . . . . . . . . . . . . . . . . . . . . . . . . . . . . . . . . . . . . . . . 369 Fred M. Kusumoto, Jennifer Crain, and Nora Goldschlager Case 91 . . . . . . . . . . . . . . . . . . . . . . . . . . . . . . . . . . . . . . . . . . . . . . . . . . . . . . . . . . . . . . 371 M. Eyman Mortada, Jasbir S. Sra, and Masood Akhtar Case 92 . . . . . . . . . . . . . . . . . . . . . . . . . . . . . . . . . . . . . . . . . . . . . . . . . . . . . . . . . . . . . . 373 Amin Al-Ahmad Case 93 . . . . . . . . . . . . . . . . . . . . . . . . . . . . . . . . . . . . . . . . . . . . . . . . . . . . . . . . . . . . . . 375 M. Eyman Mortada, Jasbir S. Sra, and Masood Akhtar Case 94 . . . . . . . . . . . . . . . . . . . . . . . . . . . . . . . . . . . . . . . . . . . . . . . . . . . . . . . . . . . . . . 379 Kenneth A. Ellenbogen and Rod Bolanos Case 95 . . . . . . . . . . . . . . . . . . . . . . . . . . . . . . . . . . . . . . . . . . . . . . . . . . . . . . . . . . . . . . 383 Byron K. Lee Case 96 . . . . . . . . . . . . . . . . . . . . . . . . . . . . . . . . . . . . . . . . . . . . . . . . . . . . . . . . . . . . . . 387 Amin Al-Ahmad and Paul J. Wang Case 97 . . . . . . . . . . . . . . . . . . . . . . . . . . . . . . . . . . . . . . . . . . . . . . . . . . . . . . . . . . . . . . 389 Kenneth A. Ellenbogen and Rod Bolanos Case 98 . . . . . . . . . . . . . . . . . . . . . . . . . . . . . . . . . . . . . . . . . . . . . . . . . . . . . . . . . . . . . . 391 Kenneth A. Ellenbogen Case 99 . . . . . . . . . . . . . . . . . . . . . . . . . . . . . . . . . . . . . . . . . . . . . . . . . . . . . . . . . . . . . . 395 Amin Al-Ahmad and Paul J. Wang Case 100 . . . . . . . . . . . . . . . . . . . . . . . . . . . . . . . . . . . . . . . . . . . . . . . . . . . . . . . . . . . . . 399 Kenneth A. Ellenbogen Case 101 . . . . . . . . . . . . . . . . . . . . . . . . . . . . . . . . . . . . . . . . . . . . . . . . . . . . . . . . . . . . . 403 Amin Al-Ahmad and Paul J. Wang Case 102 . . . . . . . . . . . . . . . . . . . . . . . . . . . . . . . . . . . . . . . . . . . . . . . . . . . . . . . . . . . . . 405 Kenneth A. Ellenbogen Case 103 . . . . . . . . . . . . . . . . . . . . . . . . . . . . . . . . . . . . . . . . . . . . . . . . . . . . . . . . . . . . . 407 Amin Al-Ahmad and Paul J. Wang Case 104 . . . . . . . . . . . . . . . . . . . . . . . . . . . . . . . . . . . . . . . . . . . . . . . . . . . . . . . . . . . . . 411 Kenneth A. Ellenbogen Case 105 . . . . . . . . . . . . . . . . . . . . . . . . . . . . . . . . . . . . . . . . . . . . . . . . . . . . . . . . . . . . . 415 Kenneth A. Ellenbogen and Rod Bolanos Case 106 . . . . . . . . . . . . . . . . . . . . . . . . . . . . . . . . . . . . . . . . . . . . . . . . . . . . . . . . . . . . . 419 Kenneth A. Ellenbogen
xvi
Case 107 . . . . . . . . . . . . . . . . . . . . . . . . . . . . . . . . . . . . . . . . . . . . . . . . . . . . . . . . . . . . . 423 Paul A. Friedman and Charles D. Swerdlow Case 108 . . . . . . . . . . . . . . . . . . . . . . . . . . . . . . . . . . . . . . . . . . . . . . . . . . . . . . . . . . . . . 427 Paul A. Friedman and Charles D. Swerdlow Case 109 . . . . . . . . . . . . . . . . . . . . . . . . . . . . . . . . . . . . . . . . . . . . . . . . . . . . . . . . . . . . . 431 Paul A. Friedman and Charles D. Swerdlow Case 110 . . . . . . . . . . . . . . . . . . . . . . . . . . . . . . . . . . . . . . . . . . . . . . . . . . . . . . . . . . . . . 433 Paul A. Friedman and Charles D. Swerdlow Case 111 . . . . . . . . . . . . . . . . . . . . . . . . . . . . . . . . . . . . . . . . . . . . . . . . . . . . . . . . . . . . . 439 Paul A. Friedman and Charles D. Swerdlow Case 112 . . . . . . . . . . . . . . . . . . . . . . . . . . . . . . . . . . . . . . . . . . . . . . . . . . . . . . . . . . . . . 441 Paul A. Friedman and Charles D. Swerdlow Case 113 . . . . . . . . . . . . . . . . . . . . . . . . . . . . . . . . . . . . . . . . . . . . . . . . . . . . . . . . . . . . . 443 Paul A. Friedman and Charles D. Swerdlow Case 114 . . . . . . . . . . . . . . . . . . . . . . . . . . . . . . . . . . . . . . . . . . . . . . . . . . . . . . . . . . . . . 447 Paul A. Friedman and Charles D. Swerdlow Case 115 . . . . . . . . . . . . . . . . . . . . . . . . . . . . . . . . . . . . . . . . . . . . . . . . . . . . . . . . . . . . . 451 Paul A. Friedman and Charles D. Swerdlow Case 116 . . . . . . . . . . . . . . . . . . . . . . . . . . . . . . . . . . . . . . . . . . . . . . . . . . . . . . . . . . . . . 453 Paul A. Friedman and Charles D. Swerdlow Case 117 . . . . . . . . . . . . . . . . . . . . . . . . . . . . . . . . . . . . . . . . . . . . . . . . . . . . . . . . . . . . . 455 Paul A. Friedman and Charles D. Swerdlow Case 118 . . . . . . . . . . . . . . . . . . . . . . . . . . . . . . . . . . . . . . . . . . . . . . . . . . . . . . . . . . . . . 459 Paul A. Friedman and Charles D. Swerdlow Case 119 . . . . . . . . . . . . . . . . . . . . . . . . . . . . . . . . . . . . . . . . . . . . . . . . . . . . . . . . . . . . . 465 M. Eyman Mortada, Jasbir S. Sra, and Masood Akhtar Case 120 . . . . . . . . . . . . . . . . . . . . . . . . . . . . . . . . . . . . . . . . . . . . . . . . . . . . . . . . . . . . . 469 Paul A. Friedman and Charles D. Swerdlow Case 121 . . . . . . . . . . . . . . . . . . . . . . . . . . . . . . . . . . . . . . . . . . . . . . . . . . . . . . . . . . . . . 471 Kenneth A. Ellenbogen Case 122 . . . . . . . . . . . . . . . . . . . . . . . . . . . . . . . . . . . . . . . . . . . . . . . . . . . . . . . . . . . . . 473 Anurag Gupta Case 123 . . . . . . . . . . . . . . . . . . . . . . . . . . . . . . . . . . . . . . . . . . . . . . . . . . . . . . . . . . . . . 477 Ronald Lo, Amin Al-Ahmad, and Paul J. Wang
Contents
Contents
xvii
Case 124 . . . . . . . . . . . . . . . . . . . . . . . . . . . . . . . . . . . . . . . . . . . . . . . . . . . . . . . . . . . . . 481 Ronald Lo, Amin Al-Ahmad, and Paul J. Wang Case 125 . . . . . . . . . . . . . . . . . . . . . . . . . . . . . . . . . . . . . . . . . . . . . . . . . . . . . . . . . . . . . 483 Ronald Lo, Amin Al-Ahmad, and Paul J. Wang Section 3 Clinical Cases Case 126 . . . . . . . . . . . . . . . . . . . . . . . . . . . . . . . . . . . . . . . . . . . . . . . . . . . . . . . . . . . . . 487 Mehmet K. Aktas, Abrar H. Shah, and James P. Daubert Case 127 . . . . . . . . . . . . . . . . . . . . . . . . . . . . . . . . . . . . . . . . . . . . . . . . . . . . . . . . . . . . . 491 Loren P. Budge and John P. DiMarco Case 128 . . . . . . . . . . . . . . . . . . . . . . . . . . . . . . . . . . . . . . . . . . . . . . . . . . . . . . . . . . . . . 495 David J. Callans Case 129 . . . . . . . . . . . . . . . . . . . . . . . . . . . . . . . . . . . . . . . . . . . . . . . . . . . . . . . . . . . . . 497 Andrew E. Darby and John P. DiMarco Case 130 . . . . . . . . . . . . . . . . . . . . . . . . . . . . . . . . . . . . . . . . . . . . . . . . . . . . . . . . . . . . . 501 Thomas J. Sawyer, Burr W. Hall, and James P. Daubert Case 131 . . . . . . . . . . . . . . . . . . . . . . . . . . . . . . . . . . . . . . . . . . . . . . . . . . . . . . . . . . . . . 505 John P. DiMarco Case 132 . . . . . . . . . . . . . . . . . . . . . . . . . . . . . . . . . . . . . . . . . . . . . . . . . . . . . . . . . . . . . 509 John P. DiMarco Case 133 . . . . . . . . . . . . . . . . . . . . . . . . . . . . . . . . . . . . . . . . . . . . . . . . . . . . . . . . . . . . . 511 John P. DiMarco Case 134 . . . . . . . . . . . . . . . . . . . . . . . . . . . . . . . . . . . . . . . . . . . . . . . . . . . . . . . . . . . . . 513 John P. DiMarco Case 135 . . . . . . . . . . . . . . . . . . . . . . . . . . . . . . . . . . . . . . . . . . . . . . . . . . . . . . . . . . . . . 517 John P. DiMarco Case 136 . . . . . . . . . . . . . . . . . . . . . . . . . . . . . . . . . . . . . . . . . . . . . . . . . . . . . . . . . . . . . 521 John P. DiMarco Case 137 . . . . . . . . . . . . . . . . . . . . . . . . . . . . . . . . . . . . . . . . . . . . . . . . . . . . . . . . . . . . . 523 John P. DiMarco Case 138 . . . . . . . . . . . . . . . . . . . . . . . . . . . . . . . . . . . . . . . . . . . . . . . . . . . . . . . . . . . . . 527 John P. DiMarco Case 139 . . . . . . . . . . . . . . . . . . . . . . . . . . . . . . . . . . . . . . . . . . . . . . . . . . . . . . . . . . . . . 529 John P. DiMarco
xviii
Case 140 . . . . . . . . . . . . . . . . . . . . . . . . . . . . . . . . . . . . . . . . . . . . . . . . . . . . . . . . . . . . . 531 John P. DiMarco Case 141 . . . . . . . . . . . . . . . . . . . . . . . . . . . . . . . . . . . . . . . . . . . . . . . . . . . . . . . . . . . . . 535 John P. DiMarco Case 142 . . . . . . . . . . . . . . . . . . . . . . . . . . . . . . . . . . . . . . . . . . . . . . . . . . . . . . . . . . . . . 539 John P. DiMarco Case 143 . . . . . . . . . . . . . . . . . . . . . . . . . . . . . . . . . . . . . . . . . . . . . . . . . . . . . . . . . . . . . 545 John P. DiMarco Case 144 . . . . . . . . . . . . . . . . . . . . . . . . . . . . . . . . . . . . . . . . . . . . . . . . . . . . . . . . . . . . . 551 John P. DiMarco Case 145 . . . . . . . . . . . . . . . . . . . . . . . . . . . . . . . . . . . . . . . . . . . . . . . . . . . . . . . . . . . . . 555 John P. DiMarco Case 146 . . . . . . . . . . . . . . . . . . . . . . . . . . . . . . . . . . . . . . . . . . . . . . . . . . . . . . . . . . . . . 559 Brett A. Faulknier, David T. Huang, and James P. Daubert Case 147 . . . . . . . . . . . . . . . . . . . . . . . . . . . . . . . . . . . . . . . . . . . . . . . . . . . . . . . . . . . . . 565 Stefan H. Hohnloser and Joachim R. Ehrlich Case 148 . . . . . . . . . . . . . . . . . . . . . . . . . . . . . . . . . . . . . . . . . . . . . . . . . . . . . . . . . . . . . 567 Joachim Ehrlich and Stefan H. Hohnloser Case 149 . . . . . . . . . . . . . . . . . . . . . . . . . . . . . . . . . . . . . . . . . . . . . . . . . . . . . . . . . . . . . 571 Bradley P. Knight Case 150 . . . . . . . . . . . . . . . . . . . . . . . . . . . . . . . . . . . . . . . . . . . . . . . . . . . . . . . . . . . . . 575 Bradley P. Knight Case 151 . . . . . . . . . . . . . . . . . . . . . . . . . . . . . . . . . . . . . . . . . . . . . . . . . . . . . . . . . . . . . 577 Bradley P. Knight Case 152 . . . . . . . . . . . . . . . . . . . . . . . . . . . . . . . . . . . . . . . . . . . . . . . . . . . . . . . . . . . . . 579 Andrew D. Krahn Case 153 . . . . . . . . . . . . . . . . . . . . . . . . . . . . . . . . . . . . . . . . . . . . . . . . . . . . . . . . . . . . . 583 Byron K. Lee, Melvin M. Scheinman, and Zian H. Tseng Case 154 . . . . . . . . . . . . . . . . . . . . . . . . . . . . . . . . . . . . . . . . . . . . . . . . . . . . . . . . . . . . . 585 Srijoy Mahapatra Case 155 . . . . . . . . . . . . . . . . . . . . . . . . . . . . . . . . . . . . . . . . . . . . . . . . . . . . . . . . . . . . . 589 Pamela K. Mason Case 156 . . . . . . . . . . . . . . . . . . . . . . . . . . . . . . . . . . . . . . . . . . . . . . . . . . . . . . . . . . . . . 593 Pamela K. Mason
Contents
Contents
xix
Case 157 . . . . . . . . . . . . . . . . . . . . . . . . . . . . . . . . . . . . . . . . . . . . . . . . . . . . . . . . . . . . . 595 Pamela K. Mason Case 158 . . . . . . . . . . . . . . . . . . . . . . . . . . . . . . . . . . . . . . . . . . . . . . . . . . . . . . . . . . . . . 597 Pamela K. Mason Case 159 . . . . . . . . . . . . . . . . . . . . . . . . . . . . . . . . . . . . . . . . . . . . . . . . . . . . . . . . . . . . . 599 Lisa G. Umphrey and John Paul Mounsey Case 160 . . . . . . . . . . . . . . . . . . . . . . . . . . . . . . . . . . . . . . . . . . . . . . . . . . . . . . . . . . . . . 603 James A. Reiffel Case 161 . . . . . . . . . . . . . . . . . . . . . . . . . . . . . . . . . . . . . . . . . . . . . . . . . . . . . . . . . . . . . 607 James A. Reiffel Case 162 . . . . . . . . . . . . . . . . . . . . . . . . . . . . . . . . . . . . . . . . . . . . . . . . . . . . . . . . . . . . . 611 Jens Seiler and William G. Stevenson Case 163 . . . . . . . . . . . . . . . . . . . . . . . . . . . . . . . . . . . . . . . . . . . . . . . . . . . . . . . . . . . . . 617 Jens Seiler and William G. Stevenson Case 164 . . . . . . . . . . . . . . . . . . . . . . . . . . . . . . . . . . . . . . . . . . . . . . . . . . . . . . . . . . . . . 625 Jens Seiler and William G. Stevenson Case 165 . . . . . . . . . . . . . . . . . . . . . . . . . . . . . . . . . . . . . . . . . . . . . . . . . . . . . . . . . . . . . 631 J. Jason West and John Paul Mounsey Case 166 . . . . . . . . . . . . . . . . . . . . . . . . . . . . . . . . . . . . . . . . . . . . . . . . . . . . . . . . . . . . . 635 Darren Traub, James P. Daubert, and Spencer Rosero Case 167 . . . . . . . . . . . . . . . . . . . . . . . . . . . . . . . . . . . . . . . . . . . . . . . . . . . . . . . . . . . . . 639 Vikas P. Kuriachan and George D. Veenhuyzen Case 168 . . . . . . . . . . . . . . . . . . . . . . . . . . . . . . . . . . . . . . . . . . . . . . . . . . . . . . . . . . . . . 643 Vikas P. Kuriachan and George D. Veenhuyzen Case 169 . . . . . . . . . . . . . . . . . . . . . . . . . . . . . . . . . . . . . . . . . . . . . . . . . . . . . . . . . . . . . 647 Jeffrey D. Booker and George D. Veenhuyzen Case 170 . . . . . . . . . . . . . . . . . . . . . . . . . . . . . . . . . . . . . . . . . . . . . . . . . . . . . . . . . . . . . 649 M. Eyman Mortada, Jasbir S. Sra, and Masood Akhtar Case 171 . . . . . . . . . . . . . . . . . . . . . . . . . . . . . . . . . . . . . . . . . . . . . . . . . . . . . . . . . . . . . 653 M. Eyman Mortada, Jasbir S. Sra, and Masood Akhtar Index . . . . . . . . . . . . . . . . . . . . . . . . . . . . . . . . . . . . . . . . . . . . . . . . . . . . . . . . . . . . . . . . 655
Section Ablation
I
Case 1 Amin Al-Ahmad
Case Summary A 72-year-old male with a history of aortic sinus of valsalva rupture and repair 30 years prior presents with palpitations. The patient reports having symptoms of shortness of breath and fatigue with palpitations. He is currently taking only aspirin. He has had cardioversions in the past for this arrhythmia and would like to consider ablation. He does not want to take anti-arrhythmic medications. His 12-lead ECG (Fig. 1.1) shows a 2:1 atrial tachyarrhythmia at a rate of approximately 180 bpm. The P-waves in the inferior leads are negative; however there is an isoelectric segment between P-waves.
What maneuvers would be important to elucidate the diagnosis during electrophysiology study and ablation?
Case Discussion This tachycardia may represent a focal atrial tachycardia given the isoelectric segment on the 12-lead ECG. However, in patients with prior cardiac surgery, atrial flutters should be considered. The possibility that this tachycardia is typical isthmus-dependent atrial flutter should be explored using entrainment pacing at the tricuspid valve (TV) to inferior
Fig. 1.1 Twelve-lead ECG showing atrial arrhythmia. Note the negative atrial deflections in the inferior leads and the isoelectric segment between them
A. Al-Ahmad School of Medicine, Stanford University, 300 Pasteur Drive, H-2146, Stanford, CA 94305 A. Natale et al. (eds.), Cardiac Electrophysiology, DOI: 10.1007/978-1-84996-390-9_1, © Springer-Verlag London Limited 2011
3
4
A. Al-Ahmad
vena cava (IVC) isthmus. Reverse typical atrial flutter would be unlikely given the morphology of the flutter waves. Scarrelated atrial flutters should also be considered. Ablation of all potential atrial flutter circuits in patients post cardiac surgery may reduce the likelihood of recurrence.1
In this case, entrainment at the TV-IVC isthmus demonstrated a near perfect PPI and ablation at the TV-IVC isthmus terminated the atrial flutter (Figs. 1.2 and 1.3). The isoelectric segment between flutter waves likely represents slow atrial conduction in atrial flutter rather than atrial tachycardia.
200ms
I aVF V6 hRA p HIS p HIS d CS 9,10 CS 7,8 CS 5,6 CS 3,4 CS 1,2 RV p ABL
PPI
TCL
ABL d Stim 4
12:22:16 PM
12:22:17 PM
Fig. 1.2 Pacing using the ablation catheter positioned at the tricuspid valve-inferior vena cava (TV-IVC) isthmus. Concealed entrainment and a near perfect post-pacing interval are noted
Case 1
5 500ms
I
aVF hRA p HIS p HIS d CS 9,10 CS 7,8 CS 5,6 CS 3,4 CS 1,2 RV p ABL ABL d Stim 4
1:01:47 PM
1:01:48 PM
1:01:49 PM
Fig. 1.3 Application of RF energy in the TV-IVC area leads to termination of the arrhythmia
Reference 1. Verma A, Marrouche NF, Seshadri N, Schweikert RA, Bhargava M, Burkhardt JD, Kilicaslan F, Cummings J, Saliba W, Natale A. Importance of ablating all potential right atrial flutter circuits in postcardiac surgery patients. J Am Coll Cardiol. July 2004;44(2): 409–414.
1:01:50 PM
1:01:51 PM
Case 2 Michel Haissaguerre
Case Summary A 47-year-old male with a 4-year history of symptomatic, drug-resistant lone paroxysmal atrial fibrillation (AF) was referred for a first ablation procedure. He suffered from daily AF episodes that lasted a maximum of 8 h. Episodes of
AF always started following monomorphic atrial ectopy (Fig. 2.1). A decapolar catheter (Xtrem, ELA Medical, Le-PlessisRobinson, France) was inserted inside the coronary sinus (CS) while the ablation catheter (Thermocool Biosense Webster, Diamond Bar, CA) and a decapolar circumferential
I II III aVR aVL aVF V1 V2 V3 V4 V5 V6
Fig. 2.1 Twelve-lead ECG. Sinus rhythm and short coupling atrial ectopies (with functional left bundle branch block [LBBB])
M. Haissaguerre Department of Cardiology and Electrophysiology, CHU de Bordeaux, Hospital Cardiologique de Haut Leveque, Avenue de Magellan, Pessac 33604, France A. Natale et al. (eds.), Cardiac Electrophysiology, DOI: 10.1007/978-1-84996-390-9_2, © Springer-Verlag London Limited 2011
7
8
M. Haissaguerre
catheter (Lasso, Biosense Webster, Diamond Bar, CA) were introduced through a long sheath in the left atrium (LA). The circumferential catheter was placed in the left superior pulmonary vein (LSPV) and recorded venous and atrial potentials (Fig. 2.2). What mechanism is illustrated and what action is required?
Case Discussion Figure 2.1 shows a recording of atrial ectopics with a short coupling interval (also note the functional left bundle branch block [LBBB]). The P-wave morphology during ectopy is flat in the lateral leads, positive in the inferior leads and in lead V1, suggesting they originate from the LSPV. Endocardial tracings from the LSPV (Fig. 2.2) show two separated potentials during sinus rhythm (Fig. 2.2, first complex). The first potential (white star) represents activation of
the adjacent LA and is synchronous with the second half of the P-wave (in the right PVs it should be the first part of the P-wave). The second potential reflects local activity from the PV striated musculature (black star). When ectopy occurs in the PV (Fig. 2.2, second complex), there is a reversal of the described activation sequence, with the PV potential preceding the atrial potential. This pattern of reverse activation in a dead-end structure during ectopic triggered AF evidence for the arrhythmogenic potential of that PV. Mapping of the earliest site of activity during ectopy allows identification of discrete sites inside the vein, while the atrial exit site is dependent on the anatomy of the PV-LA connecting fascicles. Given that arrhythmia recurrence can occur from either the pulmonary vein that is active at the time of the procedure or any other PV, complete electrical isolation of all PVs has to be carried out with a series of coalescent RF applications using a dedicated PV circumferential catheter to help with mapping. Ablation is performed outside the vein (within 1–2 cm of the PV ostia) for right PVs and for the posterior part
I II V1
PV 1-2 PV 2-3 PV 3-4 PV 4-5 PV 5-6 PV 6-7 PV 7-8 PV 8-9 PV 9-10 CS 1-2 CS 3-4 CS 5-6 CS 7-8 CS 9-10
Fig. 2.2 Endocardial tracings from the left superior pulmonary vein (LSPV). Recording from a decapolar circumferential catheter inserted inside the LSPV (PV 1–2 to PV 9–10) and a decapolar catheter inserted into the coronary sinus (CS 1–2 to CS 9–10). During sinus rhythm (first
complex), there existed the presence of two separated potentials (one atrium, white star followed by one venous, black star). During ectopy (second complex), reversal of the described activation sequence with the PV potential preceding the atrial potential occurred
Case 2
9
of the left PVs; however, due to the ridge between the left pulmonary veins and the left atrial appendage (LAA), catheter stability is an issue for ablation of the anterior aspect of the left PV’s, and ablation is often within 1 mm of the veins. In this case, ablation was started at the low anterior LSPV (pole 5) where the earliest activity was located and where a reverse in PV polarity was observed, both criteria pointing at a local anatomical connection (Fig. 2.3, panel A). Ablation at this point delayed the venous potentials (Fig. 2.3, panel B), and a second anatomical breakthrough was subsequently ablated at the upper part of the vein (pole 1). The ectopic beats stopped (Fig. 2.3, panel C) and the venous potentials became dissociated (Fig. 2.4).Ablation of the left inferior PV (LIPV) was performed in the same way. For the right veins,
A
segmental or circumferential ablation with a continuous circular lesion can be performed depending on the operator’s preference. When doing continuous circumferential lesions, it is unusual to achieve PV isolation without further ablation targeting the earliest PV activity or sites of reverse PV polarity as recorded on the circumferential mapping catheter, as in Fig. 2.2, indicating a residual anatomical connection on the line of ablation. After ablation, nine attempts at induction (with bursts up to a cycle length of 200 ms) at three different places (CS and both appendages) could not induce sustained arrhythmia, predicting a favorable clinical outcome. This case illustrates a typical ablation of paroxysmal AF where it was clearly demonstrated that the arrhythmogenic ectopic beats triggering AF originated from the LSPV.
B
C
I II
V1
PV 1-2 PV 2-3 PV 3-4 PV 4-5 PV 5-6 PV 6-7 PV 7-8 PV 8-9 PV 9-10 CS 1-2 CS 3-4 CS 5-6 CS 7-8 CS 9-10
Fig. 2.3 Endocardial tracings recorded during sinus rhythm from the left superior pulmonary (LSPV) vein. Recording from a decapolar circumferential catheter inserted inside the LSPV (PV 1–2 to PV 9–10) and a decapolar catheter inserted into the coronary sinus (CS 1–2 to CS
9–10). During ablation targeting the earliest venous potential (black star), progressive slowing of the conduction to the vein (panel A and B) to the complete block (panel C)
10
M. Haissaguerre
I
17
18
19
20
21
22
23
24
25
26
II
V1
PV 1-2 PV 2-3 PV 3-4 PV 4-5 PV 5-6 PV 6-7 PV 7-8 PV 8-9 PV 9-10 CS 1-2 CS 3-4 CS 5-6 CS 7-8 CS 9-10
Fig. 2.4 Endocardial tracings recorded during sinus rhythm from the left superior pulmonary vein (LSPV). Recording from a decapolar circumferential catheter inserted inside the LSPV (PV 1–2 to PV 9–10)
and a decapolar catheter inserted into the coronary sinus (CS 1–2 to CS 9–10). Dissociation of the venous potential (black star) with a slow automatic activity
Bibliography
Haïssaguerre M, Hocini M, Sanders P, et al. Localized sources maintaining atrial fibrillation organized by prior ablation. Circulation. 2006;113(5):616-625. Sanders P, Hocini M, Jais P, et al. Characterization of focal atrial tachycardia using high-density mapping. J Am Coll Cardiol. 2005 December 6;46(11):2088-2099. Mohamed U, Skanes AC, Gula LJ, et al. A novel pacing maneuver to localize focal atrial tachycardia. J Cardiovasc Electrophys. 2007;18(1):1-6.
Haïssaguerre M, Shah DC, Jaïs P, et al. Electrophysiological breakthroughs from the left atrium to the pulmonary veins. Circulation. 2000;102:2463-2465. Haissaguerre M, Sanders P, Hocini M, et al. Catheter ablation of longlasting persistent atrial fibrillation: critical structures for termination. J Cardiovasc Electrophys. 2005;16(11):1125-1137.
Case 3 Amin Al-Ahmad
Case Summary A 51-year-old male with a history of pulmonary fibrosis and a single lung transplant is found to have atrial flutter on routine follow-up. Initially he is unaware of this rhythm, but later recalls an increase in fatigue over the previous week. He is referred for evaluation by an electrophysiologist. A Holter monitor shows his heart rate to be approximately 90 on average. He has periods of normal sinus rhythm and periods of atrial flutter. A 12-lead ECG during atrial flutter is shown in Fig. 3.1. Over the next few months his atrial flutter becomes persistent and he continues to complain of fatigue. He is talking multiple medications post transplant to prevent rejection and has some renal insufficiency. Drug therapy with anti-
arrhythmic medications does not seem to be an attractive option given his multiple co-morbid conditions. And despite better rate control he remains symptomatic and is taken to the electrophysiology suite. Where is this tachycardia origin, and what pacing maneuvers may help determine the best area to ablate?
Case Discussion Atrial flutter can be common immediately after lung transplantation. Canine models have suggested a substrate for atrial flutter in the left atrium due to suture lines. In addition,
Fig. 3.1 Twelve-lead ECG of atrial flutter. Note the peaked and strongly positive flutter waves in lead V1
A. Al-Ahmad School of Medicine, Stanford University, 300 Pasteur Drive, H-2146, Stanford, CA 94305 A. Natale et al. (eds.), Cardiac Electrophysiology, DOI: 10.1007/978-1-84996-390-9_3, © Springer-Verlag London Limited 2011
11
12
A. Al-Ahmad
right-sided atrial flutters may also develop as these patients can commonly have right atrial dilation due to potentially long-standing increased right-sided pressures. To differentiate right- and left-sided atrial flutter, the flutter wave morphology can be useful. In this case a strongly positive flutter wave in the anterior precordial leads suggests that the flutter is left-sided. In addition, entrainment from areas on the right side can be done quickly and can help
determine if the flutter is left-sided or right-sided. Entrainment can further distinguish the critical isthmus for the atrial flutter and help guide ablation. In this case, entrainment on the right side clearly demonstrated that the flutter was not right-sided (Fig. 3.2). Entrainment in the left atrium near the right inferior pulmonary vein was near perfect. Ablation in that area to create a line to the mitral valve caused the flutter to terminate (Figs. 3.3 and 3.4).
200ms
I II V1 V6
S
RA 9,10 RA 7,8 RA 5,6
PPI
RA 3,4
TCL
RA 1,2 CS 9,10 CS 7,8 CS 5,6 CS 3,4 CS 1,2 HIS p HIS m
C
HIS d ABL ABL d Stim 3
S1
S1
S1 9:42:27 AM
S1
S1 9:42:28 AM
Fig. 3.2 Pacing in the right atrium shows a long post-pacing interval. This was the case from multiple areas in the right atrium
Case 3
13 200ms
I II V1 V6 RA 9,10 RA 7,8 RA 5,6 RA 3,4 RA 1,2 CS 9,10 CS 7,8 CS 5,6
T
CS 3,4 CS 1,2
ABL ABL d Stim 4
PPI
S1 S1 12:55:47 PM
S1
TCL 12:55:48 PM
12:55:49 PM
Fig. 3.3 Pacing in the left atrium near the right inferior pulmonary vein yields a near-perfect PPI
Bibliography Nielsen TD, Bahnson T, Davis RD, Palmer SM. Atrial fibrillation after pulmonary transplant. Chest. August 2004;126(2):496-500.
Fig. 3.4 Right anterior oblique view of the ablation catheter in the left atrium near the right inferior pulmonary vein. CS coronary sinus, ICE intracardiac echo, ABL ablation catheter
Case 4 Michel Haissaguerre
Case Summary A 56-year-old male was referred for ablation of persistent drug-resistant atrial fibrillation (AF). He had a 17-year history of AF, which had been persistent for the last 12 months. The atrial fibrillation cycle length (AFCL) measured during electrophysiological study was 154, 158, and 152 ms on surface ECG, left and right atrial appendages, respectively (Fig. 4.1). Our approach in persistent AF is to first isolate the pulmonary veins (PVs), before targeting complex fractionated atrial
electrograms (CFAEs) and finally performing linear lesions. Any resulting atrial tachycardias (ATs) are mapped and ablated. In this case, following PV isolation there was prolongation of the AFCL to 165 and 159 ms at left and right appendages, respectively. A NavX fractionation map of the LA was performed using a 20-pole high-density mapping catheter. CFAEs were distributed throughout the LA and the CS, as shown in Fig. 4.2. In cases like this, which CFAE should be targeted, and what is the value in measuring AFCL?
I II V1
LAA
Fig. 4.1 Recording from left (LAA) and right (RAA) atrial appendages, and the coronary sinus (CS 1–2 to 9–10). While fractionation is present in the coronary sinus, preventing an easy measurement of the cycle length, potentials at the top of both appendages are almost always of high voltage and unambiguous
RAA CSd
CSp
M. Haissaguerre Department of Cardiology and Electrophysiology, CHU de Bordeaux, Hospital Cardiologique de Haut Leveque, Avenue de Magellan, Pessac 33604, France A. Natale et al. (eds.), Cardiac Electrophysiology, DOI: 10.1007/978-1-84996-390-9_4, © Springer-Verlag London Limited 2011
15
16
M. Haissaguerre 7CFE Mean Map
120 mS 65 mS
0 mS
Fig. 4.2 NavX (EnSite System, St. Jude Medical, MN, USA) fractionation map of the left atrium and coronary sinus after isolation of the four pulmonary veins showing ubiquitous distribution of complex fractionated electrograms
Case Discussion Atrial electrograms in persistent AF are complex and cannot be reliably evaluated in terms of CL except in either right or left appendage. We routinely place a catheter in the right atrial appendage (RAA) and LAA to measure simultaneously the left and right AFCLs at the start of the procedure (Fig. 4.1). Using custom analysis software (Bard EP, Lowell, MA, USA), the mean AFCL for the selected window is calculated. The electrogram annotation is then verified manually online to ensure accuracy (Fig. 4.1). The electrograms measured at the top of both appendages are almost always discrete and of high amplitude, thereby facilitating unambiguous automatic annotation. In the absence of this software, the mean AFCL can be easily estimated by measuring the total duration required for 10–30 cycles (or longer) and then dividing that number by the number of cycles. Often, the AFCL is also reasonably estimated from V1 on the 12-lead ECG. Following PV isolation and throughout the procedure, the circumferential mapping catheter is placed within the RAA, and the ablation catheter in the LAA, and the AFCL may then be assessed simultaneously to measure the impact of ablation in both chambers. Initial AFCL has been shown to be the strongest predictor of success for AF ablation (AF of less than 5 years
continuous duration). AFCL of less than 140 ms is associated with AF termination in less than 69%, while a higher AFCL like in our case is associated with more than 89% of AF termination. Furthermore, the impact of ablation of each region during electrogram-based ablation can be quantified in both atria by AFCL monitoring. After each step of ablation, a gradual prolongation of AFCL is usually observed (a change in AFCL greater than 6 ms is considered significant). Conversion to sinus rhythm or AT occurs when AFCL reaches between 180 and 200 ms and, conversely, rarely occurs when the AFCL is shorter than this. Concerning the appropriate sequence of ablation during the electrogram-based part of the procedure, the main issue is to distinguish active from passive patterns, which still remains one of the major obstacles to minimal effective ablation of persistent AF. This is particularly striking when there are multiple CFAEs distributed throughout the atrium, as in this case. From a purely anatomic perspective, in addition to PVs, ablation at structures annexed to the LA, namely the interface between the inferior LA and the CS and the base of the LAA, have been shown to have the greatest impact on the AF, as measured by AFCL.1 The endpoint of ablation during electrogram-based ablation remains imprecise; however, the organization and slowing of local potentials by ablation seems for us to be preferable to complete elimination of local potentials. In this case, following PV isolation, ablation started along the inferior LA where continuous activity was recorded (Fig. 4.3, panel A). Ablation at the endocardial interface of the CS aims at interrupting the muscular fascicles connecting the LA and the CS and organizing the chaotic activity recorded within the CS. This resulted in a prolongation of the AFCL to 171 ms in the LAA and 168 ms in the RAA. The following step consisted of the ablation at the posterior part of the LAA, where consistent distal-to-proximal electrograms suggesting centrifugal activation were recorded (Fig. 4.3, panel B); this resulted in a prolongation of both AFCLs to 176 ms. Ablation was performed along the roof of the LA where almost continuous electrograms were recorded (Fig. 4.3, panel C), and the AFCL was prolonged to 188 and 183 ms in the right and left appendages, respectively. Finally, ablation of continuous electrical activity at the anteroseptal LA resulted in restoration of sinus rhythm (Fig. 4.4). During sinus rhythm, PV isolation was confirmed and the roofline was completed.
Case 4
Fig. 4.3 Recording of the ablation catheter (RF) and the coronary sinus (CS 1–2 to 9–10). Panel A: Ablation along the inferior left atrium targeting continuous electrical activity. Panel B: Ablation at the posterior part of the left atrial appendage targeting centrifugal activation with
17
consistent distal-to-proximal fractionated electrograms. Panel C: Ablation was then performed along the roof targeting almost continuous fractionated electrograms
18
M. Haissaguerre
I
I
II II V1
V1
RFp CS 1-2
RFd CS 3-4
CS 1-2
CS 5-6
CS 3-4 CS 7-8 CS 5-6 CS 7-8 CS 9-10 CS 9-10
Fig. 4.4 Recording of the ablation catheter (RF) and the coronary sinus (CS 1–2 to 9–10). Ablation of fractionated electrical activity at the anterseptal LA (left panel) resulted in direct restoration of sinus rhythm (right panel, black star)
Bibliography Haïssaguerre M, Shah DC, Jaïs P, et al. Electrophysiological breakthroughs from the left atrium to the pulmonary veins. Circulation. 2000;102:2463-2465. Haissaguerre M, Sanders P, Hocini M, et al. Catheter ablation of longlasting persistent atrial fibrillation: critical structures for termination. J Cardiovasc Electrophys. 2005;16(11):1125-1137.
Haïssaguerre M, Hocini M, Sanders P, et al. Localized sources maintaining atrial fibrillation organized by prior ablation. Circulation. 2006;113(5):616-625. Mohamed U, Skanes AC, Gula LJ, et al. A novel pacing maneuver to localize focal atrial tachycardia. J Cardiovasc Electrophys. 2007; 18(1):1-6. Sanders P, Hocini M, Jais P, et al. Characterization of focal atrial tachycardia using high-density mapping. J Am Coll Cardiol. 6 December 2005;46(11):2088-2099.
Case 5 Anurag Gupta and Amin Al-Ahmad
Case Summary A 21-year-old male with Wolf–Parkinson–White syndrome is referred for electrophysiology study given worsening episodes of palpitations. A 12-lead electrocardiogram (ECG) during sinus rhythm shows pre-excitation (Fig. 5.1). Diagnostic electrode catheters are then placed for recording in the His bundle area and right ventricle, as well as a 20-pole Halo catheter for recording in the right atrium adjacent to the tricuspid valve annulus. Ventricular pacing shows the retrograde atrial
activation sequence (Fig. 5.2). Can this patient be treated with a single application of radiofrequency energy? What maneuvers may be helpful in determining this?
Case Discussion Analysis of the delta waves) on the 12-lead ECG during sinus rhythm (Fig. 5.1), including R/S <1 in V2, negative D in III, positive D in V1, and negative D in aVF, is suggestive
Fig. 5.1 Twelve-lead ECG during sinus rhythm showing ventricular pre-excitation
A. Gupta (*) and A. Al-Ahmad Cardiac Electrophysiology Service, Division of Cardiology, Department of Medicine, Stanford University Hospital and Clinics, 300 Pasteur Drive, Room H2146, Stanford, CA e-mail:
[email protected];
[email protected] A. Natale et al. (eds.), Cardiac Electrophysiology, DOI: 10.1007/978-1-84996-390-9_5, © Springer-Verlag London Limited 2011
19
20
A. Gupta and A. Al-Ahmad
Fig. 5.2 Ventricular pacing from the RV apex. Note the change in the retrograde activation sequence. In the first beat, earliest atrial activation is at pole RA 9,10. However, in the following beat, atrial activation at pole 9,10 is late, whereas atrial activation recorded from the His bundle catheter is earliest. TV tricuspid valve, CS coronary sinus, RA right atrium, HBE His bundle electrogram, P proximal electrode pair, M middle electrode pair, D distal electrode pair, RVA right ventricular apex
Fig. 5.3 Induced SVT. Note the change in the retrograde atrial activation. The SVT continues at the same rate. Is this evidence for another accessory pathway?
of a right posterior/right posterolateral accessory pathway. During electrophysiology study, initiation of orthodromic AVRT is observed at the end of atrial extrastimuli testing; the retrograde atrial activation is observed to change spontaneously during tachycardia (Fig. 5.3). A VPC during the period that the His bundle is refractory advances the following atrial beat and resets the tachycardia. With RF application near Halo RA 9,10 at approximately 8:00 with reference to the left anterior oblique view with the
tricuspid valve depicted as a clockface, the SVT is terminated and pre-excitation is eliminated. However, a second supraventricular tachycardia is subsequently induced with atrial extrastimuli testing. The second SVT has a concentric retrograde activation pattern consistent with the other retrograde pattern seen intermittently during the initial SVT. Para-Hisian pacing is performed (Fig. 5.4).Ventricular pacing next to the His bundle and proximal to the right bundle branch at high output directly captures the His bundle, thus
Case 5
21
Fig. 5.4 Para-Hisian pacing is performed. The first QRS complex obtained with highoutput pacing is narrow, while the second QRS complex obtained with low-output pacing is wide with LBBB morphology and longer activation time of RV apex, which is indicative of the loss of His bundle capture. While earliest retrograde activation occurs at the proximal His bundle electrogram in both cases, conduction over an accessory pathway is indicated by minimal change in stimulus to atrial interval and in atrial activation sequence
activating the ventricles through the His–Purkinje system with resultant narrow QRS complex. Ventricular pacing at low output does not capture the His–Purkinje system, as evidenced by left bundle branch block (LBBB) QRS morphology with later activation of the RV apex. However, despite loss of His bundle capture, there is minimal change in the stimulus to atrial interval and activation sequence, which is indicative of another retrograde accessory pathway. RF application of an anteroseptal pathway terminated the tachycardia, and no further SVT was inducible.
Bibliography Chiang CE, Chen SA, Teo WS, et al. An accurate stepwise electrocardiographic algorithm for localization of accessory pathways in patients with Wolff–Parkinson–White syndrome from a comprehensive analysis of delta waves and R/S ratio during sinus rhythm. Am J Cardiol. 1995;76:40-46. Hirao K, Otomo K, Wang X, et al. Para-Hisian pacing: a new method for differentiating retrograde conduction over an accessory AV pathway from conduction over the AV node. Circulation. 1996;94:1027-1035.
Case 6 Michel Haissaguerre
Case Summary A 62-year-old male was referred for a second ablation of persistent atrial fibrillation (AF). He had a 5-year history of AF, which had been persistent for the previous 5 months. The first procedure consisted of pulmonary vein (PV) isolation, ablation of the inferior left atrium (LA) and coronary sinus (CS), electrogram-based ablation at the posterior and anterior LA and a linear lesion at the LA roof. This terminated the AF to an intermediate atrial tachycardia (AT), which was subsequently ablated anterior to the ostium of the left atrial appendage (LAA) with subsequent restoration of sinus rhythm. Unfortunately, a few days later, he had a recurrence, which was considered to be AF (Fig. 6.1). There was relatively organized atrial activity in V1, and the atrial cycle length was 230 ms with slight irregularity and variability in the F-wave morphology. A quadripolar catheter was placed into the CS and this demonstrated a consistent proximal-todistal activation pattern during arrhythmia. A quadripolar cooled-tip ablation catheter was then inserted into the LA. LA endocardial activation was sequentially mapped and appeared to be organized most of the time (Figs. 6.2–6.4). What inferences can be made from Figs. 6.1–6.4 with relevance to the mechanism of atrial arrhythmia?
time period. Mapping of such organized AF can be performed conventionally and allows in most cases to determinate the atrial area from which the centrifugal activation emanates. Then, the area is closely mapped to find either a discrete site of early activity (focal point), or a small circuit (where “early” and “late” cannot, therefore, be defined) representing localized re-entry. In our case, global activation demonstrated a consistent septal-to-lateral activation sequence anteriorly and posteriorly (Fig. 6.2, panel A), a high-to-low activation sequence in the anterior (Fig. 6.3, panel A) and septal LA (Fig. 6.2, panel A), and a low-to-high sequence in the posterior LA (Fig. 6.3, panel B). This activation sequence was compatible with centrifugal activation from a localized source located anterior and septal to the roofline in the presence of roofline block. (A macro re-entrant AT was excluded by the irregularity of the cycle length.) The ablation catheter was placed at the high anteroseptal LA (Fig. 6.4) where the electrograms showed a significant conduction delay between the distal and proximal bipoles. Furthermore, a pause in electrical activity confirmed that the first electrogram to show variation was the distal electrode
I II III
Case Discussion Organized AF may be defined as AF displaying irregular atrial cycle lengths with beat-to-beat variations of ³20 ms, but with discrete atrial complexes having a consistent activation sequence for (arbitrarily) ³75% of the time over a 10-min
AVR AVL AVF VI V2 V3 V4 V5
M. Haissaguerre Department of Cardiology and Electrophysiology, CHU de Bordeaux, Hospital Cardiologique de Haut Leveque, Avenue de Magellan, Pessac 33604, France
V6
Fig. 6.1 Electrocardiogram of the atrial arrhythmia with relatively organized atrial activity in V1 despite slight irregularity
A. Natale et al. (eds.), Cardiac Electrophysiology, DOI: 10.1007/978-1-84996-390-9_6, © Springer-Verlag London Limited 2011
23
24
A
B LAA Septum
Fig. 6.2 Schematic representation of the LA including the left septum and the left atrial appendage (LAA), after the first atrial fibrillation ablation. Representation of previous encirclement of the right (RPV) and left (LPV) pulmonary veins (continuous lines), and the previous ablation of the roofline (dotted line). Electrograms on distal (RFd) and proximal (RFp) poles of the ablation at the mouth of the left appendage (panel A), and at the left septum (panel B). Recording of V1 and electrogram on distal coronary sinus (CS)
M. Haissaguerre
RPV
LPV
CS
V1
RFd
RFp
CSd 100 ms
pair of the mapping catheter, while the other electrograms followed with the same activation sequence. After these observations, other electrograms were recor ded within 10–15 mm of the original site, and the whole span of the cardiac cycle was mapped in this area: the initial part of the circuit, the second part of the circuit, and throughout the circuit (Fig. 6.5). These findings were compatible with a localized re-entrant circuit with centrifugal activation of the LA, except for the posterior wall because of the roofline block. Ablation at this site slowed and terminated the tachycardia. Conduction block over the roofline was confirmed in sinus rhythm. Localized sources perpetuating AF have been described during ablation procedures as persistent fibrillatory activity confined within isolated LA areas after restoration of sinus rhythm. These sources play an important role in AF maintenance, in addition to pulmonary vein (PV) activity and re-entrant loops. After pulmonary vein isolation (PVI) and linear lesions, ablation of these sources remains the last step of substrate elimination and results in AT conversion or restoration of sinus rhythm.
A localized source may be a discrete point or a small area. During organized AF, when atrial activation mapping converges towards the origin of activity, a small area displaying centrifugal activation and including the localized source can be analyzed. This area may include a discrete point – harboring less than 75% of the cycle length (CL) – which allows tracking of the earliest activity and where local electrograms display centrifugal activation with or without one-to-one conduction to the adjacent atrium (i.e., the same or faster cycle length of the source compared to the atrial fibrillation cycle length [AFCL], respectively). In the majority of cases, the source is not a discrete point but a small area of localized re-entry, as in this case. This area of earliest activation displays electrical activity covering 75–100% of the cycle length, suggesting a localized small circuit. By mapping with a conventional quadripolar ablation catheter, local potentials will display either continuous activity spanning most of the CL or temporally alternating potentials between distal and proximal bipoles (depending on the re-entrant circuit size and properties). By mapping with a high-density 20-pole catheter, local activity will span most or all of the CL.
Case 6
25
Fig. 6.3 Same graphic as Fig. 6.2, with the addition of an anterior wall. Ablation catheter positioned at the anterior (panel A) and posterior (panel B) left atrium
A
B
V1
RFd
RFp
CSd 100 ms
Indeed, irregularity was >15% of the CL (inset ECG) suggesting, in fact, focal AT. Furthermore, post pacing interval (PPI) at the posterior wall was short (+20 ms), but the anterior PPI was very long (+200 ms), ruling out macro re-entrant roof-dependant AT. Since the roofline was previously blocked, it suggested a focal origin from the high posterior LA. Mapping this area close to the roof line revealed very low voltage local activity spanning most of the CL at three different spots very close to each other (Fig. 6.3, POST 1–3). This voltage was only 0.04 mV, which can be easily hidden in case of electrical noise. Ablation of a very low voltage continuous activity site located in between the three described spots terminated the tachycardia (Fig. 6.4). This case emphasizes the importance of localized re-entrant AT in the context of prior AF ablation. In this context, 53% of AT are focal, and localized re-entries are the
most frequent AT mechanism, representing 71% of focal ATs (and 37% of total ATs). The preferential regions for focal ATs are the PV–LA junction, left septum, and the mouth of the LAA. Focal ATs are generally localized to sites targeted during electrogram-based AF ablation. Injury or oedema to the atrial tissue induced by RF applications could generate the substrate for further arrhythmia by creating an anchoring point potentially able to maintain re-entry. Therefore the mechanism of AT after AF ablation may be different to that of spontaneous AT. The presence of pre-existing LA scar (which may be idiopathic or related to underlying structural heart disease) may also result in local slow conduction areas predisposing to re-entry. Mapping and ablation of focal or macro re-entrant AT is a crucial step in the AF ablation process, and often represents the difference between procedural success and failure.
26
M. Haissaguerre
Fig. 6.4 Same graphic as Fig. 6.2. Ablation catheter positioned a nterior and septal to the roofline
LAA LPV
Septum
RPV
CS
V1
RFd
0, 100 mV RFd RFp
CSd 100 ms
V1
0, 100 mV RFd
RFd1
LAA
POST RFd2
pt um Se
Fig. 6.5 Scanning the area within 10–15 mm of the site showed in Fig. 6.4: Electrograms show activation compatible with the entire circuit (initial part in RF2, second part in RF3 and throughout the circuit in RF1). These data are most compatible with left atrial activation coming centrifugally from a small re-entrant source central to the excursion of the catheter in this small area
RPVs
RFd3
CS
CSd
100 ms
LPVs
Case 6
Bibliography Haïssaguerre M, Hocini M, Sanders P, et al. Localized sources maintaining atrial fibrillation organized by prior ablation. Circulation. 2006;113(5):616-625. Haïssaguerre M, Shah DC, Jaïs P, et al. Electrophysiological breakthroughs from the left atrium to the pulmonary veins. Circulation. 2000;102:2463-2465.
27 Haissaguerre M, Sanders P, Hocini M, et al. Catheter ablation of longlasting persistent atrial fibrillation: critical structures for termination. J Cardiovasc Electrophys. 2005;16(11):1125-1137. Sanders P, Hocini M, Jais P, et al. Characterization of focal atrial tachycardia using high-density mapping. J Am Coll Cardiol. 6 December 2005;46(11):2088-2099. Mohamed U, Skanes AC, Gula LJ, et al. A novel pacing maneuver to localize focal atrial tachycardia. J Cardiovasc Electrophys. 2007;18(1):1-6.
Case 7 Michel Haissaguerre
Case Summary A 67-year-old male with an 8-year history of persistent atrial fibrillation (AF) and three external direct current (DC) cardioversions was referred for ablation. The inital procedure consisted of pulmonary vein (PV) isolation, extensive ablation based on fractionated electrograms in the left atrium (LA) and the right atrium (RA), and linear lesions at the roof and the left mitral isthmus. AF was not terminated and an external cardioversion was required. Linear lesions (left atrial roofline and mitral isthmus line) were completed during sinus rhythm. Early recurrence of AT necessitated a second ablation procedure one month later. A decapolar catheter was placed into the CS and showed a proximal-to-distal activation pattern. A quadripolar cooled-tip ablation catheter was then inserted into the LA. AT cycle length was 260 ms with 10% irregularity. PV isolation was confirmed. Extensive anterior scar was noted, and anterior endocardial activation was difficult to assess, but it appeared to be septal to lateral. Post pacing interval (PPI) at the RA was long (+130 ms), PPI at the posterior LA was long (+120 ms). PPI in the LAA was long (+60 ms), while PPI at the anterior LA was good (+10 ms). The anterior part of the LAA showed almost continuous activity (Fig. 7.1). Can this site be considered for ablation?
Case Discussion In the case of extensive scarring or ablation, mapping of AT can be challenging with the practical algorithm. In this situation, three methods can be used to facilitate identification
and ablation of the AT (Fig. 7.2). Entrainment manoeuvres performed during the arrhythmia point towards a focal origin by demonstrating long return cycle lengths around the mitral annulus, the roofline or the tricuspid annulus except at sites adjacent to the focus (by definition, less than two segments). For localized re-entry, local activity spanning all of the AT CL can be demonstrated at the area of earliest activity. For focal point AT, an area of early activity may exhibit middiastolic potential. These tracings illustrate a case with prior extensive LA ablation including the left mitral isthmus line. Conduction around the mitral annulus was compatible with a perimitral circuit (lateral to septal anteriorly and septal-to-lateral posteriorly). The PPI was <30 ms at the anterior annulus, but >30 ms at the CS, which made a macro re-entrant peri-mitral circuit unlikely and suggested a focal AT located lateral to the complete left mitral isthmus line. Mapping was performed in this area where a site displaying most of the CL was found at the anterior mouth of the LAA (Fig. 7.1); however, the PPI was 100 ms at this site, suggesting it was far from the origin of the AT. A second site displaying most of the CL with an activation gradient on the electrogram recording and a short PPI (10 ms) was mapped on the anterior LA (Fig. 7.3). A threedimensional reconstruction of this spot with activation mapping confirmed that the activity spanned all the cycle lengths in a very localized region (Fig. 7.4). Ablation at this site restored sinus rhythm, and a local double potential was visualized after ablation (Fig. 7.5, arrows) in sinus rhythm. This case illustrates how useful the PPI can be in difficult cases. During entrainment, attention must be paid to avoid induction of AF at short CLs and one must be aware of the possibility of conversion to another AT.
M. Haissaguerre Department of Cardiology and Electrophysiology, CHU de Bordeaux, Hospital Cardiologique de Haut Leveque, Avenue de Magellan, Pessac 33604, France A. Natale et al. (eds.), Cardiac Electrophysiology, DOI: 10.1007/978-1-84996-390-9_7, © Springer-Verlag London Limited 2011
29
30
M. Haissaguerre I
55
56
II V1
RFd
RFp
CS 1-2 CS 3-4 CS 5-6 CS 7-8 CS 9-10
Fig. 7.1 Recordings of the radiofrequency RF ablation catheter and a decapolar catheter inserted into the coronary sinus (CS 1–2 to 9–10). Site of high-voltage continuous electrograms displaying almost all the cycle length
PPI to assess number of segment(s) involved
Fig. 7.2 Approach to atrial tachycardia in the context of atrial fibrillation (AF), in the case of extensive scar or ablation. See text for details
≤2 segments FOCAL
>2 segments MACRO RE-ENTRY
• EGM spanning all the AT cycle length in the involved segment(s) • Mid diastolic potentials
Activation compatible with: • Perimitral macro re-entry • Roof dependant macro re-entry • Peritricuspid macro re-entry More complex macro re-entry
Case 7
31 I
29
30
II V1
RFd
RFp
CS 1-2
CS 3-4
CS 5-6
CS 7-8 CS 9-10
Fig. 7.3 Recordings of the RF ablation catheter and a decapolar catheter inserted into the coronary sinus (CS 1–2 to 9–10). Site of complex fractionated electrograms with a gradient of activation between the proximal and distal bipoles (black arrows)
32
M. Haissaguerre
Fig. 7.4 NavX (EnSite System, St. Jude Medical, MN, USA) activation map (antero-posterior view) showing local activity spanning all the cycle lengths in a very localized region below the left atrial appendage
I
27
II V1
RFd 154 ms
CS 1-2 CS 3-4
Fig. 7.5 Recordings of the RF ablation catheter and a decapolar catheter inserted into the coronary sinus (CS 1–2 to 9–10). After restoration of sinus rhythm, local double potential visualized after ablation
CS 5-6 CS 7-8 CS 9-10
28
Case 7
Bibliography Haïssaguerre M, Shah DC, Jaïs P, et al. Electrophysiological breakthroughs from the left atrium to the pulmonary veins. Circulation. 2000;102:2463-2465. Haissaguerre M, Sanders P, Hocini M, et al. Catheter ablation of longlasting persistent atrial fibrillation: critical structures for termination. J Cardiovasc Electrophys. 2005;16(11):1125-1137.
33 Haïssaguerre M, Hocini M, Sanders P, et al. Localized sources maintaining atrial fibrillation organized by prior ablation. Circulation. 2006;113(5):616-625. Mohamed U, Skanes AC, Gula LJ, et al. A novel pacing maneuver to localize focal atrial tachycardia. J Cardiovasc Electrophys. 2007;18(1):1-6. Sanders P, Hocini M, Jais P, et al. Characterization of focal atrial tachycardia using high-density mapping. J Am Coll Cardiol. 2005 December 6;46(11):2088-2099.
Case 8 Michel Haissaguerre
Case Summary A 54-year-old female with persistent atrial fibrillation (AF) for 12 months, a 10-year history of AF, a dilated left atrium (LA) (54 mm diameter), and having failed five external cardioversions and amiodarone, was referred for ablation. Baseline AF cycle length (CL) before ablation was 135 ms in the left atrial appendage (LAA) and 132 ms in the right atrial appendage (RAA). The first procedure consisted of PV isolation, extensive ablation based on electrograms in the LA, and linear lesions at the roof and the left mitral isthmus. Then electrogram-based ablation in the RA resulted in a transient conversion to atrial tachycardia (AT). Early recurrence of AF
I II V1
A
I II V1
B
required external cardioversion to restore sinus rhythm. During sinus rhythm, both linear lesions were assessed and completed. Four months later, the woman developed an AT and she therefore had a second ablation procedure. A decapolar catheter was placed into the coronary sinus (CS) and demonstrated a colliding activation pattern in the proximal CS. A quadripolar cooled-tip ablation catheter was then inserted into the LA. AT cycle length was 271 ms with irregularity reaching 22% of the mean AT cycle length. Pulmonary vein isolation (PVI) was confirmed, and the endocardial activation was determined by moving the ablation catheter sequentially in the anterior LA (Fig. 8.1, panel A, low anterior; and
I II V1
C
I II V1
Abl
Abl
Abl
Abl
CS
CS
CS
CS
B
A
D
C
D
Fig. 8.1 Location of different recordings in the anterior and posterior left atrium, high and low
M. Haissaguerre Department of Cardiology and Electrophysiology, CHU de Bordeaux, Hospital Cardiologique de Haut Leveque, Avenue de Magellan, Pessac, 33604, France A. Natale et al. (eds.), Cardiac Electrophysiology, DOI: 10.1007/978-1-84996-390-9_8, © Springer-Verlag London Limited 2011
35
M. Haissaguerre
36
Fig. 8.1, panel B, high anterior) and in the posterior LA (Fig. 8.1, panel C, high posterior; and Fig. 8.1, panel D, low posterior). Activation was ascending in the anterior wall (Fig. 8.1, panels A and B) and descending at the posterior wall (Fig. 8.1, panels C and D) with the activation sequence covering all the AT cycle length. What inferences can be made from Fig. 8.1 with relevance to the mechanism of AT, and what maneuver has to be done to confirm the diagnosis?
Case Discussion First, for the purposes of describing the approach to ATs arising in the context of catheter ablation of AF, the following definitions are employed: macro re-entry is defined as a circuit involving more than three atrial segments, usually greater than 2 cm in diameter and where more than 75% of the circuit is mapped. Focal point tachycardia is defined as centrifugal activation originating from a discrete site and includes automaticity, triggered activity, and re-entrant mechanisms where <75% of the cycle can be mapped in the chamber of interest. Localized re-entry constitutes a circuit involving one or two adjacent segments, usually smaller than 2 cm in diameter and spanning more than 75% of the CL within the involved segment(s). A diagnostic algorithm has been developed by our group to allow easy and accurate mapping of AT arising in the context of prior AF ablation (Fig. 8.2). After confirmation of PVI, it consists of 1. Assessment of AT cycle length variability: if >15%, suggestive of focal AT and if <15%, not of discriminating value (CL variability is calculated by dividing the CL range by the mean CL averaged over 30 cycles)
Yes
CL Irregularity >15%
No
Focal
Focal or macroreentry
Map earliest region
Activation compatible with: • Perimitral macro re-entry • Roof dependant macro re-entry • Peritricuspid macro re-entry
• Localized re-entry • Focal point AT No
• ~100% cycle • Entrainment Yes Macro re-entry
Fig. 8.2 Practical approach to atrial tachycardia (AT) in the context of atrial fibrillation (AF) ablation. See text for details
2. Macro re-entry is first investigated by assessing the activation in the LA, in order to determine the likelihood of perimitral, roof-dependent, or peritricuspid circuit (entrainment maneuvers are used to confirm the diagnosis) 3. In the case of the absence of macro re-entrant AT, focal point/area AT is determined by tracking the earliest area of activity (entrainment maneuvers can be used to evaluate the site of origin). In the case of localized re-entry, long-duration fractionated potentials spanning most or all of the CL are tracked. Otherwise, the earliest activity is targeted 4. In the case of nonconsistency, more complex tachycardia are investigated, or a change in AT (mechanical or due to stimulation) is searched In our case, the activation front was consistent with a roofdependent macro-re-entry (from A to D): ascending the anterior wall (solid line, A to B) and descending the posterior wall (dashed line, C to D). The mapped activity spanned the entire CL (potentially compatible with roof dependent AT), but slow conduction in the anteroseptal area (A) could be misleading. Indeed, irregularity was >15% of the CL (inset ECG), suggesting in fact focal AT. Furthermore, PPI at the posterior wall was short (+20 ms), but the anterior PPI was very long (+200 ms), ruling out macro re-entrant roof-dependant AT. Because the roof line was previously blocked, it suggested a focal origin from the high posterior LA. Mapping this area close to the roof line revealed very low voltage local activity spanning most of the CL at three different spots very close to each other (Fig. 8.3, POST 1 to 3). This voltage was only 0.04 mV, which can be easily hidden in case of electrical noise. Ablation of a very low voltage continuous activity site located in between the three described spots terminated the tachycardia (Fig. 8.4). This case emphasizes the importance of localized re-entrant AT in the context of prior AF ablation.1 In this context, 53% of AT are focal and localized re-entries are the most frequent AT mechanism, representing 71% of focal ATs (and 37% of total ATs). The preferential regions for focal ATs are the PV–LA junction, left septum, and the mouth of the LAA. Focal AT are generally localized to sites targeted during electrogram-based AF ablation. Injury or edema to the atrial tissue induced by RF applications could generate the substrate for further arrhythmia by creating an anchoring point potentially able to maintain re-entry. Therefore the mechanism of AT after AF ablation may be different to that of spontaneous AT. The presence of preexisting LA scar (which may be idiopathic or related to underlying structural heart disease) may also result in local slow conduction areas predisposing to re-entry. Mapping and ablation of focal or macro re-entrant AT is a crucial step in the AF ablation process, and often represents the difference between procedural success and failure.
Case 8
37
Fig. 8.3 Antero-posterior fluoroscopic view of the left atrium (LA). Mapping of three close posterior spots (post 1–3). Local activity (within red rectangles) spanning the entire CL (pink bars)
I II V1
Post 1
Post 2
Post 3
CS
A
I II V1 0.05 mV Abl
CS
18
19
20
21
B
I II V1
CS 1-2 CS 3-4 CS 5-6 CS 7-8 CS 9-10
Fig. 8.4 Antero-posterior fluoroscopic view of the left atrium (LA) with a decapolar catheter placed into the coronary sinus (CS). Panel A: Ablation catheter (Abl) placed in the posterior LA and recording a very low voltage
(<0.05 mV) signal spanning all of the CL. Panel B: Change of the atrial tachycardia (AT) (black star) during radiofrequency application at this posterior wall
38
Bibliography Haïssaguerre M, Shah DC, Jaïs P, et al. Electrophysiological breakthroughs from the left atrium to the pulmonary veins. Circulation. 2000;102:2463-2465. Haissaguerre M, Sanders P, Hocini M, et al. Catheter ablation of longlasting persistent atrial fibrillation: critical structures for termination. J Cardiovasc Electrophys. 2005;16(11):1125-1137.
M. Haissaguerre Haïssaguerre M, Hocini M, Sanders P, et al. Localized sources maintaining atrial fibrillation organized by prior ablation. Circulation. 2006;113(5):616-625. Mohamed U, Skanes AC, Gula LJ, et al. A novel pacing maneuver to localize focal atrial tachycardia. J Cardiovasc Electrophys. 2007;18(1):1-6. Sanders P, Hocini M, Jais P, et al. Characterization of focal atrial tachycardia using high-density mapping. J Am Coll Cardiol. 2005 December 6;46(11):2088-2099.
Case 9 Alberto Diaz, Dimpi Patel, William R. Lewis, and Andrea Natale
Case Summary The patient is a 55-year-old male with a history of hypertension, left ventricular hypertrophy, and paroxysmal atrial fibrillation (AF) since 2001; both Flecainide and Sotalol had failed. He underwent AF ablation in August 2007. He was placed on Sotalol after radiofrequency ablation (RFA). A month later he had recurrent palpitations that required electrical cardioversion. His arrhythmia was exacerbated after trying to come off the Sotalol. He was evaluated for a repeat ablation. His echocardiogram showed a normal left ventricular ejection fraction (LVEF), normal left atrial size, no valvular abnormalities. His adenosine stress test showed no evidence of ischemia with a LVEF of 74%. He was brought to the EP lab. A catheter was placed in the CS via the internal jugular vein; two transseptal sheaths were used to advance the ablation catheter and the Lasso catheter. The right and left superior pulmonary veins (RSPV and LSPV) appeared to have recovered and were re-ablated (Figs. 9.1 and 9.2.).
Following ablation, isoproterenol was given up to a dose of 20 mcg/kg/min. Following that, the PVs were mapped again to check for electrical isolation. The EGMs below were observed placing the Lasso catheter in the RSPV and the LSPV. Based on the recordings, what would you consider? a) Ablate the left superior PV b) Ablate the right superior PV c) Ablte both PVS d) No additional ablation
Case Discussion Both left and right PVs showed dissociated firing, which confirms both entry and exit blocks. During isoproternol infusion, AF started inside the right upper PV without affecting the atrial chambers, which remained in sinus rhythm. This confirms that there is no reason for additional ablation.
A. Diaz () Heart and Vascular Department, Metro Health Medical Center, 2500 Metro Health Drive, Cleveland, OH, 44109 e-mail:
[email protected] D. Patel St. David’s, Texas Cardiac Arrhythmia Institute, 1015 E. 32nd St, #516, OH Austin, TX 78705 W.R. Lewis Clinical Cardiology, Case Western Reserve University, Heart and Vascular Center, MetroHealth Medical Center, 2500 MetroHealth Drive, Suite H322, Cleveland, OH 44109 A. Natale Texas Cardiac Arrhythmia Institute, St. David’s Medical Center, 1015 East 32nd Street, Suite 516, Austin, TX 78705
A. Natale et al. (eds.), Cardiac Electrophysiology, DOI: 10.1007/978-1-84996-390-9_9, © Springer-Verlag London Limited 2011
39
40
Fig. 9.1 Lasso in the left superior pulmonary vein (LSPV)
Fig. 9.2 Lasso in the right superior pulmonary vein (RSPV)
A. Diaz et al.
Case 10 Antonio Rossillo, Sakis Themistoclakis, Aldo Bonso, Andrea Corrado, and Antonio Raviele
Case Summary The patient is a 13-year-old male with prior 8-year history of palpitations during exercise. No structural heart disease. The transesophageal EP study easily demonstrated the induction of sustained SVT during minimal effort (AA300ms). The echocardiography evaluation was normal and 24 h-ECG Holter documented two episodes of SVT which began after a physiologic increase in heart rate.
After examining Figs. 10.1–10.3, what is the likely diagnosis?
Case Discussion This case demonstrates a narrow complex tachycardia that is not 1:1; with A > V. This excludes AVRT and makes AVNRT less likely unless there is block below the lower common pathway (see Figs. 10.4–10.8).
Fig. 10.1 12-lead resting ECG (paper speed 25 mm/s) showing normal sinus rhythm with a ventricular rate of 66 bpm and a normal PR interval (136 ms) and a QRS width of 100 ms
A. Rossillo (), S. Themistoclakis, A. Bonso, A. Corrado, and A. Raviele Ospedale dell’Angelo, Mestre, Venice, Italy A. Natale et al. (eds.), Cardiac Electrophysiology, DOI: 10.1007/978-1-84996-390-9_10, © Springer-Verlag London Limited 2011
41
42
A. Rossillo et al.
A
B
Fig. 10.2 Panel A: 12-lead ECG taken during atrial tachycardia (paper speed 25 mm/s). Panel B: a detail of lead D2, aVL, and V1
Case 10 Fig. 10.3 Intracardiac recordings taken during the electrophysiology study (paper speed 100 mm/s) revealing SVT with a cycle length of 300 ms. Four surface ECG leads (I, aVF, V1, V6), three bipolar recordings from the His bundle region (distal= HIS D, intermediate = HIS I, and proximal = HIS P), two bipolar recordings from the coronary sinus (CS prox = proximal coronary sinus and CS dist = distal coronary sinus), and the unipolar (MC U-CATH) and the distal bipolar recording of the mapping catheter (MC D)
Fig. 10.4 The electroanatomical mapping of the tachycardia showed the origin of the arrhythmia coming from the His bundle region (the red area in the LAO view). The orange tags show the area where it was possible to record the His bundle potentials
43
44
A. Rossillo et al.
Fig. 10.5 Electroanatomical mapping with CARTO: detailed remap of the area of interest; the orange tag shows the His bundle potential and the blue tag shows the site of earliest activation during tachycardia
Fig. 10.6 Electroanatomical mapping with CARTO: detailed remap of the area of interest; the orange tag reflects the His bundle potential, the blue tag shows the site of earliest activation during tachycardia, and the red tag shows where a single RF lesion is applied
Case 10 Fig. 10.7 Intracardiac recordings taken at the ablation site (paper speed 100 mm/s). Same display as that shown in Fig. 10.3. A atrium, V ventricle
Fig. 10.8 Intracardiac recordings taken at the ablation site (paper speed 100 mm/s) during RF application. A single RF application of 30 s with titration of the power starting from 30 W resulted in sinus rhythm restoration. RF energy was stopped as soon as the His bundle potential appeared on the ablation catheter. Same display as that shown in Fig. 10.3. A atrium, V ventricle, H His bundle
45
46
Bibliography Chen SA, Chiang CE, Yang CJ, Cheng CC, et al. Sustained atrialtachycardia in adult patients. Electrophysiological characteristics, pharmacological response, possible mechanisms, and effects of radiofrequency ablation. Circulation. 1994;90:1262-1278. Frey B, Kreiner G, Gwechenberger M, Gossinger H. Ablation of atrial tachycardia originating from the vicinity of the atrioventricular node: significance of mapping both sides of the interatrial septum. J Am Coll Cardiol. 2001;38:394-400. Kalman JM, Olgin JE, Karch MR, et al. “Cristal tachycardias”: origin of right atrial tachycardias from the crista terminalis identified by intracardiac echocardiography. J Am Coll Cardiol. 1998;31:451-459. Kay GN, Chong F, Epstein AE, Dailey SM, Plumb VJ. Radiofrequency ablation for treatment of primary atrial tachycardias. J AM Coll Cardiol. 1993;21:901-909. Knight BP, Zivin A, Souza J, et al. A technique for the rapid diagnosis of atrial tachycardia in the electrophysiology laboratory. J Am Coll Cardiol. 1999;33:775-781.
A. Rossillo et al. Lai LP, Lin GL, Chen TF, et al. Clinical electrophysiological characteristics and radiofrequency catheter ablation of atrial tachicardia near the apex of Koch’s triangle. PACE. 1998;21:367-374. Lesh MD, Van Hare GF, Epstein LM, et al. Radiofrequency Catheter ablation of atrial arrhythmias: results and mechanisms. Circulation. 1994;89:1074-1089. Pappone C, Stabile G, De Simone A, et al. Role of catheter-induced mechanical trauma in localisation of target sites of radiofrequency ablation of automatic atrial tachycardia. J Am Coll Cardiol. 1996;27:1090-1097. Poty H, Saudi N, Haissaguerre M, et al. Radiofrequency catheter ablation of atrial tachycardias. Am Heart J. 1996;131:481-489. Tang CW, Scheinmann MM, Van Hare GF, et al. Use of P wave configuration during atrial tachycardia to predict site of origin. J Am Coll Cardiol. 1995;26:1315-1324. Tracy CM, Swartz JF, Fletcher RD, et al. Radiofrequency catheter ablation of ectopic atrial tachycardia using paced activation sequence mapping. J Am Coll Cardiol. 1993;21:910-917.
Case 11 Yash Y. Lokhandwala, Anoop K. Gupta, and Ranjan K. Thakur
Case Summary
Case Discussion
A 17-year-old male with a normal heart has an electrocardiogram (ECG) showing intermittent pre-excitation suggestive of left free-wall accessory pathway (AP) and supraventricular tachycardia (SVT). The induced tachycardia is a narrow QRS regular tachycardia (Fig. 11.1) with ST depression in inferior leads. The patient is taken to the electrophysiological (EP) lab. In Fig. 11.2 the ablation catheter is positioned at the lateral tricuspid annulus (9 o’clock). Where is the site of the successful ablation?
The first two complexes show earliest A in CS 1-2 and in RFD; after this the coronary sinus (CS) activation reverses, but the A remains earliest in Radiofrequency Distal (RFD), suggesting that the orthodromic atrioventricular reentrant tachycardia (AVRT) is mediated by a right free-wall AP with a bystander left-lateral AP. Ablation of the right-sided AP resulted in success.
Y.Y. Lokhandwala () KEM Hospital, Parel, Mumbai, India A.K. Gupta Apollo Hospital, Ahmedabad, India R.K. Thakur Thoracic and Cardiovascular Institute Sparrow Health System, Michigan State University 405 West Greenlawn, Suite 400, Lansing, MI 48910 A. Natale et al. (eds.), Cardiac Electrophysiology, DOI: 10.1007/978-1-84996-390-9_11, © Springer-Verlag London Limited 2011
47
48
Y.Y. Lokhandwala et al. 2008021
I II III AVR AVL AVF V1 V2 V3 V4 V5 V6 10 mm/mV 25 mm/s
Fig. 11.1 The electrocardiogram (ECG) shows induced tachycardia
Case 11
I
49 2008021
AVF V1 V6 CS910 CS78 CS56 CS34 CS12 HISD HISP RFD RFP 10 mm/mV 200 mm/s
Fig. 11.2 Intracardiac recordings. CS 1-2 is distal. The ablation catheter (RFD) is placed along the lateral tricuspid annulus
Case 12 Eric Buch, Shiro Nakahara, Marmar Vaseghi, Noel G. Boyle, and Kalyanam Shivkumar
Case Summary A 59-year-old male with ischemic cardiomyopathy who underwent heart transplantation three years earlier was repeatedly hospitalized for atrial flutter (AFL) (Fig. 12.1, panel A) refractory to rate control and antiarrhythmic drugs. Left ventricular ejection fraction was normal by echocardiography. He was referred for electrophysiology study and ablation. Intracardiac recordings showed AFL with tachycardia cycle length 268 ms and earliest left atrial activation from the distal coronary sinus (CS), consistent with a left atrial circuit (Fig. 12.1, panel B). Entrainment pacing from the mid-CS A
performed during flutter demonstrated a long post-pacing interval of 442 ms. Noteworthy was that during entrainment pacing from the high right atrial (HRA) catheter, 2:1 block of right-to-left atrial conduction was observed without terminating the tachycardia (Fig. 12.1, panel C). Where is the likely circuit of this flutter?
Case Discussion Based on Fig. 12.1, which shows the left atrium dissociated from the flutter circuit during tachycardia, left AFL was excluded from the differential diagnosis. Electroanatomic
B
C
Fig. 12.1 Cardiac tracings recorded during the clinical atrial flutter (AFL). Panel A: Surface 12-lead ECG recorded during AFL. Panel B: Intracardiac tracings and ECG leads II, V2, and V5 showing earliest left atrial activation in the distal CS. Panel C: Intracardiac tracings and ECG leads II, V2, and V5 showing entrainment pacing from the HRA, and 2:1 block of the right-to-left atrial conduction. CS coronary sinus, HRA high right atrium, CS (HB) d,p the distal and proximal electrode pairs of the coronary sinus (His bundle) catheter
E. Buch (*), S. Nakahara, M. Vaseghi, N.G. Boyle, and K. Shivkumar UCLA Cardiac Arrhythmia Center, David Geffen School of Medicine at UCLA, 10833 Le Conte Ave, A2-237 CHS, Los Angeles, CA 90095-1679 e-mail:
[email protected] A. Natale et al. (eds.), Cardiac Electrophysiology, DOI: 10.1007/978-1-84996-390-9_12, © Springer-Verlag London Limited 2011
51
52
E. Buch et al.
mapping of the right atrium (RA) showed large areas of lowvoltage scar (colored gray in Fig. 12.2). Activation mapping revealed a macro-reentrant AT utilizing a channel in the lateral wall of the RA (Fig. 12.2). A line of ablation along the lateral RA wall, shown in Fig. 12.2, successfully terminated the tachycardia. It is known that conduction from RA to LA
occurs over multiple septal pathways, including Bachmann’s bundle, the foramen ovale, and CS.1 In this case, due to scarring over a large part of the RA, conduction through Bachmann’s bundle was more rapid than the lower septal sites, resulting in distal to proximal of LA activation as seen in the CS catheter.
200 ms
Fig. 12.2 Electroanatomic map of the right atrium during the clinical AFL. Red areas indicate earliest endocardial activation; orange, yellow, green, blue, and purple indicate progressively delayed activation. The scar is shown in gray (<0.1 mV). White tags indicate the ablation sites. The black solid arrows show the activation sequence within the tachycardia circuit, and the black dotted arrows show bystander activation of right atrium as well as left atrium via Bachmann’s bundle. TA tricuspid annulus, RA right atrium, AFL atrial flutter
0 ms
−200 ms
Reference 1. Roithinger FX, Cheng J, SippensGroenewegen A, et al. Use of electroanatomic mapping to delineate transseptal atrial conduction in humans. Circulation. 1999;100:1791-1797.
Conduction via Bachmann’s bundle
CS catheter
Case 13 Bradley P. Knight
Case Summary A 54-year-old female with mental retardation presented to the emergency department with a recurrent supraventricular tachycardia (SVT) at 180 bpm. An electrophysiology (EP) study was performed. The baseline rhythm was sinus rhythm without ventricular preexcitation. Her clinical tachycardia was induced when an atrial extrastimulus was delivered with a coupling interval of 340 ms during a drive train with a cycle length (CL) of 550 ms (Fig. 13.1). There was a one-to-one relationship between the atrium and the ventricle. The ventriculoatrial (VA) interval associated with the first beat of the tachycardia is nearly identical to the remaining VA intervals. Can atrial tachycardia be excluded?
is “VA linking.” This observation strongly suggests that the atrial depolarization is dependent on the ventricular depolarization, meaning that the atrial rhythm is dependent on VA conduction. This should not be present with an atrial tachycardia, because the first beat of the tachycardia should have no relationship to the prior QRS, which was a result of conduction of the atrial extrastimulus to the ventricle. This observation can be useful, but it is important to recognize that it is not absolute and should be reproducible. There is always a small chance that the first VA interval will be the same or similar as that during tachycardia by chance alone. Indeed, in this case the tachycardia was actually a left atrial septal tachycardia. Figure 13.2 shows a spontaneous initiation of the same tachycardia that is shown in Fig. 13.1 by a premature atrial beat during sinus rhythm. This event shows that at times the tachycardia does not have a constant VA interval and occasionally displays Wenckebach AV block.
Case Discussion When a tachycardia with a constant VA interval is induced with an atrial premature beat and the first VA interval is the same as the subsequent VA intervals, it can be said that there
B.P. Knight Division of Cardiology, Northwestern Medical Center, 676 N. St. Clair, Suite 600, Chicago, IL 60611 A. Natale et al. (eds.), Cardiac Electrophysiology, DOI: 10.1007/978-1-84996-390-9_13, © Springer-Verlag London Limited 2011
53
54
I
B.P. Knight ARRHYTHMIA INDUCTION RA_d S1 = 550 S2 = 340 PRINT REVIEW FIT
II 190
360
190
550
550
420
360
350
380
360
V1
V6
550
HRA p
550
340
1520
360
ABL d
ABL m
ABL p
540
RVA p
Stim
550
550
550
550
550
400
370
730
360
340
University of Chicago
Fig. 13.1 Induction of a supraventricular tachycardia with a single atrial extrastimulus. Shown are surface electrograms from leads I, II, V1, and V6, and the intracardiac electrograms from the high right
atrium (HRA), the ablation catheter (ABL) positioned at the His bundle location, and the right ventricular apex (RVA)
Case 13
I
55
PRINT REVIEW FIT
II
610
610
610
440
410
400
410
490
V1
V6
HRA p
610
610
300
790
370
360
370
ABL d
ABL m
ABL p
RVA p
620
610
600
430
420
390
400
Stim
University of Chicago
Fig. 13.2 A different induction of the same supraventricular tachycardia shown in figure 13.1 showing a variable ventriculo-atrial relationship. The format is the same as in Figure 13.1
Case 14 M. Eyman Mortada, Jasbir S. Sra, and Masood Akhtar
Case Summary A 27-year-old male complained of multiple episodes of palpitations. His echocardiogram (ECG) and baseline ECG were within normal limits. His palpitations were captured with the 12-lead ECG shown in Fig. 14.1. In the electrophysiology (EP) lab, the same tachycardia was induced. After pacing from the right ventricle during the tachycardia shown in Fig. 14.1, the rhythm changed to the ECG shown in Fig. 14.2. During the ventricular programmed stimulation, the results shown in Fig. 14.3 were observed. The tachycardia was initiated in the lab with the atrial programmed stimulation shown in Fig. 14.4. Two ventricular extrastimuli were performed from the right ventricular apex during the tachycardia, showing the results in Fig. 14.5. Three ventricular extrastimuli were performed during the tachycardia, which showed the results in Fig. 14.6. What can be excluded? What is the diagnosis?
Case Discussion The initial presentation of this healthy young male is a wide complex tachycardia with right bundle branch block (RBBB) morphology and right axis deviation. The duration of the QRS is 145 ms. It is difficult to assess if there are P-waves. If the small deflection between the two QRS complexes is a P-wave, there is an AV association. Interestingly, there is an obvious electrical alternans present, occurring simultaneously with a CL alternans. These findings favor supraventricular tachycardia (SVT) rather than ventricular tachycardia (VT) or atrial flutter (AFL). However, we cannot exclude any of the possibilities.
M.E. Mortada (*), J.S. Sra, and M. Akhtar Department of Electrophysiology, Aurora Cardiovascular Services, Aurora Sinai/Aurora St. Luke’s Medical Centers, University of Wisconsin School of Medicine and Public Health, 2801 W. Kinnickinnic River Parkway, #777, Milwaukee, WI 53215
After pacing from the right ventricle, the QRS narrows down to a normal duration (72 ms). The electrical alternans remains the same. However, the CL of the tachycardia accelerates from 259 to 240 ms. This indicates the presence of an accessory pathway (AP) ipsilateral to the bundle branch block (right), which is part of the electrical circuit of the tachycardia. Hence the tachycardia is due to orthodromic AV reentry with aberrancy. The aberrancy disappears due to early activation of the right bundle by pacing from the right ventricle, leading to a break-in-the-link phenomenon and early recovery of the right bundle, in readiness for anterograde activation from the next beat, causing narrow QRS morphology.1 The narrow complex tachycardia (Fig. 14.2) has an incomplete RBBB with QRS at 80 ms and a rate of 240 beats per minute. The P-wave is in the ST-T segment, suggestive of short RP tachycardia. The most likely diagnosis is either AP-mediated tachycardia or atrial tachycardia. Though, one cannot exclude AV-nodal reentry tachycardia or para-Hisian ventricular tachycardia with one-to-one VA conduction — but this phenomenon is very rare. During ventricular pacing (Fig. 14.3), the VA conduction was retrograde over the AV node during the first two beats. When a ventricular extrastimuli was introduced (third beat), the VA conduction continued retrogradely over the AV node with a longer VA duration, due to the decremental property of the AV node rather than to a jumping phenomenon (dual physiology). The fourth ventricular beat is either a ventricular repetitive response or a bundle branch reentry beat, which does not result in retrograde atrial activation. The absence of VA conduction over an AP during ventricular pacing and at the time of AV nodal block makes it less likely that AP-mediated tachycardia is the cause – but it does not exclude the possibility. Some APs have multiple fibers at their ventricular insertion sites where they might interact during the electrical propagation, leading to block and prevention of retrograde conduction over the AP.2, 3 The possibility that this VT is para-Hisian in origin is less likely, due to the weak retrograde conduction over the AV node. Therefore, none of the findings in Fig. 14.3 help define the cause of the tachycardia.
A. Natale et al. (eds.), Cardiac Electrophysiology, DOI: 10.1007/978-1-84996-390-9_14, © Springer-Verlag London Limited 2011
57
58
M.E. Mortada et al.
Fig. 14.1 The patient’s palpitations were captured with this 12-lead ECG
Fig. 14.2 The same tachycardia was induced in the electrophysiology lab. After pacing from the right ventricle during the tachycardia, the patient’s rhythm changed, as shown in this ECG
Case 14
Fig. 14.3 Tracing of patient’s response to ventricular programmed stimulation with one ventricular extrastimuli. The recording in this figure and other figures are (from top to bottom): surface ECG with leads I, II and V1, high right atrial recording, proximal coronary sinus (CS
59
9,10) to distal coronary sinus (CS 1, 2), His bundle recording and right ventricular recording. HRA: high right atrium; CS: coronary sinus; HISp: proximal His, HISm: middle His; HISd, distal His, RVa: apical right ventricle
Fig. 14.4 Tracing demonstrating the initiation of the tachycardia with atrial programmed stimulation
60
Fig. 14.5 Two ventricular extrastimuli during the tachycardia
Fig. 14.6 Three ventricular extrastimuli during the tachycardia
M.E. Mortada et al.
Case 14
One extrastimuli from the high right atrium (HRA) induced tachycardia with right bundle branch block morphology and right axis deviation (Fig. 14.4). The activation sequence of the tachycardia is: left atrium (LA) to right atrium (RA), then His, followed by ventricular activation (VA). The most likely cause for this rhythm is left atrial tachycardia or orthodromic AV reentry tachycardia (AVRT) via the left free-wall AP. Antidromic AVRT, in which the electrical activation should be V-H-A, is ruled out in this case due to the sequence of the electrical activation (A-H-V) and the fact that the tachycardia is initiated by antegrade activation over the AV node. Induction of VT via extrastimuli at the high right atrium (HRA) can be seen in bundle branch reentry or fascicular ventricular tachycardia. Atrial eccentric activation usually does not rule out AVNRT. Six to eight percent of AVNRTs display eccentric atrial activation.4, 5 However, the eccentric activation occurs with earliest atrial activation coming from an area not more than 4 mm from the AV node. Hence, AVNRT can be ruled out in this case because the earliest atrial activation was at CS 1-2 (left atrial free wall), which is more than 4 mm away from the AV node. A study of the ventricular extrastimuli present during supraventricular tachycardia (SVT) at the time of His refractoriness is sometimes helpful in defining the cause of the tachycardia. In this case (Fig. 14.5), two extrastimuli are seen. The first was late and fused with the QRS of the tachycardia. The second came a few milliseconds prior to the His activation (His refractory period). If the next atrial activation occurred earlier, the presence of AP could be confirmed. However, the atrial activation did not change, as occurs in both atrial tachycardia and AVNRT.
61
AP-mediated tachycardia cannot be excluded, because the distance from the right ventricular apex to the assumed AP, which should be on the left ventricular free wall in this scenario, far from the tachycardia circuit. Therefore, pacing with one or two extrastimuli, from the right ventricular apex, may not enter and entrain the tachycardia circuit. Three ventricular extrastimuli were performed at cycle lengths of 210 ms to entrain the tachycardia (cycle length, 245 ms), which showed VAV response, thus atrial tachycardia was excluded. The post-pacing interval was 330 ms, thus the pacing site is near to the circuit and the diagnosis is orthodromic AVRT via the left ventricular free-wall AP.
References 1. Lehmann MH, Denker S, Mahmud R, Addas A, Akhtar M. Linking: a dynamic electrophysiologic phenomenon in macroreentry circuits. Circulation 1985;71:254-265. 2. Rordorf R, Vicentini A, Petracci B, Landolina M. Intermittent ratedependent retrograde conduction over a concealed atrioventricular accessory pathway: what is the mechanism? Europace 2008;Dec 3(epub ahead of print). 3. Bai R, Tritto M, Di Biase L, Salerno-Uriarte JA. Pacing site and bradycardia dependent retrograde conduction block over an atrioventricular accessory pathway. Europace 2006;8:438-442. 4. Ong MG, Lee PC, Tai CT, Lin YJ, Hsieh MH, Chen YJ, Lee KT, Tsao HM, Kuo JY, Chang SL, Chen SA. The electrophysiologic characteristics of atrioventricular nodal reentry tachycardia with eccentric retrograde activation. Int J Cardiol 2007; 120:115-122. 5. Katritsis DG, Ellenbogen KA. Eccentric retrograde atrial activation in atrioventricular nodal reentrant tachycardia. Heart Rhythm 2005;2:1394-1395.
Case 15 Eric Buch, Shiro Nakahara, Marmar Vaseghi, Noel G. Boyle, and Kalyanam Shivkumar
Case Summary
Case Discussion
A 75-year-old male with history of orthotopic heart transplantation with a cavo-caval anastomosis initially developed atrial flutter 13 years post-transplantation with an episode of mild rejection. He was successfully treated for rejection and a right atrial cavotricuspid isthmus dependent atrial flutter was ablated. Two years later, presented with palpitations and shortness of breath and was found to have recurrent atrial flutter. Cardiac catheterization and biopsy did not show evidence of transplant coronary artery vasculpathy or acute rejection. What is the most likely mechanism of atrial flutter in this patient?
Electroanatomic mapping demonstrated recurrent cavotricuspid isthmus dependent atrial flutter (Fig. 15.1) and an ablation line from the inferior vena cava to the tricuspid valve terminated the tachycardia. He has had no recurrence in 3 years of follow-up. Atrial flutter is the most common arrhythmia in post-cardiac transplant patients. Of all supraventricular tachycardias occurring in stable transplant patients without evidence of rejection or vasculopathy, cavotricuspid isthmus dependent atrial flutter is most common. Furthermore, the type of anastomosis at the time of transplant, whether cavo-caval or atrio-atrial, does not affect the incidence of atrial flutter in these patients.
E. Buch (*), S. Nakahara, M. Vaseghi, N.G. Boyle, and K. Shivkumar UCLA Cardiac Arrhythmia Center, David Geffen School of Medicine at UCLA, 10833 Le Conte Ave, A2-237 CHS, Los Angeles, CA 90095-1679 e-mail:
[email protected] A. Natale et al. (eds.), Cardiac Electrophysiology, DOI: 10.1007/978-1-84996-390-9_15, © Springer-Verlag London Limited 2011
63
64
E. Buch et al.
A
B
Fig. 15.1 (Panel A) Activation map: this patient, despite a cavo-caval anastomosis, developed atrial flutter. The activation map demonstrated a right atrial flutter with the wave-front propagating superiorly along the interatrial septum, around the annulus, and inferiorly along the lateral wall of the right atrium. (Panel B): A duodecapolar catheter was
placed in the right atrium with bipole D10 placed along the lateral wall and D1 along the interatrial septum, showing propagation from D1 to D10 in the right atrium. D duodecapolar, LAO left atrium, RAO right atrium, CS coronary sinus
Case 16 Antonio Rossillo, Sakis Themistoclakis, Aldo Bonso, Andrea Corrado, and Antonio Raviele
Case Summary
Case Discussion
The patient is a 27-year-old women with a history of palpitations. No heart disease was documented. Event recorder documented sustained SVT. Examine Figs. 16.1–16.3. What are the likely possibilities?
This patient has a normal 12-lead ECG. The induced narrow complex tachycardia is regular. The possibilities include AVNRT, AT, and AVRT. The atrial activation sequence during SVT is eccentric indicating either a left-sided AT or a left-sided AP. In this case a left-sided AP was successfully ablated (Figs. 16.4–16.6).
Fig. 16.1 Twelve-lead resting ECG (paper speed 25 mm/s) showing normal sinus rhythm
A. Rossillo (*), S. Themistoclakis, A. Bonso, A. Corrado and A. Raviele Ospedale dell’Angelo, Mestre, Venice, Italy A. Natale et al. (eds.), Cardiac Electrophysiology, DOI: 10.1007/978-1-84996-390-9_16, © Springer-Verlag London Limited 2011
65
66
A. Rossillo et al.
Fig. 16.2 Twelve-lead ECG taken during SVT (paper speed 25 mm/s)
Fig. 16.3 Intracardiac recordings taken during the electrophysiology study (paper speed 100 mm/s). Five surface ECG leads (II, III, aVF, V1, V6), two bipolar recordings from the pacing catheter placed in right atrium or ventricle (HRA), three bipolar recordings from the His bundle
region (HIS PROX, MED, and DIST), five bipolar recordings from the coronary sinus (CS), and a bipolar recording from the distal mapping catheter (MAP D). AVRT (cycle length 408 ms) with earliest activation on CS 1-2. A atrium, V ventricle, H His
Case 16
67
Fig. 16.4 Intracardiac recordings taken during RF energy delivery (paper speed 100 mm/s). Same display as shown in Fig. 16.3. The red arrow showed termination of the tachycardia. A atrium, V ventricle, H His
Fig. 16.5 Intracardiac recordings taken during RF energy delivery (paper speed 12.5 mm/s). Same display as shown in Fig. 16.3. The red arrows point at the restoration of sinus rhythm after 5 s of RF delivery
68
A. Rossillo et al.
Fig. 16.6 Intracardiac recordings (paper speed 100 mm/s) taken during programmed ventricular stimulation showing the absence of VA conduction (paper speed 100 mm/s). Same display as shown in Fig. 16.3. A atrium, V ventricle
Bibliography Calkins H, Langberg J, Sousa J, et al. Radiofrequency catheter ablation of accessory atrioventricular connections in 250 patients. Abbreviated therapeutic approach to Wolff-Parkinson-White syndrome. Circulation. 1992;85:1337-1346. Haïssaguerre M, Dartigues JF, Warin JF, Le Metayer P, Montserrat P, Salamon R. Electrogram patterns predictive of successful catheter ablation of accessory pathways. Value of unipolar recording mode. Circulation. 1991;84:188-202.
Jackman WM, Wang XZ, Friday KJ, et al. Catheter ablation of accessory atrioventricular pathways (Wolff-Parkinson-White syndrome) by radiofrequency current. N Engl J Med. 1991;324:1605-1611. Scheinman MM, Wang YS, Van Hare GF, Lesh MD. Electrocardio graphic and electrophysiologic characteristics of anterior, midseptal and right anterior free wall accessory pathways. J Am Coll Cardiol. 1992;20:1220-1229.
Case 17 Bradley P. Knight
Case Summary A 14-year-old male underwent an electrophysiology procedure for recurrent left atrial tachycardia (AT). A prior attempt at ablation was unsuccessful. After transeptal catheterization, an electroanatomic map of the left atrium (LA) and left atrial appendage (LAA) was created during AT. The mechanism of the tachycardia did not appear to be macroreentrant. The origin of the tachycardia appeared to be the left atrial appendage. However, the electroanatomic map showed a relatively broad area of early activation and failed to identify a clear focus of origin. What additional strategy could be used to increase the likelihood of successful ablation?
Case Discussion
Fig. 17.1 Fluoroscopic view during ablation of a left atrial appendage tachycardia. The view is left anterior oblique. A coronary sinus electrode catheter can be seen placed from a superior approach. An ablation catheter and the 20-electrode circular mapping catheter positioned in the appendage, placed via double transseptal catheterization, can also be seen
A multielectrode circular mapping catheter, which is usually used to map pulmonary vein (PV) ostia, was subsequently placed in the LAA (Fig. 17.1). Using the bipolar signals from this catheter, the LAA was explored until the earliest activation site was identified. Figure 17.2 shows the earliest atrial activation recorded from bipole 13–14. The ablation catheter was then positioned adjacent to the earliest bipole and radiofrequency current was delivered. The tachycardia terminated and was noninducible. Advanced computerized three-dimensional (3D) mapping systems have greatly improved the ability to tackle
complex tachycardias. However, it is important to recognize the limitations of such systems, and to occasionally confirm the findings using conventional mapping techniques. In this case, the complex structure of the LAA and its trabeculations proved to be a challenge for the electroanatomic contact mapping system to reconstruct the chamber accurately enough to provide a clear ablation target. The circular mapping catheter may have also distended the appendage and moved trabeculations that may have interfered with localization.
B.P. Knight Division of Cardiology, Northwestern Medical Center, 676 N. St. Clair, Suite 600, Chicago, IL 60611 A. Natale et al. (eds.), Cardiac Electrophysiology, DOI: 10.1007/978-1-84996-390-9_17, © Springer-Verlag London Limited 2011
69
70
Fig. 17.2 This tracing was recorded during ablation of a left atrial appendage tachycardia. Shown are the surface tracings from leads I and V5, and the bipolar intracardiac electrograms from the ablation catheter,
B.P. Knight
20-electrode circular mapping catheter positioned in the appendage, and the octapolar electrode catheter placed in the coronary sinus. Note termination of the atrial tachycardia during delivery of radiofrequency current
Case 18 Antonio Rossillo, Sakis Themistoclakis, Aldo Bonso, Andrea Corrado, and Antonio Raviele
Case Summary
Case Discussion
The patient is a 73-year-old female with a long history of palpitations and hypertension. Echo reveals normal LVEF with an LA diameter of 38 mm. The transesophageal EP study revealed SVT, which was induced with minimum effort (Figs. 18.1 and 18.2). What is the likely mechanism of the SVT?
Although the patient is elderly, the ECG shows a regular narrow complex tachycardia. Atrial tachycardia, AVNRT, and AVRT should all be considered. In this case, no visible P waves are seen during the SVT. This suggests either AVNRT or AT with a long PR interval. In this case, AVNRT was induced in the EP lab and successfully ablated (Figs. 18.3–18.16).
Fig. 18.1 The 12-lead resting ECG (paper speed 25 mm/s) showed sinus rhythm with a ventricular rate of 80 bpm a short PR interval (102 ms) and a QRS width of 80 ms
A. Rossillo (*), S. Themistoclakis, A. Bonso, A. Corrado, and A. Raviele Ospedale dell’Angelo, Mestre, Venice, Italy A. Natale et al. (eds.), Cardiac Electrophysiology, DOI: 10.1007/978-1-84996-390-9_18, © Springer-Verlag London Limited 2011
71
72 Fig. 18.2 12-lead ECG taken during SVT (paper speed 25 mm/s) with a heart rate of 186 bpm
Fig. 18.3 Intracardiac recordings taken at baseline during the electrophysiology study (paper speed 200 mm/s). Four surface ECG leads (I, aVF, V1, V6), one bipolar recording from the high right atrium (HRA), three bipolar recordings from the His bundle region (distal = HIS D, intermediate = HIS I and proximal = HIS P), two bipolar recordings from the coronary sinus (CS prox = proximal coronary sinus and CS dist = distal coronary sinus), and the distal bipolar recording of the mapping catheter (MC D). A atrium, V ventricle, H His bundle
A. Rossillo et al.
Case 18 Fig. 18.4 (a, b). Intracardiac recordings taken during programmed atrial stimulation (paper speed 200 mm/s). Same display that is shown in Fig. 18.3. (a) With a coupling interval of 410 ms the AH interval is 188 ms and (b) with a coupling interval of 280 ms it suddenly increased to 307 ms (ERP of the fast pathway with a jump of 120 ms) A atrium, V ventricle, H His bundle
73
a AVN RT Stim A2 1–1
Mestre Studio 200 mm/s
SAP100
S: 36: 45: 408
100 ms
2–avF
198 ms
3–V1 4–V6 5–hRA
V 6–His D
A
H
7–His I 8–His P 9–CS Prox 10–CS dist 11–MC D B1 15–STIM A2
S1
S2
b AVN RT Stim A2 1–1
Mestre Studio 200 mm/s
jump
S: 36: 58: 779
100 ms
2–avF 3–V1
307 ms
4–V6 5–hRA
V
6–His D
A 7–His I 8–His P 9–CS Prox 10–CS dist 11–MC D B1 15–STIM A2 S1
S2
H
74 Fig. 18.5 Intracardiac recordings taken during programmed atrial stimulation with two beats with retrograde conduction through the fast pathway (S slow pathway and F fast pathway) (paper speed 100 mm/s). Same display as that shown in Fig. 18.3
Fig. 18.6 Intracardiac recordings taken during programmed atrial stimulation (paper speed 200 mm/s) showing the atrial ERP with a coupling interval of 200 ms. Same display as that shown in Fig. 18.3. A atrium, V ventricle, H His bundle
A. Rossillo et al.
Case 18 Fig. 18.7 Intracardiac recordings taken during continuous atrial stimulation (paper speed 100 mm/s) with induction of non-sustained AVNRT. A atrium, V ventricle, H His bundle
Fig. 18.8 Intracardiac recordings taken during programmed atrial stimulation with an infusion of isoproterenol IV (paper speed 100 mm/s) showing induction of AVNRT (cycle length 300 ms). Same display as shown in Fig. 1.19.3. A atrium, V ventricle, H His bundle
75
76 Fig. 18.9 Intracardiac recordings (paper speed 200 mm/s). Same display as shown in Fig. 18.3. Pace mapping of the anteroseptal region of Koch’s triangle with a stim-H interval of 78 ms. St pacing, A atrium, V ventricle, H His bundle
Fig. 18.10 Intracardiac recordings (paper speed 200 mm/s). Same display as Fig. 1.19-3. Pace mapping of the midseptal region of Koch’s triangle with a stim-H interval of 81 ms. St pacing, A atrium, V ventricle, H His bundle
A. Rossillo et al.
Case 18 Fig. 18.11 Intracardiac recordings (paper speed 200 mm/s). Same display as shown in Fig. 18.3. Pace mapping of the posteroseptal region of Koch’s triangle with a stim-H interval of 106 ms. St pacing, A atrium, V ventricle, H His bundle
Fig. 18.12 Intracardiac recordings (paper speed 200 mm/s). Same display as shown in Fig. 18.3. Ablation site slow pathway potential
77
78 Fig. 18.13 Intracardiac recordings during radiofrequency delivery (paper speed 100 mm/s) with induction of slow junctional beats in fusion with a sinus beat. Same display as shown in Fig. 18.3
Fig. 18.14 Intracardiac recordings taken during programmed atrial stimulation with infusion of isoproterenol IV (paper speed 100 mm/s) with a very slow anterograde conduction (AH interval 280 ms). Same display as shown in Fig. 18.3. A atrium, V ventricle, H His bundle
A. Rossillo et al.
Case 18
79
Fig. 18.15 Intracardiac recordings taken during continuous atrial stimulation with an infusion of isoproterenol IV (paper speed 100 mm/s) showing the Wenckebach point and without induction of tachycardia. Same display as shown in Fig. 18.3. A atrium, V ventricle, H His bundle
Fig. 18.16 RAO and LAO of mapping catheter at the ablation site in the posteroseptal part of Koch’s triangle
RAO
Bibliography Delise P, Gianfranchi L, Paparella N, et al. Clinical usefulness of slow pathway ablation in patients with both paroxysmal atrioventricular nodal reentrant tachycardia and atrial fibrillation. Am J Cardiol. 1997;79:1421-1423. Delise P, Sitta N, Bonso A, et al. Pace mapping of Koch’s triangle reduces risk of atrioventricular block during ablation of atrioventricular nodal reentrant tachycardia. J Cardiovasc Electrophysiol. 2005;16:30-35. Haissaguerre M, Gaita F, Fischer B, et al. Elimination of atrioventricular nodal reentrant tachycardia using discrete slow potentials to
LAO
guide application of radiofrequency energy. Circulation. 1992;85: 2162-2175. Haïssaguerre M, Jaïs P, Shah DC, et al. Analysis of electrophysiological activity in Koch’s triangle relevant to ablation of the slow AV nodal pathway. Pacing Clin Electrophysiol. 1997;20:2470-2481. Jackman WM, Beckman KJ, McClelland JH, et al. Treatment of supraventricular tachycardia due to atrioventricular nodal reentry, by radiofrequency catheter ablation of slow-pathway conduction. N Engl J Med. 1992;327:313-318. Natale A, Greenfield RA, Geiger MJ, et al. Safety of slow pathway ablation in patients with long PR interval: further evidence of fast and slow pathway interaction. Pacing Clin Electrophysiol. 1997;20: 1698-1703.
Case 19 Yash Y. Lokhandwala, Anoop K. Gupta, and Ranjan K. Thakur
Case Summary A 50-year-old female with a normal heart presents with a wide complex tachycardia (Fig. 19.1). The patient is taken to the electrophysiology (EP) lab (Fig. 19.2). What clues to the diagnosis are present in the electrocardiogram (ECG)? Is this supraventricular tachycardia (SVT) or ventricular tachycardia (VT)? What is the mechanism of the tachycardia?
Case Discussion The tachycardia has right bundle branch block (RBBB) morphology with QRS axis +90. There is a sudden change in cycle length (CL), but it is not a multiple of the shorter
CL. The bundle branch block has a typical morphology and R > S in V6. Hence, this is SVT, not VT. The diagnosis of SVT confirmed, note that the HV interval is normal. The first two complexes show A-waves in the coronary sinus (CS) simultaneous with the QRS, suggesting that this is most likely slow/fast atrioventricular nodal reentrant tachycardia (AVNRT). Then, two premature atrial complexes (PACs) are delivered from CS 5-6. The first PAC does not reset the tachycardia. The second PAC conducts with a longer AH interval, and subsequently the tachycardia has a longer CL, but the retrograde atrial activation remains the same, suggesting the presence of a different slow pathway. There is baseline RBBB. The patient has slow–fast AVNRT with two slow pathways, participating in AVNRT; hence, she has two CLs.
Y.Y. Lokhandwala (*) KEM Hospital, Parel, Mumbai, India A.K. Gupta Apollo Hospital, Ahmedabad, India R.K. Thakur Thoracic and Cardiovascular Institute, Sparrow Health System, Michigan State University, 405 West Greenlawn, Suite 400, Lansing, MI 48910
A. Natale et al. (eds.), Cardiac Electrophysiology, DOI: 10.1007/978-1-84996-390-9_19, © Springer-Verlag London Limited 2011
81
82
Y.Y. Lokhandwala et al. 2008070
I II III AVR AVL AVF V1 V2 V3 V4 V5 V6 10 mm/mV 25 mm/s
Fig. 19.1 Electrocardiogram (ECG) of wide complex tachycardia
2008070 I AVF V1 V6 CS910 CS78 CS56 CS34 CS12 HISD HISP
Fig. 19.2 Intracardiac recordings. CS 1-2 is distal
Case 20 Marmar Vaseghi, Noel G. Boyle, and Kalyanam Shivkumar
Case Summary A 21-year-old female with a history of viral cardiomyopathy status post orthotopic heart transplantation (OHT) in June 2002 presented with recurrent palpitations three years post transplant. She had received a right atrio–atrial anastomosis at the time of transplantation. She was found to have an atrial flutter (AFL) on surface electrocardiogram (ECG) that proved refractory to medications. Her cardiac catheterization and biopsy showed no evidence of transplant coronary artery vasculopathy or acute rejection, respectively. She was referred for ablation of her AFL.Fluoroscopic position of the duodecapolar catheter in the right anterior oblique (RAO) view and the corresponding ECG recordings are shown in
A
Fig. 20.1 Panel A: Fluoroscopic RAO view shows the position of the duodecapolar catheter. Panel B: Intracardiac activation sequence. CS coronary sinus, D1–D10 duodecapolar bipoles 1–10, DDC duodecapolar catheter, RV right ventricle, LV left ventricle
Fig. 20.1. Duodecapolar bipoles 6–9 were positioned more posteriorly at the native atrium. What type of AFL is demonstrated by the intracardiac duodecapolar ECGs? Is there activation of the native atrium? What is the rate of the native atrial rhythm?
Case Discussion The duodecapolar recordings from the donor atrium showed a typical tricuspid annulus dependent counterclockwise AFL; Fig. 20.2 (top panels). This is the most common type of supraventricular tachycardia (SVT) in stable OHT patients.1
B I AVF V1 V6 D10 D9 D8 D7 D6 D5 D4 D3 D2 D1
200ms
M. Vaseghi (*), N.G. Boyle, and K. Shivkumar UCLA Cardiac Arrhythmia Center, David Geffen School of Medicine at UCLA, 10833 Le Conte Ave, A2-237 CHS, Los Angeles, CA 90095-1679 e-mail:
[email protected] A. Natale et al. (eds.), Cardiac Electrophysiology, DOI: 10.1007/978-1-84996-390-9_20, © Springer-Verlag London Limited 2011
83
84 Fig. 20.2 Panel A: Fluoroscopic RAO view shows the position of the duodecapolar catheter with the distal bipoles (D1–D3) along the tricuspid valve isthmus and the proximal electrode (D10) along the superior aspect of the interatrial septum. Panel B: Intracardiac activation sequence in the donor atrium confirms counterclockwise activation. CS coronary sinus, D1–D10 duodecapolar bipoles 1–10, DDC duodecapolar catheter, RV right ventricle, LV left ventricle
M. Vaseghi et al.
A
B I AVF V1 V6 D10 D9 D8 D7 D6 D5 D4 D3 D2 D1
200ms
Fig. 20.3 Electroanatomic maps of atrio-atrial anastamosis (gray areas identify native atrial tissue). The flutter wavefront involves the donor atrium only and is propagating in a counterclockwise fashion along the tricuspid annulus
Poles D6–D9 are in contact with the native atrial tissue, which is not in flutter, and is in a dissociated regular bradycardic rhythm of 40 bpm (beats per minute) (arrow), separate from the tachycardia. A critical isthmus is noted on the intracardiac ECGs at duodecopolar position D5. The activation map is shown in Fig. 20.3. An inferior vena cava (IVC) – tricuspid isthmus ablation line terminated the donor atrial tachycardia. No ablation was performed on the native atrium.
Reference 1. Vaseghi M, Boyle NG, Kedia R, et al. Supraventricular tachycardia following orthotopic heart transplantation. J Am Coll Cardiol. 2008;51:2241-2249.
Case 21 Bradley P. Knight
Case Summary A young man underwent an electrophysiology (EP) procedure for recurrent supraventricular tachycardia (SVT). The recording obtained after induction of the tachycardia is shown in Fig. 21.1. Surface intracardiac recordings from the His bundle electrogram (HBE), mapping catheter (Map), coronary sinus (CS), and right ventricle (RV) are shown. What is the mechanism of the tachycardia?
Case Discussion The initial part of the tracing shows a wide complex tachycardia with a cycle length (CL) of 320 ms. There is a His-bundle recording before the QRS complex, which has a typical left bundle branch block (LBBB) pattern. Therefore, the tachycardia is consistent with a supraventricular mechanism. The
earliest atrial activation can be seen in the proximal CS at electrode pair CS 3-4. The QRS complex, however, normalizes during the second part of the recording. An important measurement to make – whenever a SVT is associated with both a normal QRS complex and a bundle branch block – is the VA interval. The reason for this is that when the tachycardia mechanism is orthodromic reentrant tachycardia (ORT), and a rate-related bundle branch block develops on the same side as the accessory pathway (AP), the reentrant circuit will increase in length (due to transeptal conduction) and results in a longer VA interval during the ipsilateral bundle branch block. In this case, the VA interval is clearly longer during the LBBB compared to when the QRS complex normalizes. This is diagnostic for ORT using a left-sided AP. This case is also interesting, because the tachycardia CL does not change significantly during the LBBB, despite an increase in the VA interval. This is because there is a compensatory decrease in the AH interval during the LBBB.
B.P. Knight Division of Cardiology, Northwestern Medical Center, 676 N. St. Clair, Suite 600, Chicago, IL 60611 A. Natale et al. (eds.), Cardiac Electrophysiology, DOI: 10.1007/978-1-84996-390-9_21, © Springer-Verlag London Limited 2011
85
86
B.P. Knight I II III V1 V5 HBE d HBE p Map d Map p CS 1-2 CS 3-4 CS 5-6 CS 7-8 CS 9-10 RV d Stim
Fig. 21.1 This tracing was obtained during an ablation procedure for supraventricular tachycardia. Note the change in the ventricular atrial interval when the wide-QRS complex normalizes. The vertical line
depicts the onset of the surface QRS complex in each case and the arrow points to the earliest intracardiac atrial activation
Case 22 Antonio Rossillo, Sakis Themistoclakis, Aldo Bonso, Andrea Corrado, and Antonio Raviele
Case Summary
Case Discussion
The patient was a 34-year-old female with a history of palpitations. No heart disease was documented. Two months prior to the ablation the arrhythmia episodes became more frequent. After examining Figs. 22.1–22.4, will ablation at a single site eliminate the patient’s palpitations?
In this case, catheter manipulation induced AVNRT. Ventricular pacing demonstrated retrograde conduction via a left-sided accessory pathway. AVRT was also easily induced and was consistent with the clinical tachycardia. Ablation of the left-sided AP and the slow pathway were performed (see Figs. 22.5–22.11).
I II III aVR aVL aVF V1 V2 V3 V4 V5 V6
Fig. 22.1 Twelve-lead resting ECG (paper speed 25 mm/s) showing short PQ interval with ventricular pre-exitation with an early transition of R wave between lead V1 and V2 and negative wave in leads III and aVF
A. Rossillo (), S. Themistoclakis, A. Bonso, A. Corrado, and A. Raviele Ospedale dell’Angelo, Mestre, Venice, Italy A. Natale et al. (eds.), Cardiac Electrophysiology, DOI: 10.1007/978-1-84996-390-9_22, © Springer-Verlag London Limited 2011
87
88
Fig. 22.2 Intracardiac recordings at baseline during the electrophysiology study (paper speed 100 mm/s). Four surface ECG leads (I, aVF, V1, V6), two bipolar recordings from the pacing catheter placed in right
A. Rossillo et al.
atrium or ventricle (HRA), three bipolar recordings from the His bundle region (HIS PROX, MED, and DIST), and five bipolar recordings from the coronary sinus (CS). A Atrium, V Ventricle, H His bundle
Fig. 22.3 Intracardiac recordings taken during SVT induced with catheter manipulation (paper speed 100 mm/s, cycle length 330 ms). Same display as that shown in Fig. 22.2. A Atrium, V Ventricle, H His bundle
Case 22
89
Fig. 22.4 Intracardiac recordings taken during programmed ventricular stimulation with induction of SVT (paper speed 100 mm/s, cycle length 290 ms) with earliest activation on CS 3. Same display as that shown in Fig. 22.2. A Atrium, V Ventricle
Fig. 22.5 Intracardiac recordings taken during AVRT (paper speed 100 mm/s, cycle length 264 ms). Same display as that shown in Fig. 22.2 and a bipolar recording from the distal mapping catheter (ABL dist).
A spontaneous ventricular ectopic beat reduced the cycle length of the tachycardia and the anterograde conduction after the ectopic beat is through nodal decremental fibers
90
A. Rossillo et al.
Fig. 22.6 Intracardiac recordings taken during sinus rhythm (paper speed 100 mm/s). Same display as that shown in Fig. 22.5. Ablation site
III aVF V1 V6 HRA 1-2 HRA 3-4 HIS PROX HIS MED HIS DIST CS 1 CS 2 CS 3 CS 4 CS 5 ABL dist
Fig. 22.7 Intracardiac recordings taken during RF energy delivery (paper speed 100 mm/s). Same display as that shown in Fig. 22.5. The red arrows show the absence of the ventricular pre-excitation
Case 22
91
Fig. 22.8 Intracardiac recordings taken during RF energy delivery (paper speed 25 mm/s). Same display as that shown in Fig. 22.5. The red arrows show the absence of ventricular pre-excitation
III aVF V1 V6 HRA 1-2 HRA 3-4 HIS PROX HIS MED HIS DIST CS 1 CS 2 CS 3 CS 4 CS 5 ABL dist V
A
V
A
Fig. 22.9 Intracardiac recordings (paper speed 100 mm/s) taken during programmed ventricular stimulation showing a decremental VA conduction (paper speed 100 mm/s). Same display as that shown in Fig. 22.5. A Atrium, V Ventricle
92
A. Rossillo et al.
Fig. 22.10 Intracardiac recordings (paper speed 100 mm/s) taken during programmed atrial stimulation with two extrastimuli showing the ERP of the AV node (paper speed 100 mm/s). Same display as that shown in Fig. 22.5. A Atrium, V Ventricle
I II III aVR aVL aVF V1 V2 V3 V4 V5 V6
Fig. 22.11 12-lead resting ECG (paper speed 25 mm/s) showing normal PQ interval with the absence of ventricular pre-excitation with negative T waves in the inferior leads
Case 22
Bibliography Calkins H, Langberg J, Sousa J, et al. Radiofrequency catheter ablation of accessory atrioventricular connections in 250 patients. Abbreviated therapeutic approach to Wolff-Parkinson-White syndrome. Circulation. 1992;85:1337-1346. Haïssaguerre M, Dartigues JF, Warin JF, Le Metayer P, Montserrat P, Salamon R. Electrogram patterns predictive of successful catheter
93 ablation of accessory pathways. Value of unipolar recording mode. Circulation. 1991;84:188-202. Jackman WM, Wang XZ, Friday KJ, et al. Catheter ablation of accessory atrioventricular pathways (Wolff-Parkinson-White syndrome) by radiofrequency current. N Engl J Med. 1991;324:1605-1611. Scheinman MM, Wang YS, Van Hare GF, Lesh MD. Electrocardio graphic and electrophysiologic characteristics of anterior, midseptal and right anterior free wall accessory pathways. J Am Coll Cardiol. 1992;20:1220-1229.
Case 23 Yash Y. Lokhandwala, Anoop K. Gupta, and Ranjan K. Thakur
Case Summary
Case Discussion
The patient is a 58-year-old male with a normal heart. The resting electrocardiogram (ECG) is normal. He has a narrow QRS tachycardia. He is taken to the electrophysiology (EP) lab; see Figs. 23.1 and 23.2. What is the likely mechanism of this tachycardia?
The ventriculo-atrial (VA) conduction pattern and VA time are different in the two beats in Fig. 23.1. The first retrograde A is bracketed, earliest in coronary sinus (CS) 3-4 and 5-6; the second retrograde A is also bracketed, but earliest in CS 5-6 and 7-8. Figure 23.2 shows para-Hisian pacing. The VA
Fig. 23.1 Ventricular pacing
Y.Y. Lokhandwala (*) KEM Hospital, Parel, Mumbai, India A.K. Gupta Apollo Hospital, Ahmedabad, India R.K. Thakur Thoracic and Cardiovascular Institute, Sparrow Health System, Michigan State University, 405 West Greenlawn, Suite 400, Lansing, MI 48910 A. Natale et al. (eds.), Cardiac Electrophysiology, DOI: 10.1007/978-1-84996-390-9_23, © Springer-Verlag London Limited 2011
95
96
Y.Y. Lokhandwala et al.
Fig. 23.2 Para-Hisian pacing
time clearly is shortest with His bundle capture (second complex). The VA pattern of the first complex is different from the other two complexes. The first retrograde A is earliest in CS 5-6. The second and third retrograde A are earliest in CS 7-8. Thus the second and third retrograde A are via the AV node; the first is likely via a left AP. The tachycardia displays varying degrees of right bundle branch block (RBBB) during SVT (Fig. 23.3) (normal HV). The earliest retrograde A is consistently in CS 5-6, with a constant VA interval. CS 9-10 was at CS os (ostium). Thus the retrograde A is via a concealed left posterior AP.
In Fig. 23.4, tachycardia has a left bundle branch block (LBBB) morphology. Compared to the previous tracing, the VA time is clearly longer in this case. This is consistent with our prior diagnosis of a left-sided AP. Ablation was performed during right ventricular (RV) pacing (Fig. 23.5) because this allows for catheter stability. If ablation is performed during reentrant tachycardia, the catheter may move suddenly when RT terminates during RF application. So, catheter stability can be obtained by entraining the RT at 10–20 ms faster than the tachycardia CL. The VA conduction during RV pacing changes from left AP to AV node.
Case 23
Fig. 23.3 Development of a right bundle branch block (RBBB)
97
98
Fig. 23.4 Development of a left bundle branch block (LBBB)
Y.Y. Lokhandwala et al.
Case 23
Fig. 23.5 Ablation of AP
99
Case 24 Marmar Vaseghi, Noel G. Boyle, and Kalyanam Shivkumar
Case Summary
graphy demonstrated a normal ejection fraction at the time of arrhythmia. Surface electrocardiogram (ECG) was consistent with atypical atrial flutter (AFL). The patient was taken to the electrophysiology (EP) laboratory. Intracardiac ECGs along with location of the catheters are shown in (Fig. 24.1). What is the likely mechanism of this arrhythmia based on the duodecapolar and coronary sinus (CS) catheter recordings?
A 60-year-old male status post orthotopic heart transplantation (OHT) in June 1999 presented with drug refractory palpitations 8 years after his transplantation. Cardiac catheterization and biopsy showed no evidence of transplant vasculopathy or acute rejection, respectively. Echocardio
A
B
I aVF V6
200 msec
D1 D2 D3 D4 D5 D6 D7 D8 D9 D10 CS 9,10 CS 7,8 CS 5,6 CS 3,4 CS 1,2 RVa
Fig. 24.1 Panel A: Fluoroscopic LAO view: The position of the duodecapolar and CS catheters are shown in right atrium. Panel B: Intracardiac tracings obtained from the duodecapolar and CS catheters are shown.
CS coronary sinus, D1–D10 duodcapolar bipoles 1–10, LAO left anterior oblique, RVa RV catheter bipole
M. Vaseghi (*), N.G. Boyle, and K. Shivkumar UCLA Cardiac Arrhythmia Center, David Geffen School of Medicine at UCLA, 10833 Le Conte Ave, A2-237 CHS, Los Angeles, CA 90095-1679 e-mail:
[email protected] A. Natale et al. (eds.), Cardiac Electrophysiology, DOI: 10.1007/978-1-84996-390-9_24, © Springer-Verlag London Limited 2011
101
102
Case Discussion Although typical clockwise AFL is the most common type of supraventricular tachycardia (SVT) in stable post-OHT patients, this patient’s intracardiac tracings show earliest activation in the distal CS consistent with a left AFL. Further, the flutter pattern spreads from the distal CS to the proximal CS and then to the right atrium where D10 (or septal activation is earliest). This is followed by activation
Fig. 24.2 Activation map: the flutter circuit begins in the inferior wall of the LA (pink) close to the CS 3-4 bipole, and then propagates across to the RA and around the tricuspid annulus, as well as around and over the mitral annulus. LA left atrium, LAA left atrial appendage, LIPV left inferior pulmonary vein, LSPV left superior pulmonary vein, RA right atrium, RSPV right superior pulmonary vein, SVC superior vena cava
M. Vaseghi et al.
around the tricuspid annulus. Hence this left AFL spreads in a figure-eight fashion around both the tricuspid and mitral annuli. Mapping with ablation catheters showed earliest activation at CS 1-2 and ostium of the left inferior pulmonary vein. The activation map obtained via the ablation catheter and the ablation line, which successfully terminated the tachycardia, are shown in (Fig. 24.2). The patient has been without recurrence during the 10 months of follow-up.
Case 25 Bradley P. Knight
Case Summary A female marathon runner without structural heart disease had a documented regular wide complex tachycardia (Fig. 25.1). At the time of electrophysiology (EP) testing, she had normal sinus and atrioventricular (AV) node function, no ventricular preexcitation, and no inducible ventricular tachycardia (VT). A wide complex tachycardia was induced with an atrial premature extrastimulus during sinus rhythm (Fig. 25.2). The QRS morphology was a right bundle branch block (RBBB) pattern with an inferior axis. There was not His-bundle potential recordable before the QRS during the tachycardia. What is the most likely diagnosis?
tachycardia was induced by an atrial premature beat that conducts to the ventricle, with a QRS morphology that is the same as that during tachycardia, makes VT very unlikely. SVT with aberrancy can also be excluded because a Hisbundle potential could not be recorded before the QRS complex. Therefore, the most likely diagnosis is ART. In a patient with no preexcitation during sinus rhythm, the most common mechanism of ART is a slowly conducting, right-sided atriofascicular accessory pathway (AP). A study of 384 patients with a single AP found that anterograde decremental conduction was seen only in the right free wall location. However, the QRS morphology in this patient is not consistent with a right-sided AP. In this case, the earliest ventricular activation was along the mitral annulus where ablation was successful (Fig. 25.3). This represents a very unusual location for a slowly conducting anterograde AP.
Case Discussion The differential of a wide complex tachycardia includes VT, supraventricular tachycardia (SVT) with aberrancy, and antidromic AV reentrant tachycardia (ART). The fact that this
B.P. Knight Division of Cardiology, Northwestern Medical Center, 676 N. St. Clair, Suite 600, Chicago, IL 60611 A. Natale et al. (eds.), Cardiac Electrophysiology, DOI: 10.1007/978-1-84996-390-9_25, © Springer-Verlag London Limited 2011
103
104
B.P. Knight
Fig. 25.1 Shown is a two lead rhythm strip showing the onset of a sustained wide-QRS complex tachycardia. The tracing was recorded from an ambulatory event recorder
Case 25
Fig. 25.2 This tracing was recorded during an ablation procedure in a patient with a wide complex tachycardia. Shown are surface electrograms from leads I, II, V1, and V3, and the intracardiac electrograms from the high right atrium (HRA), the ablation catheter (ABL) posi-
105
tioned at the His bundle location, and the right ventricular apex (RVA). Note the initiation of a wide complex tachycardia with a single atrial extrastimulus delivered during sinus rhythm
106
B.P. Knight
Fig. 25.3 This tracing was recorded after the ablation catheter was positioned along the mitral annulus at the site of the earliest ventricular local activation. The format is the same as figure 25.2
Case 26 Richard H. Hongo and Andrea Natale
Case Summary The patient is a 53-year-old woman with recurrent palpitations since age 16 with documented episodes of tachyarrhythmia (Fig. 26.1). Discrete P-waves at a rate of approximately 180 bpm have been apparent with atrioventricular (AV) node blockade from adenosine (Fig. 26.2).
She has been treated with a variety of antiarrhythmic agents including verapamil, sotalol, and amiodarone with variable success. She presented for electrophysiology (EP) study with possible catheter ablation because of highly symptomatic episodes of atrial arrhythmia that continued to recur despite medical therapy. High-dose isoproterenol infusion (20 mcg/min)
Fig. 26.1 Continuous 3-lead (II, V1, V5) ECG capturing recurrent episodes of narrow-complex regular tachyarrhythmia at a rate of approximately 180 bpm
R.H. Hongo Sutter Pacific Medical Foundation, California Pacific Medical Center, 2100 Webster Street, Suite 521, San Francisco, CA 94115, USA e-mail:
[email protected] A. Natale () Texas Cardiac Arrhythmia Institute, St. David’s Medical Center, 1015 East 32nd Street, Suite 516, Austin, TX 78705 e-mail:
[email protected] A. Natale et al. (eds.), Cardiac Electrophysiology, DOI: 10.1007/978-1-84996-390-9_26, © Springer-Verlag London Limited 2011
107
108
during EP study induced a tachyarrhythmia with a ventricular rate of 150 bpm (Fig. 26.3). Double transseptal puncture was performed and a 20-mm 10-electrode lasso catheter and a 3.5-mm F-curve saline irrigated ablation catheter were advanced into the left atrium (LA). Intracardiac activation of the atrial arrhythmia was recor ded (Fig. 26.4) with the lasso catheter at the os of the right superior pulmonary vein (RSPV) and the ablation catheter approximately 6 mm within the same vein (Fig. 26.5). A 20-electrode deflectable catheter was introduced from the right internal jugular vein and placed along the lateral wall of the right atrium (RA) with the distal 10 electrodes in the coronary sinus (CS). What is the tachyarrhythmia based on the surface electrocardiogram (ECG) before and during adenosine, and assuming it is the same arrhythmia induced with isoproterenol? What is the tachyarrhythmia based on the intracardiac ECGs? What is the most appropriate next step?
R.H. Hongo and A. Natale
Case Discussion AV block during adenosine infusion (Fig. 26.2) uncovers discrete P-waves most consistent with atrial tachycardia. Distinct isoelectric segments between the P-waves in all twelve leads of the ECG during isoproterenol infusion (Fig. 26.3) makes both typical and atypical flutter less likely. Loss of 1:1 AV association eliminates atrioventricular reciprocating tachycardia (ART) as a diagnosis. Intracardiac ECGs (Fig. 26.4) reveal the earliest activation to be at the distal ablation catheter positioned within the RSPV (Fig. 26.5), clearly before the activation around the lasso catheter at the vein os. The most appropriate ablation strategy is to isolate the RSPV. Ablation at the earliest activation site within the vein should be avoided because of the risk of PV stenosis. If either atrial fibrillation (AF) or typical atrial flutter (AFL) were observed clinically, isolation of all four pulmonary veins and cavotricuspid isthmus ablation, respectively, would also be appropriate.
Fig. 26.2 Continuous 3-lead (II, V1, V5) ECG during adenosine-induced AV block. Uncovered discrete P-waves are most consistent with atrial tachycardia
Case 26
109
Fig. 26.3 12-lead ECG during isoproterenol infusion (20 mcg/min) demonstrates sustained atrial tachyarrhythmia. Distinct isoelectric PR segments are seen in all leads I
200 ms
aVF V1
S
V6 HRA 19,20 HRA 17,18 HRA 15,16 HRA 13,14 HRA 11,12 ABL p ABL d CS 9,10 CS 7,8
C
CS 5,6
C
CS 3,4
C
CS d
C
Ls 1,2 Ls 2,3 Ls 3,4 Ls 4,5 Ls 5,6 Ls 6,7 Ls 7,8 Ls 8,9 Ls 9,10
10:06:09 AM
10:06:10 AM
CardioLab v6.5 GE Medical Systems
Fig. 26.4 Simultaneous surface and intracardiac ECGs during the atrial tachyarrhythmia reveal the earliest activation to be at the distal electrode pair of the ablation catheter (ABL d), clearly in front of the activation seen on the Lasso catheter. HRA = high right atrial catheter; CS = coronary sinus catheter; Ls = Lasso catheter
110
Fig. 26.5 Fluoroscopic image in shallow LAO projection, showing the position of the catheters that correspond with the intracardiac ECGs from Fig. 26.4. The Lasso catheter is placed at the right superior pulmonary vein (RSPV) os. The ablation catheter is shown positioned within the RSPV
R.H. Hongo and A. Natale
Case 27 M. Eyman Mortada, Jasbir S. Sra, and Masood Akhtar
Case Summary A 63-year-old female with a history of tachycardia 7 years ago with the electrocardiogram (ECG) shown in Fig. 27.1 presented complaining of recurrent palpitations with the tachycardia shown in Fig. 27.2. During an electrophysiology (EP) study, the responses from premature ventricular beat and ventricular pacing were recorded during the tachycardia, as shown in Figs. 27.3 and 27.4. What is the diagnosis?
Case Discussion The presenting tachycardia is characterized by a long RP, narrow complex tachycardia, with low septal to high atrial activity (negative P-wave in lead II and positive P-waves in leads aVR and aVL). Therefore, the differential diagnoses include: low septal atrial tachycardia, atypical AV nodal reentry tachycardia (AVNRT), and persistent junctional reciprocating tachycardia.
Seven years prior to this presentation, the patient had short RP and narrow complex tachycardia. The differential diagnoses of that rhythm include: typical AVNRT, atrial tachycardia (AT) with long PR interval, and AV reentry tachycardia via an accessory pathway (AP). The presence of both rhythms in one patient means this is probably AVNRT. The first rhythm was probably a typical tachycardia. The patient now presents with atypical AVNRT. The first intracardiac tracing reveals earliest atrial activation is in the proximal coronary sinus (CS), suggesting an AT originating in the lower septum, retrograde activation over the AV node, or retrograde activation over a septal AP. A spontaneous premature ventricular capture (PVC) during the tachycardia, at the time of the His refractory, was observed. At that time, the tachycardia cycle length (CL) was not changed. The presence of a PVC reduces the likelihood that the mechanism is an AV reentry tachycardia via a septal AP. The second intracardiac tracing illustrates ventricular entrainment during the tachycardia. Post–ventricular pacing shows a V-A-V response, excluding the diagnosis of a low septal AT. Thus, the diagnosis is atypical AVNRT.
M.E. Mortada (*) J.S. Sra, and M. Akhtar Department of Electrophysiology, Aurora Cardiovascular Services, Aurora Sinai/Aurora St. Luke’s Medical Centers, University of Wisconsin School of Medicine and Public Health, 2801 W. Kinnickinnic River Parkway, #777, Milwaukee, WI 53215 A. Natale et al. (eds.), Cardiac Electrophysiology, DOI: 10.1007/978-1-84996-390-9_27, © Springer-Verlag London Limited 2011
111
112
Fig. 27.1 Surface ECG of the tachycardia seven years prior to the current presentation
Fig. 27.2 Surface ECG of the current tachycardia
M.E. Mortada et al.
Case 27
113
Fig. 27.3 Spontaneous PVC during the tachycardia at the His refractory period. From top to bottom: surface ECG with leads I, II and V1, followed by intracardiac tracing of HRA, proximal to distal CS, and finally His catheters
Fig. 27.4 Response from ventricular entrainment of the tachycardia
Case 28 Yash Y. Lokhandwala, Anoop K. Gupta, and Ranjan K. Thakur
Case Summary A 21-year-old female with a normal heart comes to the electrophysiology (EP) lab because of palpitations. A diagnosis of typical atrioventricular reentrant tachycardia (AVNRT) is made (Fig. 28.1). However, after atrial pacing, long PR and sudden left bundle branch block (LBBB) – like morphology is seen (Figs. 28.2 and 28.3). What maneuver is being performed? What does this say about the tachycardia mechanism?
during tachycardia; this caused advancement of the next QRS. The QRS advancement by a “His-refractory” APD proves the presence of an extranodal (atriofascicular or atrioventricular accessory pathway) (Fig. 28.4). The HA during tachycardia is identical to HA during RV pacing; thus the Mahaim-like accessory pathway is not a bystander. Final diagnosis: (1) Right lateral Mahaim-like AP with true antidromic AVRT and (2) slow–fast, typical AVNRT (Figs. 28.5 and 28.6).
Case Discussion An atrial premature depolarization (APD) was delivered from a radiofrequency (RF) catheter in the right atrium (RA),
Y.Y. Lokhandwala () KEM Hospital, Parel, Mumbai, India A.K. Gupta Apollo Hospital, Ahmedabad, India R.K. Thakur Thoracic and Cardiovascular Institute, Sparrow Health System, Michigan State University, 405 West Greenlawn, Suite 400, Lansing, MI 48910
A. Natale et al. (eds.), Cardiac Electrophysiology, DOI: 10.1007/978-1-84996-390-9_28, © Springer-Verlag London Limited 2011
115
116
Y.Y. Lokhandwala et al.
RUBY HALL CLINIC I
2008051
AVF V1 V6
CS910
CS78
CS56
CS34
CS12
HISP
RFD
RFP 10 mm/mV 100 mm/s
Fig. 28.1 Typical atrioventricular reentrant tachycardia (AVNRT)
Version WIN2000/XP : EPTRACER V0.77
Case 28 RUBY HALL CLINIC 2008051 I II III AVR AVL AVF V1 V2 V3 V4 V5 V6 10 mm/mV 25 mm/s
Fig. 28.2 Change to left bundle branch block (LBBB) morphology is seen with atrial pacing
117 Version WIN2000/XP : EPTRACER V0.77
118
Y.Y. Lokhandwala et al. Version WIN2000/XP : EPTRACER V0.77
I
2008051
AVF V1 V6
CS910
CS78
CS56
CS34
CS12
HISP
RFD
10 mm/mV 100 mm/s
Fig. 28.3 A diagnostic maneuver. Does this help with a diagnosis?
Case 28
119 Version WIN2000/XP : EPTRACER V0.77
I
2008051
AVF V1 V6
CS910
CS78
CS56
CS34
CS12
HISP
RFD
10 mm/mV 200 mm/s
Fig. 28.4 APC delivered from the RA free wall
120
Y.Y. Lokhandwala et al.
RUBY HALL CLINIC 2008051 I AVF V1 V6 CS78 CS56 CS34 CS12 HISP HISM HISD RBP RBD RFD 10 mm/mV 200 mm/s
Fig. 28.5 Intracardiac recording during SVT
Version WIN2000/XP : EPTRACER V0.77
Case 28
121 Version WIN2000/XP : EPTRACER V0.77
2008051 I AVF V1 V6 CS78 CS56 CS34 CS12 HISP HISM HISD RBP RBD RFD 10 mm/mV 200 mm/s
Fig. 28.6 RV pacing
Case 291 Eric Buch, Shiro Nakahara, Marmar Vaseghi, Noel G. Boyle, and Kalyanam Shivkumar
Case Summary
Case Discussion
A 57-year-old female with recurrent atrial flutter (AFL) was referred for catheter ablation. The surface electrocardiogram (ECG) suggested clockwise AFL with positive F-waves in II, III, and AVF, and negative F-wave in V1. The complete absence of an R-wave in lead I was noted. The chest x-ray demonstrated dextrocardia, and echocardiography established complete situs inversus with otherwise normal intracardiac anatomy. Based on the ECG, what is the likely diagnosis?
A duodecapolar (DD) catheter was positioned around the tricuspid annulus in the anatomical right atrium (RA) with the distal tip located at the medial cavotricuspid isthmus (Fig. 29.1). Electrophysiological mapping of the AFL demonstrated earliest activation in the electrogram recorded from DD poles 19,20 and latest activation at DD poles 1,2. Top left image shows an LAO view of the heart (camera in the RAO position relative to the torso) and direction of flutter (white arrows) as shown by
Fig. 29.1 Fluoroscopy of intracardiac catheters. Top left panel shows LAO view of the heart (camera in the RAO position relative to the torso). Top right panel shows the RAO view of the heart (camera in the
LAO position relative to the torso). CS coronary sinus catheter, Abl ablation catheter, HRA high right atrial catheter, His His bundle catheter
This case is adapted from heart rhythm volume 2, issue 6, pages 673-674 (June 2005)
1
E. Buch (), S. Nakahara, M. Vaseghi, N.G. Boyle, and K. Shivkumar UCLA Cardiac Arrhythmia Center,David Geffen School of Medicine at UCLA, 10833 Le Conte Ave, A2-237 CHS, Los Angeles, CA 90095-1679 e-mail:
[email protected] A. Natale et al. (eds.), Cardiac Electrophysiology, DOI: 10.1007/978-1-84996-390-9_29, © Springer-Verlag London Limited 2011
123
124
E. Buch et al.
the intracardiac tracings, demonstrating early activation in DD19, 20 and counterclockwise rotation of flutter in the RA (Fig. 29.2). Entrainment pacing demonstrated that the
tricuspid valve – inferior vena cava (IVC) isthmus was a part of the circuit and this tachycardia was cured by ablation of the isthmus.
I II III aVF V1 HRA DD19,20 DD17,18 DD15,16 DD13,14 DD11,12 DD9,10 DD7,8 DD5,6 DD3,4 DD1,2 HIS p HIS m HIS d CS 9,10 CS 7,8 CS 5,6 CS 3,4 CS 1,2 RVa Stim 3
200 ms
12:32:15 PM
12:32:16 PM
Fig. 29.2 Intracardiac electrograms. In this patient with mirror-image dextrocardia, typical flutter was counterclockwise (CCW) in the RA, as demonstrated by the wave front of activation on duodecapolar (DD) catheter. However, the inferior leads of the EKG inscribe F-waves that are positive, which is suggestive of clockwise (CW) flutter. This discrepancy
12:32:17 PM
is attributed to the fact that in the situs inversus heart, because of the rotated position of the atria and ventricles (mirror image of a situs solitus heart), CCW flutter appears CW (its mirror image) on the surface EKG. Hence, the position of the heart in the thorax must be kept in mind when evaluating CW versus CCW flutter based on a surface EKG
Case 30 Luis C. Sáenz and Miguel A. Vacca
Case Summary The patient is a 5-year-old male with a past medical history significant for paroxysmal palpitations which became incessant 8 months ago despite taking various anti-arrhythmic agents (AADs). The patient had worsening congestive heart failure and was hospitalized. He had no other significant past medical history. The tachycardia EKG taken in the emergency room is showed in Fig. 30.1. His echocardiogram showed left ventricular dilatation and ejection fraction of 38%. While the patient was in the hospital another tracing was taken showing spontaneous changes in his electrical rhythm (Fig. 30.2a and b). Variable atrioventricular conduction during the tachycardia with an isoelectric interval between the p waves is showed in Fig. 30.2a suggesting an atrial tachycardia as the mechanism of the arrhythmia. After
increasing the doses of the AAD medications, a transitory and regular rhythm with “1 per 1” atrioventricular conduction was registered, and is showed in Fig. 30.2b. Considering tachycardiomyopathy and the failure of the AAD to prevent arrhythmia recurrences, the patient was brought to the electrophysiology lab for ablation. CARTO was used during the procedure. A decapolar multielectrode catheter was introduced into the coronary sinus for mapping. In Fig. 30.3, the coronary sinus channels are showed from proximal (SC5) to distal (SCD). The third coronary sinus channel is not showed due to electrical noise. The intra-cardiac electrograms from the roving CARTO mapping and ablation catheter are showed as RFD and RFP for the bipolar electrograms that were taken from the distal1,2 and proximal3,4 par of electrodes, respectively. The unipolar electrogram from the electrodes 1 and 3 of the CARTO roving catheter are showed as 1 and 3, respectively. A CARTO
Fig. 30.1 EKG of tachycardia taken in the emergency room L.C. Sáenz (*) Jefe Servicio de Electrofisiología, Cardiólogo-Electrofisiólogo, Fundación Cardio Infantil-Instituto de Cardiología, Calle 163A No 28-60, Bogotá, Colombia M.A. Vacca Cardiac Electrophysiologist, Centro Internacional de Arritmias FCI, Fundacion Cardioinfantil, Instituto de Cardiologia, Calle 163 A No. 13B-60, Bogota, Colombia A. Natale et al. (eds.), Cardiac Electrophysiology, DOI: 10.1007/978-1-84996-390-9_30, © Springer-Verlag London Limited 2011
125
126
L.C. Sáenz and M.A. Vacca
Fig. 30.2 (a and b) Spontaneous changes in the electrical rhythm
right atrial and superior vena cava activation map was constructed during the tachycardia and shown in Fig. 30.3. The CARTO map suggested a focal origin of the tachycardia and showed the site of earliest electrical activation (in red) of the mapped chambers over the posterior aspect of the superior vena cava where fractionated and almost double electrograms were found and marked as blue points, Fig. 30.3.
Remarkably, the electrograms registered from the coronary sinus are almost isochronic with the electrograms taken from the earliest CARTO zone in the posterior aspect of the superior vena cava, Fig. 30.3. In order to simplify the case, the CARTO activation and the sequence of the propagation maps (from left to right) of the right atrium, superior vena cava, and coronary sinus is shown in Fig. 30.4.
Case 30
127
Fig. 30.3 Right atrium and superior vena cava CARTO activation map and intra-cardiac electrograms registered during tachycardia
shows a really broad zone of early activation (in red) suggesting a passive activation from another site.2 Moreover, the isochronal activation between the distal coronary sinus electrode and the earliest electrogram over the superior vena cava suggests that the cardiac site responsible for originating the tachycardia is activating these structures almost at the same time. In normal conditions this place would be equidistant between the distal coronary sinus and the superior vena cava. In this way, the origin of the tachyCase Discussion cardia would be in the left atrium.3 In fact, the EKG of the tachycardia (Fig. 30.1) shows a positive p wave in the preThe accuracy and reliability of a CARTO map are dependent cordial and inferior leads with negative p wave in aVL and on having an adequate number of contact points taken in iso-biphasic in DI suggesting that the origin of the tachycarorder to construct the chamber(s) of interest. The CARTO dia is from the left pulmonary veins.4 system constructs a map solely with information taken from A CARTO left atrial activation map integrating the elecsites where the tip of the roving catheter contacts cardiac tis- trical information of the right atrium, superior vena cava, and sue. The system can construct a map that provides informa- coronary sinus was performed and showed in Fig. 30.5. This tion about the site of earliest activation. When using the map showed the actual site of earliest activity to be over the CARTO system it is important to correlate the 3D color maps Carina between the left pulmonary veins as shown by the with information provided from the electrograms taken from tachycardia EKG. different cardiac places and verify and appropriate informaThe corresponding propagation CARTO map is showed tion provided by the system. in Fig. 30.6. The sequence shows a focal origin of the tachyIn Fig. 30.3, the CARTO activation map shows the ear- cardia between the left pulmonary veins (photo 1 of the liest activation zone over the posterior aspect of the supe- sequence) with spreading of the activation to the posterior rior vena cava. However, the intra-cardiac electrograms in wall and the roof of the left atrium (photo 2 and 3 of the Fig. 30.3 showed an almost isochronal activation between sequence). The activation reaches the anterior wall of the LA the distal coronary sinus electrode and the earliest electro- and the posterior aspect of the superior vena cava and almost gram over the superior vena cava. Moreover, the coronary at the same time the distal coronary sinus is depolarized from sinus electrodes show a discrete distal to proximal activa- the posterior wall of the LA (photo 4 of the sequence). tion. The bipolar electrograms registered from the posteFinally, the RA is passively activated through an isorior aspect of the superior vena cava are double potentials chronic front of waves from the posterior aspect of the supeof high amplitude and just 13 ms before the onset of the p rior vena cava and the distal coronary sinus (photo 4 of the wave. The corresponding unipolar electrograms from this sequence). place showed an r/S pattern. The analysis of the intra- The passive activation of one atrial chamber with the eleccardiac bipolar and unipolar electrograms suggests that trical activity originating in the other atrium is a phenomenon despite the CARTO showing that the earliest activation is that can cause confusion during mapping. Unnecessary radi over the posterior aspect of the superior vena cava, it would ofrequency applications maybe made targeting the passive be a passive electrical activation from another cardiac site.1 activation sites without having an effect on the tachycardia. Furthermore, the CARTO map of the superior vena cava Inter-atrial connections have been described and are By analyzing the EKG in Fig. 30.1, what is the possible origin of the tachycardia? Considering both the electrograms and CARTO maps, why does the posterior aspect of the superior vena cava and the distal coronary sinus have almost the same time of activation? What is the best next step to do in this case?
128
L.C. Sáenz and M.A. Vacca
Fig. 30.4 CARTO activation and propagation maps of the right atrium, superior vena cava, and coronary sinus shown in a posterior view
Posterior view
Fig. 30.5 The CARTO activation map integrating the electrical information of the right atrium, superior vena cava, coronary sinus, and the left atrium
Anterior view
Case 30
129
Fig. 30.6 CARTO propagation map integrating the electrical information of the right atrium, superior vena cava, coronary sinus, and the left atrium shown as a sequence of pictures from the superior to inferior views
130
L.C. Sáenz and M.A. Vacca
responsible for this phenomenon.5 The most recognized interatrial connection between the atrial chambers is through the Bachmann Bundle, which connects the left atrial appendage and the superior portion of the Crista Terminalis in the anterior aspect of the superior vena cava, Fig. 30.7. Another connection is between the posterior wall of the LA and the coronary sinus and between the coronary sinus and the RA, Fig. 30.7. There are other less recognized inter-atrial connec-
tions as through the fossa ovalis and between the right pulmonary veins and the superior vena cava, Fig. 30.8. Remarkably, in this particular case the earliest activation of the RA was founded over the posterior aspect of the superior vena cava. So, it seems to correspond to a passive activation through a connection between the right superior pulmonary vein and this part of the superior vena cava as showed in Fig. 30.8.
Anterior-superior view
LAO view
Bachmann CT
Fig. 30.7 Anatomical and CARTO pictures showing the inter-atrial connections
LAO view
Fig. 30.8 Drawing and CARTO pictures showing the inter-atrial connections
Posterior view
Case 30
The atrial tachycardia was slowed by radiofrequency applications over the carina between the left pulmonary veins as showed in Fig. 30.9. This site revealed atrial bipolar electrogram of low amplitude, fractionated, and earlier 52 ms than the onset of the p wave. The corresponding unipolar electrogram showed a QS pattern. See Fig. 30.3 for comparison. The EKG post ablation showed a p wave with a positive/ negative morphology in V1, positive in lead I and aVL, and was predominantly negative in the inferior leads suggesting an escape low CT rhythm, Fig. 30.10a. Retrospectively, the
131
organized and 1:1 atrioventricular conduction rhythm (showed in Fig. 30.2b and magnified in Fig. 30.10a) would be the same ablated atrial tachycardia with fixed exit block of the conduction from the pulmonary vein to the LA that can be confused with a normal sinus rhythm. This case points out that although electroanatomic mapping is a useful tool, the information obtained by the system needs to be validated with electrograms and entrainment. The inter-atrial connections can muddy the interpretation of the activation map. Because of that registration of electrical activity from only one chamber can be misleading.
Fig. 30.9 CARTO and ICE showing the location of successful RF ablation
132
a
L.C. Sáenz and M.A. Vacca Pre ablation
Post ablation I II III AVR AVL AVF V1 V2 V3 V4 V5 V6
Fig. 30.10 (a and b) Comparison between the pre- and post-ablation p waves morphology
References 1. Stevenson WG, Soejima K. Recording techniques for clinical electrophysiology. J Cardiovasc Electrophysiol. 2005;16:1017-1022. 2. Markowitz SM, Lerman BB. How to interpret electroanatomic maps. Heart Rhythm. 2006;3:240-246. 3. Lemery R, Soucie L, Martin B, et al. Human study of biatrial electrical coupling. Circulation. 2004;110:2083-2089.
4. Kistler PM, Roberts-Thomson KC, Haqqani HM, et al. P-wave morphology in focal atrial tachycardia. J Am Coll Cardiol. 2006;48:1010-1017. 5. Ho SY, Sanchez-Quintana D, Cabrera JA, et al. Anatomy of the left atrium: implications for radiofrequency ablation of atrial fibrillation. J Cardiovasc Electrophysiol. 1999;10:1525-1533.
Case 31 Bradley P. Knight
Case Summary A young woman with a permanent form of reciprocating tachycardia (PJRT) underwent an electrophysiology
procedure. The 12-lead ECG is shown in Fig. 31.1. The results of pacing maneuvers were consistent with orthodromic AV reentry using a slowly conducting accessory pathway (AP). Where is the AP most likely located?
Fig. 31.1 A 12-lead rhythm strip showing a nearly incessant, supraventricular tachycardia with a long RP interval
B.P. Knight Division of Cardiology, Northwestern Medical Center, 676 N. St. Clair, Suite 600, Chicago, IL 60611 A. Natale et al. (eds.), Cardiac Electrophysiology, DOI: 10.1007/978-1-84996-390-9_31, © Springer-Verlag London Limited 2011
133
134
B.P. Knight
Case Discussion The most likely location for an AP with decremental conduction that participates in PJRT is the posterior septum. Therefore, efforts to map the atrial insertion during tachycardia should initially focus in this region. However, it is not uncommon for these pathways to be located elsewhere. At least two studies have found that the site of earliest retrograde atrial activation occurs at annular locations other than the posteroseptal region in approximately one fourth of cases. In this case, the earliest retrograde activation during single echo beats was along the lateral mitral annulus across from the distal coronary sinus electrode (Fig. 31.2). Radiofrequency ablation at this site eliminated AP conduction. Less than 5% of cases are found in the left lateral location.
Fig. 31.2 This tracing was recorded during catheter ablation of a supraventricular tachycardia. Shown are surface electrograms from leads I, II, III, V1, and V5, and the intracardiac electrograms from the high right atrium (HRA), the ablation catheter (Abl) positioned at the lateral mitral annulus where the earliest local activation could be recorded, and the coronary sinus (CS). The vertical line shows that the earliest local atrial activation precedes the onset of the p-wave on the surface tracing
Bibliography Gaita F, Haissaguerre M, Giustetto C, et al. Catheter ablation of PJRT with RF current. J Am Coll Cardiol. 1995;25:648-654. Meiltz A, Weber R, Halimi F, et al. PJRT in adults: Peculiar Features and Results of RF Ablation. Europace. 2006;8:21-28.
Case 32 Luigi Di Biase, Rodney P. Horton, and Andrea Natale
Case Summary A 66-year-old male with essential hypertension and a history of bilateral hernia surgery, was self-referred to our institution regarding persistent shortness of breath status post-“redo” pulmonary vein isolation performed at another institution. At admission he denied any chest discomfort, palpitations, presyncope, or syncope. The patient began experiencing symptomatic episodes of atrial fibrillation in 2003. His echocardiogram and blood screening resulted within the normal range. The arrhythmia was initially treated both with propafenone and flecainade at the appropriate dosages. These AADs were ineffective. Since his AF became more persistent, the patient underwent several cardioversions. In 2006 he finally underwent pulmonary vein isolation (PVI). The procedure had no complications and a CT scan performed 3 months after the ablation showed no pulmonary vein stenosis. Five months following the ablation, the patient went into atrial flutter which required cardioversion. In 2008 the patient underwent a repeat procedure. One month following the second ablation procedure, he started experiencing progressively worsening dyspnea on exertion with daily activities such as walking across the street. He also reported orthopnea. At admission, his blood screening and physical examination resulted normal.
L. Di Biase (*) Department of Electrophysiology, St. David’s Medical Center, 1015 32nd Street, Suite 506, Austin, TX 78705 email:
[email protected] R.P. Horton Department of Electrophysiology, Texas Cardiac Arrhythmia Institute, 1015 East 32nd Street, Austin, TX 78705 A. Natale Texas Cardiac Arrhythmia Institute, St. David’s Medical Center, 1015 East 32nd Street, Suite 408, Austin, TX 78705
Which should be the diagnostic and therapeutic steps for this patient? a) Perform an Echocardiogram b) Perform a Transesophageal Echocardiogram c) Perform a CT scan e/o MRI scan d) Perform a CT scan and a Transesophageal Echocardiogram e) Perform a ventilation/perfusion scan f) Perform a chest x ray g) Perform all the above
Case Discussion The patient had a CT scan and an echocardiogram 9 months after the redo procedure. The echocardiogram showed normal EF and an enlarged pulmonary artery with increased pulmonic valve velocities consistent with pulmonary hypertension. Additionally there was mild right ventricular hypertrophy, and mild tricuspid regurgitation. The TEE was technically difficult and with suboptimal echocardiographic images. No thrombus was appreciable in the left atrium and the left atrium appendage. Solely the left superior pulmonary vein was visualized. EF was estimated at 60%. Generally all the structures were poorly visualized. The Doppler echocardiogram showed increased velocities in the distal portion of the left superior pulmonary vein, consistent with a significant stenosis. The CT scan showed complete occlusion of the LIPV and severe stenosis of both left and right superior pulmonary veins. An enlarged pulmonary artery was also noted (Fig. 32.1 and 32.2). Based on the CT reports which should be the treatment stategy? a) Anticoagulation and clinical follow up b) Dilation with baloon and anticoagulation c) Dilation with balloon, anticoagulation and follow up with VQ/scan d) None of the above e) All the above
A. Natale et al. (eds.), Cardiac Electrophysiology, DOI: 10.1007/978-1-84996-390-9_32, © Springer-Verlag London Limited 2011
135
136
L. Di Biase et al.
There were no significant gradient across these stenosis following the dilation. Based on the good angiographic results of the balloon angiography the patient should: a) Stop anticoagulation therapy b) Continue anticoagulation therapy and repeat the CT scan at follow up c) Continue anticoagulation therapy and repeat TEE at follow up d) All the above e) None of the above
Fig. 32.1 CT scan showing complete occlusion of the LIPV and severe stenosis of both left and right superior pulmonary veins
Fig. 32.2 CT scan showing an enlarged pulmonary artery with severe stenosis of the right and left superior pulmonary veins
Because of the significant dyspnea and pulmonary hypertension related to pulmonary vein stenosis of at least three pulmonary veins, the decision was made to undergo balloon angioplasty and possible stenting. The left superior, left inferior, and right superior pulmonary veins were found severely occluded (³90%) at the angiograms and were dilated with balloon angioplasty.
The reported incidence of PV stenosis/occlusion defined as >70% narrowing or ³90% affects 3.4% of patients following catheter ablation of atrial fibrillation.1–3 The incidence has decreased in the most recent years due the utilization of different techniques for the PV isolation that limit the burnings at the antrum of the pulmonary veins which is at a considerable distance from the true PV ostia.1 The clinical presentation of these patients is variable; from totally asymptomatic or with mild dyspnea to severe dyspnea with hemoptysis, fever, or pleuritic chest pain.1,3 The pathogenesis is due to an initial ablative insult that precipitates a healing reaction culminating in an endovascular contraction and proliferation of the elastic lamina/intima. Misdiagnosis is very common in these patients (pulmonary embolism, lung cancer, pneumonia, and new onset of asthma, are the most common1,3), because symptoms may occur far from the procedural time. This is why imaging following catheter ablation is crucial also in asymptomatic patient. In fact, PV stenosis may progress insidiously. Patients can present with a variety of respiratory symptoms, but may also remain asymptomatic especially when only one vein demonstrates severe stenosis. Chest x-ray is usually not helpful in diagnosing this condition. As shown in this case, TEE does not always provide clear images of the pulmonary veins. CT scan and/or magnetic resonance imaging are the best tools for diagnosis.4,5 Many groups suggest that the assessment of PV diameter using CT scan or MRI 3 months after ablation provides the best identification of PV stenosis since the caliber remain relatively stable beyond 3 months after ablation. However, late progression from a mild to a severe PV stenosis has been described and a repeat imaging is required for any patient who develops new symptoms consistent with stenosis.1 In patients with moderate to severe stenosis, ventilation/perfusion (V/Q) scan may be useful because it provides a good measure of the lung functionality. The CSI index (cumulative stenosis index [CSI] = sum of the percent stenosis of the unilateral veins divided by the total number of ipsilateral veins) has been proposed with a cutoff value of 75% to identify patients at greatest risk of severe symptoms and lung diseases. In these patients,
Case 32
early and, when required, repeated PV intervention should be considered for restoration of pulmonary flow and prevention of associated lung disease.1 Late opening of the vessel although feasible will not be able to reduce patient’s symptoms and restore lung functionality. In the absence of symptoms, there is no consensus on the best treatment strategy and the CSI index may be useful to identify patients at higher risk. Balloon angioplasty with or without stenting has been shown to achieve satisfactory results although restenosis requiring repeat intervention is necessary in nearly 45–50% of patients.1,6 As shown in this case, after the second ablation, the patient did not undergo CT scan. As described earlier in the text, imaging is very important to detect stenosis even in asymptomatic patients. The dyspnea reported by the patient was underestimated for several months. This resulted in severe pulmonary hypertension and pulmonary artery dilatation (as demonstrated by the CT scan). In such a case, late dilatation of the occluded pulmonary veins although angiographically successful may not solve the patient’s symptoms.
137
References 1. Di Biase L, Fahmy TS, Wazni OM, et al. Pulmonary vein total occlusion following catheter ablation for atrial fibrillation: clinical implications after long-term follow-up. J Am Coll Cardiol. 2006;48: 2493-2499. 2. Saad EB, Marrouche NF, Saad CP, Natale A et al. Pulmonary vein stenosis after catheter ablation of atrial fibrillation: emergence of a new clinical syndrome. Ann Intern Med. 2003 Apr 15; 138(8):634-638. 3. Saad EB, Rossillo A, Saad CP, et al. Pulmonary vein stenosis after radiofrequency ablation of atrial fibrillation: functional characterization, evolution, and influence of the ablation strategy. Circulation. 2003;108:3102-3107. 4. Packer DL, Keelan P, Munger TM, et al. Clinical presentation, investigation, and management of pulmonary vein stenosis complicating ablation for atrial fibrillation. Circulation. 2005;111: 546-554. 5. Neumann T, Sperzel J, Dill T, et al. Percutaneous pulmonary vein stenting for the treatment of severe stenosis after pulmonary vein isolation. J Cardiovasc Electrophysiol. 2005;16:1180-1188. 6. Qureshi AM, Prieto LR, Latson LA, et al. Transcatheter angioplasty for acquired pulmonary vein stenosis after radiofrequency ablation. Circulation. 2003;108:1336-1342.
Case 33 M. Eyman Mortada, Jasbir S. Sra, and Masood Akhtar
Case Summary A 55-year-old male presented with recurrent symptoms of dizziness and palpitations. He was found to have stable vitals and abnormal rhythm (Fig. 33.1). The 12-lead ECG at baseline during sinus rhythm is shown in Fig. 33.2. The patient underwent a complex electrophysiology study. The intracardiac tracing shown in Fig. 33.3 demonstrates the wide complex tachycardia rhythm.
What is the diagnosis? Radiofrequency ablation was performed successfully. Where was the ablation performed?
Case Discussion In defining a wide complex tachycardia, it is important to differentiate between ventricular tachycardia (VT) and supraventricular tachycardia (SVT) (associated with aberrancy,
Fig. 33.1 At presentation, the patient was found to have stable vitals and abnormal rhythm demonstrated in this 12-lead ECG
M.E. Mortada (*), J.S. Sra, and M. Akhtar Department of Electrophysiology, Aurora Cardiovascular Services, Aurora Sinai/Aurora St. Luke’s Medical Centers, University of Wisconsin School of Medicine and Public Health, 2801 W. Kinnickinnic River Parkway, #777, Milwaukee, WI 53215 e-mail:
[email protected] A. Natale et al. (eds.), Cardiac Electrophysiology, DOI: 10.1007/978-1-84996-390-9_33, © Springer-Verlag London Limited 2011
139
140
preexcitation, preexistent wide QRS complex, or antidromic AV reentry tachycardia). All of these are possible diagnostic options in this case. The seventh complex on the wide QRS complex tachycardia is premature and results in an earlier initiation of the tachycardia after the premature beat. This response fits all of the above mentioned diagnostic possibilities. If this is a VT, the premature beat could be a premature ventricular capture (PVC) that entrained the ventricular tachycardia and accelerated the rhythm for that one beat (Fig. 33.4). If this is atrial flutter with 2:1 AV conduction or reentry atrial tachycardia associated with left bundle branch block, the premature beat could be a premature atrial capture (PAC) that entrained the reentry circuit and accelerated the rhythm for that one beat (though atrial flutter option is the least possible, due to the fast rate of the atrial flutter, 366 bpm [160 ms]) (Fig. 33.5). If this is an orthodromic AV reentry tachycardia with left bundle branch block, the premature beat could be a PAC or PVC that entrained the circuit and accelerated the rhythm for that one beat (Fig. 33.6). If this is an AV nodal reentry tachycardia with left bundle branch block, the premature beat could be a PAC that entrained the circuit and accelerated the rhythm for that one
Fig. 33.2 The patient’s 12-lead ECG at baseline during sinus rhythm
M.E. Mortada et al.
beat (Fig. 33.7). Intravenous adenosine affects the atrioventricular node; hence it affects almost all SVTs by terminating the tachycardia or producing atrioventricular dissociation. Few ventricular tachycardias are sensitive to adenosine (e.g., normal heart VT), thus response to adenosine is no help in making a diagnosis. The absence of structural heart disease is an important piece of information, increasing the likelihood that the tachycardia is SVT rather than VT in origin. The most helpful piece of information for differential diagnosis in this case was the baseline 12-lead ECG, which showed the same QRS morphology. Therefore, VT could be easily excluded. Acceleration of the tachycardia by one premature beat, as seen in the first ECG (the seventh beat), favors the diagnosis of orthodromic AV reentry tachycardia (AVRT) over AV nodal reentry tachycardia (AVNRT). Intracardiac electrocardiograms taken during the tachycardia revealed the activation of the atria to be from the distal to the proximal coronary sinus, followed by the high right atrium and His (septum). It also revealed constant VA linking. These findings confirmed the diagnosis of an orthodromic AVRT using a left lateral free-wall accessory pathway and the best location for successful radiofrequency ablation is the lateral side of the mitral valve annulus.
Case 33
141
Fig. 33.3 Intracardiac tracing demonstrating the wide complex tachycardia rhythm (Rhythm tracing from top to bottom: Lead I, Lead II, Lead V1, high right atrium recording “HRA,” proximal coronary sinus recording (CS 9,10) to distal coronary sinus recording (CS 1,2), and His bundle recording “proximal HSp, and distal HSd.”) HRAp: proximal high right atrium; CS: coronary sinus; HSp: proximal His; HSd: distal His
Fig. 33.4 The seventh complex on the wide QRS complex tachycardia is premature and results in an earlier initiation of the tachycardia after the premature beat. If this patient has a ventricular tachycardia (VT), the premature beat could be a premature ventricular capture that entrained the VT and accelerated the rhythm for that one beat
Fig. 33.5 If this patient has atrial flutter with 2:1 AV conduction associated with left bundle branch block, the premature beat could be a premature atrial capture that entrained the atrial flutter and accelerated the rhythm for that one beat (though this option is the least possible, due to the fast rate of the atrial flutter--366 bpm “160 ms”)
Fig. 33.6 If this patient has an orthodromic AV reentry tachycardia with left bundle branch block, the premature beat could be a PAC or PVC that entrained the circuit and accelerated the rhythm for that one beat
Fig. 33.7 If this patient has an AV nodal reentry tachycardia with left bundle branch block, the premature beat could be a PAC that entrained the circuit and accelerated the rhythm for that one beat
Case 34 Antonio Rossillo, Sakis Themistoclakis, Aldo Bonso, Andrea Corrado, and Antonio Raviele
Case Summary
Case Discussion
The patient is a 28-year-old female with frequent runs of SVT. No relevant structural changes on echocardiography. Based on Figs. 34.1 and 34.2, what is the likely mechanism of the tachycardia?
This tachycardia is a long RP tachycardia, thus the differential diagnosis includes atypical AVNRT, atrial tachycardia, or AVRT that utilizes a slowly conducting AP. When a single VPC is delivered the tachycardia terminates without reaching
Fig. 34.1 12-lead ECG taken during SVT (paper speed 25 mm/s) showing a negative P wave in the inferior leads with a short PQ interval and a heart rate of 120 bpm
A. Rossillo (*), S. Themistoclakis, A. Bonso, and A. Corrado Cardiovascular Department, Ospedale dell’Angelo, Mestre, Venice, Italy A. Raviele Chief of Cardiology, Chief of Cardiovascular, Cardiovascular Department, Ospedale dell’Angelo, Mestre, Venice, Italy A. Natale et al. (eds.), Cardiac Electrophysiology, DOI: 10.1007/978-1-84996-390-9_34, © Springer-Verlag London Limited 2011
143
144
the atrium. This makes AT unlikely. Distinguishing AVRT that utilizes a slowly conducting AP versus AVNRT can be difficult, but as the tachycardia terminated during a HIS
Fig. 34.2 Intracardiac recordings taken at baseline during the electrophysiology study (paper speed 100 mm/s). Four surface ECG leads are shown (I, aVF, V1, V6), one bipolar recording from the pacing catheter (HRA), three bipolar recordings from the distal His bundle region (HIS D = distal, HIS I = medial, and HIS P = proximal), two bipolar recordings from the coronary sinus (CS1 = distal CS and CS4 prox CS), and the distal bipolar recording of the mapping catheter (MC D Bi). A single paced beat from the right ventricle with a coupling interval allows for anterograde activation of the His bundle which interrupted the tachycardia twice. (A atrium, V ventricle, and H His bundle)
Fig. 34.3 Intracardiac recordings (paper speed 200 mm/s). Same display as shown in Fig. 34.2. Ablation site: earliest activation on MC D Bi
A. Rossillo et al.
refractory VPC, the likely diagnosis is AVRT or PJRT. This tachycardia is often clinically frequent or incessant. In Fig. 34.3 and 34.4 the ablation site is shown.
Case 34
145
Fig. 34.4 RAO and LAO projection of mapping catheter at the ablation site
RAO
Bibliography Coumel PH, Attuel P, Leclerque JF. Permanent form of junctional reciprocating tachycardia: mechanism, clinical and therapeutic impli cation. In: Narula OS, ed. Cardiac Arrhythmias. Electrophysiology, Diagnosis and Management. Baltimore/London: Williams & Wilkins; 1979:347-363. Critelli G, Scherillo M, Monda V, D’Ascia C, Musumeci S, Antignano A. Transvenous catheter ablation of the accessory atrioventricular
LAO
p athway in the permanent form of junctional reciprocating tachycardia. Am J Cardiol. 1985;55:1639-1641. Farré J, Ross D, Wiener I, Bär FW, Vanagt EJ, Wellens HJ. Reciprocal tachycardias using accessory pathways with long conduction times. Am J Cardiol. 1979;44:1099-1109. Smith RT, Gillette PC, Massumi A, McVey P, Garson A Jr. Transcatheter ablative techniques for treatment of the permanent form of junctional reciprocating tachycardia in young patients. J Am Coll Cardiol. 1986;8:385-390.
Case 35 Yash Y. Lokhandwala, Anoop K. Gupta, and Ranjan K. Thakur
Case Summary
Case Discussion
A 47-year-old man presented with recurrent tachycardia (Fig. 35.1) which is responsive to verapamil. The patient has a normal heart. His ECG is normal in sinus rhythm. In the EP lab he is diagnosed with AVRT. Explain what occurs during RF delivery in Fig. 35.2.
Local AP conduction in the RF channel is eliminated evidenced by prolongation of the local VA interval. Yet, an eccentric atrial activation suggesting a left-sided AP persists in the CS recordings. RF delivery at a different location resulted in VA block (Fig. 35.3). There were two left-sided APs: the first was anterolateral and the second was posterolateral; they were 3–4 cm away from each other.
Y.Y. Lokhandwala (*) KEM Hospital, Parel, Mumbai, India e-mail:
[email protected] A.K. Gupta Apollo Hospital, Ahmedabad, India e-mail:
[email protected] R.K. Thakur Thoracic and Cardiovascular Institute, Sparrow Health System, Michigan State University, 405 West Greenlawn, Suite 400, Lansing, MI 48910 A. Natale et al. (eds.), Cardiac Electrophysiology, DOI: 10.1007/978-1-84996-390-9_35, © Springer-Verlag London Limited 2011
147
148
Y.Y. Lokhandwala et al. Your Hospital Name in File HEADING.TXT
Version WIN200 : EPTRACER V0.48
2008017 I II III AVR AVL AVF V1 V2 V3 V4 V5 V6 10 mm/mV 25mm/s
Fig. 35.1 The ECG suggests that this is likely AVRT, with a left-sided AP (long RP, ST elevation in aVR and ST depression in lead I)
Case 35
149 Your Hospital Name in File HEADING.TXT
I
2008017
AVF V1 V6 CS910 CS78 CS56 CS34 CS12 HISD HISP RFD RFP 10 mm/mV 100mm/s
Fig. 35.2 Intracardiac EGMs during RF energy application. What happens?
Version WIN200 : EPTRACER V0.48
150
Y.Y. Lokhandwala et al. Your Hospital Name in File HEADING.TXT
Version WIN200 : EPTRACER V0.48
2008017 I AVF V1 V6 CS910 CS78 CS56 CS34 CS12 HISD HISP RFD RFP 10 mm/mV 100mm/s Offline printed on 17-05-2008 at 14:23:17.
Fig. 35.3 VA block is achieved
Case 36 Eric Buch, Shiro Nakahara, Marmar Vaseghi, Noel G. Boyle, and Kalyanam Shivkumar
Case Summary A 60-year-old female with a history of left atrial myxoma, resected 9 years ago, subsequently developed palpitations, fatigue, and tachycardia. Despite antiarrhythmic medications, including sotalol and amiodarone, and multiple
cardioversions, she continued to experience tachycardia and was referred for electrophysiology study and catheter ablation. Surface electrocardiogram showed wide-complex tachycardia with a ventricular rate of 115 beats per minute (Fig. 36.1). What is the differential diagnosis of this arrhythmia based on the electrocardiogram?
Fig. 36.1 Surface electrocardiogram shows a regular wide-complex tachycardia at approximately 115 beats per minute
E. Buch (*), S. Nakahara, M. Vaseghi, N.G. Boyle, and K. Shivkumar UCLA Cardiac Arrhythmia Center, David Geffen School of Medicine at UCLA, 10833 Le Conte Ave, A2-237 CHS, Los Angeles, CA 90095-1679 e-mail:
[email protected] A. Natale et al. (eds.), Cardiac Electrophysiology, DOI: 10.1007/978-1-84996-390-9_36, © Springer-Verlag London Limited 2011
151
152
Case Discussion Diagnostic electrophysiology catheters were placed in the high right atrium, at the His bundle, the right ventricular apical septum, and in the coronary sinus. Intracardiac electrograms were recorded during tachycardia (Fig. 36.2).
Fig. 36.2 Intracardiac recordings during tachycardia. Left atrial activation as seen in coronary sinus catheter proceeds from medial to lateral, consistent with a focus or reentrant circuit in the right atrium
E. Buch et al.
Electroanatomic mapping was used to show the right atrial activation sequence. The tachycardia circuit was around the lateral right atriotomy scar from previous cardiac surgery. Ablation was performed at the superior margin of the scar and the tachycardia terminated (Fig. 36.3). It could not be reinduced on multiple attempts.
Fig. 36.3 Left anterior oblique fluoroscopic view of intracardiac electrophysiology catheters. The tip of the ablation catheter is in the lateral right atrium at the superior margin of the atriotomy scar, at the site of tachycardia termination
Case 37 Bradley P. Knight
Case Summary A 25-year-old woman with Wolff–Parkinson–White syndrome was referred for catheter ablation. The 12-lead EKG is consistent with an anteroseptal accessory pathway (AP) (Fig. 37.1). A previous attempt at ablation of the AP was aborted due to its proximity to the His bundle. Another attempt at ablation was made. The baseline rhythm was sinus with ventricular preexcitation. Orthodromic AVRT was inducible and the AP had a short refractory period that was less than 250 ms. The earliest ventricular activation during sinus rhythm was at a site where there was also a large His bundle recording during block in the AP. How would you proceed?
Case Discussion In this case, the patient has symptomatic WPW with a high risk AP. There is a significant risk of causing irreversible AV block with RF current. Cryoenergy ablation is an
alternative to radiofrequency ablation. One advantage of cryoenergy over radiofrequency current is the ability to “cryo-map” to minimize the likelihood of inadvertent AV block.1 As the ablation electrode temperature is lowered below freezing, the tissue reaches a temperature where temporary conduction block occurs well before it reaches a temperature where permanent tissue destruction occurs. This permits the ability to monitor during ablation for undesirable results, such as AV block, before a permanent lesion is created. Fig. 37.2 shows early activation at the para-Hisian region with an activation time of -30 ms. Fig. 37.3 shows disappearance of ventricular preexcitation when the temperature is lowered during cryoablation. As an iceball forms on the tip of the electrode, the electrogram is lost. After rewarming of the electrode occurs, a large His bundle recording can be seen (Fig. 37.4). After elimination of the AP, AV conduction was intact and there was no VA conduction. The AP did not recover during long-term follow-up. Cryoablation should be considered when attempting to ablate an AP that is located in the para-Hisian or midseptal region, to minimize the risk of AV block.
B.P. Knight Division of Cardiology, Northwestern Medical Center, 676 N. St. Clair, Suite 600, Chicago, IL 60611 e-mail:
[email protected] A. Natale et al. (eds.), Cardiac Electrophysiology, DOI: 10.1007/978-1-84996-390-9_37, © Springer-Verlag London Limited 2011
153
154
B.P. Knight
Fig. 37.1 This 12-lead EKG shows ventricular preexcitation consistent with an anteroseptal accessory pathway
Successful cryo site
Fig. 37.2 This tracing was recorded during catheter ablation of an anteroseptal accessory pathway. Shown are surface electrograms from leads I, II, III, V1, and V5, and the intracardiac electrograms from the high right atrium (HRA), the radiofrequency ablation catheter (Abl) positioned near the His bundle region, the cryo catheter (CRYO) with the tip positioned at the earliest local ventricular activation, and the right ventricular apex (RVA). The local ventricular activation recorded from the cryo catheter is 30 msec prior to the onset of the delta wave
−30 ms
University of Chivago
Case 37
155
Fig. 37.3 This figure shows disappearance of preexcitation during cryo-ablation. The format is the same as figure 37.2
During cryoablation
University of Chicago
Appearance of his bundle during rewarming after cryo
Fig. 37.4 This figure shows the reappearance of the His-bundle recording after rewarming of the cryo-ablation electrode. The format is the same as figure 37.2
University of Chicago
Reference 1. Gaita F et al. Safety and efficacy of cryoablation of accessory pathways adjacent to the normal conduction system. J Cardiovasc Electrophysiol. 2003;14:825-829.
Case 38 Luigi Di Biase, Rodney P. Horton, and Andrea Natale
Case Summary A 65-year-old male with a 15-year history of symptomatic persistent atrial fibrillation was referred to our institution. The patient had a normal left ventricular ejection fraction. The patient denied chest discomfort, presyncope, or syncope. Atrial fibrillation was initially chemically cardioverted with Quinidine and Diltiazem. A few years later the drugs were ineffective and the patient underwent three electrical cardioversions. Quinidine and Diltiazem were replaced with Tikosyn. Because Tikosyn was ineffective, the patient decided to undergo pulmonary vein isolation. Five months following the procedure, the patient experienced symptoms related to atrial flutter with rapid ventricular response. For these reasons the patient underwent a second ablation procedure. During the second procedure, the patient did not show any recovery of conduction around the pulmonary veins. The presenting arrhythmia was atrial flutter, which was terminated during ablation in the coronary sinus. Two months after the second procedure the patient developed recurrence of left atrial flutter. The CT scan after the second procedure is shown (Figs. 38.1 and 38.2). What do you propose next for management of this patient’s rhythm abnormality, would a repeat ablation be an option, if so, what is the ablation target? What do you propose next? a) Clinical follow-up with AADs for rate control b) Propose a MAZE or COX procedure
L. Di Biase (*) Department of Electrophysiology, St. David’s Medical Center, 1015 32nd Street, Suite 506, Austin, TX 78705 e-mail:
[email protected] R.P. Horton Department of Electrophysiology, Texas Cardiac Arrhythmia Institute, 1015 East 32nd Street, Austin, TX 78705 e-mail:
[email protected] A. Natale Texas Cardiac Arrhythmia Institute, St. David’s Medical Center, 1015 East 32nd Street, Suite 408, Austin, TX 78705
c) Repeat catheter ablation targeting non PV site d) AV node ablation and pacing e) None of the above
Case Discussion The patient was taken to the electrophysiology laboratory for a repeat ablation. During the third procedure, before ablation in the left atrium, ablation in the persistent left superior vena cava (SVC) was performed. Once isolation was achieved, the ablation catheter was moved to the left atrium. The atrial flutter was prolonged and interrupted during ablation along the mitral annulus. During and after the infusion of high-dose isoproterenol (20 mcg/min), there were no evidence of additional firing sites. After 9 months of follow-up, the patient was asymptomatic and free from arrhythmia recurrences. This case shows that in a number of patients electrical disconnection of the pulmonary veins is not sufficient to prevent arrhythmic recurrences. This is possible in patients with left atrial scar preceding the ablation procedure, non-paroxysmal AF patients, females,1 and in patients with venous anomaly including persistent left SVC.2 In these patients, isolation of the left SVC is necessary together with the isolation of the pulmonary veins to prevent recurrences. Whether isolation of the left SVC without isolation of the PVs would have been sufficient to treat these patients is not known. Persistence of the left SVC is a congenital anomaly resulting from an abnormal development of the coronary sinus. In these patients, the left cardinal vein does not obliterate during the fetal life, and on the contrary, persists draining into either the left atrium or the right atrium through an enlarged CS. The described prevalence of left SVC persistence varies between 0.3% and 2% in individuals with a normal heart.3,4 A persistence of the left SVC can be diagnosed with echocardiography based an unusual enlarged CS, but the gold standards are CT scan and MRI.
A. Natale et al. (eds.), Cardiac Electrophysiology, DOI: 10.1007/978-1-84996-390-9_38, © Springer-Verlag London Limited 2011
157
158 Fig. 38.1 CT scan showing the left atrium, the pulmonary veins, and the persistent LSVC
L. Di Biase et al.
In the above-mentioned case, spontaneous ectopies originating at different levels in the left SVC were present. It is always important to look for unusual sources of AF/Aflutter especially in patients presenting for repeat procedures. Unusual sources of AF should be considered after the failure of one or more procedures for AF ablation. Isolation of the PVs is not sufficient to prevent recurrences when a left SVC is present. Thus, diagnosis and isolation of the left SVC appears critical to avoid AF recurrence in all patients with AF and with this venous anomaly.
References
Fig. 38.2 CT scan showing a persistent LSVC draining into the coronary sinus
The left SVC when present is always a source of ectopies that can initiate AF through the electrical connections with the LA and CS. Hsu et al.5 suggested that as a consequence of the abnormal embryologic development, the persistent of the left SVC may be associated with the presence of electrical tissue, responsible for arrhythmias.
1. Verma A, Wazni OM, Marrouche NF, et al. Pre-existent left atrial scarring in patients undergoing pulmonary vein antrum isolation: an independent predictor of procedural failure. J Am Coll Cardiol. 2005;45:285-292. 2. Elayi CS, Fahmy TS, Wazni OM, Patel D, Saliba W, Natale A. Left superior vena cava isolation in patients undergoing pulmonary vein antrum isolation: impact on atrial fibrillation recurrence. Heart Rhythm. 2006;3:1019-1023. 3. Gonzalez-Juanatey C, Testa A, Vidan J, et al. Persistent left superior vena cava draining into the coronary sinus: report of 10 cases and literature review. Clin Cardiol. 2004;27:515-518. 4. Fraser RS, Dvorkin J, Rossall RE, Eidem R. Left superior vena cava: a review of associated congenital heart lesions, catheterization data and roentgenologic findings. Am J Med. 1961;31:7 11-716. 5. Hsu LF, Jais P, Keane D, et al. Atrial fibrillation originating from persistent left superior vena cava. Circulation. 2004;109: 828-832.
Case 39 M. Eyman Mortada, Jasbir S. Sra, and Masood Akhtar
Case Summary A 38-year-old female complained of recurrent episodes of lightheadedness and palpitations over several months. She came to the emergency room with the same symptoms, at which time her 12-lead ECG was as follows (Figs. 39.1–39.4). Two ventricular extrastimuli (S1) at cycle lengths of 400 ms were administered during the tachycardia (Fig. 39.5). What is the best initial management, and what is the most likely diagnosis?
Case Discussion The P-wave occurring between the two R-waves, seen in the first 12-lead ECG of this young female with documented normal PR interval during sinus rhythm in the past, suggests a diagnosis of supraventricular tachycardia, specifically AV nodal reentry tachycardia (AVNRT) with 2:1 AV conduction. Since we were unable to exclude other possibilities, such as atrial tachycardia a complex electrophysiology study was required. When the patient’s rhythm spontaneously changed from a 460- to a 230-ms R-R cycle length (Figs 39.1 and 39.3), it revealed the presence of 2:1 AV conduction, which then
M.E. Mortada (*), J.S. Sra, and M. Akhtar Department of Electrophysiology, Aurora Cardiovascular Services, Aurora Sinai/Aurora St. Luke’s Medical Centers, University of Wisconsin School of Medicine and Public Health, 2801 W. Kinnickinnic River Parkway, #777, Milwaukee, WI 53215 e-mail:
[email protected]
changed to 1:1 AV conduction. This excludes the diagnosis of accessory pathway-mediated tachycardia, as it would not occur in the presence of 2:1 AV block. The patient was symptomatic with her tachycardia, so the best next step in management is to convert her to normal rhythm and then perform further work-up to diagnose and treat her condition. She was hemodynamically was stable, therefore DC cardioversion is not indicated, but adenosine IV bolus is the best first-line approach. b-blockers or calcium channel blockers are acceptable therapies, yet, complex electrophysiology study post conversion is more appropriate to confirm the diagnosis and cure the patient’s arrhythmia by ablation therapy. The first intracardiac tracing (Fig. 39.4) showed narrow complex tachycardia with two atrial activities recorded for each incident of ventricular activity. When there ventricular activity, the VA relationship was fixed. Notice that the His was present before each atrial activity, which suggests the location of the block is in the His–Purkinje system in a 2:1 pattern due to refractoriness. The first ventricular extrastimuli penetrated the Purkinje system but not the His. Therefore, it did not engage the circuit and did not change the cycle length of the tachycardia. The second ventricular extrastimuli was given earlier in the tachycardia cycle length and it penetrated also to a level below the His. Hence, the cycle length of activation at this vulnerable location of block has shortened. The His-Purkinje system’s refractoriness is directly related to the duration of proceeding cycle length1. Therefore, the site of block’s refractory period is also shortened. That leads to the penetration of the next atrial activity into the His-Purkinje system and setting a 1:1 AV conduction. Notice the fixed VA relationship before and after the ventricular extrastimulation. This linking suggests that the tachycardia diagnosis is most likely AV nodal reentry tachycardia.
A. Natale et al. (eds.), Cardiac Electrophysiology, DOI: 10.1007/978-1-84996-390-9_39, © Springer-Verlag London Limited 2011
159
160
M.E. Mortada et al.
Fig. 39.1 The patient’s 12-lead ECG upon admission to emergency room with symptoms of lightheadedness and palpitations
Fig. 39.2 The patient’s previous baseline 12-lead ECG
Case 39
161
Fig. 39.3 The patient spontaneously switched into the rhythm shown in this 12-lead ECG, accompanied by worsening symptoms, but she remained hemodynamically stable
Fig. 39.4 Intracardiac tracing of the same tachycardia the patient presented first to the ER. (Rhythm tracing from top to bottom: lead I, Lead II, Lead V1, high right atrium recording “HRA”, His bundle recording
“HIS”, and right ventricular apex recording “RVa”. Labels used in this figure are similar to the labels used in subsequent figures)
162
M.E. Mortada et al.
Fig. 39.5 Two ventricular extrastimuli (S1) at cycle lengths of 400 ms were administered during the tachycardia. The tachycardia accelerated from 2:1 AV conduction to 1:1 AV conduction
Reference 1. Akhtar M, Mahmud R, Tchou P, Denker S, Gilbert CJ. Normal electrophysiologic responses of the human heart. Cardiol Clin 1986;4: 365–386.
Case 40 Antonio Rossillo, Sakis Themistoclakis, Aldo Bonso, Andrea Corrado, and Antonio Raviele
Case Summary
Case Discussion
The patient is a 40-year-old female with a prior history of palpitations and is being medically managed for mild hypertension. The episodes became more frequent 2 months prior to the time of ablation. The ECG recordings were taken while the patient was having palpitations. The ECG revealed both orthodromic and antidromic tachycardia. Where is the likely position of the AP?
This bypass tract is likely located in the posterior septal space given the morphology of the maximally preexcited tracing (Figs. 40.1, 40.2) and it was successfully as shown in figs. 40.3–40.14.
A. Rossillo, S. Themistoclakis, A. Bonso, A. Corrado, and A. Raviele (*) Cardiovascular Department, Ospedale dell’Angelo, Mestre, Venice, Italy e-mail:
[email protected] A. Natale et al. (eds.), Cardiac Electrophysiology, DOI: 10.1007/978-1-84996-390-9_40, © Springer-Verlag London Limited 2011
163
164
A. Rossillo et al.
Fig. 40.1 The 12-lead resting ECG (paper speed 25 mm/s) showing normal sinus rhythm with a ventricular rate of 109 bpm with a short PR interval (118 ms) and a QRS width of 81 ms with mild delta wave in the initial portion of the QRS
Case 40
165
Fig. 40.2 12-lead ECG taken during atrial pacing (paper speed 25 mm/s) showing maximum ventricular preexitation with an early transition of the R wave between leads V1 and V2 and negative wave in the inferior leads
166
Fig. 40.3 Intracardiac recordings taken at baseline during the electrophysiology study (paper speed 200 mm/s). Four surface ECG leads are shown (I, aVF, V1, V6): (1) bipolar recording from the high right atrium (HRA), (2) bipolar recording from the distal His bundle region (HIS D)
A. Rossillo et al.
(3) five unipolar recordings from the coronary sinus (CS), (4) the unipolar (MC U-CATH) and the distal bipolar recording of the mapping catheter (MC D). (A atrium, V ventricle and H His bundle)
Case 40
167
Fig. 40.4 Intracardiac recordings taken during baseline while ventricular pacing (paper speed 200 mm/s). Same display as shown in Fig. 40.3 (A atrium, V ventricle)
168
A. Rossillo et al.
A Fig. 40.5 Intracardiac recordings taken during programmed atrial stimulation (paper speed 100 mm/s). Same display as shown in Fig. 40.3. Panel A: At a coupling interval of 290 ms the ventricular
preexitation was still present and in Panel B: With a coupling interval of 280 ms suddenly disappeared (ERP of bypass fiber). (A atrium, V ventricle and H His bundle)
Case 40
B Fig. 40.5 (continued)
169
170
A. Rossillo et al.
A Fig. 40.6 Intracardiac recordings taken during programmed ventricular stimulation (paper speed 200 mm/s). Same display as shown in Fig. 40.3. Panel A: At a coupling interval of 280 ms, the Stim-A interval
was 130 ms and suddenly increased and in Panel B to 180 ms with a coupling interval of 270 ms (retrograde ERP of a bypass fiber). (A atrium, V ventricle)
Case 40
B Fig. 40.6 (continued)
171
172
A. Rossillo et al.
Fig. 40.7 Intracardiac recordings taken during programmed atrial stimulation (paper speed 200 mm/s). Same display as shown in Fig. 40.3. AVRT induction (AA 280 ms) with a single extrastimulus (drive 500 ms and S1–S2 230 ms). (A atrium, V ventricle)
Case 40
Fig. 40.8 Intracardiac recordings (paper speed 200 mm/s). Same display as shown in Fig. 40.3. Ablation site. The bipolar recording of the ablation catheter shows presence of an atrial signal and of the bypass
173
fiber without ventricle activity that is present only in the unipolar recording. (A atrium, K Kent bundle, and V ventricle)
174
Fig. 40.9 Intracardiac recordings (paper speed 100 mm/s). Same display as shown in Fig. 40.3. Ablation site. The bipolar recording of the ablation catheter shows presence of an atrial signal and of the bypass
A. Rossillo et al.
fiber without ventricle activity that is present only in the unipolar recording (A atrium, K Kent bundle, and V ventricle)
Case 40
Fig. 40.10 Intracardiac recording taken during radiofrequency delivery (paper speed 200 mm/s). Same display as shown in Fig. 40.3. The second beat disappeared during ventricular preexitation and the intrac-
175
ardiac recording showed a clear separation between atrial (A) and ventricular signals (V). H His signal
176
Fig. 40.11 Intracardiac recordings taken during radiofrequency delivery (paper speed 25 mm/s). Same display as in Fig. 40.3. The arrow shows the elimination of the ventricular preexcitation. The circle shows
A. Rossillo et al.
the ventricle signal on the ablation catheter before and after the elimination of the bypass fibers
Case 40
177
A Fig. 40.12 Intracardiac recordings taken during programmed ventricular stimulation (paper speed 200 mm/s). Same display as shown in Fig. 40.3. Panel A: At a coupling interval of 280 ms, the Stim-A interval
was 160 ms and Panel B: At a coupling interval of 270 a decremental VA conduction was present (A atrium, V ventricle)
178
B Fig. 40.12 (continued)
A. Rossillo et al.
Case 40
179
Fig. 40.13 Intracardiac recording taken at the end of the procedure (paper speed 200 mm/s). The same display as shown in Fig. 40.3 (A atrium, V ventricle, H His bundle)
180
A. Rossillo et al.
Bibliography
A
RAO
Calkins H, Langberg J, Sousa J, et al. Radiofrequency catheter ablation of accessory atrioventricular connections in 250 patients. Abbreviated therapeutic approach to Wolff-Parkinson-White syndrome. Circulation. 1992;85:1337-1346. Haïssaguerre M, Dartigues JF, Warin JF, Le Metayer P, Montserrat P, Salamon R. Electrogram patterns predictive of successful catheter ablation of accessory pathways. Value of unipolar recording mode. Circulation. 1991;84:188-202. Jackman WM, Wang XZ, Friday KJ, et al. Catheter ablation of accessory atrioventricular pathways (Wolff-Parkinson-White syndrome) by radiofrequency current. N Engl J Med. 1991;324:1605-1611. Scheinman MM, Wang YS, Van Hare GF, Lesh MD. Electrocardio graphic and electrophysiologic characteristics of anterior, midseptal and right anterior free wall accessory pathways. J Am Coll Cardiol. 1992;20:1220-1229.
LAO
MV
H RA TT
CS MCV
B
TV
Fig. 40.14 Panel A: RAO and LAO of the mapping catheter at the ablation site in the proximal coronary sinus. Panel B: LAO picture of the mapping catheter at the ablation site in the proximal coronary sinus (RA right atrium, TT Todaro’s tendon, TV tricuspid valve, CS coronary sinus, MCV middle cardiac vein, H His bundle, MV mitral valve)
Case 41 Yash Y. Lokhandwala, Anoop K. Gupta, and Ranjan K. Thakur
Case Summary
Case Discussion
A 36-year-old woman with a normal heart had this ECG in the lab (Fig. 41.1). A narrow QRS tachycardia is seen without visible P waves; sharp J point in V1; this likely represents AVNRT. Multiple APDs change or accelerate tachycardia – how?
Tachycardias, pre- and post-APDs, are AVNRT. The APDs block in the first slow pathway and conduct by another slow pathway (Fig. 41.2). RF energy delivered in slow pathway region eliminated dual AVN physiology and inducibility.
Y.Y. Lokhandwala (*) KEM Hospital, Parel, Mumbai, India e-mail:
[email protected] A.K. Gupta Apollo Hospital, Ahmedabad, India e-mail:
[email protected] R.K. Thakur Thoracic and Cardiovascular Institute, Sparrow Health System, Michigan State University, 405 West Greenlawn, Suite 400, Lansing, MI 48910 e-mail:
[email protected] A. Natale et al. (eds.), Cardiac Electrophysiology, DOI: 10.1007/978-1-84996-390-9_41, © Springer-Verlag London Limited 2011
181
182
Y.Y. Lokhandwala et al. 2008083
I II III AVR AVL AVF V1 V2 V3 V4 V5 V6 10 mm/mV 25 mm/s
Fig. 41.1 SVT with change in cycle length
Case 41 2008083 I AVF V1 V6
CS78
CS34
CS12
HISD
HISP
rfD
rfP 10 mm/mV 50 mm/s
Fig. 41.2 SVT with change in antegrade limb resulting in faster cycle length
183
Case 42 Eric Buch, Shiro Nakahara, Marmar Vaseghi, Noel G. Boyle, and Kalyanam Shivkumar
Case Summary A 59-year-old male was referred for radiofrequency catheter ablation of atrial flutter and atrial fibrillation. He had a 3-year history of highly symptomatic paroxysmal atrial fibrillation that failed to respond to multiple medications. He underwent catheter-based radiofrequency ablation (RFA) with pulmonary vein isolation and subsequently a modified left- and right-sided surgical maze procedure after recurrence of atrial fibrillation. Following the maze procedure, patient developed incessant atrial flutter causing palpitations and fatigue. He was referred for catheter ablation. Diagnostic electrophysiology catheters were placed at the His bundle, the right ventricular apical septum, and in the coronary sinus. A multipolar halo catheter was placed in the right atrium with the distal electrodes in the coronary
sinus, middle electrodes along the lateral right atrial wall, and proximal electrodes near the roof of the right atrium. Intracardiac activation map showed atrial flutter with distal to proximal coronary sinus activation and passive counterclockwise activation of the right atrium (Fig. 42.1). After transseptal catheterization, electroanatomic mapping of the left atrium suggested that the tachycardia circuit utilized an isthmus between the mitral annulus and the left inferior pulmonary vein. Entrainment mapping confirmed that this left-sided isthmus was critical to the tachycardia circuit. During ablation in the region between the left inferior pulmonary vein and the mitral annulus (Fig. 42.2b), the intracardiac activation sequence changed abruptly, yet tachycardia persisted (Fig. 42.3). What is responsible for this change in sequence?
E. Buch (*), S. Nakahara, M. Vaseghi, N.G. Boyle, and K. Shivkumar UCLA Cardiac Arrhythmia Center, David Geffen School of Medicine at UCLA, 10833 Le Conte Ave, A2-237 CHS, Los Angeles, CA 90095-1679 e-mail:
[email protected] A. Natale et al. (eds.), Cardiac Electrophysiology, DOI: 10.1007/978-1-84996-390-9_42, © Springer-Verlag London Limited 2011
185
186 Fig. 42.1 Intracardiac recordings during tachycardia. Coronary sinus activation is from distal to proximal, consistent with a left atrial tachycardia circuit. Right atrial activation is earliest at the high septum (HALO 10) due to conduction through Bachman’s bundle, then conducts along the roof and down the lateral right atrial wall (HALO 9 to HALO 5). There is competing activation through the coronary sinus (HALO 1 to HALO 4), resulting in fusion of right atrial activation
E. Buch et al. I aVF V1 V6 ABL d HIS p HIS m HIS d HALO 10 HALO 9 HALO 8 HALO 7 HALO 6 HALO 5 HALO 4 HALO 3 HALO 2 HALO 1 CS 9, 10 CS 7, 8 CS 5, 6 CS 3, 4 CS 1, 2
Case Discussion Successful ablation of the mitral isthmus, which was critical for perpetuation of the left atrial macroreentrant tachycardia, resulted in termination of that rhythm. However, it unmasked an underlying typical counterclockwise right
atrial flutter with a longer tachycardia cycle length. In effect, the typical right atrial flutter had been entrained by the left atrial flutter. The patient then underwent further ablation at the cavotricuspid isthmus, resulting in termination of the atrial flutter, bidirectional isthmus block, and noninducibility of either tachycardia.
Case 42 Fig. 42.2 (a) Three-dimensional voltage map of the left atrium, left posterior oblique view. Large areas of scar are seen (in red), likely from prior catheter and surgical ablation procedures. (b) Posteroanterior view of same patient’s left atrium, showing ablation sites (red points) between left inferior pulmonary vein and mitral annulus. Pacing at these sites showed entrainment of the atrial flutter with postpacing interval near tachycardia cycle length
187
a
b
188 Fig. 42.3 Intracardiac recordings during ablation between left inferior pulmonary vein and mitral annulus. Tachycardia showed abrupt change in activation sequence and cycle length during ablation, with counterclockwise right atrial activation followed by medial to lateral coronary sinus activation. The last few beats are consistent with typical counterclockwise right atrial flutter
E. Buch et al. I aVF V1 V6 ABL d HIS p HIS m HIS d HALO 10 HALO 9 HALO 8 HALO 7 HALO 6 HALO 5 HALO 4 HALO 3 HALO 2 HALO 1 CS 9, 10 CS 7, 8 CS 5, 6 CS 3, 4 CS 1, 2
TCL 228 ms
TCL 280 ms
Case 43 Bradley P. Knight
Case Summary A patient with recurrent PSVT undergoes an electrophysiology procedure. A regular narrow QRS complex tachycardia is induced with a CL 360 ms and a 1:1 AV relationship with simultaneous atrial and ventricular activation. Figure 43.1 shows the response immediately after overdrive atrial pacing at a CL of 350 ms. Shown are the surface recordings I, II, V1, and V5 and intracardiac recordings from the high right atrium (HRA), at the His bundle region with the ablation catheter (Abl), and right ventricular apex (RV). A stimulation channel is also displayed. What is the diagnosis of the tachycardia?
Case Discussion During a regular supraventricular tachycardia with a 1:1 AV relationship, the differential diagnosis includes AV nodal reentry (AVNRT), orthodromic AV reentry (ORT), and atrial
tachycardia (AT). Atrial overdrive pacing can be used to help “rule in” or “rule out” atrial tachycardia as the mechanism.1,2 The principle is that during atrial overdrive pacing just slightly faster than the tachycardia, a 1:1 AV relationship can sometimes be maintained. In the case of an AT, the first atrial depolarization immediately after atrial pacing is stopped will occur independent of the last QRS complex that was a result of AV conduction during atrial pacing. Therefore the first VA interval would most likely differ from the VA interval during tachycardia. In contrast, in the case of an AV nodal– dependent tachycardia, such as AVNRT or ORT, the VA interval of the first return beat is usually the same as the VA interval during tachycardia because the atrial depolarization is dependent on VA conduction. However, there is always a small chance that the first VA interval will be the same or similar as that during tachycardia by chance alone. In this example, the AH interval is increasing during atrial pacing, but 1:1 AV conduction remains intact. The first VA interval after pacing is the same as that during tachycardia. Because the short VA interval excludes ORT as a mechanism,2 the diagnosis is most likely AVNRT.
B.P. Knight Division of Cardiology, Northwestern Medical Center, 676 N. St. Clair, Suite 600, Chicago, IL 60611 e-mail:
[email protected] A. Natale et al. (eds.), Cardiac Electrophysiology, DOI: 10.1007/978-1-84996-390-9_43, © Springer-Verlag London Limited 2011
189
190
B.P. Knight
Fig. 43.1 Example of atrial overdrive pacing during tachycardia to determine the mechanism of tachycardia. See text for format and abbreviations
References 1. Knight BP, Zivin A, Souza J, Flemming M, Pelosi F, Goyal R, Man KC, Strickberger SA, Morady F. A technique for the rapid diagnosis of atrial tachycardia in the electrophysiology laboratory. J Am Coll Cardiol 1999;33:775-81.
2. Knight BP, Ebinger M, Oral H, Kim MH, Sticherling C, Flemming M, Pelosi F, Michaud GF, Strickberger SA, Morady F. Diagnostic value of tachycardia features and pacing maneuvers during paroxysmal supraventricular tachycardia. J Am Coll Cardiol 2000;36:574-82.
Case 44 Antonio Rossillo, Sakis Themistoclakis, Aldo Bonso, Andrea Corrado, and Antonio Raviele
Case Summary The patient is a 76-year-old male with history of AVNRT since childhood and very frequent episodes occurring in the last 2 months. Ischemic heart disease with mild reduction of LVEF was present. Coronary angiography showed a moderate stenosis in the right and circumflex coronary and no significant stenosis of LAD (Fig. 44.1). An EP study was scheduled to define the arrhythmia (Fig. 44.2). What type of tachycardia is this? What ablative strategy would you choose?
Case Discussion The tachycardia is clearly an AVNRT and the presence of a first-degree AV block suggests the absence or pathological
anterograde conduction through the fast pathway. According to this finding, a standard ablative approach targeting the slow pathway may result in complete AV block. Pace mapping Koch’s triangle to define anterograde and retrograde structures of the AV node is useful in preventing this complication. In this case, pace mapping showed an absence of an anterograde fast pathway and the presence of a retrograde fast pathway in the anteroseptal region of the Koch’s triangle. Therefore, a single RF lesion was applied in the region of the fast pathway, and Fig. 44.3 shows a junctional rhythm during the ablation. The red arrow shows when the retrograde conduction skips from the fast to the slow pathway (A = atrium, V = ventricle). A transient complete AV Block (15s) was present at the end of RF energy delivery. The EP study performed at the end of the procedure showed a normal Wenckebach point and the tachycardia was no longer inducible.
A. Rossillo (*), S. Themistoclakis, A. Bonso, A. Corrado, and A. Raviele, Cardiovascular Department, Ospedale dell’Angelo, Mestre, Venice, Italy e-mail:
[email protected] A. Natale et al. (eds.), Cardiac Electrophysiology, DOI: 10.1007/978-1-84996-390-9_44, © Springer-Verlag London Limited 2011
191
192
A. Rossillo et al.
Fig. 44.1 12-lead resting ECG (paper speed 25 mm/s) showing sinus rhythm with first-degree AV block
Fig. 44.2 Intracardiac recordings taken during programmed atrial stimulation (paper speed 200 mm/s) with the induction of the tachycardia with a cycle length of 428 ms study (A atrium, V ventricle, and H His bundle)
Case 44
193
Fig. 44.3 A junctional rhythm during the ablation. The red arrow shows when the retrograde conduction skips from the fast to the slow pathway (A atrium, V ventricle)
Bibliography Delise P, Gianfranchi L, Paparella N, et al. Clinical usefulness of slow pathway ablation in patients with both paroxysmal atrioventricular nodal reentrant tachycardia and atrial fibrillation. Am J Cardiol. 1997;79:1421-1423. Delise P, Sitta N, Bonso A, et al. Pace mapping of Koch’s triangle reduces risk of atrioventricular block during ablation of atrioventricular nodal reentrant tachycardia. J Cardiovasc Electrophysiol. 2005;16:30-35. Haissaguerre M, Gaita F, Fischer B, et al. Elimination of atrioventricular nodal reentrant tachycardia using discrete slow potentials to
guide application of radiofrequency energy. Circulation. 1992;85: 2162-2175. Haïssaguerre M, Jaïs P, Shah DC, et al. Analysis of electrophysiological activity in Koch’s triangle relevant to ablation of the slow AV nodal pathway. Pacing Clin Electrophysiol. 1997;20:2470-2481. Jackman WM, Beckman KJ, McClelland JH, et al. Treatment of supraventricular tachycardia due to atrioventricular nodal reentry, by radiofrequency catheter ablation of slow-pathway conduction. N Engl J Med. 1992;327:313-318. Natale A, Greenfield RA, Geiger MJ, et al. Safety of slow pathway ablation in patients with long PR interval: further evidence of fast and slow pathway interaction. Pacing Clin Electrophysiol. 1997; 20:1698-1703.
Case 45 Yash Y. Lokhandwala, Anoop K. Gupta, and Ranjan K. Thakur
Case Summary
Case Discussion
A 25-year-old man with normal heart presents with SVT. His tachycardia is initiated with ventricular pacing (Figs. 45.1 and 45.2). The response to ventricular pacing is shown in Fig. 45.3. What is the diagnosis?
Tachycardia is instantiated by VPDs. VA interval is long during tachycardia and the atrial pattern during tachycardia is similar to that of the retrograde A waves (in the limited leads available). V pacing at a faster rate than the tachycardia shows VA dissociation suggesting that this is atrial tachycardia.
Y.Y. Lokhandwala (*) KEM Hospital, Parel, Mumbai, India e-mail:
[email protected] A.K. Gupta Apollo Hospital, Ahmedabad, India e-mail:
[email protected] R.K. Thakur Thoracic and Cardiovascular Institute, Sparrow Health System, Michigan State University, 405 West Greenlawn, Suite 400, Lansing, MI 48910, USA e-mail:
[email protected] A. Natale et al. (eds.), Cardiac Electrophysiology, DOI: 10.1007/978-1-84996-390-9_45, © Springer-Verlag London Limited 2011
195
196
Y.Y. Lokhandwala et al.
Ruby hall clinic 2008046 I AVF V1 V6
RAD
RAP
HISD
HISP
10 mm/mV 100 mm/s
Fig. 45.1 Initiation of SVT
Version WIN2000/XP : EPTRACER V0.77
Case 45 2008046
I
AVF
V1
V6
RAD
RAP
HISD
HISP
10 mm/mV 100 mm/s
Fig. 45.2 Activation sequence of the SVT
197
198
Y.Y. Lokhandwala et al. 2008046
I AVF V1 V6
RAD
RAP
HISD
HISP
10 mm/mV 100 mm/s
Fig. 45.3 Ventricular pacing during SVT
Case 46 Eric Buch, Shiro Nakahara, Marmar Vaseghi, Noel G. Boyle, and Kalyanam Shivkumar
Case Summary A 79-year-old male with a history of hypertension, coronary artery bypass grafting, and implantable cardioverterdefibrillator for ventricular fibrillation, presented with symptomatic supraventricular tachycardia. He had failed antiarrhythmic medications and cardioversion attempts, and was referred for electrophysiology study and catheter ablation. Twelve-lead electrocardiogram showed organized
atrial activity with positive flutter waves in lead V1 and inferior leads, not suggestive of typical counterclockwise atrial flutter (Fig. 46.1). Diagnostic electrophysiology catheters were placed at the His bundle, the right ventricular apical septum, and in the coronary sinus. Intracardiac activation map obtained during tachycardia showed counterclockwise activation of the right atrium. Ablation of the cavotricuspid isthmus was performed without tachycardia termination. Why did the tachycardia persist?
Fig. 46.1 Surface electrocardiogram of clinical tachycardia. There is an organized atrial rhythm with low-amplitude flutter waves that are predominantly positive in lead V1. Variable conduction to the ventricles results in an irregular ventricular rhythm
E. Buch (*), S. Nakahara, M. Vaseghi, N.G. Boyle, and K. Shivkumar UCLA Cardiac Arrhythmia Center, David Geffen School of Medicine at UCLA, 10833 Le Conte Ave, A2-237 CHS, Los Angeles, CA 90095-1679 e-mail:
[email protected] A. Natale et al. (eds.), Cardiac Electrophysiology, DOI: 10.1007/978-1-84996-390-9_46, © Springer-Verlag London Limited 2011
199
200
E. Buch et al.
Case Discussion Entrainment mapping from the septal side of the cavotricuspid isthmus and the distal coronary sinus were performed. Although the tachycardia could be entrained from both sites, the post-pacing interval was much shorter from the distal coronary sinus, suggesting a left atrial flutter circuit with passive counterclockwise activation of the right atrium. After transseptal catheterization, electroanatomic mapping of the left atrium was performed. Voltage mapping revealed an area of scar on the left atrial roof. Activation a
map shown in Fig. 46.2a demonstrates a left atrial tachycardia with centrifugal activation of the left atrium originating near a scar on the left atrial roof. Several viable sites were found adjacent to the scar, with early activation (Fig. 46.2b). Pacing from these locations resulted in entrainment with concealed fusion, and post-pacing interval near tachycardia cycle length. During ablation (Fig. 46.3) the tachycardia spontaneously terminated, and could not be reinduced. This case illustrates that typical counterclockwise activation of the right atrium does not prove that it is part of the tachycardia circuit. b
Fig. 46.2 (a) Three-dimensional electroanatomical activation map of the left atrium during tachycardia, anteroposterior view. Earliest activation of the left atrium is at the roof (red area), just lateral to a region of scar (gray
area) near the superior aspect of the left atrial septum. (b) Right posterior oblique view. Radiofrequency energy was applied at viable sites adjacent to the scar (red points within red area) with low-amplitude potentials
ISI
200 ms
aVF V1 V5 ABL d ABL p HIS p
C
HIS m
C
HIS d
C
CS 9, 10
Fig. 46.3 During ablation at one of the sites shown in Fig.46.2b, the tachycardia terminated and could not be reinduced. Note the resumption of atrioventricular paced rhythm at the right side of the tracing
CS 7, 8 CS 5, 6 CS 3, 4 CS 1, 2
11:34:13 AM
11:34:14 AM
Case 47 Bradley P. Knight
Case Summary A patient with recurrent PSVT undergoes an electrophysiology procedure. Fig. 47.1 shows a recording during the procedure. Shown are the surface recordings and intracardiac recordings from the His bundle electrogram (HBE) ablation catheter (Abl), coronary sinus (CS), and right ventricular apex (RV). What is the diagnosis of the tachycardia?
Case Discussion The differential diagnosis of a regular narrow QRS complex tachycardia in the EP lab includes AV nodal reentry, orthodromic AV reentry, and atrial tachycardia. The transition zones during tachycardia can be very useful when trying to make a diagnosis. In this case the tachycardia terminates. Before termination, one can see that there is simultaneous
activation of the atrium and ventricle with a septal VA interval that is close to zero. This excludes AV reentry using an accessory pathway. When attempting to measure the septal VA interval, it can be difficult at times to differentiate the atrial signal from the ventricular signal on the His bundle recording. In this example, the proximal coronary sinus recordings (CS9-10) can be used to identify the timing of septal atrial activation. The termination of the tachycardia is associated with spontaneous AV block. This observation excludes an AT which would be expected to conduct to the ventricle upon termination if there is a 1:1 conduction during the tachycardia. In this case the tachycardia actually slows slightly before termination. Therefore, it would be extremely unlikely that an atrial tachycardia that spontaneously slows just before termination would be associated with AV block on the last beat. Another observation that excludes an atrial tachycardia is that as the tachycardia cycle length increases the VA interval remains constant. This fixed relationship suggests an AV nodal–dependent tachycardia.
B.P. Knight Division of Cardiology, Northwestern Medical Center, 676 N. St. Clair, Suite 600, Chicago, IL, 60611 e-mail:
[email protected] A. Natale et al. (eds.), Cardiac Electrophysiology, DOI: 10.1007/978-1-84996-390-9_47, © Springer-Verlag London Limited 2011
201
202
B.P. Knight I
320
330
340
400
780
430
680
390
780
450
II III V1 V5 HBE d HBE p Map d Map p CS 1-2 CS 3-4 CS 5-6 CS 7-8 CS 9-10 RV d Stim
Fig. 47.1 Tracing recorded during an electrophysiology procedure in a patient with recurrent paroxysmal supraventricular tachycardia. Spontaneous termination of tachycardia occurs. See text for format and abbreviations
Case 48 Antonio Rossillo, Sakis Themistoclakis, Aldo Bonso, Andrea Corrado, and Antonio Raviele
Case Summary A 47-year-old male patient without any structural heart disease presented with a history of palpitations since childhood. All ECGs recorded while the patient experienced palpitations showed a wide QRS complex tachycardia with a left bundle branch block pattern. The arrhythmia was always
sustained and external DC shock was necessary on three occasions to restore sinus rhythm. Pharmacologic treatment (including amiodarone and propafenone) was unsuccessful and the two last episodes of tachycardia were reported to be faster than usual and poorly tolerated (Figs. 48.1–48.8). Based on the previous tracings, what is the tachycardia mechanism?
Fig. 48.1 12-lead resting ECG (paper speed 25 mm/s) showing sinus rhythm, a normal PR interval (170 ms), and a QRS width of 100 ms with slight slurring of the initial portion of the QRS
A. Rossillo (*), S. Themistoclakis, A. Bonso, A. Corrado, and A. Raviele Cardiovascular Department, Ospedale dell’Angelo, Mestre, Venice, Italy A. Natale et al. (eds.), Cardiac Electrophysiology, DOI: 10.1007/978-1-84996-390-9_48, © Springer-Verlag London Limited 2011
203
204
Fig. 48.2 Intracardiac recordings at baseline during the electrophysiology study (paper speed 100 mm/s). Four surface ECG leads (I, aVF, V1, V6), two bipolar recordings from the His bundle region (HIS D for distal and HIS P for proximal), seven bipolar recordings from the epi-
A. Rossillo et al.
cardial side of the tricuspid annulus obtained by a Cardima catheter positioned in the right coronary artery (C1–C7), and one bipolar recording from the coronary sinus (SC). AH interval is 80 ms and HV interval 30 ms
Case 48
Fig. 48.3 Intracardiac recordings taken during programmed atrial stimulation (paper speed 200 mm/s). Same display as shown in Fig. 48.2. During programmed atrial stimulation (coupling interval between S1 and S2: 375 ms) A-H interval increased from 105 to 125 ms,
205
H-V interval decreased from 30 to 10 ms, and the QRS complex on the surface ECG became wider with a left bundle branch block morphology
206
A. Rossillo et al.
Fig. 48.4 Intracardiac recordings taken during programmed atrial stimulation (paper speed 200 mm/s). Same display as shown in Fig. 48.2. At the coupling interval of 300 ms, atrioventricular node duality was demonstrated with an A-H interval which increased suddenly from 111 to 192 ms
Case 48
Fig. 48.5 Intracardiac recordings taken during atrial stimulation (cycle length 410 ms; paper speed 100 mm/s). Same display as shown in Fig. 48.2. Ventricular preexcitation is present with a negative (−10 ms)
207
H-V interval, a His bundle potential identified at the very beginning of the QRS complex, and a left bundle branch block morphology on the surface ECG
208
A. Rossillo et al.
Fig. 48.6 Intracardiac recordings during atrial stimulation (paper speed 100 mm/s). Same display as that shown in Fig. 48.2. When atrial pacing at a cycle length of 270 ms is abruptly interrupted, two atrioventricular node echoes appeared (slow–fast type)
Case 48
Fig. 48.7 Intracardiac recordings taken during the tachycardia induced during programmed atrial stimulation (paper speed 100 mm/s). Same display as Fig. 48.2. The tachycardia had a cycle length of 330 ms and
209
a left bundle branch block morphology. Intracardiac recordings showed that the site of earliest atrial activation occurred in the His bundle region with a retrograde His deflection
210
A. Rossillo et al.
Fig. 48.8 12-lead ECG taken during the induced tachycardia (rate: 186 bpm; paper speed 25 mm/s) showing left bundle branch block morphology. In fact, the induced tachycardia on 12-lead ECG is identical to the one observed during rapid atrial stimulation (see Fig. 48.6)
Case Discussion The patient has an atriofascicular pathway or so-called Manheim. Based on Fig. 48.9 this is likely antidromic tachycardia with retrograde conduction over either the fast or slow pathway (Figs. 48.10–48.13).
Case 48
Fig. 48.9 Intracardiac recordings taken during the tachycardia (paper 200 mm/s). Same display as shown in Fig. 48.2. A spontaneous change in H-A interval was observed (from 120 to 69 ms) without any change in atrial retrograde activation pattern but with shortening of the tachy-
211
cardia cycle length (from 670 to 570 ms). This figure illustrates an abrupt switch from the slow to the fast retrograde pathway excluding the slow–fast atrioventricular node reentrant tachycardia as the mechanism of the induced tachycardia
212
Fig. 48.10 Intracardiac recordings taken during mapping of the tricuspid annulus during sinus rhythm. Same representation as shown in Fig. 48.2, except that a bipolar recording obtained from the tip of the
A. Rossillo et al.
ablation catheter (MC BIP) is now displayed. In the lateral region of the tricuspid annulus a specific potential was recorded (M potential)
Case 48
Fig. 48.11 Intracardiac recordings taken during mapping of the tricuspid annulus during sinus rhythm. Same representation as shown in Fig. 48.2. When mapping the region where the M potential was
213
recorded, mechanical block occurred (normalization of the QRS complex on the surface ECG, normalization of H-V interval) (arrow). Ablation was performed at this site
214
A. Rossillo et al.
Fig. 48.12 12-lead ECG recorded after the ablation procedure (paper speed 25 mm/s). Radiofrequency current was applied on the site shown in Fig. 48.12. The 12-lead ECG is now normal without any ventricular preexcitation
Case 48
215
Bibliography Ao
Cdx H
VM
M VT
SC
Cdx
Fig. 48.13 Picture showing the circuit of the arrhythmia (VT = tricuspid valve, MV = mitral valve, Ao = Aorta, Cdx = right coronary artery, H = His bundle, and M = mahaim fibers)
Aliot E, de Chillou C, Revault d’Allones G. Mahaim tachycardias. Eur Heart J. 1998;19(suppl E):E25-E31. Ellenbogen KA, Ramirez NM, Packer DL, et al. Accessory nodoventricular (Mahaim) fibers: a clinical review. PACE. 1986;9:868-875. Gallagher JJ, Smith WM, Kasell JH, et al. Role of Mahaim fibers in cardiac arrhythmias in man. Circulation. 1981;64:176-184. Haissaguerre M, Cauchemez B, Marcus F, et al. Characteristics of the ventricular insertion sites of accessory pathways with anterograde decremental conduction properties. Circulation. 1995;91:10771085. Josephson ME. Nodoventricular and fasciculoventricular bypass tracts. In: Josephson ME, ed. Clinical Cardiac Electrophysiology: Techniques and Interpretation. 2nd ed. Malvern: Lea & Febiger; 1993:396-416. Mounsey JP, Griffith MJ, McComb JM, et al. Radiofrequency ablation of Mahaim fiber following localization of Mahaim pathway potentials. J Cardiovasc Electrophysiol. 1994;5:432-434. Tchou P, Lehmann MH, Jazayeri M, et al. Atriofascicular connections or a nodoventricular Mahaim fiber? Electrophysiologic elucidation of the pathway and associated reentrant circuit. Circulation. 1988;77:837-841.
Case 49 Eric Buch, Shiro Nakahara, Marmar Vaseghi, Noel G. Boyle, and Kalyanam Shivkumar
Case Summary
Case Discussion
A 30-year-old female with primary pulmonary hypertension had recurrent atrial flutter despite amiodarone treatment and multiple cardioversions. Because of worsened dyspnea resulting from atrial flutter, she was referred for electrophysiology study and catheter ablation. Preprocedural surface electrocardiograms, both recorded during the month prior to ablation, showed organized atrial activity with variable flutter wave morphology (Fig. 49.1). Based on the 12-lead ECG, what is the diagnosis? Do these arrhythmias arise from multiple flutter circuits?
By definition, typical atrial flutter is a macroreentrant tachycardia bound anteriorly by the tricuspid isthmus and posteriorly by the inferior vena cava, the terminal crest, and most commonly, the superior vena cava.1 The majority of these tachycardias are counterclockwise (CCW), with activation descending the anterolateral and ascending the septal RA. Less commonly, isthmus-dependent atrial flutter is clockwise (CW), with activation descending the septal and ascending the anterolateral RA. Multiple studies have attempted to characterize the surface ECG appearance of
Fig. 49.1 Surface electrocardiogram showing flutter waves that are predominantly negative in lead V1 and biphasic in the inferior leads, not typical of cavotricuspid isthmus dependent counterclockwise atrial flutter
E. Buch (*), S. Nakahara, M. Vaseghi, N.G. Boyle, and K. Shivkumar UCLA Cardiac Arrhythmia Center, David Geffen School of Medicine at UCLA, 10833 Le Conte Ave, A2-237 CHS, Los Angeles, CA 90095-1679 e-mail:
[email protected] A. Natale et al. (eds.), Cardiac Electrophysiology, DOI: 10.1007/978-1-84996-390-9_49, © Springer-Verlag London Limited 2011
217
218
CW and CCW flutter.2 It is currently accepted that predominantly negative F wave deflections in the inferior leads and in lead V6 and positive deflection in V1 indicate CCW flutter, while CW flutter has positive F waves in the inferior leads and V6, and negative F waves in lead V1.1,2 In this
E. Buch et al.
case, negative F wave deflection in V1 and positive deflection in inferior leads (Fig. 49.1a) predicted CW flutter, which was established by intracardiac mapping (Fig. 49.2). The other surface ECG (Fig. 49.1b) was likely CCW flutter. Neither arrhythmia was inducible after ablation at the
a
b
Fig. 49.2 (a) Left anterior oblique fluoroscopic view of intracardiac electrophysiology catheters. The duopecapolar catheter (D1–D10) is curled in the right atrium, with the proximal bipoles (D9, D10) near the right atrial roof, the middle bipoles (D4–D8) along the lateral right atrial wall, and the distal bipoles (D2, D3) near the cavotricuspid
isthmus, with the most distal bipole (D1) in the proximal coronary sinus. (b) Surface and intracardiac recordings. The activation sequence proceeds from distal to proximal, consistent with clockwise isthmusdependent atrial flutter
Case 49
219
I aVF V1 V6 aVR ABL d ABL p D10 D9 D8 D7 D6 D5 D4 D3 D2 D1
200 ms
11:15:07 AM
11:15:08 AM
Fig. 49.3 With the ablation catheter positioned as shown in Fig. 49.2 above, radiofrequency energy was applied. Intracardiac recordings during ablation, showing termination of tachycardia
cavotricuspid isthmus (Fig. 49.3). Of note, F wave morphology not adhering to the above criteria has been described in a minority of patients when the left atrium is activated by Bachmann’s bundle. In this case, CCW flutter can manifest as positive F waves in the inferior leads, and clockwise flutter can manifest as negative F waves in the inferior leads. (Figs 49.2 and 49.3).3
References 1. Saoudi N, Cosio F, Waldo A, et al. Classification of atrial flutter and regular atrial tachycardia according to electrophysiologic mechanism and anatomic bases: a statement from a joint expert group
from the Working Group of Arrhythmias of the European Society of Cardiology and the North American Society of Pacing and Electrophysiology. J Cardiovasc Electrophysiol. 2001;12: 852-866. 2. Milliez P, Richardson AW, Obioha-Ngwu O, Zimetbaum PJ, Papageorgiou P, Josephson ME. Variable electrocardiographic characteristics of isthmus-dependent atrial flutter. J Am Coll Cardiol. 2002;40:1125-1132. 3. Oshikawa N, Watanabe I, Masaki R, et al. Relationship between polarity of the flutter wave in the surface ECG and endocardial atrial activation sequence in patients with typical counterclockwise and clockwise atrial flutter. J Interv Card Electrophysiol. 2002; 7:215-223.
Case 50 Bradley P. Knight
Case Summary
Left-sided slow pathway ablation
An attempt was made to ablate the slow pathway in a 45-year-old woman with recurrent SVT and reproducibly inducible typical AV nodal reentry. Using a standard 4-mmtip electrode ablation catheter, a total of 55 applications of RF current were delivered using 50 W at several sites just outside and slightly within the coronary sinus ostium. Despite adequate tissue heating and the occurrence of an accelerated junctional rhythm with most lesions, the tachycardia remained inducible. What other approach could be used?
RF site
Case Discussion A very small number of patients who have what appear to be the slow–fast type of AV nodal reentry cannot be successfully ablated using a right-sided approach. It is likely that there are left-sided inputs to the AV node that require ablation to eliminate slow pathway function. Using a retrograde aortic approach the left posterior septal aspect of the mitral annulus was explored (Fig. 50.1). A site was identified that was associated with a complex, fractionated atrial electrogram (Fig. 50.2). A single application of RF current at this site resulted in an accelerated junctional rhythm and elimination of slow pathway function (Fig. 50.3).
Fig. 50.1 A fluoroscopic image in a left anterior oblique projection during ablation of a slow pathway using a left-sided approach. Three catheters can be seen in the right heart including a right ventricular, His-bundle, and coronary sinus catheter. There is also a left sided ablation catheter that has been positioned along the posteroseptal mitral annulus using a retrograde aortic approach
B.P. Knight Division of Cardiology, Northwestern Medical Center, 676 N. St. Clair, Suite 600, Chicago, IL 60611, USA e-mail:
[email protected] A. Natale et al. (eds.), Cardiac Electrophysiology, DOI: 10.1007/978-1-84996-390-9_50, © Springer-Verlag London Limited 2011
221
222
Fig. 50.2 Tracing showing the electrogram at the successful ablation site during sinus rhythm in a patient undergoing a left-sided slow AV nodal pathway ablation. Shown are surface recordings from leads I, II, V1, and V5, and the intracardiac recordings from high right atrium (HRA), His-bundle electrogram region (HBE), left ventricle (LV), and right ventricular apex (RVA)
Bibliography Katritsis DG, Becker AE, Ellenbogen KA, et al. Right and left inferior extensions of the atrioventricular node may represent the anatomic substrate of the slow pathway in humans. Heart Rhythm. 2004;1:582-586.
B.P. Knight
Fig. 50.3 Tracing demonstrating an accelerated junctional rhythm during delivery of radiofrequency current at the successful ablation site in a patient undergoing a left-sided slow AV nodal pathway ablation. The format and abbreviations are the same as in Figure 50.2
Case 51 Yash Y. Lokhandwala, Anoop K. Gupta, and Ranjan K. Thakur
Case Summary A 54-year-old woman with a normal heart presents to the EP lab with an SVT (Fig. 51.1). The tachycardia could be induced with three APCs or with ventricular premature beats (Figs. 51.2 and 51.3). Explain the mechanism of the tachycardia?
Tachycardia is initiated by a decrementally conducting PVC. VA time is long; earliest in proximal CS. With PVC placed when the His was refractory, no atrial preexcitation was observed, suggesting that the likely mechanism was atypical AVNRT. There is slow–intermediate and slow–slow AVNRT, hence ablation of the slow pathway was performed; no tachycardia was observed after this. The VA time at initiation of even typical AVNRT often shortens over the initial few complexes, more so in older patients; hence an apparently atypical AVNRT may later appear typical (Fig. 51.4).
Case Discussion With application of three atrial extrastimuli, tachycardia is induced. Induction is preceded by lengthening of the AH interval; this suggests that the likely mechanism is AVNRT.
Y.Y. Lokhandwala (*) KEM Hospital, Parel, Mumbai, India e-mail:
[email protected] A.K. Gupta Apollo Hospital, Ahmedabad, India e-mail:
[email protected] R.K. Thakur Thoracic and Cardiovascular Institute, Sparrow Health System, Michigan State University, 405 West Greenlawn, Suite 400, Lansing, MI 48910, USA e-mail:
[email protected] A. Natale et al. (eds.), Cardiac Electrophysiology, DOI: 10.1007/978-1-84996-390-9_51, © Springer-Verlag London Limited 2011
223
224
Y.Y. Lokhandwala et al.
RUBY HALL CLINIC 2008045 I II III AVR AVL AVF V1 V2 V3 V4 V5 V6 10 mm/mV 25 mm/s
Fig. 51.1 SVT
Version WIN2000/XP : EPTRACER V0.77
Case 51
225 2008045
I AVF V1 V6 CS910 CS78 CS56 CS34 CS12 HISD HISP RFD RFP 10 mm/mV 100 mm/s
Fig. 51.2 Three PACs initiate tachycardia
226
Y.Y. Lokhandwala et al. 2008045
I AVF V1 V6
CS34
CS12
HISD
HISP
RFD
10 mm/mV 100 mm/s
Fig. 51.3 Tachycardia initiation with ventricular premature beat
Case 51
227 2008045
I AVF V1 V6 CS910 CS78 CS56 CS34 CS12 HISD HISP RFD
Fig. 51.4 The patient later showed this pattern – typical AVNRT
Case 52 Eric Buch, Shiro Nakahara, Marmar Vaseghi, Noel G. Boyle, and Kalyanam Shivkumar
Case Summary A 79-year-old male with a history of hypertension, diabetes, coronary artery bypass grafting, recently diagnosed with atrial flutter and referred for electrophysiology study and catheter ablation. Initial 12-lead electrocardiogram (Fig. 52.1) showed organized atrial activity with a cycle length of
approximately 220 ms with negative F waves in leads II, III, and aVF, positive F waves in V1, suggesting typical counterclockwise atrial flutter. Conduction to ventricles was predominantly 4:1. Electroanatomic mapping suggested macroreentrant arrhythmia in the right atrium. As seen in Fig. 52.2, the direction of activation was counterclockwise, and the circuit
Fig. 52.1 Surface electrocardiogram of clinical tachycardia. Atrial flutter is seen, with F waves that are predominantly negative in the inferior leads and upright in V1, consistent with typical counterclockwise atrial flutter
E. Buch (*), S. Nakahara, M. Vaseghi, N.G. Boyle, and K. Shivkumar UCLA Cardiac Arrhythmia Center, David Geffen School of Medicine at UCLA, 10833 Le Conte Ave, A2-237 CHS, Los Angeles, CA 90095-1679 e-mail:
[email protected] A. Natale et al. (eds.), Cardiac Electrophysiology, DOI: 10.1007/978-1-84996-390-9_52, © Springer-Verlag London Limited 2011
229
230
E. Buch et al.
Fig. 52.2 Three-dimensional electroanatomic mapping during tachycardia. Anteroposterior view of right atrium shows the reentrant circuit, with early meeting late activation in a circuit around the tricuspid annulus. Pacing from the tip of the ablation catheter, which was positioned at the cavotricuspid isthmus, showed entrainment with concealed fusion and a postpacing interval of 10 ms longer than tachycardia cycle length
involved the cavotricuspid isthmus. What maneuvers are useful in proving that the cavotricuspid isthmus is part of the tachycardia?
Case Discussion One maneuver that can show whether a given location is close to the arrhythmia circuit is entrainment mapping.1 This is accomplished by pacing slightly faster than the tachycardia cycle length to repeatedly reset the reentrant circuit. If paced P wave morphology is identical to that observed during atrial flutter, this is termed entrainment with concealed fusion. If tachycardia continues after cessation of pacing, the first postpacing interval will be close to the tachycardia cycle length, provided that the pacing site is near the reentrant circuit.2 Postpacing interval is defined as the time from last paced stimulus to the next local electrogram during tachycardia, measured at the site of pacing.
In this case, pacing from the cavotricuspid isthmus showed entrainment with concealed fusion, with identical surface flutter wave morphology and intracardiac activation pattern as compared to tachycardia. The postpacing interval was within 10 ms of the flutter cycle length. This showed that the cavotricuspid isthmus was likely part of the tachycardia circuit, and therefore radiofrequency ablation of the isthmus was performed. The ablation catheter was advanced into the right ventricle, curved, and pulled inferiorly to seat it on the cavotricuspid isthmus, with the catheter tip at 6 o’clock in the left anterior oblique view. A series of radiofrequency applications were delivered to create a line of conduction block across the cavotricuspid isthmus. During ablation, the cycle length suddenly prolonged and the arrhythmia terminated with a long pause (Fig. 52.3). Normal sinus node function and atrioventricular conduction resumed after a few seconds, and atrial flutter was no longer inducible at the end of the ablation procedure.
Case 52
231
Fig. 52.3 Three-dimensional electroanatomic mapping during radiofrequency ablation, caudal left anterior oblique view. As the line of conduction block across the cavotricuspid isthmus was completed, the flutter terminated spontaneously and could not be reinduced
References 1. Cosio FG, Lopez Gil M, Arribas F, Palacios J, Goicolea A, Nunez A. Mechanisms of entrainment of human common flutter studied with multiple endocardial recordings. Circulation. 1994;89:2117-2125.
2. Stevenson WG, Khan H, Sager P, et al. Identification of reentry circuit sites during catheter mapping and radiofrequency ablation of ventricular tachycardia late after myocardial infarction. Circulation. 1993;88:1647-1670.
Case 53 Bradley P. Knight
Case Summary A patient is scheduled to undergo a catheter ablation procedure for recurrent PSVT, which appears most likely to be typical AV nodal reentry. You discover that the patient underwent insertion of an inferior vena cava filter 2 years ago after a pulmonary embolism. How would you proceed with this case?
Case Discussion When performing a slow pathway ablation procedure, it is important to monitor VA conduction during the accelerated junctional rhythm to avoid AV block. Therefore, it is usually necessary to place two catheters in the heart during the procedure. Unfortunately, some patients represent vascular access challenges. In this case, the patient has a filter in the
inferior vena cava. Although there are some risks associated with crossing the filter with a catheter, it is clear that an electrophysiology procedure can be performed safely in these patients from the femoral vein.1 It is helpful to be certain that the vena cava is patent before the procedure with a CT scan or venography. But assuming that the IVC is patent, long guiding sheaths and electrophysiology catheters can be passed safely through the filter. Figure 53.1 shows a long sheath already through the filter, and a second one being advanced over a guidewire that has been passed through the filter. There are times, however, when the vena cava is completely obstructed. In this situation, options include a superior or transhepatic central venous approach. One reasonable approach that has been shown to be successful in patients with AV nodal reentry is to place two central venous sheaths in the right internal jugular vein.2 This allows placement of a diagnostic catheter in the high right atrium and an ablation catheter along the tricuspid annulus (Fig. 53.2).
B.P. Knight Division of Cardiology, Northwestern Medical Center, 676 N. St. Clair, Suite 600, Chicago, IL 60611 e-mail:
[email protected] A. Natale et al. (eds.), Cardiac Electrophysiology, DOI: 10.1007/978-1-84996-390-9_53, © Springer-Verlag London Limited 2011
233
234
Fig. 53.1 Fluoroscopic image during catheter placement from a femoral vein approach through an inferior vena cava filter. Two sheaths were advanced over a guidewire to access the heart with two electrophysiology catheters. See text for details
References 1. Sinha SK, Harnick D, Gomes JA, Mehta D. Electrophysiologic interventions in patients with inferior vena cava filters: safety and efficacy of the transfemoral approach. Heart Rhythm. 2005;2:15-18.
B.P. Knight
Fig. 53.2 Right anterior oblique fluoroscopic view after placement of an ablation catheter and diagnostic quadrapolar catheter via the internal jugular vein, during a catheter ablation procedure for typical atrioventricular nodal reentry in a patient without inferior vena cava access 2. Salem Y, Burke MC, Morady F, Knight BP. Slow pathway ablation for atrioventricular nodal reentry using a right internal jugular vein approach: a case series. Pacing Card Electrophysiol. 2006; 29:59-62.
Case 54 Yash Y. Lokhandwala, Anoop K. Gupta, and Ranjan K. Thakur
Case Summary An 18-year-old young man with normal heart presents to the EP lab. His initial tracing shows AF followed by an SVT (Fig. 54.1). Figures 54.2 and 54.3 show the response to ventricular premature beats and ventricular pacing. Does this help elucidate the cause of the tachycardia?
Case Discussion Even a very early PVC from the right ventricle does not advance the “A.” This makes AVRT extremely unlikely because (1) left-sided AP is ruled out by the activation
sequence and (2) a right-sided AP should have caused atrial preexcitation. This PVC does not rule out AVNRT because to do so, the PVC must be delivered when the His is refractory. Thus, the differential diagnoses at this point include AVNRT and atrial tachycardia. Ventricular pacing is performed during tachycardia. VA dissociation is seen with continuation of tachycardia, proving a diagnosis of atrial tachycardia. The tachycardia itself does not lend itself to analysis of the P waves. The tachycardia continues in spite of bursts of PVCs, suggesting that this is atrial tachycardia. P wave morphology is revealed after the bursts; upright in I, II, III and aVF, suggesting high right atrial origin (Fig. 54.4)
Y.Y. Lokhandwala (*) KEM Hospital, Parel, Mumbai, India e-mail:
[email protected] A.K. Gupta Apollo Hospital, Ahmedabad, India e-mail:
[email protected] R.K. Thakur Thoracic and Cardiovascular Institute, Sparrow Health System, Michigan State University, 405 West Greenlawn, Suite 400, Lansing, MI 48910, USA e-mail:
[email protected] A. Natale et al. (eds.), Cardiac Electrophysiology, DOI: 10.1007/978-1-84996-390-9_54, © Springer-Verlag London Limited 2011
235
236
Y.Y. Lokhandwala et al. 2008043
I AVF V1 V6 CS910 CS56 CS12 HISD HISP
Fig. 54.1 The left half shows AF which initiates a narrow complex SVT. AF termination is followed by a junctional beat and then, SVT. SVT starts with an “A”
Case 54
237 2008043
I AVF V1 V6 CS910 CS56 CS12 HISD HISP RFD RFP 10 mm/mV 300 mm/s
Fig. 54.2 A VPC is delivered during the tachycardia
238
Y.Y. Lokhandwala et al. 2008043
I AVF V1 V6 CS910 CS56 CS12 HISD HISP RFD RFP 10 mm/mV 150 mm/s
Fig. 54.3 V pacing during the tachycardia
Case 54
239 2008043
I AVF V1 V6 CS910 CS56 CS12 HISD HISP RFD RFP 10 mm/mV 150 mm/s
Fig. 54.4 P wave morphology can often be uncovered after ventricular pacing
Case 55 Eric Buch, Shiro Nakahara, Marmar Vaseghi, Noel G. Boyle, and Kalyanam Shivkumar
Case Summary A 52-year-old male with history of hypertension, diabetes, and coronary artery disease status post coronary artery bypass grafting was recently seen for palpitations and dyspnea. Surface
electrocardiogram showed a regular supraventricular tachycardia at a rate of approximately 250 per minute, with 2:1 conduction to the ventricles (Fig. 55.1). He was referred for electrophysiology study and catheter ablation. Based on this ECG tracing, what is the most likely mechanism of this tachycardia?
Fig. 55.1 Surface electrocardiogram of the clinical tachycardia. Sawtooth flutter waves are predominantly negative in the inferior leads, upright in V1, and inverted in V6
E. Buch (*), S. Nakahara, M. Vaseghi, N.G. Boyle, and K. Shivkumar UCLA Cardiac Arrhythmia Center, David Geffen School of Medicine at UCLA, 10833 Le Conte Ave, A2-237 CHS, Los Angeles, CA 90095-1679 e-mail:
[email protected] A. Natale et al. (eds.), Cardiac Electrophysiology, DOI: 10.1007/978-1-84996-390-9_55, © Springer-Verlag London Limited 2011
241
242
E. Buch et al.
a
b I aVF V1 V6 HRA HIS p HIS m HIS d CS 9,10 CS 7,8 CS 5,6 CS 3,4 CS 1,2 DD 19,20 DD 17,18 DD 15,16 DD 13,14 DD 11,12 DD 9,10 DD 7,8 DD 5,6 DD 3,4 DD 1,2 RVA
100 ms
T C C C
Fig. 55.2 (a) Fluoroscopic left anterior oblique view showing catheter positions. The duodecapolar (DD) catheter is curled in the right atrium such that the proximal, mid, and distal bipoles of the catheter are near the right atrial roof, anterolateral wall, and coronary sinus os, respectively. Arrows show the direction of activation. (b) Intracardiac tracings.
3:15:00 PM
Right atrial activation is seen to proceed from septal to lateral along the superior right atrium, down the anterolateral wall to the cavotricuspid isthmus area, then up the atrial septum to complete the circuit. As seen in the CS catheter, bystander left atrial activation occurs simultaneously with right atrial septal activation
Case 55
Case Discussion Diagnostic electrophysiology catheters were placed at the right atrium, the His bundle, the right ventricular septum, and in the coronary sinus. A multipolar deflectable catheter was curled in the right atrium with its distal bipoles near the mouth of the coronary sinus, and the proximal bipoles near the roof of the right atrium (Fig. 55.2a). Intracardiac recordings from these catheters show that right atrial activation proceeds in a counterclockwise direction in the left anterior oblique view, from proximal to distal bipole on the duodecapolar catheter (Fig. 55.2b). An ablation catheter was advanced into the right ventricle, pulled back slowly to seat the tip on the cavotricuspid isthmus. Beginning at the edge of the tricuspid annulus, where the ventricular electrogram from the distal bipole of the ablation catheter was larger than the atrial electrogram, a line of
References 1. Disertori M, Inama G, Vergara G, Guarnerio M, Del Favero A, Furlanello F. Evidence of a reentry circuit in the common type of atrial flutter in man. Circulation. 1983;67:434-440. 2. Cabrera JA, Sanchez-Quintana D, Ho SY, Medina A, Anderson RH. The architecture of the atrial musculature between the orifice of the inferior caval vein and the tricuspid valve: the
243
ablation was created across the isthmus, connecting the tricuspid annulus to the inferior vena cava. In the right anterior oblique projection, the ablation catheter was seen to be well separated from the His bundle catheter. In the left anterior oblique projection, the ablation catheter was near the 6 o’clock position. During ablation, the flutter terminated. After ablation, bidirectional conduction block across the isthmus was verified by pacing from either side of the isthmus. The common form of atrial flutter is a macroreentrant right atrial arrhythmia with a counterclockwise pattern of activation as seen from the left anterior oblique view.1 Critical to perpetuation of the arrhythmia is a narrow area of slow conduction bounded by the inferior vena cava and tricuspid valve, called the cavotricuspid isthmus.2 Surgical or catheterbased ablation of the isthmus can result in conduction block and effective treatment of this arrhythmia.3,4
anatomy of the isthmus. J Cardiovasc Electrophysiol. 1998;9: 1186-1195. 3. Klein GJ, Guiraudon GM, Sharma AD, Milstein S. Demonstration of macroreentry and feasibility of operative therapy in the common type of atrial flutter. Am J Cardiol. 1986;57:587-591. 4. Feld GK, Fleck RP, Chen PS, et al. Radiofrequency catheter ablation for the treatment of human type 1 atrial flutter. Identification of a critical zone in the reentrant circuit by endocardial mapping techniques. Circulation. 1992;86:1233-1240.
Case 56 Bradley P. Knight
Case Summary A regular supraventricular tachycardia with a 1:1 AV relationship and a CL of 420 ms is induced in the EP lab. Activation of the high right atrium is coincident with the
QRS complex. Ventricular overdrive pacing is delivered from the right ventricular apex during tachycardia at a cycle length 30 ms less than the tachycardia CL. Figure 56.1 shows the response immediately upon cessation of ventricular pacing. What is the mechanism of tachycardia?
I II V1 V5
Fig. 56.1 This tracing was recorded when ventricular overdrive pacing was stopped after being delivered during a regular supraventricular tachycardia with 1:1 atrioventricular conduction. Shown are surface recordings from leads I, II, V1, and V5, and the intracardiac recordings from high right atrium (HRA), His-bundle electrogram region (HBE), and right ventricular apex (RVA). Note the response after pacing can be described as a VAV response. See text for further description
HRA
390
390
480
420
430
420
420
HBEd HBEm HBEp RVA
390
390
Stim University of Chicago
B.P. Knight Division of Cardiology, Northwestern Medical Center, 676 N. St. Clair, Suite 600, Chicago, IL 60611, USA e-mail:
[email protected] A. Natale et al. (eds.), Cardiac Electrophysiology, DOI: 10.1007/978-1-84996-390-9_56, © Springer-Verlag London Limited 2011
245
246
Case Discussion During a regular supraventricular tachycardia with a 1:1 AV relationship, the differential diagnosis includes AV nodal reentry (AVNRT), orthodromic AV reentry (ORT), and atrial tachycardia (AT). The ventricular overdrive pacing can be used in the EP laboratory to help “rule in” or “rule out” atrial tachycardia as the mechanism.1,2 The principle involved is that when ventricular pacing during tachycardia often results in 1:1 retrograde conduction, the atrial rate accelerates to the ventricular pacing rate until ventricular pacing is stopped. In the case of an AT, the last atrial depolarization that is a result of retrograde VA conduction will block antegrade, because the AV node will be refractory in the antegrade direction after recent retrograde activation. After resumption of the AT, the first atrial depolarization will conduct to the ventricle. After ventricular pacing is stopped, the sequence of activation can be described as atrial–atrial–ventricular (VAAV; Figs. 56.1 and 56.2). In contrast, in the case of an AV nodal dependent tachycardia, such as AVNRT or ORT, ventricular pacing with 1:1 VA conduction entrains the tachycardia which, if it does not terminate, will conduct anterograde over the AV node as soon as the tachycardia resumes. After entraining an AV nodal dependent tachycardia, the sequence A
V
Fig. 56.2 Ladder diagram showing the response to overdrive ventricular pacing during an atrial tachycardia. The response after pacing is a VAAV response. See text for more detail
B.P. Knight A
V
Fig. 56.3 Ladder diagram showing the response to overdrive ventricular pacing during an atrial tachycardia. The response after pacing is a VAV response. This ladder diagram best describes the response seen in Fig. 56.1. See text for more detail
of activation can be described as atrial–ventricular (VAV; Fig. 56.3). When analyzing the response to ventricular pacing the following steps should be taken: 1. Confirm ventricular capture. 2. Confirm acceleration of atrial rate to pacing rate. 3. Identify last atrial depolarization arising from last ventricular paced beat. 4. Identify the next conducted ventricular beat. 5. Categorize the sequence as VAV or VAAV. In this case, the response immediately after ventricular pacing is VAV, which rules out AT. Because the short VA interval excludes ORT as a mechanism,2 the diagnosis must be AVNRT.
References 1. Knight BP, Zivin A, Souza J, et al. A technique for the rapid diagnosis of atrial tachycardia in the electrophysiology laboratory. J Am Coll Cardiol. 1999;33:775-781. 2. Knight BP, Ebinger M, Oral H, et al. Diagnostic value of tachycardia features and pacing maneuvers during paroxysmal supraventricular tachycardia. J Am Coll Cardiol. 2000;36:574-582.
Case 57 Yash Y. Lokhandwala, Anoop K. Gupta, and Ranjan K. Thakur
Case Summary A 23-year-old man with recurrent episodes of paroxysmal palpitations associated with near-syncope was referred for evaluation. Cardiac evaluation showed a normal heart.
Y.Y. Lokhandwala (*) KEM Hospital, Parel, Mumbai, India e-mail:
[email protected] A.K. Gupta Apollo Hospital, Ahmedabad, India e-mail:
[email protected] R.K. Thakur Thoracic and Cardiovascular Institute, Sparrow Health System, Michigan State University, 405 West Greenlawn, Suite 400, Lansing, MI 48910, USA e-mail:
[email protected]
Figure 57.1 shows the tachycardia induced during EP study. What is the interpretation?
Case Discussion The tachycardia is an orthodromic AVRT mediated by a right-sided accessory pathway because the tachycardia becomes faster with the disappearance of RBBB. This conclusion is based on the fact that if the bundle branch block is on the same side as the accessory pathway, then the tachycardia path is longer during bundle branch block, leading to a slower tachycardia rate during BBB and faster rate with its disappearance (Fig. 57.2). Parahisian pacing was also performed before and after successful ablation (Figs. 57.3 and 57.4).
A. Natale et al. (eds.), Cardiac Electrophysiology, DOI: 10.1007/978-1-84996-390-9_57, © Springer-Verlag London Limited 2011
247
248
Y.Y. Lokhandwala et al.
RUBY HALL CLINIC 2008037 I II III AVR AVL AVF V1 V2 V3 V4 V5 V6 10 mm/mV 25 mm/s
Fig. 57.1 Induced SVT
Version WIN2000/XP : EPTRACER V0.77
Case 57 RUBY HALL CLINIC
249 Version WIN2000/XP : EPTRACER V0.77
2008037 I
AVF
V1
V6
CS34
CS12
HISD
HISP
RFD
10 mm/mV 100 mm/s
Fig. 57.2 Intracardiac signals during transition. The A is earliest in the RF catheter, followed by the His, and still later in the CS. The two programmed PVCs eliminate the RBBB by depolarizing the IVS and
breaking the transseptal linking which was perpetuating the RBBB. The VA time shortens with the disappearance of RBBB
250
Y.Y. Lokhandwala et al.
RUBY HALL CLINIC
Version WIN2000/XP : EPTRACER V0.77
2008037 I
AVF
V1
V6
CS34
CS12
HISD
HISP
RFD
10 mm/mV 300 mm/s
Fig. 57.3 Para-Hisian pacing before RF. The second complex shows His bundle capture evidenced by a narrower QRS complex. The VA time does not shorten with His bundle capture. Thus the VA conduction is via an AP
Case 57 RUBY HALL CLINIC
251 Version WIN2000/XP : EPTRACER V0.77
2008037 I
AVF
V1
V6
CS34
CS12
HISD
HISP
RFD
10 mm/mV 100 mm/s
Fig. 57.4 Para-Hisian pacing after ablation. There was no VA conduction after ablation; with isoproterenol, VA conduction was seen and, therefore, para-Hisian pacing was performed. Second through sixth
complexes show His capture evidenced by narrower complexes. The VA time shortens with His bundle capture suggesting that the VA conduction is via the AV node
Case 58 Roopinder Sandhu, Dimpi Patel, William R. Lewis, and Andrea Natale
Case Presentation A 22-year-old male with no significant medical history presented with recurrent palpitations, dizziness, and chest discomfort. The patient reports having symptoms of shortness of breath and fatigue. He is currently not taking any medications. His 12-Lead ECG (Fig. 58.1) shows ventricular preexcitation. The baseline intracardiac electrograms prior to atrial pacing are shown in Fig. 58.2. The baseline intervals are: PR, 143; QRS, 104; HV, 27. Figures 58.3 and 58.4 show programmed atrial stimulation. The accessory pathway ERP is 330 ms. Figure 58.5 shows that atrial fibrillation was induced with atrial pacing. Figure 58.6 illustrates that the shortest preexcited RR interval is 298 ms or 201 BPM. These two electrograms suggest that the accessory pathway is not a high-risk pathway. Figure 58.7 shows that during ventricular pacing at 700 ms retrograde conduction is occurring over the AV node and not the accessory pathway. Figure 58.8 shows that VA block while pacing at 680 ms. Figure 58.9 reveals that a wide complex tachycardia was inducible. Figure 58.10 shows the 12-lead ECG of the wide QRS tachycardia. R. Sandhu (*) Department of Cardiology, University of Alberta, Walter Mackenzie Center, Suite 2C2, 8440 112 St, Edmonton, AB T6G 2B7, Canada e-mail:
[email protected] D. Patel St. David’s Texas Cardiac Arrhythmia Institute, 1015 E. 32nd St. #516, Austin, TX 78705, USA e-mail:
[email protected] W.R. Lewis Heart and Vascular Center, MetroHealth Medical Center, Case Western Reserve University, 2500 MetroHealth Drive, Suite H322, Cleveland, OH 44109, USA e-mail:
[email protected] A. Natale Texas Cardiac Arrhythmia Institute, St. David’s Medical Center, 1015 East 32nd Street, Suite 516, Austin, TX 78705 e-mail:
[email protected]
Figure 58.11 shows an atrial extrastimulus (pac) during tachycardia. What does the preceding pacing maneuver prove?
Case Discussion The differential diagnosis for patients with a preexcited tachycardia include: atrial arrhythmias, true antidromic AV reciprocating tachycardia, AV nodal reentry with a bystander pathway, or antidromic tachycardia using multiple pathways. The atrial extrastimulus at the time of the refractory His advances the ventricular activation which demonstrates the presence of an accessory pathway and its participation in the tachycardia circuit. The retrograde activation is concentric and a retrograde His prior to atrial activation is shown. The AV node is the likely retrograde conducting pathway. This is not consistent with an atrial tachycardia. We cannot exclude the presence of another pathway (ex concealed accessory pathway not participating in this reentry circuit or a slow conducting retrograde accessory pathway). The optimal ablation site is the region of earliest ventricular activation during maximal preexcitation. Presence of an accessory pathway potential can be guided by a unipolar recording. Figure 58.12 shows three sites that may eliminate the preexcitation. Site three is the most likely to eliminate preexcitation. Figure 58.13 shows application of RF at the preexcitation site and Fig. 58.14 shows that the PR, 176; QRS,100; HV, 50 after ablation. Figure 58.15 shows the post-ablation ECG. In summary, the criteria for diagnosing an antidromic AVRT includes that the QRS configuration during the tachycardia is identical to that obtained during maximal preexcitation; demonstrate the ventricles participate during the tachycardia (1:1 conduction, atrial premature depolarization advances the tachycardia with an identical morphology); the participation of the atria in the tachycardia; and to exclude atrial tachycardia, atrial flutter, or AVNRT using a bystander pathway.
A. Natale et al. (eds.), Cardiac Electrophysiology, DOI: 10.1007/978-1-84996-390-9_58, © Springer-Verlag London Limited 2011
253
254 Fig. 58.1 Baseline ECG
Fig 58.2 Baseline intervals: PR, 143; QRS, 104; HV, 27
R. Sandhu et al.
Baseline ECG
Case 58
Fig. 58.3 Atrial extrastimuli
Fig. 58.4 AP ERP/AV ERP: ³600/330
255
256
Fig. 58.5 Atrial fibrillation
Fig. 58.6 Shortest preexcited RR
R. Sandhu et al.
Case 58
Fig. 58.7 Ventricular pacing: 700 ms
Fig. 58.8 VA block: 680 ms
257
258
Fig. 58.9 Wide complex tachycardia
Fig. 58.10 Antidromic AVRT
R. Sandhu et al.
Case 58
259
Fig. 58.11 Atrial S2 advances tachycardia. CS distal pacing
Site 1
Fig. 58.12 Which site would you select for ablation?
Site 2
Site 3
260
Fig. 58.13 Application of RF energy
Fig. 58.14 Post-ablation intervals: Pr, 176msec; QRS 100 msec; HV 50 msec
R. Sandhu et al.
Case 58
Fig. 58.15 Post-ablation ECG
Bibliography Yee R, Klein GJ, Sharma AD, et al. Tachycardia associated with accessory atrioventricular pathways. In: Zipes DP, Jalife J, eds. Cardiac Electrophysiology. Philadelphia, PA: WB Saunders; 1990:463.
261
Case 59 M. Eyman Mortada, Jasbir S. Sra, and Masood Akhtar
Case Summary
Case Discussion
During ventricular programmed stimulation, the following response was observed (note CS 1,2 is distal) (Fig. 59.1). What are the mechanisms of retrograde conduction seen in this tracing?
To recognize the direction of the atrial activation, it is essential to be acquainted with the normal atrial activation originating from the sinus node “high right atrium.” In this patient, the last atrial beat comes from the high right atrium “sinus node” (Fig. 59.2).
Fig. 59.1 ECG demonstrating the response of program stimulation from the right ventricle with one premature beat after pacing in a steady cycle for six beats. At the top is Lead V1 from surface ECG, followed by the intracardiac tracing. From top to bottom: tracing from the right atrial appendage “HRA,” then proximal coronary sinus to
d istal coronary sinus recording, followed by the His recording, and finally the recording from the right ventricular apex. HRA high right atrium; CS coronary sinus; HISp proximal His; HISm mid His; RVa right ventricular apex
M.E. Mortada (*), J.S. Sra , and M. Akhtar Department of Electrophysiology, Aurora Cardiovascular Services, Aurora Sinai/Aurora St. Luke’s Medical Centers, University of Wisconsin School of Medicine and Public Health, 2801 W. Kinnickinnic River Parkway, #777, Milwaukee, WI 53215, USA e-mail:
[email protected] A. Natale et al. (eds.), Cardiac Electrophysiology, DOI: 10.1007/978-1-84996-390-9_59, © Springer-Verlag London Limited 2011
263
264
M.E. Mortada et al.
Fig. 59.2 Response of program stimulation from the right ventricle with one premature beat after pacing in a steady cycle for 6 beats, with illustration of the activation sequence (arrows). At the top are three surface ECGs including lead I, II, and V1, followed by the intracardiac tracing. From top to bottom: tracing from the right atrial appendage “HRA,” then proximal coronary sinus to distal coronary sinus recording,
followed by the His recording, and finally the recording from the right ventricular apex. HRA high right atrium; CS coronary sinus; HISp proximal His; HISm mid His; HISd distal His; RVa right ventricular apex; H annotation of the His deflection; A annotation of the atrial deflection; V annotation of the ventricular deflection
The first atrial beat appears after a ventricular pacing rhythm, with earliest activity seen in the His, suggestive of retrograde activity over the atrioventricular node (AVN). However, the atrial activity in the second complex is clearly retrograde over the AVN, with earliest activation in the His, followed by the proximal coronary sinus, then the high right atrium. Therefore, the first atrial beat is a fusion beat between the sinus node (the activity in the high right atrium appears prior to the activity in the proximal coronary sinus) and the retrograde activation over the AVN (the activity in the His is earliest). The third atrial beat starts after premature ventricular extrastimuli. Conduction is seen from the distal to the proximal coronary sinus followed by the high right atrium and His, suggestive of retrograde activation over a left free-wall accessory pathway. The fourth atrial beat follows a left bundle QRS complex with the same VA duration and activation sequence as the third atrial complex; hence conduction is retrograde over the left free-wall accessory pathway. The fifth atrial beat has the same activation sequence as the previous two atrial complexes, but the VA duration is shorter. The rationale behind the difference in VA duration is
that the initial electrical activation for the third and fourth ventricular complexes is in the right ventricle apex, which is far from the left free-wall accessory pathway. On the other hand, the initial electrical activation of the fifth ventricular complex is over the normal conduction system to both ventricles simultaneously; thus the electrical activity reaches the accessory pathway rapidly and, subsequently, the VA duration becomes shorter. The origin of the first three ventricular complexes (left bundle branch block morphology) is right ventricular pacing. To understand the fourth ventricular complex, it is first necessary to evaluate the fifth ventricular complex. The fourth atrial complex, as explained previously, has been activated retrogradely through the left free-wall accessory pathway. After activating the atrium, it comes down through the AVN and His-Purkinje system to both ventricles. The AH duration is longer than the baseline sinus rhythm AH (compared to the sixth complex) due to the decremental property of the AVN node. The HV duration in this complex (the fifth complex) is the most accurate duration from the His to the earliest ventricular activation over the normal conduction system. The fourth ventricular complex has left bundle branch block morphology. It is preceded by a His deflection that
Case 59
comes prior to atrial activation. Therefore, this complex did not result from atrial activation. It is either a premature ventricular capture, or it is related to the previous ventricular activity (e.g. bundle branch reentry or repetitive ventricular response). The HV duration is slightly longer than the baseline HV duration when compared to the fifth complex, which is considered the most accurate HV duration over the normal conduction system, as previously mentioned. Hence, the source of this complex is probably bundle branch reentry.
265
Finally, the sixth ventricular complex comes after a sinus beat, which conducts over the AV node, down to the HisPurkinje system, then to both ventricles. The possibility of a ventricular fusion beat occurring over the AV node and the left free-wall accessory pathway in the sixth ventricular complex is excluded due to the fact that the HV duration of this beat is the same as the duration of the most accurate HV duration over the AVN and His-Purkinje system (the fifth beat).
Case 60 Matthew D. Hutchinson
Case Summary A 41-year-old woman without structural heart disease presents with recurrent palpitations due to ventricular bigeminy. She had failed multiple antiarrhythmic medications, including amiodarone. She was referred to our institution for consideration of ablation. Her clinical PVC is shown in Fig. 60.1; the morphology is left bundle, inferior axis with a precordial transition at V3. Based upon the morphology of the PVCs in Fig. 60.1, what is the probable site of origin of the patient’s arrhythmia?
Case Discussion The patient presents with frequent PVCs of a left bundle, inferior axis morphology. When differentiating sites of origin for outflow tract tachycardias, it is most important to consider the precordial transition in light of the anatomical relationships between the RV and LV outflow tracts (RVOT, LVOT). The RVOT is positioned anterior to the LVOT relative to the chest wall; thus the precordial transition for RVOT sites tends to be later than LVOT sites (typically V4 or later).1 Early transitions at V1 or V2 suggest an origin more posterior than the RVOT, most commonly the aortic sinuses of Valsalva (ASOV) or the LVOT. Due to the higher prevalence of RV outflow tract tachycardias, the majority of left bundle inferior axis tachycardias with precordial transition at V3 will originate from the septal (posterior or anterior) aspect of the RV outflow tract. This is because the septal aspect is frequently crescent shaped and both extreme posterior and anterior aspects of the septal surface of the outflow region
M.D. Hutchinson Cardiovascular Division, Department of Medicine, University of Pennsylvania, 3400 Spruce Street, 9 Founders, Philadelphia, PA 19104, USA e-mail:
[email protected]
may be recorded to the left of the midseptum. Approximately 20% of outflow tract tachycardias will originate from left sided structures such as: the ASOV, the LVOT, the coronary arterial or coronary venous musculature, or the epicardial surface. Pacemaps from the ASOV produce very different and characteristic morphologies. The non-coronary cusp sits adjacent to the inter-atrial septum, and a large atrial signal (and occasionally a His bundle defection) is recorded there. The right cusp is situated anterior and rightward to the left cusp, producing morphologies with later precordial transitions (usually at V2 or V3) and a larger R wave in lead I. Pacing more leftward within the right cusp as well as within the left cusp produces a progressively more rightward frontal plane axis, a more prominent rS complex in lead I, and an earlier precordial transition (at V1 or V2). Tachycardias originating from between the right and left cusps have a characteristic notching in the S wave in V1. The most characteristic feature of outflow-type tachycardias originating from epicardial structures is delayed initial activation in the precordial leads. Daniels and colleagues describe the maximum deflection index as the ratio of the longest time from QRS onset to maximal QRS deflection (either positive or negative) in any precordial lead to the total QRS duration.2 Ratios of ³0.55 were predictive of epicardial origin with high sensitivity and specificity. Bazan and colleagues further described the presence of Q wave in lead I and the absence of Q waves in the inferior leads as highly predictive of epicardial origin from the basal superior LV.3 Thus for our patient, the precordial transition at V3 presents a variety of possibilities for the site of origin.4 A combination of activation and pacemapping is utilized in these cases. Activation times were later than the onset of the QRS in the RVOT, and the pacemap was a poor match (Fig. 60.2). Detailed mapping of the ASOV was performed which revealed an excellent pacemap from the left cusp with an activation time 31 ms pre-QRS; however ablation at this site was unsuccessful (Fig. 60.2). We then performed mapping of the distal coronary sinus near the origin of the anterior interventricular vein (AIV); the activation time was also 31 ms pre-QRS, and the pacemap was a near perfect match
A. Natale et al. (eds.), Cardiac Electrophysiology, DOI: 10.1007/978-1-84996-390-9_60, © Springer-Verlag London Limited 2011
267
268 Fig. 60.1 12-lead ECG revealing left bundle, inferior axis PVCs in a bigeminal pattern
M.D. Hutchinson I II III aVR aVL aVF V1 V2 V3 V4 V5 V6
a
b LCC activation
Clinical
I
I
II
II
III
RVOT
RCC
LCC
III
aVR aVL aVF
aVL
V1
aVF
V2
V1
V3 V4
Fig. 60.2 (a) Clinical PVC with activation time 31 ms pre-QRS as recorded from the left coronary cusp (LCC). (b) Clinical PVC with pacemaps obtained from the RV outflow tract (RVOT), the right and the left coronary cusps (RCC, LCC)
aVR
V5 V6
V2 V3 V4
ABld
V5
ABlp
V6
(Fig. 60.3). Ablation in this location abolished the patient’s PVCs. In summary, the 12-lead ECG provides important information about the site of origin of outflow tract ventricular
ectopy. However, due to individual variability in the anatomical relationships between these complex structures, detailed activation and pacemapping are of critical importance when ablating outflow tract ectopy.
Case 60 Fig. 60.3 (a) Clinical PVC with activation time 31 ms pre-QRS as recorded from the proximal portion of the anterior interventricular vein (AIV). (b) Clinical PVC with pacemap obtained from the AIV. (c) Fluoroscopy obtained in the right and left anterior oblique projections demonstrating the proximity of two catheters placed in the LCC and proximal AIV
269
a
b
c
CS activation
GCV/AIV
I
I
II
II
III aVR aVL
III
aVL
V1
aVF
V2
V1
V4 V5 V6
RAO
aVR
aVF
V3
Clinical
V2 V3
LAO
V4
Abld
V5
Ablp
V6
References 1. Dixit S, Gerstenfeld EP, Callans DJ, Marchlinski FE. Electro cardiographic patterns of superior right ventricular outflow tract tachycardias: distinguishing septal and free-wall sites of origin. J Cardiovasc Electrophysiol. 2003;14(1):1-7. 2. Daniels DV, Lu YY, Morton JB, et al. Idiopathic epicardial left ventricular tachycardia originating remote from the sinus of Valsalva: electrophysiological characteristics, catheter ablation, and identification from the 12-lead electrocardiogram. Circulation. 2006; 113(13):1659-1666.
3. Bazan V, Gerstenfeld EP, Garcia FC, et al. Site-specific twelve-lead ECG features to identify an epicardial origin for left ventricular tachycardia in the absence of myocardial infarction. Heart Rhythm. 2007;4(11):1403-1410. 4. Tanner H, Hindricks G, Schirdewahn P, et al. Outflow tract tachycardia with R/S transition in lead V3: six different anatomic approaches for successful ablation. J Am Coll Cardiol. 2005;45(3): 418-423.
Case 61 Ronald Lo, Henry H. Hsia, and Amin Al-Ahmad
Case Summary
Case Discussion
A 55-year-old woman with no significant past medical history has been complaining of palpitations for the past 3 years. These symptoms are associated with fatigue, activity intolerance, chest discomfort, and lightheadness. A Holter monitor was ordered which demonstrated greater than 20,000 PVCs per day. These accounted for approximately 25% of her daily heart beats. Her electrocardiogram is shown in Fig. 61.1. Her echocardiogram demonstrated an ejection fraction of approximately 45% with normal valves and cardiac chamber sizes. She was tried on beta blockers without any effect with respect to symptoms. Where is the origin of her premature ventricular contractions?
Analysis of the electrocardiogram demonstrates ventricular bigeminy with left bundle left inferior axis morphology PVCs. This suggested a possible right ventricular outflow tract origin of the PVCs; however, the early transition raises the possibility of a left ventricular outflow tract or aortic cusp site of origin. Initial mapping of the right ventricle and the right ventricular outflow tract did not locate any points of earliest activation that were earlier than ventricular activation. Mapping was then performed in the left ventricle and the aortic root along with the aortic cusps. Careful mapping using CARTO demonstrated a site between the left and right aortic cusps that was −22 ms presystolic to the earliest ventricular electrogram. The unipolar electrogram also demonstrated a QS signal as shown in Fig. 61.2. Pace mapping was also performed in the areas around the aortic cusps, with the best pace mapped site being in between the right and the left aortic cusps. Further analysis of the electrocardiogram demonstrated a qrS pattern in leads V1–V3, which appear to be due to a site of activation in between the right and left coronary cusps. The activation pattern is due to direction of the propagating wave from the right and left coronary cusps as a q wave in lead V1, and the anterior activation pattern from the RVOT to the aortic root as an r wave in V1. The remainder of the ventricular activation produced a large S wave resulting in the qrS pattern seen on the electrocardiogram. Catheter ablation in this region (Fig. 61.3) produced successful termination of the spontaneous PVCs and repeat Holter monitors afterward did not demonstrate any PVCs consistent with the ablated morphology.
R. Lo (*) Riverside Electrophysiology, 4000 14th Street, Suite 209, Riverside, CA 92501, USA e-mail:
[email protected] H.H. Hsia Department of Cardiovascular Medicine, Stanford University, 300 Pasteur Drive, H2146, Stanford, CA 94305, USA e-mail:
[email protected] A. Al-Ahmad School of Medicine, Stanford University, 300 Pasteur Drive, H-2146, Stanford, CA 94305, USA e-mail:
[email protected]
A. Natale et al. (eds.), Cardiac Electrophysiology, DOI: 10.1007/978-1-84996-390-9_61, © Springer-Verlag London Limited 2011
271
272
R. Lo et al.
Fig. 61.1 12-lead ECG with ventricular bigeminy with left bundle left inferior PVCs
Unipolar and bipolar activation mapping
43 ms
LAT 2-Map > 134 Points
AoV
−114 ms R
Fig. 61.2 Left ventricular unipolar and bipolar activation mapping demonstrating earliest activation in the region between the left and right coronary cusp and the earliest activation at that site −22 ms presystolic
L 0.70 cm
Case 61
273 Aortogram
Successful ablation site
Bibliography Yamada T, Yoshida N, Murakami Y, et al. Electrocardiographic characteristics of ventricular arrhythmias originating from the junction of the left and right coronary sinuses of Valsalva in the aorta: the activation pattern as a rationale for the electrocardiographic characteristics. Heart Rhythm. 2008;5:184-192.
LAO
Fig. 61.3 Successful ablation site of premature ventricular contraction in between the left and right coronary cusp
Case 62 Richard H. Hongo and Andrea Natale
Case Summary The patient is a 57-year-old woman with newly diagnosed nonischemic dilated cardiomyopathy (LVEF 25–30%) and >11,000 uniform VPCs during a 24-h Holter monitor. She presented for electrophysiology study for ablation of the VPC focus (Fig. 62.1) and to assess for inducible sustained ventricular tachycardia. 3-D electroanatomic activation mapping was performed with a 4-mm Navistar RMT catheter utilizing the CARTO and Stereotaxis remote magnetic navigation systems. Earliest activation (0 ms presystolic) of the recurring VPCs within the right heart was localized to the anteroseptal RVOT, just below the pulmonary valve. Pacemapping was also performed and an 11/12-lead morphology match was achieved at the same site as the earliest activation. Ablation was performed at this site with the 4-mm RMT catheter with temperature and power limited to 52°C and 50 W, respectively. Despite multiple ablations (Fig. 62.2) achieving adequate temperature and power, there were no flurries of ventricular beats during ablations and the VPCs continued to recur. What is the most appropriate next step? Should this patient receive an ICD?
Case Discussion Prominent R waves in precordial leads V1 and V2 is most consistent with an LVOT VPC focus. Right heart electroanatomic mapping reveals earliest activation at the anteroseptal aspect of the RVOT just below the pulmonary valve. The earliest site, however, is on time with, but does not precede, the
R.H. Hongo Sutter Pacific Medical Foundation, California Pacific Medical Center, 2100 Webster Street, Suite 521, San Francisco, CA 94115, USA e-mail:
[email protected]
onset of the P wave. Pacemapping at this site is only an 11-out-of-12 leads match with the VPC morphology. Despite the less than ideal activation time and pacemapping match, attempting ablation with the 4-mm catheter is appropriate before proceeding with the higher risk left-sided mapping and ablation. Once, however, the lack of VPC flurries and the persistence of VPCs are apparent, further ablation with higher powered catheters is unlikely to be successful, and the next most appropriate step is to proceed with mapping of the LVOT; the LVOT is more approachable than the epicardium, and the VPC ECG morphology is more suggestive of an LVOT focus. Following transseptal puncture, 3-D activation mapping of the recurring VPCs was performed with remote magnetic navigation within the LVOT. A region of early activation was found just below the aortic valve, along the posteroseptal aspect of the LVOT, but was no earlier than the earliest site in the RVOT (0 ms presystolic). Within this region, however, there was a single site that demonstrated presystolic (−20 ms) activation (Fig. 62.3). The first ablation at this site resulted in a brief flurry of VPCs that was followed by complete cessation of VPCs (Fig. 62.4). Several ablations were performed adjacent to the first ablation, targeting fractioned electrogram signal. There were no recurrent VPCs observed overnight on telemetry monitoring, and the VPC focus has not recurred after 6 months. The LVEF has remained depressed (<30%), however, and she has undergone insertion of an implantable cardioverter-defibrillator for primary prevention of sudden cardiac arrest. Elimination of the VPC focus by ablation should precede insertion of an implantable cardioverter-defibrillator in this patient because there is a reasonable possibility that the newly diagnosed dilated cardiomyopathy is caused by the excessive burden of VPCs. Persistence of systolic dysfunction (LVEF of 35% or less) for over 3 months, however, prompted her to be reconsidered for implantable cardioverter-defibrillator therapy.
A. Natale (*) Texas Cardiac Arrhythmia Institute, St. David’s Medical Center, 1015 East 32rd Street, Suite 516, Austin, TX 78705, USA e-mail:
[email protected] A. Natale et al. (eds.), Cardiac Electrophysiology, DOI: 10.1007/978-1-84996-390-9_62, © Springer-Verlag London Limited 2011
275
276
Fig. 62.1 12 lead ECG showing morphology of VPC
Fig. 62.2 CARTO map showing ablation lesions in the RVOT
R.H. Hongo and A. Natale
Case 62 Fig. 62.3 Site of earliest activation in the LVOT
Fig. 62.4 CARTO map showing site of successful ablation
277
Case 63 David J. Callans
Case Summary A 46-year-old woman with a history of renal failure status post failed transplant received an ICD after an episode of ventricular fibrillation (VF). Evaluation at that time revealed normal LV function and no coronary artery disease. She was well for 6 months, but then presented with syncope and ICD shocks. Interrogation of the ICD showed 60 episodes detected in the VF zone over the previous 4 days, most of them nonsustained, but four resulting in shock delivery. Telemetry monitoring showed frequent episodes of nonsustained polymorphic VT. She was transferred to our hospital for further evaluation. 12-lead ECG monitoring at this hospital showed frequent PVCs with several different right bundle, left superior axis morphologies which frequently started episodes of
nonsustained and sustained polymorphic VT (Fig. 63.1). What is the best strategy for ablation in this patient?
Case Discussion In the electrophysiology laboratory, she had multiple morphologies of both left and right bundle PVCs, all of which were preceded by an apparent Purkinje potential (Fig. 63.2). Catheter ablation was targeted at elimination of Purkinje potentials in the area around earliest onset of PVCs. Afterward, no further spontaneous PVCs were noted despite isoproterenol infusion (Fig. 63.3). The patient has been well, without recurrent syncope or ICD shocks over 6 months follow-up.
D.J. Callans (*) Department of Cardiology, University of Pennsylvania, 3400 Spruce Street, Philadelphia, PA 19104, USA e-mail:
[email protected] A. Natale et al. (eds.), Cardiac Electrophysiology, DOI: 10.1007/978-1-84996-390-9_63, © Springer-Verlag London Limited 2011
279
280
D.J. Callans
Fig. 63.1 Frequent PVCs, with slightly different right bundle, left superior axis morphologies, often causing runs of nonsustained and sustained polymorphic ventricular tachycardia
Case 63 Fig. 63.2 Left and right bundle PVCs with intracardiac recordings from a mapping catheter (Carto P, Carto D), and RV apical catheter. Note that PVCs were preceded by a sharp potential, presumably from the Purkinje system
281
I II III aVR aVL aVF V1 V2 V3 V4 V5 V6 Carto P Carto D RVa
9:55:26 AM 10:10:30 AM
Fig. 63.3 Left ventricular voltage mapping in the RAO and LAO projections. The voltage maps are essentially normal. Although some abnormalities are suggested at the junction of the inferior wall and the septum,
the electrograms recorded in this region were normal in morphology. Red dots represent individual ablation lesions which were targeted by finding Purkinje-like potentials in sinus rhythm and before PVCs
Case 64 J. David Burkhardt, Dimpi Patel, and Andrea Natale
Case Summary A 48-year-old Hispanic male presents with a history of dyspnea on exertion that has been present for two months. He has been evaluated by an internist who discovered on monitoring that the patient had frequent PVCs and runs of monomorphic ventricular tachycardia. He had an echocardiogram that revealed an EF of 25% without segmental wall motion abnormality. A nuclear stress test revealed no scar or ischemia. A 24-h Holter monitor revealed frequent runs of ventricular tachycardia at a rate of 150 BPM up to 12 beats and 45,000 PVCs. All of the PVCs and VT are of the same morphology. An EKG is included below (Fig. 64.1). Laboratory tests including TSH were unremarkable. He denies syncope or recent illness.
What is the diagnosis? Where is the origin of the VPC? What is the best strategy for management of this patient?
Case Discussion The case is consistent with a PVC-induced cardiomyopathy. The high frequency of PVCs is the likely culprit behind the symptoms and left ventricular dysfunction. The location appears to be LVOT and specifically aortic cusp. The features include high amplitude R waves in II, III, and aVf, as well as the early transition and notched upstroke of the R wave in lead V6. With a normal stress test and typical location of “normal heart” PVCs further evaluation is not necessary. The LV
J.D. Burkhardt (*) and A. Natale Texas Cardiac Arrhythmia Institute, 1015 E. 32nd Street, Suite 505, Austin, TX 78705 e-mail:
[email protected];
[email protected] D. Patel St. David’s Texas Cardiac Arrhythmia Institute, 1015 E. 32nd St. #516, Austin, TX 78705 e-mail:
[email protected]
A. Natale et al. (eds.), Cardiac Electrophysiology, DOI: 10.1007/978-1-84996-390-9_64, © Springer-Verlag London Limited 2011
283
284
dysfunction and congestive heart failure symptoms will likely resolve with elimination of the PVC. An ICD is not indicated since this is likely a reversible condition. The patient was tried on a medication; however there was no decrease in the number of VPCs. Ablation is the next reasonable step to give the patient the best chance of full
25mm/s 10mm/mV 150Hz 005E 12SL 250 CID:19
Fig. 64.1 12-lead ECG showing the VPC
Fig. 64.2 Intracardiac echo helps with the anatomy of the aortic valve region
J.D. Burkhardt et al.
recovery. ICE may be a useful aide in this endeavor (Fig. 64.2). It is difficult to appreciate the anatomy of the aortic valve using flouroscopy or electroanatomic mapping alone. Pacemapping at the aortic cusp revealed a perfect pacemap (Fig. 64.3). This location was targeted for ablation and was successful in eliminating the VPC.
EID:23 EDT: 13:20 06-FEB-2006 ORDER: 256788740 ACCOUNT: 0999
Case 64
285 Pacemap
Successful site I
I
100 ms II
II
III
III
aVR
aVR
aVL
aVL
aVF
aVF
V1
V1
S
V2
V2
V3
V3
V4
V4
V5
V5
V6
V6
ABL 1,2 Abl uni
ABL 1,2 Abl uni
49 msec
ABL 3,4
5:58:05 PM
CardioLab v6.0 GF Medical Systems
Fig. 64.3 Spontaneous VPC vs. pacemap
ABL 3,4
5:42:29 PM
CardioLab v6.0
Case 65 Matthew D. Hutchinson
Case Presentation A 36-year-old woman was referred to our institution for repetitive, polymorphic ventricular tachycardia and ventricular fibrillation (PMVT, VF). A comprehensive cardiovascular evaluation revealed normal left ventricular structure and function. The baseline QT interval was normal and there was no family history of sudden cardiac death. She had an ICD placed due to recurrent presyncope with these episodes. Intracardiac electrograms and telemetric monitoring revealed recurrent PMVT episodes which were initiated with shortcoupled PVCs of uniform morphology. She had failed multiple antiarrhythmic agents, as well as a previous ablation attempt. The 12-lead morphology of her initiating PVC is given in Fig. 65.1. What are the common sites of origin for idiopathic PVCs initiating PMVT/VF? What is the likely site of origin for the patient’s inciting PVC, and what strategy is most effective in mapping such PVCs?
Case Discussion This case highlights the uncommon clinical entity of idiopathic PMVT/VF. These patients have normal corrected QT intervals and lack the characteristic findings of catecholaminergic polymorphic ventricular tachycardia on exercise testing. The largest experience of 27 patients by Haissaguerre
and colleagues found that the inciting PVC originated either in the right ventricular outflow tract (15%) or the Purkinje network (85%).1 Those beats originating in the LV Purkinje network were characterized by a relatively narrow QRS duration (115 ± 11 ms) compared to those from the RV (143 ± 10 ms). This patient’s PVC morphology is narrow (ms) and has a right bundle superior axis which is consistent with an origin from the left ventricular Purkinje network. In the aforementioned Haissaguerre series, the earliest Purkinje deflections for LV ectopy preceded the local ventricular activation by an average of 46 ± 29 ms (range 10–150 ms).The most common ablation strategy is to identify the earliest presystolic Purkinje activity; this is most easily accomplished via electroanatomical mapping of the Purkinje pre-potentials. Figure 65.2 demonstrates mapping along the LV septum in our patient. Note the dramatic prematurity seen in the conducted ectopic beats. Also shown in Fig. 65.2 is a more premature Purkinje depolarization which is not conducted to the local myocardium. Figure 65.3 depicts an episode of NSVT with early Purkinje pre-potentials driving the tachycardia. Ablation at this site abolished the clinical PVC; however a second PVC morphology was then seen. This PVC was mapped more apically and was noted to have extremely early Purkinje activation in this region (Fig. 65.4). Ablation at this site abolished the patient’s PVCs. The majority of these patients have an excellent long-term result after ablation, and 89% of patients in the Haissaguerre group were arrhythmia free off of antiarrhythmic drugs at a mean follow-up of 2 years.
M.D. Hutchinson Cardiovascular Division, Department of Medicine, University of Pennsylvania, 3400 Spruce Street, 9 Founders, Philadelphia, PA 19104, USA e-mail:
[email protected] A. Natale et al. (eds.), Cardiac Electrophysiology, DOI: 10.1007/978-1-84996-390-9_65, © Springer-Verlag London Limited 2011
287
288 Fig. 65.1 The patient’s dominant clinical PVC is shown in a trigeminal pattern in sinus rhythm. The ventricular ectopy is narrow, and has a right bundle superior axis
M.D. Hutchinson I II III aVR aVL aVF V1 V2 V3 V4 V5 V6
Fig. 65.2 Intracardiac recordings obtained during LV activation mapping of the clinical PVC. Bipolar recordings are shown from the His bundle position (His 1–5) and LV septum (LV 1–5). There are sharp early potentials (asterisks) on the ablation catheter poles (Abl) recorded after the first sinus beat which represent Purkinje potentials which do not activate the LV myocardium. After the second sinus beat, there are later-coupled Purkinje potentials which do conduct to the myocardium and produce the clinical PVC
I II III aVR aVL aVF V1 V2 V3 V4 V5 V6 LV5 LV4 LV3 LV2 LV1 Abld Ablp His5 His4 His3 His2 His1 RVa
Case 65
a
289
b I
II III aVR
I II III aVR aVL aVF V1 V2 V3
aVL
V4 V5
aVF
V6 LV5
V1
LV4 LV3
V2
LV2 LV1
V3 V4 V5 V6
Abld Ablp His5 His4 His3 His2 His1 RVa
Fig. 65.3 (a) 12-lead ECG demonstrating a typical episode of NSVT initiated by the clinical PVC. (b) Intracardiac recordings from the episode of NSVT pictured in panel A. Note the premature Purkinje potentials (asterisks) preceding both the clinical PVC and each beat of nonsustained VT
I II III aVR aVL aVF V1 V2 V3 V4 V5 V6 LV5 LV4 LV3 LV2 LV1 Abld Ablp His5 His4 His3 His2 His1 RVa
Fig. 65.4 Intracardiac recordings of a second PVC which was seen after ablation of the clinical PVC. Note the dramatically premature Purkinje potentials noted on the ablation catheter (asterisks). This PVC was targeted at the anteroapical LV
Reference 1. Haissaguerre M, Shoda M, Jais P, et al. Mapping and ablation of idiopathic ventricular fibrillation. Circulation. 2002;106(8): 962-967.
Case 66 Matthew D. Hutchinson
Case Presentation A 73-year-old man with a longstanding nonischemic cardiomyopathy and an implantable defibrillator placed for secondary prevention of sudden death was referred for increasingly frequent ICD shocks due to rapid VT. Interrogation of his ICD revealed three distinct intracardiac morphologies. He was referred for EP study and voltage mapping, at which time he had a single VT morphology reproducibly induced with programmed stimulation (Fig. 66.1). The intracardiac electrograms of this induced tachycardia matched one of the patient’s dominant clinical arrhythmias. Biventricular voltage maps were normal with the exception of a small area of attenuated electrograms at
Sinus rhythm
a
I
Clinical VT
b
II
I II
III
III
aVR
aVR
aVL
aVL
aVF
aVF
V1 V2 V3 V4 V5 V6
the lateral mitral annulus (Fig. 66.2). Endocardial pacemaps for the clinical VT were poor and activation times late during VT; thus no ablation was performed. The patient was taken back to the EP lab on a later date for an epicardial approach. Percutaneous epicardial access was obtained using the Sosa technique. Epicardial bipolar voltage mapping was performed which revealed a large, confluent area of low voltage along the lateral LV extending from the mitral annulus to the mid-cavity (Fig. 66.3). Despite aggressive programmed stimulation with up to triple extrastimuli and isoproterenol infusion, only polymorphic VT was inducible. What features of the clinical tachycardia suggest an epicardial site of origin? Based upon the information available, describe the most effective ablation strategy for this patient.
200 ms
V1 V2 V3 V4 V5 V6
Fig. 66.1 (a) Sinus rhythm QRS morphology. (b) Clinical VT morphology
M.D. Hutchinson Cardiovascular Division, Department of Medicine, University of Pennsylvania, 3400 Spruce Street, 9 Founders, Philadelphia, PA 19104, USA e-mail:
[email protected]
Case Discussion There are several ECG criteria for predicting LV epicardial VT origin in patients with nonischemic dilated cardiomyopathy. Based upon the ECG morphology of the VT in Fig. 66.1, one would expect a mid-inferolateral exit site. Bazan and colleagues prospectively examined several criteria to differentiate basal and apical inferior epicardial tachycardias, and found the following to have significant predictive value: the presence of Q waves in the inferior leads; a pseudodelta wave ³34 ms (measured from onset of QRS to earliest rapid deflection in precordial leads); and the shortest precordial RS complex ³121 ms (measured from onset of precordial QRS to earliest nadir of S wave). All of the above criteria are met for our patient’s tachycardia (Fig. 66.4). The most effective method of localizing the site for VT ablation is entrainment mapping; however this is not possible in many patients due either to arrhythmia noninducibility or to hemodynamic compromise during VT. In this case in which no VT is inducible, an alternate strategy utilizing substrate-based ablation is necessary. The technique of substrate-based ablation was previously described by Marchlinski
A. Natale et al. (eds.), Cardiac Electrophysiology, DOI: 10.1007/978-1-84996-390-9_66, © Springer-Verlag London Limited 2011
291
292
M.D. Hutchinson
Fig. 66.2 Bipolar LV and RV endocardial voltage maps in LAO (left) and PA (right) projections. There is a limited region of low voltage present at the basal anterolateral LV
>1.5 mV
Limited endocardial voltage abnormality
R
LAO
Lateral LV epicardium
PA
>1.0 mV
Sinus rhythm
a
I II
Base
R
III aVR
aVL
aVL
aVF
aVF
V3 Fractionated/late potentials
V4 <0.5 mV
V5 V6
Fig. 66.3 Bipolar LV epicardial voltage map with a large region of low voltage involving the lateral LV extending from the base to the midwall. Areas with abnormal electrograms (late or fractionated potentials) are labeled with a gray tag
and colleagues, and requires a detailed knowledge of the patient’s substrate.1 High-density voltage mapping to define the area of bipolar voltage attenuation is performed; bipolar signals <1.5 mV on the endocardium and <1.0 mV on the epicardium are characterized as abnormal. Sites with abnormal electrogram characteristics including fractionated, late, and split potentials are given anatomical tags; these areas represent regions of surviving myocardium within the scar which exhibit abnormal conduction properties and may participate in tachycardia circuits. When information regarding the patient’s spontaneous or induced arrhythmias is available, the approximate exit sites for these tachycardias are localized by pacing at the border zone of the bipolar voltage abnormality. Linear lesions sets are designed incorporating the individual VT exit sites identified by pacemapping and extending through the border zone into the dense scar
II
aVR
V2
200 ms
I
III
V1
Apex
b
Clinical VT PD = 52 ms
<0.5 mV
V1 V2 V3 V4 V5 V6 RS = 164 ms
Fig. 66.4 (a) The sinus rhythm QRS complex with arrows indicating the absence of Q waves in the inferior leads. (b) The patient’s VT morphology demonstrates prominent QS waves in the inferior leads. The pseudodelta wave measured in the precordial leads is 52 ms; the shortest RS complex (measured in V4) is 164 ms. These features suggest an epicardial VT exit site. See text for discussion
(defined by a bipolar voltage <0.5 mV). Whenever possible, targeting areas of markedly abnormal (late and split) electrograms is also performed. In this case, we have detailed information about the patient’s substrate from electroanatomical mapping. The gray dots in Fig. 66.3 represent abnormal bipolar electrograms within the low voltage region. We also have a 12-lead morphology of one of the patient’s clinical arrhythmias which was induced during the initial EP study. Pacemapping of this VT morphology was performed, with the best match from the inferior scar border (Fig. 66.5). Linear lesions were applied extending from the region of the best pacemap for the clinical VT into the dense scar. Additional linear lesions were given to transect regions
Case 66 Fig. 66.5 Comparison of the best pacemap morphology to the clinical VT. The pacing site is indicated by the star on the electroanatomical map
293 VT pacemap
Clinical VT
>1.0 mV
R
<0.5 mV Fractionated/late potentials VT pacemap site
a
b >1.0 mV
Fig. 66.6 (a) Lesion set design which incorporates an initial linear lesion (yellow dashed line) through the VT exit region localized by pacemapping and extending into dense scar. The other linear lesions (white dashed lines) transect regions with abnormal electrogram characteristics. (b) Final electroanatomical map with ablation lesions represented as red tags
R
R
<0.5 mV Targeting clinical VT and late potentials
with abnormal electrograms as shown in Fig. 66.6. The usual procedural endpoint of arrhythmia noninducibility is not applicable in this case; however when properly executed, substrate-based ablation strategies are as effective as entrainment-based strategies in achieving long-term arrhythmia suppression.
Reference 1. Marchlinski FE, Callans DJ, Gottlieb CD, Zado E. Linear ablation lesions for control of unmappable ventricular tachycardia in patients with ischemic and nonischemic cardiomyopathy. Circulation. 2000;101(11):1288-1296.
Case 67 Matthew D. Hutchinson
Case Presentation A 39-year-old man with familial hypercholesterolemia, prior coronary endarterectomy, mechanical aortic valve replacement, and mitral valve annuloplasty presented with symptomatic recurrent monomorphic ventricular tachycardia (MMVT). His sinus rhythm ECG indicated prior inferior infarction; his left ventricular ejection fraction was 50% with mild concentric hypertrophy. He was noted to have frequent monomorphic PVCs (right bundle, left superior axis) which appeared to initiate the episodes of MMVT (Fig. 67.1). He was then brought to the EP lab for mapping and ablation. The left ventricle was accessed by transseptal approach due to the mechanical aortic prosthesis. His bipolar voltage map revealed patchy endocardial low voltage regions involving the basal and mid inferior walls. The clinical VT (right bundle, left superior axis morphology; CL 280 ms) was reliably initiated by a distinct unifocal PVC (Fig. 67.2). The blood pressure was marginal during VT, and the patient was quite symptomatic. Based on the ECG morphologies, where are the sites of origin/exit for both the PVC and the sustained VT? What would be your ablation strategy for this VT; how can the frequent PVCs be incorporated into this strategy?
Case Discussion This patient likely has an ischemic substrate for ventricular tachycardia based on the history of premature coronary artery disease, ECG evidence of prior inferior infarction, and evidence of patchy endocardial scar by voltage mapping in the typical inferior regional distribution. Importantly, there may
be more endocardial scar than seen here with the typical CARTO color settings used (<1.5 mV), as underlying myocardial hypertrophy with larger baseline local potentials can mask superficial scar. Adjustment of the color range may be necessary at times to highlight endocardial scar in the setting of ventricular hypertrophy. Figures 67.1 and 67.2 demonstrate frequent monomorphic PVCs which appear to originate from the mid inferior wall (apical-septal aspect of this area) based on the ECG. Interestingly, the clinical VT which is initiated by this PVC has an overall similar morphology, but exits more basally (later precordial transition) and slightly further from the septum (wider QRS). The close relationship between single PVCs and reentrant VT arising from the area of scar in ischemic substrate has been described by Bogun and colleagues.1 When the PVC is frequent, it can serve as a target for mapping and ablating a potentially arrhythmogenic area of the ischemic scar, especially if the clinical VT is not sustained or tolerated to allow for more detailed entrainment mapping. In this case we performed activation and pacemapping of the PVC, leading us to focus on the area just basal and septal to the posteromedial papillary muscle. While mapping here, the clinical VT was initiated by a catheter-induced PVC, and large mid-diastolic potentials were seen at this site. Entrainment was quickly performed while the VT was still tolerated, and the response suggested that this was an isthmus site. RF delivered here terminated the VT. After termination, we performed pacemapping from the distal and proximal bipoles of the ablation catheter (spacing configuration 2 mm–5 mm–2 mm). Interestingly, pacing at threshold from the more apical bipole produced a pace match for the spontaneous PVC (Fig. 67.3), and pacing at threshold from the more basal bipole produced an excellent pacemap match for the clinical VT (Fig. 67.4). Several additional lesions in this area significantly raised the pacing threshold, eradicated the PVCs, and rendered the VT noninducible.
M.D. Hutchinson Cardiovascular Division, Department of Medicine, University of Pennsylvania, 3400 Spruce Street, 9 Founders, Philadelphia, PA 19104, USA e-mail:
[email protected] A. Natale et al. (eds.), Cardiac Electrophysiology, DOI: 10.1007/978-1-84996-390-9_67, © Springer-Verlag London Limited 2011
295
296 Fig. 67.1 12-lead surface ECG obtained at rest outside of the electrophysiology lab. Note baseline right bundle-branch block as well as Q waves in the inferior leads (arrows). There are frequent spontaneous monomorphic PVCs (stars), the last one initiating a run of monomorphic ventricular tachycardia
M.D. Hutchinson 800 ms I
II III aVR aVL aVF V1 V2
V3 V4 V5 V6
200 ms I II III aVR aVL aVF V1 V2
Fig. 67.2 12-lead surface ECG obtained in the electrophysiology lab at start of the ablation procedure. Again the sinus beat is followed by the same PVC (star) which initiates the patient’s clinical VT
V3 V4 V5 V6
Case 67 Fig. 67.3 Bipolar voltage map of the left ventricle (inferior view; purple color denotes voltage > 1.5 mV, red color denotes voltage < 0.5 mV) with site (star) of threshold pacemap compared with spontaneous PVC
297 PVC
Pacemap I
I II
II
>1.5 mV Septum
III aVR
aVR
aVL
aVL
aVF
aVF
V1
V1
V2
V2
V3
V3
V4
V4
V5
V5
V6
V6
<0.5 mV Base
III
Apex
Posteromedial papillary muscle
Pacemap
>1.5 mV Septum
<0.5 mV Base
Fig. 67.4 Bipolar voltage map of the left ventricle (inferior view; purple color denotes voltage > 1.5 mV, red color denotes voltage < 0.5 mV) with site (star) of threshold pacemap compared with clinical VT
Reference 1. Bogun F, Crawford T, Chalfoun N, et al. Relationship of frequent postinfarction premature ventricular complexes to the reentry circuit of scar-related ventricular tachycardia. Heart Rhythm. 2008; 5(3):367-374.
Apex
Posteromedial papillary muscle
VT
I
I
II
II
III
III
aVR
aVR
aVL
aVL
aVF
aVF
V1
V1
V2
V2
V3
V3
V4
V4
V5
V5
V6
V6
Case 68 Matthew D. Hutchinson
Case Presentation A 54-year-old man with CAD and previous inferior infarction and prophylactic single chamber ICD implantation was referred for recurrent, symptomatic monomorphic ventricular tachycardia and ICD shocks despite amiodarone. He was also noted to have frequent, monomorphic PVCs with an intracardiac morphology matching his clinical tachycardia. He subsequently underwent an electrophysiology study during which the clinical VT was induced and characterized with activation mapping. Activation times for the clinical PVCs of up to 30 ms pre-QRS were recorded from the right ventricular outflow tract; however ablation in this area was acutely unsuccessful. The patient was subsequently referred to our institution for a repeat ablation procedure. The patient’s admission ECG is shown in Fig. 68.1. Based upon the morphology of the PVCs on the presenting ECG (Fig. 68.1), what is the differential diagnosis for the site of origin of the patient’s arrhythmia? What are important laboratory considerations when mapping tachycardias of this type?
Case Discussion The patient presents with frequent PVCs of a left bundle, inferior axis morphology. This combination of features most commonly represents a right ventricular outflow tract origin. As discussed previously, however, when the ectopy transition is at V3 it is important to left-sided sites of origin (the aortic sinuses of valsalva, the left ventricular outflow tract, the coronary arterial or venous musculature, or the epicardial surface).
Interestingly, the patient’s PVC recorded in the EP lab demonstrated a different precordial morphology with an earlier transition at V2 (Fig. 68.2). The difference in morphology is due to erroneous placement of the precordial leads in Fig. 68.1. Leads V1 and V2 should be placed at the fourth intercostal space along the sternal borders. In this case, the lead positioning for V1 and V2 in Fig. 68.1 was most likely at the third interspace (note the similarity between leads V3 on the initial ECG and V2 on the adjacent laboratory recording). It is critically important to check the lead placement whenever using the ECG to predict tachycardia site of origin. In this case, the large R wave in lead V2 and biphasic complex in lead I suggest a more posterior site of origin from the aortic sinuses of valsalva. During the procedure, the patient had very infrequent PVCs and was noninducible for VT with programmed stimulation. The administration of sedative medications can have important implications for arrhythmia inducibility. Moreover the administration of local anesthetic agents subcutaneously can result in measurable serum concentrations which can further suppress arrhythmias. On review it was noted that the patient had been administered intravenous benzodiazepines for sedation prior to sheath insertion. After administration of isoproterenol 0.5 mcg/min, the patient had ventricular bigeminy. Activation times of 37 ms pre-QRS were recorded in the RV outflow tract; however the pacemap did not match the spontaneous PVC (Fig. 68.3). The aortic sinuses were then mapped via a retrograde approach. The earliest activation was 45 ms pre-QRS at the junction between the right and left coronary sinuses; pacemapping this region replicated perfectly the clinical PVC morphology (Fig. 68.4). Ablation at this site abolished the patient’s ectopy.
M.D. Hutchinson Cardiovascular Division, Department of Medicine, University of Pennsylvania, 3400 Spruce Street, 9 Founders, Philadelphia, PA 19104, USA e-mail:
[email protected] A. Natale et al. (eds.), Cardiac Electrophysiology, DOI: 10.1007/978-1-84996-390-9_68, © Springer-Verlag London Limited 2011
299
300
M.D. Hutchinson
Fig. 68.1 The presenting 12-lead ECG demonstrates frequent left bundle, right inferior axis ventricular ectopy
a
b I II III aVR aVL aVF V1 V2 V3 V4 V5 V6
Fig. 68.2 (a) The same ECG from Fig. 68.1 is reproduced; the black arrow indicates a large R wave in lead V3. (b) The 12-lead ECG taken in the EP lab for the same patient shows an earlier precordial transition
at V2 (arrow). This most likely represents erroneous placement of leads V1 and V2 in the third intercostal space
Case 68 Fig. 68.3 (a) Comparison of the clinical PVC morphology to the best pacemap obtained from the RV outflow tract. Although the frontal plane axis matches the clinical ectopy well, the precordial transition is much later from the RVOT. (b) The earliest activation time for the clinical PVC from the RVOT is 37 ms pre-QRS
301
a
Clinical PVC
RVOT pacemap
b
RVOT activation I
I
II
II
III III aVR aVR
aVL
aVL
aVF
aVF
V1 V2
V1
V3 V2
V4
V3
V5
V4
V6 Abld
V5
Ablp V6
a
RVa
Clinical PVC
R/LCC pacemap
b
R/LCC activation
I
I
II
II
III
III aVR
aVR aVL aVF
aVL aVF V1 V2
V1
Fig. 68.4 (a) Comparison of the clinical PVC morphology to the best overall pacemap obtained from the junction of the right and left coronary cusps. (b) The earliest overall activation time for the clinical PVC obtained from the junction of the right and left coronary cusps with a sharp pre-potential 45 ms pre-QRS
V3
V2
V4
V3
V5
V4
V6 Abld
V5 V6
Ablp RVa
Case 69 Matthew D. Hutchinson
Case Summary A 57-year-old man with history of isolated septal myocardial infarction had a CRT-D device implanted several years ago for ventricular tachycardia and congestive heart failure. He presented now with repeated ICD shocks despite treatment with amiodarone. Interrogation of the ICD demonstrated two predominant VT morphologies with similar cycle lengths (520–550 ms). The patient was brought to the electrophysiology lab for mapping and ablation. Voltage mapping demonstrated a moderately large area of basal septal scar, and programmed stimulation with single extrastimuli induced two VT morphologies (Figs. 69.1 and 69.2) which matched with the ICD electrograms of the patient’s spontaneous tachycardias. Based on the ECG morphologies, where are the likely sites of origin/exit for each VT? How might these VTs be associated with the scar, and what would be your ablation strategy?
Case Discussion Despite the atypical scar location, this patient has an ischemic substrate due to known occlusion of a large septal perforator. The first VT (Fig. 69.1) has a left bundle, inferior axis morphology with a precordial transition at V3 suggesting exit from the mid superior septum. The second VT (Fig. 69.2) has a right bundle, left superior morphology with
precordial transition at V4 suggesting an exit from the mid inferior wall adjacent to the septum. Given that the VT circuits in ischemic patients are often large, multiple different VT morphologies may share components of the tachycardia circuit. The similar tachycardia cycle lengths also suggest a similar topography for the two VT morphologies. Localizing these VT exit sites on this patient’s voltage map indicates that they likely originate from opposite sides of the septal scar. In this case, detailed mapping of the septal scar during VT 2 revealed several sites with mid-diastolic potentials (Fig. 69.3) which were demonstrated by entrainment to be isthmus sites. RF lesions at these sites terminated VT 2. Upon repeat programmed stimulation, neither VT 2 nor VT 1 could be re-induced. Thus it appeared that the two VTs shared a common critical isthmus for reentry, with wavefronts travelling in opposing directions and exiting on opposing sides of the scar. It is important to remember that scar-related VT can take long and circuitous routes, especially if there are extensive areas of scarring. As is seen in this case, the VT cycle length can be extremely long on antiarrhythmic drug therapy, even when the scar is not extensive. Thus in patients with extensive scar, two or more VTs of potentially widely disparate morphologies are the rule. However, as in this case, some of the VTs may simply travel in opposing directions through a common isthmus, and ablation of this common isthmus would abolish the pathway for multiple reentry circuits. Certainly this is good reason not to be discouraged when multiple VTs are induced at the start of the procedure.
M.D. Hutchinson Cardiovascular Division, Department of Medicine, University of Pennsylvania, 3400 Spruce Street, 9 Founders, Philadelphia, PA 19104, USA e-mail:
[email protected] A. Natale et al. (eds.), Cardiac Electrophysiology, DOI: 10.1007/978-1-84996-390-9_69, © Springer-Verlag London Limited 2011
303
304 Fig. 69.1 12-lead surface ECG of the first ventricular tachycardia (VT) morphology induced during ablation procedure. The tachycardia cycle length as denoted is 550 ms
M.D. Hutchinson 200 ms I II III aVR aVL aVF V1
550 ms
V2 V3 V4 V5 V6
200 ms I II III aVR aVL aVF V1 V2 V3 V4
Fig. 69.2 12-lead surface ECG of the second VT morphology induced. The tachycardia cycle length as denoted is 526 ms
V5 V6
526 ms
Case 69 Fig. 69.3 Bipolar voltage map of the left ventricle illustrating the site (white star) of middiastolic potentials during VT 2 (Abl recording from ablation catheter, d distal bipole, p proximal bipole). The large white arrows represent the approximate exit sites for the two VT morphologies induced
305 I II III
VT1 RAO LV base
aVR aVL aVF V1 V2 V3 V4 V5 V6
>1.5 mV VT2
LV Apex Abld Ablp
<0.5 mV
200 ms
Case 70 Matthew D. Hutchinson
Case Presentation A 71-year-old man with history of distant inferior myocardial infarction with subsequent coronary artery bypass graft surgery and St. Jude mechanical mitral valve prosthesis initially presented with symptomatic ventricular tachycardia which was treated with amiodarone and implantation of an ICD. Now, several years later, he presented with ICD shocks for recurrent sustained monomorphic VT despite continued amiodarone therapy. He was referred for mapping and ablation of VT. His bipolar voltage mapping revealed an extensive scar involving the basal inferior and lateral LV. The clinical VT (right bundle, right inferior axis; CL 510 ms) was easily induced with programmed stimulation and was well-tolerated hemodynamically allowing detailed entrainment mapping. Several sites with mid-diastolic potentials during VT were found in the basal inferolateral wall; Figs. 70.1 and 70.2 demonstrate responses to attempted entrainment from two of these sites. Describe the response to entrainment shown in Figs. 70.1 and 70.2. Should either site be considered a good target for ablation of the clinical VT?
Case Discussion Figure 70.1 demonstrates the entrainment response at the end of pacing at a site of mid-diastolic potentials. There is relatively long stimulus-QRS (S-QRS) interval during
M.D. Hutchinson Cardiovascular Division, Department of Medicine, University of Pennsylvania, 3400 Spruce Street, 9 Founders, Philadelphia, PA 19104, USA e-mail:
[email protected]
pacing with concealed entrainment – suggesting that the paced beats are exiting the scar in the same fashion as the VT. However, the local return cycle length is significantly longer than the tachycardia cycle length (Fig. 70.3). This entrainment response suggests that the sharp potential on the ablation catheter does not directly participate in perpetuating the VT; it is being activated passively as the wavefront travels through the scar, thus producing mid-diastolic potentials unrelated to the critical circuit. Ablation at this location would be unlikely to disrupt the reentrant wavefront. Figure 70.2 is obtained at another site of mid-diastolic potentials. In this case, an early-coupled extrastimulus terminates the tachycardia without activating the left ventricular myocardium. The response to this so-called non-propagated extrastimulus suggests that the pacing stimulus is delivered within the protected isthmus of the reentrant circuit, thereby extinguishing the reentrant wavefront in both an orthodromic (encountering tissue still refractory after the previous reentrant beat) and an antidromic (colliding with the subsequent reentry wavefront as it travels into the isthmus) manner. Interrupting the reentrant circuit with this extrastimulus also implies that the critical isthmus traversing the pacing electrode is relatively narrow, thus susceptible to disruption. It is also noted that the subsequent pacing stimulus reproduces the clinical VT morphology perfectly, and activates the myocardium with a long stimulus-QRS interval. This suggests that the pacing stimulus is delivered proximally within the reentrant circuit and without antidromic capture. In this case, VT was repeatedly reinduced with programmed stimulation and terminated with nonpropagated extrastimuli. VT was subsequently terminated and rendered noninducible after a single RF lesion in this location. In summary, locating early diastolic activity during VT identifies regions of viable myocardium within the scar; however these potentials may or may not participate in the tachycardia circuit. The accurate interpretation of the entrainment response can easily differentiate the critical tachycardia isthmus from a passively-activated bystander loop.
A. Natale et al. (eds.), Cardiac Electrophysiology, DOI: 10.1007/978-1-84996-390-9_70, © Springer-Verlag London Limited 2011
307
308 Fig. 70.1 12-lead surface ECG and intracardiac recordings of the response to entrainment at a site of mid-diastolic potentials during ventricular tachycardia. See text for discussion. Abl ablation catheter electrogram, d distal bipole, p proximal bipole, RVa electrogram recording from the right ventricular apex
M.D. Hutchinson 200 ms I II III aVR aVL aVF V1 V2 V3 V4 V5 V6 Abld Ablp RVa
400 ms I II III aVR aVL aVF V1 V2 V3 V4 V5 V6
Fig. 70.2 ECG and intracardiac recordings from another site of mid-diastolic potentials during ventricular tachycardia
Abld Ablp RVa
Case 70 Fig. 70.3 Same snapshot as Fig. 70.1, with measured time intervals as denoted (ms)
309 200 ms I II III aVR aVL aVF V1 V2 V3 V4 V5 V6 Abld Ablp RVa
551
504
Case 71 Matthew D. Hutchinson
Case Summary A 69-year-old man with ischemic cardiomyopathy and remote anterior infarction presented to the Emergency Department with palpitations. He was noted on ECG to have monomorphic ventricular tachycardia (left bundle, left superior axis; CL 420 ms). He was treated with intravenous amiodarone and lidocaine; however he had recurrent arrhythmias which uniformly responded to anti-tachycardia pacing. An example of the 12-lead rhythm strip and intracardiac recordings obtained during the pace termination of an episode of VT is included in Fig. 71.1. Coronary angiography revealed no significant disease progression since his last evaluation. He was referred for EP study and ablation.
In the EP lab, bipolar voltage mapping revealed a large area of low voltage involving the distal anterior wall and LV apex. The patient’s clinical VT was easily inducible (CL 410 ms) with single extrastimuli. As the VT was well-tolerated hemodynamically, detailed activation and entrainment mapping was performed. Figures 71.2 and 71.3 depict the response to entrainment at the sites indicated in the electroanatomical map. What information about the VT circuit is gained by examination of the 12-lead and intracardiac recordings in Fig. 71.1? Describe the response to entrainment in Figs. 71.2 and 71.3. Ablation at which of these sites would be expected to abolish the tachycardia? If neither, where would one expect
I II
A
III
B
aVR aVL
Marker channel
aVF V1
VEGM
V2
Fig. 71.1 Panel A 12-lead ECG illustrating overdrive pacing during VT from the ICD lead. Panel B Intracardiac recording simultaneous with ECG in panel A. See text for discussion. VEGM local bipolar ventricular electrogram
V3 V4 V5 V6
M.D. Hutchinson Cardiovascular Division, Department of Medicine, University of Pennsylvania, 3400 Spruce Street, 9 Founders, Philadelphia, PA 19104, USA e-mail:
[email protected] A. Natale et al. (eds.), Cardiac Electrophysiology, DOI: 10.1007/978-1-84996-390-9_71, © Springer-Verlag London Limited 2011
311
312 Fig. 71.2 Panel A Electroanato mical map with large area of low voltage involving LV anterior wall and apex. Panel B Overdrive pacing during VT performed at the site indicated with star in panel A
M.D. Hutchinson 200 ms
A
B AP view
I II III aVR aVL aVF V1 V2 V3 V4 V5
Septum >1.5 mV
V6 Abld
Apex
Ablp
<0.5 mV
A
B AP view
200 ms I II III aVR aVL aVF V1 V2 V3 V4 V5
Septum >1.5 mV
Fig. 71.3 Panel A Electroanato mical map as described in Fig. 71.2. Panel B Overdrive pac ing during VT performed at the site indicated with star in panel A
Apex
V6 Abld Ablp
<0.5 mV
to find the most appropriate ablation site based upon the information presented?
Case Discussion This case illustrates a common presentation of stable, MMVT occurring after healed anterior myocardial infarction. Figure 71.4 demonstrates the response to overdrive pacing from
the ICD lead (the local bipolar electrogram [B, bottom panel] and the marker channel [A, top channel] are shown). Importantly, examination of the morphology and post-pacing interval (PPI) on the local bipolar electrogram reveals intracardiac fusion and a postpacing interval (PPI) <30 ms of the tachycardia cycle length (TCL). This suggests that the RV lead pacing site represents an outer loop of the tachycardia circuit. Examination of the 12-lead ECG obtained during ATP confirms these findings. These findings can be particularly helpful when the 12-lead morphology of the clinical VT is not available.
Case 71 Fig. 71.4 Panel A 12-lead ECG from Fig. 71.1 with QRS fusion during pacing and a postpacing interval within 30 ms of the tachycardia cycle length (TCL). Thus, pacing from the RV apex is an outer loop site for the clinical VT. Panel B Intracardiac recording confirming the observations in panel A. VEGM local bipolar ventricular electrogram
313 I
460 ms 440 ms
II
A
B
III aVR
Marker channel
aVL aVF V1
VEGM
V2
460 ms
V3
440 ms
V4 V5 V6
200 ms
A
B AP view
I II III aVR aVL aVF V1 V2 V3 V4 V5
Septum
Fig. 71.5 Panel A Electroanato mical map as described in Fig. 71.2. Panel B Overdrive pacing during VT performed at the site indicated with star in panel A. This represents an entrance site for the clinical VT. See text for discussion
>1.5 mV
Apex
V6 Abld Ablp
300 423
300 414
<0.5 mV
Pacing within a protected part of the VT circuit produces a paced morphology identical to the spontaneous VT morphology. The degree of prematurity of the bipolar electrogram (EGM) recorded within the protected circuit relative to the onset of the surface QRS complex correlates to the degree of proximity within the circuit.1–3 Figure 71.2 demonstrates concealed entrainment with a PPI-TCL < 30 ms. The stimulus-QRS (S-QRS) interval equals the electrogram-QRS (EGM-QRS) interval and is quite long (300 ms, 73% TCL; Fig. 71.5). This
represents an entrance site for the VT which is oriented at the lateral border of the low voltage region. Figure 71.3 also shows concealed entrainment with a PPI-TCL < 30 ms. Here the stimulus-QRS interval (equal to the EGM-QRS) is 30 ms (8% TCL; Fig. 71.6); this represents an exit site which is located at the septal side of the low voltage region. Focal ablation of VT can be performed by careful mapping and targeting of the critical tachycardia isthmus. The broad nature and potential redundancy of entrance and exit
314 Fig. 71.6 Panel A Electroanato mical map as described in Fig. 71.2. Panel B Overdrive pacing during VT performed at the site indicated with star in panel A. This represents an exit site for the clinical VT. See text for discussion
M.D. Hutchinson 200 ms
A
B AP view
I II III aVR aVL aVF V1 V2 V3 V4 V5
Septum >1.5 mV
Apex
30
V6 Abld
30
404
Ablp
414
<0.5 mV
200 ms
A
B AP view
I II III aVR aVL aVF V1 V2 V3 V4 V5
Septum >1.5 mV
Apex
Fig. 71.7 Panel A Electroanatomical map as described in Fig. 71.2. Panel B Overdrive pacing during VT performed at the site indicated with star in panel A. This represents an isthmus site for the clinical VT. See <0.5 mV text for discussion
sites for a given tachycardia limits the efficacy of focal ablation at these sites. Detailed mapping and targeting of the critical isthmus is more effective in eliminating the tachycardia with focal ablation. In this case, mapping earlier diastolic activity within the dense scar more proximal to the exit site identified an isthmus site (StimulusQRS = EGM-QRS = 173 ms; 42% TCL; Fig. 71.7). Ablation at this isthmus site terminated the tachycardia and rendered it noninducible. In summary, a combination of detailed activation and entrainment mapping remains the most efficient method of ablating tolerated VT.
V6 Abld Ablp
173 405
173 414
References 1. Josephson ME. Clinical Cardiac Electrophysiology: Techniques and Interpretations, vol. 3. 3rd ed. Philadelphia, PA: Lippincott, Williams, & Wilkins; 2002. 2. Almendral JM, Gottlieb CD, Rosenthal ME, et al. Entrainment of ventricular tachycardia: explanation for surface electrocardiographic phenomena by analysis of electrograms recorded within the tachycardia circuit. Circulation. 1988;77(3):569-580. 3. Stevenson WG, Khan H, Sager P, et al. Identification of reentry circuit sites during catheter mapping and radiofrequency ablation of ventricular tachycardia late after myocardial infarction. Circulation. 1993;88(4 Pt 1):1647-1670.
Section Devices
II
Case 72 Amin Al-Ahmad and Paul J. Wang
Case Summary
Case Discussion
A 19-year-old woman with a history of cardiac transplantation 5 years prior is admitted with acute rejection. She has a dual chamber PPM, Medtronic Kappa 401. Telemetry strip revealed pacemaker artifact after the QRS. A 12-lead ECG is shown in Fig. 72.1. Her pacemaker settings are as follows:
This ECG shows normal sinus rhythm with low voltage and poor R wave progression, in addition to a rightward axis. There are pacemaker artifacts after each QRS. This indicates clear undersensing on the ventricular channel. In this tracing, the pacemaker detects the intrinsic atrial activation and because of ventricular undersensing delivers a pacemaker stimulus after the AV delay interval ends. Could this pacing artifact be an atrial stimulus? With ventricular undersensing the pacemaker may deliver an atrial stimulus, but would follow this with a ventricular stimulus. As there is only one stimulus regularly seen in this ECG, the pacemaker is adequately sensing the intrinsic atrial activation and tracking appropriately.
Mode
DDD
LRL/URL
50/150 bpm
Sensing, ventricular channel
2.8 mV
AV delay
Is the pacemaker function appropriate? Should any programming changes be made in this case?
A. Al-Ahmad and P.J. Wang (*) School of Medicine, Stanford University, 300 Pasteur Drive, H-2146, Stanford, CA 94305, USA e-mail:
[email protected] A. Natale et al. (eds.), Cardiac Electrophysiology, DOI: 10.1007/978-1-84996-390-9_72, © Springer-Verlag London Limited 2011
317
318
Fig. 72.1 12-lead ECG showing pacing artifact
A. Al-Ahmad and P.J. Wang
Case 73 Amin Al-Ahmad and Paul J. Wang
Case Summary A 32-year-old woman with a history of corrected transposition of the great vessels and a pacemaker placed for sinus node dysfunction is seen for a routine pacemaker check. She has a Medtronic Kappa Model KDR401/403. Atrial sensing was 2.0 mV, the threshold was 0.5 V @ 0.25 ms. Ventricular sensing was 4 mV and the threshold was 1.5 V @ 0.6 ms. Lead impedance on both leads was stable. Device settings are as follows: Mode
DDDR
LRL/URL
60/125 bpm
Max sensor rate
140 bpm
AV delay
260 ms
PVARP
240 ms
Several atrial high rate episodes were noted. Figure 73.1 shows a stored electrogram of an atrial high rate episode. Does this electrogram reveal an atrial tachycardia? What is the cause of the atrial high rate episodes?
a coupling interval of 398 ms; this likely represents a ventricular premature contraction (VPC) (Asterisk, Fig. 73.2). Shortly after the VPC an atrial event can be seen on the atrial electrogram, but this event is not detected by the pacemaker (the marker channel does not show any atrial events) as it is likely in the ventricular blanking period (Arrow, Fig. 73.2). This atrial event may be a retrograde atrial beat, or may simply be a sinus beat that fortuitously falls after the VPC. This atrial beat does not conduct since it is either retrograde or because it is just after the VPC. Once the atrial escape interval is over, an atrial paced event is delivered; however, this is unlikely to capture the atrium since it is delivered while the atrium is most likely refractory. After the programmed AV delay, a ventricular paced event results in a similar retrograde atrial event and the cycle continues. With an AR followed shortly by an AP, the device records this as an atrial high rate episode. It is important not to make assumptions that high rate episodes correspond to atrial tachyarrhythmias in all cases as this may lead to inappropriate use of medications. Careful examination of stored electrograms when available may be useful to better elucidate the cause of these atrial high rate episodes.
Case Discussion The stored strip shows atrial pacing followed by ventricular sensing initially. A ventricular sensed event occurs with
A. Al-Ahmad and P.J. Wang (*) School of Medicine, Stanford University, 300 Pasteur Drive, H-2146, Stanford, CA 94305 e-mail:
[email protected] A. Natale et al. (eds.), Cardiac Electrophysiology, DOI: 10.1007/978-1-84996-390-9_73, © Springer-Verlag London Limited 2011
319
320
Fig. 73.1 Stored electrogram showing the atrial EGM and the marker intervals
Fig. 73.2 Stored electrogram showing initiation of atrial high rate episode. Asterisk shows a ventricular sensed event (VPC). Arrow marks the atrial beat that follows the VPC (probably retrograde)
A. Al-Ahmad and P.J. Wang
Case 74 Kenneth A. Ellenbogen
Case Summary A 72-year-old female with a past medical history of hypertension and an “arrhythmia” is admitted for an elective cholecystectomy. While the patient is on telemetry, her pacemaker is incidentally noted to “not be working properly” and you are called to evaluate her device. Review of the admission chest film reveals a right-sided generator with a single lead positioned in the RV apex. A 12-lead EKG with a rhythm strip is requested and is shown (Fig. 74.1). What is the underlying rhythm and is there evidence of abnormal pacer function? The device is interrogated and the programmed parameters are shown (Fig. 74.2) with a programmed lower rate of 70 bpm. A “real time” intracardiac EGM of the RV and marker channel is obtained during one of the “episodes” (Fig. 74.3). There appears to be ventricular pacing shortly after a sensed QRS labeled as VR on the marker channel at less than the lower rate limit (ventricular escape interval). Is this consistent with normal pacer function and what is the management for this problem?
Case Discussion Review of the 12-lead EKG shows atrial fibrillation with evidence of pacer artifacts following several of the QRS complexes, but with only the second beat on the strip resulting in ventricular capture. Based on the tracing there is evidence of
undersensing due to pacing spikes evident shortly after several of the patient’s intrinsic beats. In regard to ventricular capture, the second beat is paced and the remaining pacer spikes fall within the T wave of the preceding beat, when the ventricle is likely refractory (“physiologic” non-capture). At this point one is likely to suspect abnormal pacer function with at least R wave undersensing and no evidence of problems with capture. Interrogation of the device provides the programmed lower rate limit (equal to the ventricular escape interval in a VVI pacemaker) of 70 bpm along with the ventricular refractory period (VRP) of 500 ms. Knowledge of these two programmed parameters along with inspection of the surface lead, RV EGM, and marker channel helps to shed light on the phenomenon seen on the initial EKG. Review of Fig. 74.3 shows that the native beats are properly sensed, but they are labeled VR on the marker channel as they fall within the 500 ms VRP initiated by the prior beat. Every paced or sensed beat reinitiates the ventricular escape interval which is the programmed lower rate of 70 bpm. The exception is if the sensed beat falls within the 500 ms VRP in which case the ventricular escape interval will not be reset. Due to the variability of the R-R intervals during atrial fibrillation and the intervals between various VR beats, subsequent pacing spikes exhibit variable ventricular capture resulting in pseudo-fused, fused, and completely paced beats. Thus, the pacemaker is exhibiting normal function. Shortening the VRP will eliminate some of the “undersensing” of native beats and will minimize the “inappropriate pacing” following these beats.
K.A. Ellenbogen Cardiac Electrophysiology and Pacing, Department of Cardiology, VCU School of Medicine, 980053, Richmond, VA 23298-0053, USA e-mail:
[email protected] A. Natale et al. (eds.), Cardiac Electrophysiology, DOI: 10.1007/978-1-84996-390-9_74, © Springer-Verlag London Limited 2011
321
322
K.A. Ellenbogen
Fig. 74.1 12 lead ECG showing pacer spikes without capture
Arrows showing pacing spikes
Fig. 74.2 Pacemaker programmed parameters
Case 74
323
Fig. 74.3 Surface ECG, EGM and marker channel while patient has the abnormality seen on the 12 lead ECG
Arrows pointing to ventricular pacing with varying degrees of capture
Case 75 Nora Goldschlager
Case Summary
Case Discussion
A 63-year-old woman with documented bradycardia– tachycardia syndrome but no structural heart disease was implanted with a dual chamber pacing system. The low rate limit was programmed at 70 ppm, the upper tracking rate to 100 ppm. Ventricular sensitivity was programmed to 2 mV and atrial sensitivity to 1 mV. Despite her history of paroxysmal atrial arrhythmias, automatic mode switch function had not been programmed on. The procedure was accomplished without incident, and antiarrhythmic drug therapy was begun to manage her tachyarrhythmias, which consisted primarily of paroxysmal atrial tachycardia and atrial fibrillation. The patient did well over the ensuing several months, but then began to have recurrent palpitations associated with breathlessness and chest discomfort. Because these symptoms were new after pacemaker placement, she was seen ahead of schedule in the cardiology clinic, where physical examination revealed no evidence of heart failure; subsequently performed echocardiography revealed normal left and right ventricular size and systolic function, normal pulmonary artery pressure, and only mild biatrial enlargement. A pharmacologic stress test showed no evidence of ischemia. Because of irregular heart rates observed during her workup she was referred to the pacemaker clinic, where, in the course of the evaluation, Fig. 75.1 was obtained. What does the figure illustrate? How does it explain the patient’s problem? What steps could be taken to solve the problem?
The figure displays some of the permanently programmed parameters, the body surface ECG, atrial intracardiac electrogram, and marker channel information. All QRS complexes are paced. Atrial pacing stimuli are being delivered at a rate of about 100/min (with the exception of the third event (“P”)); atrial capture is not occurring. The atrial intracardiac electrogram reveals regular atrial activity at a rate of about 300/min, suggesting atrial flutter. Note the variable amplitude of the atrial electrical signals. Note also that the electrogram is recorded at a gain of 2.5 mV/cm; thus the atrial electrical signals are of very low amplitude, and below the programmed sensitivity. The atrial tachyarrhythmia is therefore not sensed, resulting in delivery of atrial stimulus outputs. More importantly, even if the mode switch function had been programmed on, mode switch would not have occurred due to the atrial undersensing. Note also that event counters will (erroneously) designate these events as AV pacing. The atrial fibrillation likely explains the patient’s clinical symptoms, which could be due to the atrial arrhythmia itself or to intermittent sensing of fibrillatory impulses, resulting in rapid and/or irregular ventricular paced rates; the latter would not occur if automatic mode switch is programmed on. The patient’s symptoms resolved after reprogramming the atrial sensitivity to 0.3 mV, and the mode switch function on (DDIR), as well as optimizing her antiarrhythmic medication regimen.
N. Goldschlager Department of Cardiology, University of California, San Francisco, Cardiology Division, 5G1, San Francisco General Hospital, 1001 Potrero Avenue, San Francisco, CA 94110, USA e-mail:
[email protected] A. Natale et al. (eds.), Cardiac Electrophysiology, DOI: 10.1007/978-1-84996-390-9_75, © Springer-Verlag London Limited 2011
325
326
Fig. 75.1 Program perimeters, surface ECG, marker channel and atrial lead recording of the patient
N. Goldschlager
Case 76 Fred M. Kusumoto, Jennifer Crain, and Nora Goldschlager
Case Summary A 66-year-old man with a history of coronary artery disease comes to your office complaining of generalized fatigue. He denies chest pain or other symptoms of ischemia. He has a history of a myocardial infarction 6 years ago but has normal left ventricular function by recent echocardiogram. He underwent permanent pacemaker implantation in 2001 for a “slow heart rate.” His comorbidities include diabetes, obesity, and hypertension. He currently is being treated with extendedrelease metoprolol 50 mg daily and aspirin. On physical examination his heart rate is 60 beats per minute. His lungs are clear and he has an II/VI systolic murmur. Initial interrogation of his device is shown in Figs. 76.1–76.3. Although the patient’s nonspecific symptom of fatigue could be due to a variety of problems (diabetes, deconditioning, atypical presentation of ischemia), what specific rhythm and devicerelated issues should be considered?
Case Discussion On initial evaluation (Fig. 76.1) it is seen that the patient has a single-chamber ventricular pacemaker. Evaluation of the event counters shows that the patient is paced from the
F.M. Kusumoto (*) Department of Cardiovascular Diseases, Mayo Clinic, 4500 San Pablo Road, Jacksonville, FL 32224, USA e-mail:
[email protected] J. Crain Electrophysiology and Pacing Service, Division of Cardiovascular Diseases, Department of Medicine, Mayo Clinic, Jacksonville, FL 32224, USA N. Goldschlager Department of Cardiology, University of California, San Francisco, Cardiology Division, 5G1, San Francisco General Hospital, 1001 Potrero Avenue, San Francisco, CA 94110, USA e-mail:
[email protected]
ventricle 84% of the time (Fig 76.2). The rhythm strip (Fig 76.3) shows paced QRS complexes (“P”) and retrograde P waves. Patients with retrograde atrial activation can develop pacemaker syndrome. Originally described almost 40 years ago, this complex process can lead to a constellation of vague symptoms including syncope or presyncope, weakness, lightheadedness, orthopnea, paroxysmal nocturnal dyspnea, dizziness, and, in some cases even pulmonary edema. The pathophysiology of pacemaker syndrome is multifactorial: ventricular pacing can lead to worsening mitral regurgitation, elevated atrial natriuretic peptide, and higher sympathetic activity. Pacemaker syndrome is most commonly observed in patients with single-chamber ventricular pacemakers implanted for sinus node dysfunction, where ventricular pacing leads to retrograde atrial activation and/or AV dyssynchrony. The prevalence of pacemaker syndrome varies depending on definition, but in the Mode Selection Trial, pacemaker syndrome developed in 18% of patients with pacemakers implanted for sinus node dysfunction. Ventricular pacing can also lead to heart failure and development of atrial fibrillation. In an analysis of the MOST data, likelihood of heart failure hospitalization increased 2.5–3 fold with ventricular pacing > 80% for the VVI pacing mode, and risk of atrial fibrillation increased by approximately 20% for each additional 25% increase in cumulative ventricular pacing. The device was reprogrammed to VVI with a lower rate of 40 ppm and the beta blocker was temporarily discontinued. A rhythm strip recorded 1 week later is shown in Fig. 76.4. His symptoms had improved significantly. Beta blockers are a critical component for the treatment of patients after myocardial infarction. However, in this case, reinstitution of beta blockade therapy could lead to a higher percentage of ventricular pacing. An atrial lead could be added with a small but finite risk, and, with current battery technology, the patient might well require device replacements over his lifetime. The Pacemaker and Beta-Blocker Therapy after Myocardial Infarction (PACE-MI) Trial is
A. Natale et al. (eds.), Cardiac Electrophysiology, DOI: 10.1007/978-1-84996-390-9_76, © Springer-Verlag London Limited 2011
327
328
F.M. Kusumoto et al.
Fig. 76.1 Interrogation of baseline pacing parameters. PPM, pulses per minute; V, volt; mV, millivolt; ADL, activity of daily living
Initial interrogation report Ventricular long term histogram Sensed
Paced
Percentage of beats 50 40 30 20 10 0 < 40
60
80
100 120 140 Rate (BPM)
160
180 >
Ventricular high rate episodes: 0 Pacing (percentage of total): Sensed Paced
16.2% 83.8%
Fig. 76.2 Initial interrogation of long-term rate counters. BPM, beats per minute
currently underway and is randomizing patients after myocardial infarction with contraindications to beta blockade due to bradycardia to standard therapy or combined therapy with pacing and beta-blockade with a composite pri mary endpoint of total mortality and nonfatal myocardial infarction.
Case 76
329
Fig. 76.3 Baseline rhythm strip. Lead II and surface ventricular electrograms are shown. P, ventricular paced event
Fig. 76.4 Baseline rhythm strip (lead II) and ventricular electrograms on follow-up. S, ventricular sensed event
Bibliography Ellenbogen KA, Gilligan DM, Wood MA, et al. The Pacemaker syndrome: a matter of definition. Am J Cardiol. 1997;79:1226-1229. Link MS, Hellkamp AS, Estes NA 3rd, et al. MOST study investigators. High incidence of pacemaker syndrome in patients with sinus node
dysfunction treated with ventricular-based pacing in the Mode Selection Trial (MOST). J Am Coll Cardiol. June 2004;43(11):2066-2071. Sweeney MO, Hellkamp AS, Ellenbogen KA, et al. MOde Selection Trial Investigators. Adverse effect of ventricular pacing on heart failure and atrial fibrillation among patients with normal baseline QRS duration in a clinical trial of pacemaker therapy for sinus node dysfunction. Circulation. 2003;107:2932-2937.
Case 77 Gregory M. Marcus and Nora Goldschlager
Case Summary A 63-year-old woman with paroxysmal atrial fibrillation and rapid ventricular response and intermittent presyncope is found to have symptomatic sinus pauses >3 s on Holter monitoring as well as an intermittent accelerated junctional rhythm. In sinus rhythm, her PR interval is 180 ms, and she has normal QRS complex morphology. She undergoes placement of a St. Jude Identity ADx dual chamber pacemaker without any apparent complications. Notably, the majority of the right atrial sites tested for capture and sensing thresholds exhibited low intracardiac voltage, and an acceptable atrial sensitivity could be achieved only in the low right atrium. At implant, atrial and ventricular sensitivities are 2.7 and >12 mV, respectively. The atrial capture threshold is 1.1 V at 0.5 ms, and the ventricular capture threshold is 0.9 V at 0.5 ms. The device is programmed to DDD mode, with a base rate of 60 ppm, a paced AV delay of 275 ms, and a sensed AV delay (“PV delay”) of 250 ms. The evening after the device is placed, the on call hospitalist is contacted regarding the telemetry findings displayed in Fig. 77.1. A chest radiograph (Fig. 77.2) confirms that both leads are in appropriate positions, with the right atrial lead placed in the low right atrium for reasons mentioned above. Concerned that the pacemaker is not functioning correctly, the on call physician contacts the implanting electrophysiologist. After viewing the telemetry strips faxed to his home, the electrophysiologist reassures the on call physician that the device appears to be functioning normally
G.M. Marcus (*) Division of Cardiology, Electrophysiology Section, University of California, 500 Parnassus Avenue, MUE 434, San Francisco, CA 94143-1354, USA e-mail:
[email protected] N. Goldschlager Department of Cardiology, University of California, Cardiology Division, 5G1, General Hospital, 1001 Potrero Avenue, San Francisco, CA 94110, USA e-mail:
[email protected]
and that neither urgent device interrogation nor urgent reprogramming is necessary. How does the telemetry tracing reassure the implanting electrophysiologist?
Case Discussion This case demonstrates the importance of first assessing the patient’s underlying rhythm when interpreting paced electrocardiograms. The crucial finding (known from the history and evident in Fig. 77.1) is that the patient has an intermittent accelerated junctional rhythm. The first pacing output stimulus is associated with an inverted P wave in lead II, consistent with atrial capture from a low atrial position. The QRS complex immediately following that P wave occurs too early to represent a conducted complex and therefore likely originates in the AV junction. The following pacing output stimulus occurs nearly simultaneously with the onset of the next QRS complex (a second junctional complex). Importantly, no intervening P wave between the first atrial stimulus and this pacing output is observed, and the timing of these two stimulus outputs is exactly 1000 ms (60 ppm, the programmed lower rate limit). Therefore, this second pacing output represents a “pseudo–pseudo-fusion” complex (an atrial output stimulus occurring around the time of inscription of the QRS complex). Although atrial capture cannot be confirmed, since the P wave (if present) would be buried in the QRS complex, there is no evidence that atrial capture does not occur. Indeed, given documentation of atrial capture of the previous (paced) P wave, the likelihood is that this stimulus does result in atrial capture. Immediately following the first pseudo–pseudo-fusion complex, ventricular safety pacing is observed. Safety pacing occurs when an event is sensed in the ventricular channel during the safety pace interval, an interval in the ventricular sensing channel immediately following the ventricular blanking period and preceding the ventricular alert period.1 The purpose of safety pacing is to avoid ventricular asystole in the event that an electrical event (such as an atrial pacing output stimulus) is oversensed in
A. Natale et al. (eds.), Cardiac Electrophysiology, DOI: 10.1007/978-1-84996-390-9_77, © Springer-Verlag London Limited 2011
331
332
G.M. Marcus and N. Goldschlager
Fig. 77.1 Telemetry strip demonstrating a junctional rhythm associated with intermittent pseudo–pseudo-fusion and safety pacing (see text for discussion)
Fig. 77.2 Posteroanterior and lateral chest radiographs demonstrating a dual chamber pacing system, with one lead in the low right atrium and a second lead in the right ventricular apex
the ventricular channel, potentially inhibiting ventricular pacing stimulus output. In this case, the sensed event that triggers safety pacing may be the atrial stimulus (particularly as the atrial lead is in the low right atrium) or the sensed portion of the QRS complex. Safety pacing can be recognized by the shortened AV interval (usually between 110 and 120 ms) between atrial and ventricular stimulus outputs: In this case, the safety-pace interval is 120 ms, clearly shorter than the programmed AV interval of 275 ms described above. The fourth pacing output stimulus results in atrial capture and is associated with a spontaneous junctional complex. Importantly, the interval between the ventricular safety pacing stimulus and this stimulus is shorter than that between the two previous atrial stimuli. This provides further proof that the safety paced ventricular stimulus is indeed ventricular: The lower rate limit (base rate) is determined by the sum of the AV interval and the VA interval. We know that the AV interval is 275 ms and that the lower rate limit of 60 beats per
minute equals a cycle length of 1,000 ms; therefore, the VA interval must be 1,000 ms − 275 ms = 725 ms, the interval observed. It is therefore not surprising that the fourth QRS complex, which presumably has a P wave buried in it and occurs earlier than 1,000 ms after the last atrial paced event, is not associated with a pacing output (in fact, this provides evidence that atrial sensing is intact). The following two complexes are spontaneous, the latter likely a conducted sinus beat, followed by four pseudo– pseudo-fusion complexes. When the QRS complex occurs early enough to fall in the safety pace interval (as in the 7th, 8th, 9th, and 12th complexes), safety pacing occurs. When the QRS is sensed after the safety pace interval (and therefore in the ventricular alert period, as in the 10th and 11th complexes), the ventricular output is appropria tely inhibited (demonstrating that ventricular sensing is intact). Although the pacing system is functioning appropriately, it can be reprogrammed in the hopes of improving patient
Case 77
comfort. A simple option in this case would be to increase the lower rate limit, allowing for more consistent atrial capture. As the patient has no known AV conduction disease and as the programmed AV interval is quite long, this program change should result in conduction along her normal AV node–His–Purkinje system and a narrow QRS for each paced atrial event.
333
Reference 1. Wang PJ, MacFie J, Homoud MK, Link MS, Foote CB, Estes M. Modes of pacemaker function. In: Kusomoto FM, Goldschlager NF, eds. Cardiac Pacing for the Clinician. Philadelphia, PA: Lippincott Williams & Wilkins; 2001:63-90.
Case 78 Amin Al-Ahmad and Paul J. Wang
Case Summary A 65-year-old woman has a permanent pacemaker implanted on the previous day for episodes of AV block resulting in syncope. The device is a dual chamber Medtronic EnRhythm DR. The atrial sensing at implant was 3 mV, the threshold was 0.8 V at a pulse width of 0.5 ms, and the impedance was 556 W. The ventricular sensing at implant was 15 mV, the threshold was 0.4 V at a pulse width of 0.5 ms, and the impedance was 840 W. Telemetry shows an episode of AV block; however, there is no ventricular pacing for two P-waves (Fig. 78.1, arrow). The device is tested and the lead parameters are unchanged from implant. Device settings are as follows: Mode
AAI/DDD
LRL/URL
50/150 bpm
AV delay paced/sensed
180/150 ms
Programmed sensitivity (RV)
0.9 mV
PVARP
310 ms
What is the cause of the lack of atrial tracking in the tracing?
thresholds make this extremely unlikely. Ventricular oversensing causing inhibition from electromagnetic interference, diaphragm contraction, or intracardiac signals should be considered, in particular in pacing systems that utilize unipolar leads. However, in this case inhibition is similarly unlikely. The pacing mode AAI/DDD offers the best explanation of the tracing. This pacing mode provides AAI pacing as long as AV conduction remains intact. If there is an AA interval without a ventricular sense, the pacemaker delivers a back-up ventricular pace 80 ms after the AA escape interval. Also, if two out of four AA intervals lack a ventricular sense event consistent with AV block, the device changes to a DDD mode. The device will then periodically check for AV conduction, and if it is present switches back to an AAI mode. In this case, programming the device to a DDD mode with a long AV delay would eliminate this ECG finding and provide ventricular back-up for all episodes of AV block. Since AAI/DDD does provide back-up ventricular pacing, making no pacemaker programming changes would also be reasonable if the patient does not have symptoms due to lack of ventricular pacing that may occur on occasion of AV block.
Case Discussion In this tracing two P-waves are not followed by a paced ventricular beat. While it is possible that failure to capture can explain this, the lack of pacing artifact and the outstanding
A. Al-Ahmad and P.J. Wang (*) School of Medicine, Stanford University, 300 Pasteur Drive, H-2146, Stanford, CA 94305, USA e-mail:
[email protected] A. Natale et al. (eds.), Cardiac Electrophysiology, DOI: 10.1007/978-1-84996-390-9_78, © Springer-Verlag London Limited 2011
335
336
A. Al-Ahmad and P.J. Wang
Fig. 78.1 This figure shows telemetry strip with intermittent AV block, non-conducted P waves with no pacing stimulus following them
Case 79 Kenneth A. Ellenbogen and Rod Bolanos
Case Summary A 55-year-old woman with a single chamber pacemaker presents to clinic for her 1-month post-implant visit. She has a history of coronary artery disease and presented to an outside facility with complete heart block 5 weeks ago in the setting of an acute coronary syndrome. Her escape rhythm at the time consisted of an accelerated junctional rhythm at 65 bpm. She underwent a four vessel coronary artery bypass grafting with implantation of a 4951 RV epicardial pacing lead and Medtronic SigmaTM single-chamber pacemaker generator. She reports persistent exercise intolerance despite an uneventful post-op course and compliance with cardiac rehabilitation. Interrogation of the device reveals the following programmed parameters (Fig. 79.1) along with the accompanying rate histogram (Fig. 79.2), which reveals almost no ventricular paced events and shows primarily ventricular sensed events in a wide spectrum of rates up to 130 bpm. What are some possible explanations for these findings with the patient’s past medical history? Inspection of surface lead II along with the ventricular EGM and marker channel provides insight into the rate histogram findings (Fig. 79.3a). There are clearly more ventricular sensed events than there are QRSs on the surface lead. What else is being sensed on the ventricular channel? What are the potential management options in this particular patient and in general with this problem?
Case Discussion While the predominance of V sensed events on the histogram is surprising, it is not necessarily abnormal. One explanation K.A. Ellenbogen () Cardiac Electrophysiology and Pacing, Department of Cardiology, VCU School of Medicine, Richmond, VA 23298-0053, USA e-mail:
[email protected]
is that the patient has regained AV nodal conduction after she underwent revascularization in the setting of an acute coronary syndrome. This explanation is excluded with inspection of (Fig. 79.3a, b) the surface lead and ventricular EGM where there is not only a V sensed event on the marker channel for every QRS but also for every P wave seen on lead II. The rhythm here is sinus rhythm at 75 bpm with an accelerated junctional rhythm at 65 bpm. The ventricular histogram should have shown predominantly V sensed events in the range of 60–80 bpm based on her rhythm and programmed VVI setting at a lower rate of 50 bpm. The oversensing of farfield P waves explains the ventricular sensed events up to 130 bpm. This patient was fortunate that she had an accelerated junctional rhythm with p wave oversensing would otherwise have likely resulted in underpacing and subsequent syncope. Oversensing of P waves is uncommon with apical positioning of RV pacing leads. It can be seen though when leads are placed in a more basal location near the tricuspid valve, coronary sinus, mitral annulus or proximal RVOT, or with integrated RV defibrillator leads not positioned far enough out in the apex. In this case, the epicardial RV lead was positioned over the basal anterior aspect of the right ventricle, which is in relatively close proximity to right atrial appendage predisposing to farfield sensing of P waves. In this case the R waves measured 3.5 mV and thus decre asing the ventricular sensitivity to avoid the P wave oversensing was not a feasible management option. A noninvasive management option for dual chamber pacers with this problem is to force atrial pacing to take advantage of the cross-chamber blanking period in the ventricular channel after an atrial event and the shorter ventricular intervals which will prevent ventricular sensitivity from reaching its lowest values. Unfortu nately, with consistent P wave oversensing the RV lead usually requires repositioning. Management in this case consisted of insertion of new bipolar endocardial RV and atrial leads with upgrade to a dual chamber pacemaker programmed in the DDD mode.
A. Natale et al. (eds.), Cardiac Electrophysiology, DOI: 10.1007/978-1-84996-390-9_79, © Springer-Verlag London Limited 2011
337
338
K.A. Ellenbogen and R. Bolanos
Fig. 79.1 Programmed parameters for pacemaker
Modes Mode
Additional Features VVI
Sleep Single chamber hysteresis Transtelephonic monitor Extended telemetry Activate diagnostics?
Rates Lower rate
50 ppm
Refractory/Blanking Ventricular refractory
330 ms
Ventricular Lead Amplitude Pulse width Sensitivity Pace polarity Sense polarity Lead monitor Minimum impedance Maximum impedance
3.50 V 0.45 ms 2.00 mV Unipolar Unipolar Monitor only 200 Ω 3,000 Ω
Clinical status: 03/06/02 to 07/19/02 Ventricular histogram
Sensed Paced
% of Beats 50 40 30 20 10 0 < 40
60
80 100 120 140 160 180> Rate (bpm)
Ventricular high rate episodes: 0 Pacing (percentage of total): Sensed Paced
99.7% 0.3%
Fig. 79.2 Stored pacemaker ventricular histogram
Off Off Off Off Completed
Ventricular High Rate Episodes V. Min. detection rate V. events to detect V. events to terminate High rate collection method
220 ppm Three beats Two beats Rolling
Selectable Diagnostic Custom rate trend Rate trend collect Include refractory senses? Rate trend duration
Rolling Exclude 24 h
Case 79 Fig. 79.3 (a, b) Surface ECG along with marker channel and ventricular EGM
339 Surface lead showing hr of 65 bpm
a LEAD III 0.1 mV/mm
Marker channel
EGM 1 mV/mm
Oversensing of the atrium on ventricular lead V Rate 65 bpm
b LEAD III 0.1 mV/mm
Marker channel
EGM 1 mV/mm
Oversensing the atrium
Case 80 Nora Goldschlager
Case Summary A 73-year-old man underwent dual chamber pacing system implantation for intermittent advanced and complete AV block. His ejection fraction was normal, and he was not known to have any ventricular ectopy. Over the ensuing 6 months, pacemaker interrogation revealed normal dual chamber operation and no ventricular ectopy (designated “PVE” in this pacing system). One month after his last pacemaker clinic visit the patient began to note increasing effort fatigue and breathlessness. A thorough history and physical examination revealed no evidence of heart failure and was normal. A 12-lead ECG demonstrated sinus rhythm with first degree AV block and no evidence of atrial or ventricular pacing outputs. Interrogation of the pacemaker indicated over 700,000 PVEs occurring during the prior month, a totally unexpected finding for this patient (Fig. 80.1). Figure 80.2 was then recorded.
Explain the events illustrated in Fig. 80.2. Do these events account for the interrogated event counter information? What steps should be taken to eliminate the problem?
Case Discussion Figure 80.2 displays some programmed parameters, the body surface ECG, marker channel information, and the atrial intracardiac electrogram. (This manufacturer designates “P” as a native atrial event, “R” as a native ventricular event, “A” as an atrial pace event, and “V” as a ventricular pace event.) The initial portion of the ECG shows sinus rhythm; regular atrial activity is confirmed by the atrial electrogram. First degree AV block is present; the PR interval is about 400 ms (note the programmed PV interval is 160 ms); the P waves are not followed by paced ventricular complexes, indicating that they are not sensed. Event counts
Fig. 80.1 Event counts upon pacemaker interrogation
Rate (ppm)
PV
PR
AV
AR
PVE
30 - 54 55 - 69 70 - 89 90 - 109 110 - 129 130 - 149 150 - 179 180 - 224 225 - 249 > 250 Total:
20 64,875 319,697 132,003 19,259 4,238 1 0 0 0 540,093
0 14 2,786 7,958 7,043 3,410 868 23 0 0 22,102
636 529,998 2,730,957 1,034,390 148,900 2,645 306 0 0 0 4,447,832
0 12,999 54,422 74,729 9,597 425 4 0 0 0 152,176
0 9 246,901 376,031 112,734 7,532 1,962 955 1 0 746,125
Total event count:
5,908,328
N. Goldschlager Department of Cardiology, University of California, San Francisco, Cardiology Division, 5G1, San Francisco General Hospital, 1001 Potrero Avenue, San Francisco, CA, USA 94110 e-mail:
[email protected] A. Natale et al. (eds.), Cardiac Electrophysiology, DOI: 10.1007/978-1-84996-390-9_80, © Springer-Verlag London Limited 2011
341
342
N. Goldschlager
Fig. 80.2 Real time electrogram with marker channel
As is suggested by the horizontal lines in this marker channel, indicating (for this manufacturer) refractory periods, the native P waves are falling in PVARP (the upper horizontal line) and therefore will not be tracked. The device is therefore sensing consecutive R waves. Atrial outputs do not follow the sensed R waves since the RR cycle length is shorter than the programmed base rate; thus, the RA interval timeout is continually aborted. Consecutive R waves without an intervening atrial sensed event define a “PVE,” accounting for the erroneously overcounted PVEs, and explaining the event counter information. A premature ventricular complex in the middle of the strip is followed by a pause, allowing the RA interval to time
out, and leads to an atrial paced event, followed by a ventricular paced event. Normal P wave tracking follows. Note that the upper horizontal line representing PVARP is shorter in the PV cycles than in the RR cycles; this is due to PVARP extension after a “PVE” diagnosis is made, and is a programmable feature in this device. Thus, “PVE” sensing with automatic PVARP extension fosters continuation of the PVE diagnosis and disallows appropriate tracking. The mode of onset of this rhythm is not known from the rhythm strip shown. To eliminate the problem, however, the PVARP extension on “PVE” sensing could be programmed OFF; the P waves would then fall in the alert period of the AV interval and would be appropriately tracked.
Case 81 Fred M. Kusumoto, Jennifer Crain, and Nora Goldschlager
Case Summary An 82-year-old man with permanent atrial fibrillation has a permanent pacemaker implanted after AV nodal ablation. During a routine pacing system evaluation, interrogation of his device demonstrates a cluster of ventricular arrhythmias on one day. A stored electrogram from one of the events is shown in Fig. 81.1. Evaluation of the device demonstrates stable thresholds and lead impedances. What should be done next?
Case Discussion Evaluation of the electrograms from the episode reveals high frequency signals. The differential diagnosis includes appropriate sensing of a rapid ventricular rate due to ventricular or supraventricular arrhythmias or to sensing of other, extracardiac electrical signals (oversensing). Oversensing can be due to intracardiac electrical activity (ventricular repolarization (T waves) or atrial depolarization (P waves)), extracardiac signals (myopotentials or electromagnetic interference), or
lead/connector problems. In this case a “clean” electrogram with no baseline artifact (initial signals on the left side of the strip) is suddenly replaced by both high and low amplitude high frequency signals. This type of signal is commonly referred to as “noise” and usually is due to electromagnetic interference or lead/connector problems. Baseline evaluation of the device demonstrated no evidence of lead/connector problems (stable impedances, thresholds, normal electrograms). A lead problem that is intermittent can sometimes be identified by having the patient perform specific maneuvers such as reaching for the ceiling or placing the arm as far as possible behind his or her back. In this case, on further questioning, the patient recalled that on the day on which this figure was recorded he had undergone hernia surgery. Radiofrequency current from electrocautery use during surgery can interact with the sensing circuitry of a pacing system. The likelihood and consequences of this interaction vary from patient to patient. In general, electrocautery can be safely used if several simple recommendations are followed (Table 81.1). In this case the oversensing (Vs) led to transient inhibition of pacing stimulus output, which can lead to asystole in patients who are pacemaker dependent.
F.M. Kusumoto (*) Department of Cardiovascular Diseases, Mayo Clinic, 4500 San Pablo Road, Jacksonville, FL 32224, USA e-mail:
[email protected] J. Crain Electrophysiology and Pacing Service, Division of Cardiovascular Diseases, Department of Medicine, Mayo Clinic, Jacksonville, FL 32224, USA N. Goldschlager Department of Cardiology, University of California, San Francisco, Cardiology Division, 5G1, San Francisco General Hospital, 1001 Potrero Avenue, San Francisco, CA 94110, USA e-mail:
[email protected] A. Natale et al. (eds.), Cardiac Electrophysiology, DOI: 10.1007/978-1-84996-390-9_81, © Springer-Verlag London Limited 2011
343
344
F.M. Kusumoto et al.
Reference 1. Cohan L, Kusumoto FM, Goldschlager NF. Environmental effects on cardiac pacing systems. In: Kusumoto FM, Goldschlager N, eds. Cardiac Pacing for the Clinician. 2nd ed. New York, NY: Springer; 2008.
Fig. 81.1 Ventricular electrograms from the pacemaker during the high ventricular rate episode (V-Tachy). VS, ventricular sensed event; VP, ventricular paced event
Table 81.1 Recommendations for the use of electrocautery during surgery Rate response and any special algorithms should be programmed “off” prior to the surgical procedure. It is also important to ascertain ahead of time specific characteristics of that manufacturer’s magnet response since magnet application may be required during the surgical procedure. The pacemaker can be programmed to an asynchronous pacing mode (VOO, AOO, or DOO) just prior to surgery. Programming the pacemaker to an asynchronous mode may be preferable to placing a magnet over the device particularly if there is little or no intrinsic ventricular activity since the manufacturers’ magnet rates may be higher than desired (e.g., 100/min St. Jude devices). However, problems can arise during asynchronous pacing if the pacemaker competes with the patient’s intrinsic rhythm. This can result in the induction of tachyarrythmias. Use bipolar cautery if possible. This type of system has a short current path, which greatly reduces the area of significant electrical signal generation to roughly a 6-in. circle centered on the site of electrocautery application. In addition, an ultrasonic scalpel may reduce EMI. If a unipolar cautery system must be used, the indifferent electrode (grounding pad) should be placed such that the current flow between it and the cautery tip will not intersect the pacing system. For example, the thigh ipsilateral to the surgical site can be used in abdominal procedures. Good contact between the indifferent electrode and the skin must be maintained to reduce the chance of loss of contact, resulting in the pacing lead becoming a current sink for the electrocautery. Do not use electrocautery within 6 in. of the pulse generator. Use the minimum power settings required for adequate electrocautery. Use short bursts (preferably less than 1 s in duration) spaced more than 5 s apart. If electrocautery is causing inhibition of the pacemaker, a longer time between bursts will minimize hemodynamic effects. Monitor the patient for signs of pacemaker inhibition or triggering. If the ECG tracing is not clear because of interference from the use of cautery, the patient should be monitored manually or by some other means, such as ear or finger plethosgraphy or arterial pressure display. Provisions for alternative pacing and defibrillation should be readily available in the operating suite. Verify function of the pacemaker after the procedure with a complete pacing system interrogation and threshold determinations. With permission from Cohan et al.1
Case 82 Amin Al-Ahmad and Paul J. Wang
Case Summary An 85-year-old man with a history of coronary artery disease, aortic valve replacement, and a PPM placed for intermittent AV block comes to the emergency department with shortness of breath. The pacemaker is a dual chamber St. Jude Medical Victory DR 5810. Telemetry reveals ventricular pacing with a heart rate of 130 bpm (Fig. 82.1). Pacemaker parameters are as follows: Mode
DDD
LRL/URL
60/130 bpm
AV delay sensed/paced
150/200 ms
PVARP
250 ms
Automatic mode switch
On, detection rate 180 bpm
What are the possible causes of the rapid ventricular pacing? Should programming changes be made?
Case Discussion Ventricular pacing at the upper rate limit could be due to tracking of either sinus tachycardia or an atrial arrhythmia at that rate. If the atrial rate exceeds the maximum tracking rate, ventricular pacing will display a “Wenkeback” phenomenon and there will be regular changes in rate when the intrinsic atrial activation occurs during the pacemaker atrial refractory period (PVARP). Atrial fibrillation may also cause
pacing at the upper tracking rate, however, with automatic mode switch the device would be expected to change to a non-tracking mode (unless the atrial fibrillation electrograms are undersensed). With ventricular pacing at the upper tracking rate one must consider pacemaker-mediated tachycardia (endless-loop tachycardia). This can occur is a ventricular premature beat, results in retrograde atrial activation that is then tracked resulting in ventricular pacing, again with retrograde atrial activation, and the cycle continues. It can also occur if an atrial stimulus fails to capture the atrium, and the ventricular stimulus that follows in the DDD mode results in retrograde atrial activation, resulting in initiation of the tachycardia. The post-ventricular atrial refractory period (PVARP) decreases the likelihood of tracking of retrograde atrial beats as they often occur during this interval. In patients with a short PVARP who have retrograde conduction, PMT is possible when the retrograde conduction time is longer than the PAVRP. Current pacemakers have algorithms to prevent or terminate PMT. The PVC extension algorithm is one that results in PVARP extension after a PVC is detected by the pacemaker. In addition, if the pacemaker detects atrial sensing and ventricular pacing at a rapid rate, a PMT termination algorithm may prolong the PVARP interval for a single beat to terminate PMT. In this case, PMT was diagnosed and the PVARP was extended to 350 ms. This results in termination of PMT (Fig. 82.2). In addition, in this patient the upper tracking rate was decreased to 110 bpm given the patient’s history of coronary disease.
A. Al-Ahmad (*) School of Medicine, Stanford University, 300 Pasteur Drive, H-2146, Stanford, CA 94305, USA e-mail:
[email protected] P.J. Wang School of Medicine, Stanford University, 300 Pasteur Drive, H-2146, Stanford, CA 94305, USA e-mail:
[email protected] A. Natale et al. (eds.), Cardiac Electrophysiology, DOI: 10.1007/978-1-84996-390-9_82, © Springer-Verlag London Limited 2011
345
346 Fig. 82.1 Telemetry reveals ventricular pacing with a rate of 130 bpm
Fig. 82.2 This figure shows the termination of rapid pacing due to PMT once the PVARP is increased
A. Al-Ahmad and P.J. Wang
Case 83 Kenneth A. Ellenbogen
Case Summary A 71-year-old female has a dual chamber pacemaker implanted 3 years ago for symptomatic sinus bradycardia. She presents to the Emergency Room complaining of shortness of breath and palpitations. She is noted to be tachycardic with a regular, nonpaced rhythm of approximately 140 bpm. The patient has no known prior history of atrial arrhythmias.
Her pacemaker is interrogated revealing the following programmed parameters (Fig. 83.1). The surface lead II, atrial and ventricular marker channels, and atrial EGM are shown (Fig. 83.2) during the patient’s presenting tachycardia. What appears to be the underlying atrial rhythm based on the atrial EGM? Based on the marker channel, it appears that the device is sensing only every other atrial beat. Is this normal pacemaker function and what are the potential adverse effects from this phenomenon?
Fig. 83.1 Surface ECG, marker channel and atrial EGM
K.A. Ellenbogen Cardiac Electrophysiology and Pacing, Department of Cardiology, VCU School of Medicine, P.O. Box 980053, Richmond, VA 23298-0053, USA e-mail:
[email protected] A. Natale et al. (eds.), Cardiac Electrophysiology, DOI: 10.1007/978-1-84996-390-9_83, © Springer-Verlag London Limited 2011
347
348
K.A. Ellenbogen
Every other p wave falls in PVAB period
Fig. 83.2 Surface ECG, marker channel and atrial EGM
Case Discussion Inspection of the atrial EGM shows a regular atrial rate of 230 ms. This finding combined with the appearance of lead II is most consistent with atrial flutter as the underlying atrial rhythm. There is 2:1 conduction resulting in a ventricular rate of approximately 140 bpm. The patient’s device is exhibiting normal pacemaker function – it is inhibited, even though it is undersensing atrial flutter, and only senses every other atrial electrogram. The patient’s underlying ventricular rate of 140 bpm inhibits ventricular pacing. Every other atrial electrogram is falling within the post-ventricular atrial blanking period (PVAB) which is programmed in this device to 130 ms, and is the initial component of the post-ventricular atrial refractory period (PVARP). The PVAB is the portion of the PVARP where atrial sensed signals are “ignored.” As a result of this atrial “functional undersensing,” the device may fail to mode switch appropriately in this case from DDDR to the non-atrial tracking mode of DDI. Failure to mode switch
could result in tracking of the atrial flutter at the pacemaker’s programmed upper tracking rate if the sensed AV interval were to be exceeded or ventricular conduction absent. Additionally, the device underreports the time, duration, and frequency of atrial arrhythmias. Management in this case would first involve improved ventricular rate control, followed by adjustment of the PVAB if necessary to mitigate the observed functional oversensing. Manufacturer-specific algorithms to prevent underdetection of atrial flutter are also available, but still may lead to underdetection of atrial flutter.
Bibliography Israel CW, Barold SS. Failure of atrial flutter detection by a pacemaker with a dedicated atrial flutter detection algorithm. PACE. 2002; 25:1274-1277.
Case 84 Nora Goldschlager
Case Summary A 68-year-old woman underwent dual chamber pacing system implantation for sinus node dysfunction associated with 5–8 s pauses in atrial rhythm and severe presyncope. AV conduction was normal, and intraventricular conduction delays were not present. Pacing system implantation was accomplished without difficulty. One week later, during a routine wound check visit, a 12-lead ECG was recorded with and without magnet, and the pacemaker interrogated. Despite no known AV conduction system disease, the event counters indicated 100% ventricular pacing and 100% atrial sensing, which would not be expected in a patient with sinus node dysfunction and intact AV conduction. Fig. 84.1 shows the body surface ECG, the atrial and ventricular intracardiac electrograms, and the marker channel information. What is the problem with this pacing system and does it explain the event counter information? What would the ECG recorded in DOO mode show? How could the problem have been avoided?
designated by this manufacturer) coincides with a spontaneous QRS complex. The V pace event producing a P wave and the P sense event reflecting QRS complex sensing indicate lead reversal at the pulse generator. Since atrial pacing output is occurring through the ventricular channel, the atrial paced events are designated ‘‘V,’’ accounting for the 100% V pace events on interrogation. And since QRS sensing is occurring through the atrial channel, ‘‘P’’ is designated, accounting for the 100% P sense events on interrogation. Had a free-running and DOO mode rhythm strip and/or ECG been recorded at the time of implant and interpreted correctly, the diagnosis of lead reversal would have been made and the problem corrected immediately. An ECG recorded in DOO mode would have shown that the first of the two stimulus outputs (the atrial stimulus) was producing a QRS complex and the second stimulus output (the ventricular stimulus) was producing a P wave (if temporal opportunity existed); the first of these events would have been diagnostic of the problem.
Case Discussion Scrutiny of the Fig. 84.1 shows that the ‘‘V’’ (V pace event as designated by this manufacturer) produces a P wave, and not a QRS complex, and that the ‘‘P’’ (atrial sense event as
N. Goldschlager Department of Cardiology, University of California, San Francisco, Cardiology Division, 5G1, San Francisco General Hospital, 1001 Potrero Avenue, San Francisco, CA 94110, USA e-mail:
[email protected] A. Natale et al. (eds.), Cardiac Electrophysiology, DOI: 10.1007/978-1-84996-390-9_84, © Springer-Verlag London Limited 2011
349
350
Fig. 84.1 Program settings, surface ECG, marker channel and both the ventricular and atrial EGM
N. Goldschlager
Case 85 Anurag Gupta and Amin Al-Ahmad
Case Summary A 29-year-old female with history of transposition of the great arteries status post Mustard procedure and sick sinus syndrome status post dual chamber permanent pacemaker presents to the emergency room with 3-day history of cough and nasal congestion thought consistent with upper respiratory tract infection. However, the cardiac electrophysiology service is contacted by the emergency room providers because an ECG obtained with the patient lying supine in bed demonstrates atrial pacing at 105 ppm (Fig. 85.1). Interrogation of her Medtronic Kappa 400 ppm confirms current atrial pacing at 97 ppm (Fig. 85.2). Inhibition of pacing for assessment of underlying rhythm reveals predominantly sinus rhythm at 64 bpm. Device settings are as follows: Mode
DDDR
Mode switch
Off
Lower rate
60 ppm
Upper tracking rate
125 ppm
Upper sensor rate
140 ppm
Sensor
Integrated
Sinus preference
On
No other additional features or algorithms are programmed on. What are the possibilities accounting for her atrial pacing at an elevated rate that exceeds her lower rate limit?
Case Discussion The observation made by the emergency room providers is that she has an elevated atrial pacing rate. Notable reasons for atrial pacing at rate exceeding the lower rate limit include
but are not limited to: (a) sensor driven rate changes, discussed below; (b) programmed interventions such as rate smoothing, rate hysteresis, rate drop response, overdrive algorithms, and atrial fibrillation suppression algorithms; (c) reversion to alternate mode and/or rate, for example, as with magnet application, electromagnetic interference, and battery depletion; and (d) device malfunction including component and/or circuit failures with runaway. In this patient, interrogation of her device effectively rules out all categories except “inappropriate” sensor-driven rate. Though multiple sensing methods are employed, these notably include sensors of body motion and minute ventilation sensors, both of which are active in this patient as represented by the “integrated” designation of her sensor. More specifically, sensors of body motion generally utilize a piezoelectric crystal that reacts to mechanical stress, body vibrations, and/or acceleration. None of these stimuli are likely present in this stationary patient lying supine in her bed without any objects on her device. On the other hand, minute ventilation sensors measure thoracic impedance by repetitively emitting high frequency current (e.g., 16 times per second for Medtronic Kappa 400 ppm) and then measuring voltage between the PM can and lead electrodes; the changes in impedance amplitude and cycle length approximate changes in tidal volume and respiratory rate, respectively, and thus allow estimation of minute ventilation. In this patient, tachypnea and coughing from her respiratory infection led to increase in her minute ventilation sensor. Integrated dual sensors may mimic sinus node function response more closely and mitigate inappropriate sensing with cross-checking algorithms that detect discrepancies between the two sensors. In her case, with further change of her sensor from “integrated” to “activity” mode only (thus eliminating the minute ventilation sensor), she promptly reverted to an atrial sensed sinus rhythm at 64 bpm. This case illustrates the importance of recognizing reasons for changes in atrial rate, and demonstrates the role of sensors in pacing.
A. Gupta (*) and A. Al-Ahmad Cardiac Electrophysiology Service, Division of Cardiology, Department of Medicine, Stanford University Hospital and Clinics, 300 Pasteur Drive, Room H2146, Stanford, CA 94305-5233, USA e-mail:
[email protected] A. Natale et al. (eds.), Cardiac Electrophysiology, DOI: 10.1007/978-1-84996-390-9_85, © Springer-Verlag London Limited 2011
351
352
Fig. 85.1 12-lead ECG demonstrating atrial pacing at rate exceeding lower rate limit
Fig. 85.2 Intracardiac electrogram verifying atrial pacing at rate exceeding lower rate limit
A. Gupta and A. Al-Ahmed
Case 86 Fred M. Kusumoto, Jennifer Crain, and Nora Goldschlager
Case Summary A 64-year-old man with a history of atrial fibrillation has been receiving sotalol. A dual chamber pacemaker with antitachycardia pacing capabilities was implanted because of symptomatic bradycardia associated with sotalol therapy. Since being placed on sotalol several months ago he has had no episodes of symptomatic atrial fibrillation and wonders whether he can stop anticoagulation. He has hypertension, but otherwise does not have a history of coronary artery disease, heart failure, prior stroke or transient ischemic attack, or diabetes. Initial interrogation of his device is shown in Fig. 86.1. Does the device provide any additional clinical information that would help in patient management?
Case Discussion Interrogation of the device demonstrates a significant amount of asymptomatic atrial fibrillation. Episodes of asymptomatic atrial fibrillation are up to 12 times more common than episodes of symptomatic atrial fibrillation; and prolonged episodes (>48 h) are observed in up to 45% of patients, of which 38% are completely asymptomatic. Implantable cardiac rhythm devices provide a unique opportunity for evaluating arrhythmia burden and efficacy of therapy.
Experimental studies have suggested that antitachycardia pacing can be useful for terminating episodes of atrial fibrillation. In the Atrial Therapy Efficacy and Safety Trial (ATTEST), antitachycardia pacing was effective as defined by the device in terminating 54% of episodes of atrial fibrillation/atrial tachycardia. However, the overall atrial fibrillation burden was not decreased by the use of specialized algorithms for preventing and terminating atrial fibrillation. Thus, success defined by the device may or may not represent true efficacy. The summary and electrograms from a “successfully” treated episode in this patient is shown in Figs. 86.2 and 86.3. Notice that atrial fibrillation terminates 1 min after the pacing sequ ence. Despite being classified as “successful,” it is not clear whether the antitachycardia pacing truly had any effect on the arrhythmia. The patient later began developing side effects from the sotalol therapy and he subsequently underwent radiofrequency catheter ablation with almost complete elimination of his atrial fibrillation (Fig. 86.4). Recently published guidelines suggest that anticoagulation should be continued for at least 2 months after radiofrequency catheter ablation. After this initial period, decisions regarding continued anticoagulation should take individual risk factors into account. In this case, because his only risk factor for stroke was hypertension, after discussion with the patient the treating physician chose to discontinue anticoagulation after 2 months. The patient continues to be monitored for recurrence of atrial fibrillation through his implanted device.
F.M. Kusumoto (*) Department of Cardiovascular Diseases, Mayo Clinic, 4500 San Pablo Road, Jacksonville, FL 32224, USA e-mail:
[email protected] J. Crain Electrophysiology and Pacing Service, Division of Cardiovascular Diseases, Department of Medicine, Mayo Clinic, Jacksonville, FL 32224, USA N. Goldschlager Department of Cardiology, University of California, San Francisco, Cardiology Division, 5G1, San Francisco General Hospital, 1001 Potrero Avenue, San Francisco, CA 94110, USA e-mail:
[email protected] A. Natale et al. (eds.), Cardiac Electrophysiology, DOI: 10.1007/978-1-84996-390-9_86, © Springer-Verlag London Limited 2011
353
354
F.M. Kusumoto et al.
Initial interrogation: Cardiac compass trends
P = Program I = Interrogate _ = Remote
P
I
P
P
I
I
I
Page 1
P
PI
I
24 20 16 12 8 4 0
AT/AF total hours/day
V. rate duting AT/AF (bpm) max/day avg/day
>200 150 108 <50 Jun 2005
Aug 2005
Oct 2005
Dec 2005
Feb 2006
Apr 2006
Jun 2006
Fig. 86.1 Interrogation of the pacemaker showing amount of atrial arrhythmias: AT/AF atrial tachycardia/atrial fibrillation, bpm beats per minute
Date of Visit: 14-Jul-2006 13:05:19 9987 Software Version 1.2 Copyright Medtronic, Inc. 2002
Device: EnRhythm P1501DR
Treated AT/AF Episode #531 Type
ATP Seq
AT/AF
2
Success
ID#
Date
Time hh:mm
Yes
531
10-Jun-2006
16:31
Duration hh:mm:ss :02:25
A-A
V-V
Onset
Interval (ms) 1,500
Avg bpm A/V 221/97 AT/AF = 350 ms
Fast A.= 200 ms Detection
First ATP
19 s
Term.
1.1 min
1.1 min
1,200 900 600
400
200
−15
−10
−5
0
−5
0
0
−10
−5
0
Time (s)
Fig. 86.2 Summary of a “successfully” treated episode of atrial fibrillation. Notice that the atrial rate remains elevated (average cycle length 200 ms or approximately 300 bpm even after the antitachycardia pacing
(ATP)). Notice that the patient had an episode of atrial fibrillation that terminated spontaneously prior to the treated event
Case 86
Fig. 86.3 Electrograms from the episode. Atrial pacing (TP) does not terminate the atrial fibrillation. Notice the relatively controlled ventricular rate during the episode of atrial fibrillation. During sinus rhythm,
Fig. 86.4 The patient underwent radiofrequency catheter ablation for atrial fibrillation (arrow). After ablation a significant decrease in episodes of atrial fibrillation was recorded. I device interrogation, P device programmed
355
at the end of the strip, the patient has ventricular pacing. TD tachycardia detect, FD fibrillation detect, FS fibrillation sense
356
Bibliography Calkins H, Brugada J, Packer DL, et al. HRS/EHRA/ECAS expert consensus statement on catheter and surgical ablation of atrial fibrillation: recommendations for personnel, policy, procedures and follow-up. Heart Rhythm. 2007;4:816-861. Israel CW, Gronefeld G, Ehrlich JR, Li YG, Hohnloser SH. Long-term risk of recurrent atrial fibrillation as document by an implantable monitoring device: Implications for optimal patient care. J Am Coll Cardiol. 2004;43:47-52.
F.M. Kusumoto et al. Lee MA, Weachter R, Pollak S, et al. The effect of atrial pacing therapies on atrial tachyarrhythmia burden and frequency: results of a randomized trial in patients with bradycardia and atrial tachyarrhythmias. J Am Coll Cardiol. 2003;41:1926-1932. Page RL, Wilkinson WE, Clair EA, McCarthy EA, Pritchett EL. Asymptomatic arrhythmias in patients with symptomatic paroxysmal atrial fibrillation and paroxysmal supraventricular tachycardia. Circulation. 1994;89:224-227.
Case 87 Kenneth A. Ellenbogen
Case Summary
Case Discussion
A 54-year-old man presents to pacemaker clinic for routine device follow-up. He has a history of hypertension, typical isthmus-dependent atrial flutter, and atrial tachycardia. Six months ago he underwent implantation of a dual chamber Medtronic pacemaker with atrial antitachycardia pacing capabilities. On further questioning he reports an episode of palpitations that occurred last week “lasting several minutes.” Patient underwent interrogation of his device and the episode summary correlated with his reported symptoms. It showed an atrial tachycardia with an atrial median cycle length of 230 ms and a duration of 1.1 min. It was terminated by the first round of ramp pacing and had a ventricular rate of 120–140 bpm. The interval plot for the event is shown (Fig. 87.1) with what key event seen at the far right hand side of the interval plot? The accompanying atrial and ventricular marker channels at the onset of the atrial arrhythmia is shown (Fig. 87.2) followed by the marker channels and EGM (Atip-RV ring) during delivery of ATP therapy (Fig. 87.3) correlating with the far right hand side of the interval plot. What is the arrhythmia and what does the pacemaker do?
The episode summary reveals an atrial arrhythmia with a median cycle length of 230 ms falling in the AF zone which is successfully treated with a single sequence of ATP. Review of the interval plot (Fig. 87.1) shows the ATP delivered at the far right hand side with termination of the arrhythmia as evidenced by the “boxes” returning to an approximate atrial cycle length of approximately 800 ms. The corresponding marker channel (Fig. 87.3) shows the delivery of ATP consisting of eight pulses beginning at 190 ms with each subsequent pulse decrementing by 10 ms with successful termination of the atrial arrhythmia. Devices with ATP capability can have ATP therapy programmed either with “Ramp” where there is a train of decremental pulses or with Burst+ where there is a drive train of nondecremental pulses followed by two atrial extrastimuli. In those with atrial tachycardia cycle lengths >240 ms, Ramp ATP has been shown to be more efficacious than Burst+ with likely similar efficacy in atrial arrhythmias with cycle lengths <240 ms. In this case, the patient’s symptomatic atrial arrhythmia was effectively and consistently terminated by atrial antitachycardia pacing. This feature has proven to be less useful now, as many patients with organized atrial arrhythmias will undergo a curative ablative procedure if the arrhythmias are recurrent and symptomatic, while patients who have atrial fibrillation and more disorganized atrial arrhythmias are much less likely to have the arrhythmia terminated by pacing. However, there is still clearly a role for anti-tachycardia atrial pacing in selected patients.
K.A. Ellenbogen Cardiac Electrophysiology and Pacing, Department of Cardiology, VCU School of Medicine, P.O. Box 980053, Richmond, VA 23298-0053, USA e-mail:
[email protected]
A. Natale et al. (eds.), Cardiac Electrophysiology, DOI: 10.1007/978-1-84996-390-9_87, © Springer-Verlag London Limited 2011
357
358
Fig. 87.1 Stored interval plot of episode
K.A. Ellenbogen
Case 87
Fig. 87.2 Atrial and ventricular marker channel during episode
359
360
Fig. 87.3 Atrial EGM and marker channel during delivery of ATP
Bibliography Gulizia M, Mangiameli S, Orazi S, et al. Randomized comparison between Ramp and Burst+ atrial antitachycardia pacing therapies in patients suffering from sinus node disease and atrial fibrillation and implanted with a DDDRP device. Europace. 2006;8(7):465-73.
K.A. Ellenbogen
Case 88 Nora Goldschlager
Case Summary A 72-year-old man had a dual chamber pacing system implanted for bradycardia–tachycardia syndrome, associated with palpitations, chest discomfort, and presyncopal spells. The implant procedure was completed without difficulty. The patient remained in sinus rhythm throughout his recovery. At his first postimplant clinic visit, pacemaker interrogation revealed 100% atrial pacing and 100% spontaneous QRS complexes, considered to be appropriate for his previously documented arrhythmia. A 12-lead ECG recorded with a magnet placed over the pulse generator revealed ventricular capture; atrial stimulus outputs were not well seen and atrial capture could not be confirmed. (Note that for this manufacturer, the DOO rate is 100 ppm and the AV interval is 110 ms.) Rate programming to a low base rate was performed in order to assess the patient’s underlying rhythm and rate. Figure 88.1 illustrates the findings with the base rate programmed to 45 ppm. What is the problem? How is the marker channel information explained?
Case Discussion Figure 88.1 illustrates the body surface ECG, the ventricular intracardiac electrogram, and marker channel information. The atrial pacing stimulus (“A”) is producing a QRS complex,
indicating misconnection of the atrial and ventricular leads at the pulse generator. The atrial stimulus delivery is followed about 108 ms later by a sensed event in the ventricular channel (“R”). Because of the misconnection, the designated sensed “R” event is in fact an atrial event. This atrial event might be a native P wave buried within the paced QRS complex or a farfield signal due to the pacing stimulus delivery itself (cross talk); the differential diagnosis cannot be made from Fig. 88.1, nor would an atrial electrogram solve the problem, as it would register an electrical event in either case. Cross talk could be diagnosed if reduction in stimulus voltage output resulted in disappearance of the atrial sensed event (designated “R”), whereas a native P wave would be unaffected. Ventricular pacing through the atrial lead in this patient is not desirable, and could lead to atrial fibrillation, systolic dysfunction, AV dyssynchrony or 1:1 retrograde atrial activation with its attendant hemodynamic problems of hypotension, AV valve regurgitation, and clinical heart failure. Had the lead misconnection diagnosis been made at the time of implant the problem would have been corrected immediately, making pacing system revision unnecessary. It is mandatory to verify the origin of the depolarization resulting from pacing stimulus delivery: In this case, atrial stimulus delivery causing ventricular depolarization would have been obvious had appropriate attention been paid to the monitored rhythms during the implantation procedure.
N. Goldschlager Department of Cardiology, University of California, San Francisco, Cardiology Division, 5G1, San Francisco General Hospital, 1001 Potrero Avenue, San Francisco, CA 94110, USA e-mail:
[email protected] A. Natale et al. (eds.), Cardiac Electrophysiology, DOI: 10.1007/978-1-84996-390-9_88, © Springer-Verlag London Limited 2011
361
362
N. Goldschlager
Surface ECG Position Gain Filter Markers Position IEGM Position Gain Configuration Sweep Speed
ECG Controls
On 1 1 On On 2 On 3 10 V IEGM Bi 25
Programmed Parameters
mV/cm
Mode Base Rate A-V Delay P-V Delay Magnet Response Temporary 30
DDDR 45 250 175 Temporary Off Off
ppm ms ms
mV/cm mm/s
1.0 Second
Fig. 88.1 Programmed parameters, surface ECG, marker channel and ventricular EGM
25 Apr 2002 16:07
Case 89 Kenneth A. Ellenbogen, Rod Bolanos, and Mark A. Wood
Case Summary
Case Discussion
A 67-year-old female with a past medical history of sick sinus syndrome underwent implantation of a dual chamber pacemaker 2 years ago. The patient presented to her local emergency room (ER) after being bitten by a spider. While in the ER, she was placed on a cardiac and respiratory monitor. While receiving treatment for the spider bite, she was noted to have a wide complex tachycardia of 150 bpm on telemetry. The patient was given a 150-mg bolus of amiodarone and transferred to the nearest regional medical center for further management of her tachycardia. The 12-lead EKG of the tachycardia is shown (Fig. 89.1). Is there evidence of “normal pacemaker function” on the strip? The patient’s tachycardia suddenly stopped while en route to the regional medical center. Upon arrival there, her pacemaker was interrogated. Her device was a Guidant Pulsar MaxTM II dual chamber pacemaker with a 4469 FinelineTM 2 active fixation atrial lead and 4470 FinelineTM 2 active fixation ventricular lead. The device programmed parameters are shown (Fig. 89.2). What programmed parameter is particularly significant? Further interrogation of the device reveals the Trending Plot/Sensor Replay (Fig. 89.3) summarizing the Minute Ventilation (MV) data which in this instance was sampled by the device every 60 s over 102 h. What can one garner from the plot in terms of the onset, offset, and duration of the tachycardia, and thus the possible etiology of the patient’s tachycardia? What steps can be taken to prevent this problem in other cases?
Review of the 12-lead EKG shows a wide complex tachycardia with two pacing spikes associated with each QRS consistent with dual chamber pacing at a rate of 150 bpm. There is no evidence of undersensing or failure to capture (in the atrium or ventricle) on this strip. To know if the pacer was functioning properly, one would need access to the programmed parameters of the device (Fig. 89.2), which revealed that the device was programmed DDDR with a dual sensor rate response at a maximum sensor rate of 150 bpm matching the clinical “tachycardia.” Therefore, the pacer exhibited normal function based on its programming, but its sensor response was inappropriately triggered by a nonphysiologic cause, the respiratory monitor in the emergency room. Review of Fig. 89.3 shows that the onset and offset of the sensor-driven tachycardia along with a brief respite shown as a “blip” on the Trending Plot correlated to the patient’s exposure to the respiratory monitor and other electrical equipment in the emergency room. This patient’s sensor-driven tachycardia was initiated by the MV sensor which responds to changes in transthoracic impedance and not the accelerometer which is triggered by movement/motion affecting a piezocrystal located in the device generator. Review of the MV impedance measurements (Fig. 89.4) shows that the impedance first went up around the time of the patient’s presentation to the ER likely due to hyperventilation. The impedance rose again and remained elevated after the patient was placed on the monitor. Devices such as this patient’s may pace at the upper sensor rate in response to electrocautery, hyperventilation (physiologically appropriate), and to other medical electronic equipment such as respiratory monitors in clinical care areas and those associated with echocardiographic machines. Therefore, it is recommended that rate sensor modulated pacing features be disabled in any patient that is undergoing a surgical procedure or who is to be exposed to monitoring in a critical care unit.
K.A. Ellenbogen () Cardiac Electrophysiology and Pacing, Department of Cardiology, VCU School of Medicine, Richmond, VA 23298-0053, USA e-mail:
[email protected]
A. Natale et al. (eds.), Cardiac Electrophysiology, DOI: 10.1007/978-1-84996-390-9_89, © Springer-Verlag London Limited 2011
363
364
Fig. 89.1 12 lead ECG of presenting tachycardia
K.A. Ellenbogen et al.
Case 89 Fig. 89.2 Device programmed parameters
365 Brady Parameters Initial Value DDDR
Present Value DDDR
DYN
60 130 150 --
DYN
60 ppm 130 ppm 150 ppm -- ms
DYN
0.40 2.6 0.25 --
DYN
0.40 ms 2.6 V 0.25 mV -- ms
Mode Lower Rate Limit Max Tracking Rate Max Sensor Rate AV Delay (paced) Atrial Pulse Width Amplitude Sensitivity Refractory (PVARP) Ventricular Pulse Width Amplitude Sensitivity Refractory
AUTO
0.40 2.6 3.08 250
AUTO
0.40 ms 2.6 V 3.08 mV 250 ms
AV Delay
Dynamic AV Delay Maximum Delay Minimum Delay Sensed AV Offset AV Search Hysteresis Search Interval AV Increase
Initial Value On 270 120
Present Value On 270 ms 120 ms
−30
−30 ms
32 30
32 cycles 30 %
Sensor (s)
Accelerometer Activity Threshold Reaction Time Response Factor Recovery Time
Initial Value On Medium 30 AUTO 11 2
Present Value On Medium 30 sec AUTO 11 2 min
Minute Ventilation MV Lead Response Factor High Rate Response Factor HIgh Rate Break Point
On Ventricle 3 AUTO 85 115
On Ventricle 3 AUTO 85 % 115 ppm
Age Gender Auto Response ACC Initial Response Factor MV Initial Response Factor Sensor Rate Target Time Dependent Blend
66 Female
66 Female
On 8 3
On 8 3
110 On
110 ppm On
366
Fig. 89.3 Trending replay parameters and plot data
K.A. Ellenbogen et al.
Case 89 Fig. 89.4 Minute ventilation impedance data
Bibliography Seeger W, Kleinert M. An unexpected rate response of a minute ventilation dependent pacemaker. PACE. 1989;12:1707.
367
Case 90 Fred M. Kusumoto, Jennifer Crain, and Nora Goldschlager
Case Summary A 75-year-old man with an 8-year history of permanent atrial fibrillation comes to your office for a second opinion. He had a single chamber ventricular pacemaker placed 4 years ago for bradycardia. He began developing symptoms of increasing shortness of breath and had his pacing rate increased from 60 to 70 ppm. Progressive shortness of breath developed; an echocardiogram demonstrated severe mitral regurgitation (Fig. 90.1). He has now been referred for mitral valve replacement. What pacing system programming changes should be considered at this point? Fig. 90.1 Echocardiogram obtained during ventricular pacing
Case Discussion The patient has severe mitral regurgitation associated with ventricular pacing. The pacemaker was reprogrammed to a lower rate limit of 40 bpm to allow intrinsic ventricular activation; the mitral regurgitation improved significantly (Fig. 90.2), as did the patient’s symptoms. Several large randomized trials have reported an increased incidence in heart failure associated with right ventricular pacing. Case reports have described patients in which right
F.M. Kusumoto (*) Department of Cardiovascular Diseases, Mayo Clinic, 4500 San Pablo Road, Jacksonville, FL 32224, USA e-mail:
[email protected] J. Crain Electrophysiology and Pacing Service, Division of Cardiovascular Diseases, Department of Medicine, Mayo Clinic, Jacksonville, FL 32224, USA N. Goldschlager Department of Cardiology, University of California, San Francisco, Cardiology Division, 5G1, San Francisco General Hospital, 1001 Potrero Avenue, San Francisco, CA 94110, USA e-mail:
[email protected]
Fig. 90.2 Echocardiogram obtained during spontaneous cardiac rhythm
ventricular pacing significantly worsened mitral regurgitation. In one study of 256 patients that underwent AV nodal ablation and permanent pacing, approximately 5% of pati ents developed severe mitral regurgitation and four patients underwent mitral valve replacement; at surgery, no structural
A. Natale et al. (eds.), Cardiac Electrophysiology, DOI: 10.1007/978-1-84996-390-9_90, © Springer-Verlag London Limited 2011
369
370
abnormalities of the mitral valve were identified. Right ventricular pacing causes abnormal cardiac activation that in susceptible patients can worsen mitral regurgitation by changing the depolarization contraction timing sequence of the mitral valve apparatus and altering papillary muscle alignment. In patients that require pacing for rate support, biventricular pacing can improve right ventricular pacing–associated mitral regurgitation. In this case, ensuring minimal right ventricular pacing by simple programming changes improved the patient’s symptoms and averted the planned surgery.
F.M. Kusumoto et al.
Bibliography Berglund H, Nishioka T, Hackner E, et al. Ventricular pacing: a cause of reversible severe mitral regurgitation. Am Heart J. 1996;131:1035-1037. Sweeney MO, Hellkamp AS, Ellenbogen KA, et al. Adverse effect of ventricular pacing on heart failure and atrial fibrillation among patients with normal baseline QRS duration in a clinical trial of pacemaker therapy for sinus node dysfunction. Circulation. 2003; 107:2932-2937. Wilkoff BL, Cook JR, Epstein AE, et al. Dual-chamber pacing or ventricular backup pacing in patients with an implantable defibrillator: the Dual Chamber and VVI Implantable Defibrillator (DAVID) trial. JAMA. 2002;288:3115-3123.
Case 91 M. Eyman Mortada, Jasbir S. Sra, and Masood Akhtar
Case Summary
Case Discussion
An 83-year-old male with sick sinus syndrome received a permanent pacemaker. He has a history of coronary artery disease with preserved left ventricular function. He complained of multiple episodes of palpitations. Interrogation of his pacemaker revealed multiple episodes of tachycardia, where one of them was stored as shown in Fig. 91.1. His baseline 12-lead ECG demonstrated sinus rhythm with first-degree AV block. The tachycardia was induced in the electrophysiology lab with atrial extrastimuli. The results of ventricular entrainment of this tachycardia are shown in Fig. 91.2. What is the diagnosis and what would be the best management in this case?
Pacemaker interrogation revealed sinus rhythm in the first four beats. Then, a premature atrial contraction occurred in the post ventricular atrial refractory period (PVARP), followed by a prolonged ‘PR’ interval. This premature beat initiated the tachycardia with a very short, fixed ‘RP’ interval (70 ms) and a cycle length of 400 ms. The constant fixed ‘RP’ interval suggests ventricular-atrial linking due to retrograde activation over a pathway (AV nodal or accessory). The short ‘RP’ duration presumably excludes atypical AV reentry tachycardia and orthodromic AV reentry tachycardia via an accessory pathway. Therefore, the most likely diagnosis is typical AV nodal reentry tachycardia.
Fig. 91.1 Interrogation of the patient’s pacemaker revealed multiple episodes of tachycardia, one of which is shown here. (Simultaneous recording [from top to bottom] of: atrial bipolar recording, ventricular bipolar recording, and Marker’s channel.) AS atrial sensing; VS ventricular sensing
M.E. Mortada (*), J.S. Sra, and M. Akhtar Department of Electrophysiology, Aurora Cardiovascular Services, Aurora Sinai/Aurora St. Luke’s Medical Centers, University of Wisconsin School of Medicine and Public Health, 2801 W. Kinnickinnic River Parkway, #777, Milwaukee, WI 53215, USA e-mail:
[email protected] A. Natale et al. (eds.), Cardiac Electrophysiology, DOI: 10.1007/978-1-84996-390-9_91, © Springer-Verlag London Limited 2011
371
372
M.E. Mortada et al.
Fig. 91.2 Tachycardia was induced in the electrophysiology lab with atrial extrastimuli. Ventricular entrainment of this tachycardia is shown here. (Simultaneous recording [from top to bottom] of: surface ECG and leads I, II and V1, high right atrial recording, proximal coronary
sinus (CS 9,10) to distal coronary sinus (CS 1,2) recording, His recording, and right ventricular recording.) HRA high right atrium; CS coronary sinus; HISp proximal His; HISd distal His; RVa apical right ventricle
During the electrophysiology study, the post ventricular entrainment demonstrated ‘V-A-V’ response excluding atrial tachycardia. The short ‘VA’ duration seen during the tachycardia (46 ms), concentric atrial activation, the long postpacing interval (780 ms), and the long ventricular spike to
earliest atrial activity duration (200 ms) excludes AV reentry tachycardia via a septal accessory pathway and atypical AV nodal reentry tachycardia. Hence, the diagnosis is confirmed as typical AV nodal reentry tachycardia, and the best management in this case is AV nodal modification.
Case 92 Amin Al-Ahmad
Case Summary
Case Discussion
A 65-year-old man with a history of congestive heart failure due to ischemic cardiomyopathy and an implantable cardioverter defibrillator is admitted to the intensive care unit after a witnessed collapse. First responders found him to be in ventricular fibrillation and were able to successfully shock him to normal sinus rhythm using an external defibrillator. Interrogation of his Guidant Contak Renewal 3 HE ICD revealed an episode of ventricular fibrillation that was not successfully converted despite multiple maximum energy shocks (Fig. 92.1). Device therapy was exhausted. Records obtained from his implanting institution demonstrate failure to convert ventricular fibrillation during defibrillation threshold testing at 31 J, and failure once at 36 J. Maximum energy (41 J) was successful in terminating ventricular fibrillation. Device settings are as follows:
Defibrillation threshold testing is an important step during the implantation of ICDs. Both sensing of ventricular fibrillation and determination of the threshold energy needed to terminate ventricular fibrillation can be determined during defibrillation threshold testing. High defibrillation thresholds can be seen in younger patients, those taking amiodarone, patients with non-ischemic cardiomyopathy, and those with a lower ejection fraction.1 Patients with a defibrillation threshold that has a safety margin of less than 10 J should be considered for device modification. High defibrillation thresholds can occasionally be decrea sed by using sotalol or dofetolide. Removal of the proximal coil from the circuit or changing the polarity of shock may also be of value. Placement of a subcutaneous shock coil or a shock coil in the azygous vein may also decrease the defibrillation thresholds. Some devices allow for a change in the programmed defibrillation waveform pulse width or the tilt. It this case the best option would have been to advance the lead further into the right ventricle. Figure 92.2 shows the tip of the shock lead is placed in the right ventricular inflow area just beyond the tricuspid valve annulus. Advancement of the lead such that the tip reaches the right ventricular apex will likely improve the defibrillation threshold.
Mode
DDD
Lower rate limit/Upper rate limit
60/120 ppm
VT Zone (rate > 180 bpm)
ATP × 3 (burst), 41 J × 5
VF Zone (rate > 220 bpm)
41 J × 5
A chest X-ray taken in the intensive care unit is shown (Fig. 92.2). What steps could have been taken during the implant to prevent this from occurring?
A. Al-Ahmad School of Medicine, Stanford University, 300 Pasteur Drive, H-2146, Stanford, CA 94305, USA e-mail:
[email protected] A. Natale et al. (eds.), Cardiac Electrophysiology, DOI: 10.1007/978-1-84996-390-9_92, © Springer-Verlag London Limited 2011
373
374
A. Al-Ahmad
Fig. 92.1 Episode of ventricular fibrillation with failed 41 J shock
Reference 1. Russo AM, Sauer W, Gerstenfeld EP, et al. Defibrillation threshold testing: is it really necessary at the time of implantable cardioverter-defibrillator insertion? Heart Rhythm. 2005; 2:456-461.
Fig. 92.2 Chest X-rays showing biventricular system; note lack of left ventricular lead due to failure to place lead during implant
Case 93 M. Eyman Mortada, Jasbir S. Sra, and Masood Akhtar
Case Summary
Case Discussion
A 62-year-old male had a history of nonischemic cardiomyopathy with a left ventricular ejection fraction of 30%, congestive heart failure (New York Heart Association class II-III), and atrial fibrillation. His rhythm was converted to sinus rhythm with DC cardioversion and maintained with Tikosyn therapy. The cardiomyopathy persisted after optimizing his medical therapy for 3–4 months. Therefore, he received a dual-chamber implantable cardioverter defibrillator, and the device was programmed to suppress the atrial fibrillation by pacing the atrium 10 beats faster than his sinus rate to a maximum of 110 bpm with an AF suppression cycle of 30 beats. He has a first-degree AV block with AV delay of 205 ms, so the sensed AV delay was programmed at 250 ms and the paced AV delay was programmed at 300 ms. On an office visit, he complained of multiple episodes of palpitations. During the interrogation of the device, an episode of tachycardia had occurred, similar to multiple episodes that were disclosed on the stored EGMs (Fig. 93.1). What is the mechanism of the tachycardia? What is the best course of action? Fig. 93.2 shows a recording after shortening the PostVentricular Atrial Refractory Period (PVARP), shortening the AV delay, and turning ON the VIP (Ventricular Intrinsic Preference) mode. What is the most likely mechanism of the tachycardia? What is the best course of action?
Current models of pacemakers have advanced programming capabilities. There are multiple features that can be adapted to the individual patient’s needs. However, these sophisticated programming options can create complications of their own. This case is one example. Atrial fibrillation (AF) suppression, by pacing the atrium faster than the sinus rate, is a great tool to reduce the frequency of AF. This patient had no episodes of AF since the device was implanted. Yet, he had multiple symptomatic episodes of other types of tachycardia. The tachycardias started with a programmed pacing in the atrium at faster rates (10 bpm faster) than the sinus rate to suppress AF, and, at the same time, he had two premature ventricular captures (PVC). The second PVC had a retrograde atrial activity, which landed in the PVARP. It was then followed by an atrial pacing at the programmed cycle length (10 bpm faster than the sensed sinus rate) which did not capture the atrium due to its refractory period status (functional non-capture). After 300 ms (paced AV delay), the ventricle was paced and subsequently gave a retrograde atrial activity that landed in the PVARP again, leading to continuous-circuit symptomatic tachycardia. This manifestation had been described by Barold and Levine as pacemaker-repetitive nonreentrant ventriculoatrial synchronous rhythm1. Currently, there is no programmable algorithm available to detect and terminate this condition. There are a few algorithms which may prevent this situation from happening:
M.E. Mortada (*), J.S. Sra, and M. Akhtar Department of Electrophysiology, Aurora Cardiovascular Services, Aurora Sinai/Aurora St. Luke’s Medical Centers, University of Wisconsin School of Medicine and Public Health, 2801 W. Kinnickinnic River Parkway, #777, Milwaukee, WI 53215, USA e-mail:
[email protected]
1. Prolongation of the atrial escape interval to allow atrial myocardium time to recover: This algorithm can be programmed in St. Jude and Medtronic devices by extending the atrial escape interval to 300–350 ms from the sensed atrial activity when it lands in PVARP (e.g. PVC with retrograde atrial conduction). This would prevent the functional non-capture condition. 2. A decrease in the lower base rate (increase lower rate interval): In this scenario, it would be feasible to lower the base rate only if one turned off the atrial fibrillation suppression
A. Natale et al. (eds.), Cardiac Electrophysiology, DOI: 10.1007/978-1-84996-390-9_93, © Springer-Verlag London Limited 2011
375
376
Fig. 93.1 During the interrogation of the device, an episode of tachycardia had occurred. It was similar to multiple episodes that were disclosed on the stored EGMs. From top to bottom: Leadless ECG,
M.E. Mortada et al.
Markers’ channel, atrial bipolar recording, right ventricular bipolar recording, and key parameters. Labels used in this figure are defined in the text
Fig. 93.2 After shortening the PVARP, shortening the AV delay, and turning ON the VIP (Ventricular Intrinsic Preference) mode; this recording was observed
Case 93
mode. But that would increase the risk of atrial fibrillation. Hence, using atrial pacing for AF suppression and terminating this condition if it occurs reduces the number of pacing cycles. That, in turn, increases the frequency of detections of intrinsic atrial activity, leading to interruption of the pacemaker-repetitive nonreentrant ventriculoatrial synchronous rhythm, if it is present. 3. Shortening of the AV delay: This feature is helpful in patients who need ventricular pacing due to atrioventricular block. It allows for longer VA duration in a fixed cycle length, as in this case. When there is longer VA duration, the next atrial pacing comes further out, allowing more time for the atrial tissue to recover during the PVARP if there is atrial activity, leading to capture of the atrium and preventing this condition from happening. After shortening the PVARP, shortening the AV delay, and switching the Ventricular Intrinsic Preference (VIP) mode to ON, the patient again had two PVCs, which induced another type of tachycardia. After the second PVC (fourth sensed ventricular activity), the patient had a retrograde atrial activity during the PVARP, leading to pacemaker-repetitive nonreentrant ventriculoatrial synchronous rhythm for two beats. In these two beats, after the PVC, the paced AV delay was long, with no escaped intrinsic ventricular activity. Due to the programmed VIP, the paced AV delay was shortened, leading to termination of the above condition, but causing the retrograde atrial activity to be out of the PVARP due to the decremental VA conduction, and, therefore, tracking it with ventricular pacing and creating pacemaker-mediated tachycardia (PMT). It is less likely to be an atrial arrhythmia (atrial tachycardia) due to the fixed VA conduction time and the fixed rate at max track rate (120 bpm “500 ms”). There was no far-field ventricular sensing, since the atrial pacing was not inhibited, and there was atrial activity present at the same cycle length of the sinus rate (the sinus cycle length was 630–640 ms as seen in the first interrogation). Additionally, there were extra markers on the atrial lead at the same time as the markers on the ventricular lead. Finally,
377
the rhythm recorded over the ventricle during the tachycardia is all paced beats. Therefore, it cannot be spontaneous _ventricular tachycardia. The best course of action is to increase the PVARP to prevent the PMT from happening. However, this increases the risk of pacemaker-repetitive nonreentrant ventriculoatrial synchronous rhythm. The following modifications of the algorithm prevented the two observed tachycardias: 1. PVARP duration same as it was in the beginning. 2. Detection of PMT algorithm turned ON to terminate the tachycardia when it happens. 3. Shortened AV delay with longer ventricular intrinsic preference. 4. Extended atrial escape interval when atrial activity is sensed in the PVARP. 5. Reduced number of pacing cycles during atrial pacing in the AF suppression algorithm. Each patient is unique in regards to the function of his conduction system and its response to the different pacing maneuvers. Thus, it is essential to try the above algorithms at multiple intervals, to fit the requirements of each patient, until the issue is solved. As a last resort, it may be necessary to turn OFF the AF suppression program to prevent pacemaker-repetitive nonreentrant ventriculoatrial synchronous rhythm. If that is the case, it is important to remember to increase the PVARP Neither decreasing PVARP nor changing atrial sensitivity solves the problem of PMT2.
References 1. Barold SS, Levine PA. Pacemaker repetitive nonreentrant ventriculoatrial synchronous rhythm. A review. J Interv Card Electrophysiol 2001;5:45-58. 2. Ellenbogen KA, Kay GN, Lau CP, Wilkoff BL, Lau CP. Clinical Cardiac Pacing, Defibrillation and Resynchronization Therapy. 3rd ed. Edinburgh, UK: Saunders/Elsevier Health Science, 2006, pp 101.
Case 94 Kenneth A. Ellenbogen and Rod Bolanos
Case Summary A 68-year-old man undergoes implantation of a dual chamber ICD for sustained monomorphic ventricular tachycardia (VT) induced during electrophysiology study. His past medical history is significant for an ischemic cardiomyopathy with an LVEF of 40% and a prior history of syncope. The patient receives a Guidant VitalityTM 2 EL T 167 ICD with a 4087 FlextendTM active fixation atrial lead and a dual coil active fixation RelianceTM G 0185 Gore ICD lead. The R waves at implant were 14 mV and the P waves were 2.5 mV. The following stored EGM was recorded just prior to DFT testing (Fig. 94.1). The first and sixth QRS complexes are preceded by a V sensed event and the third QRS complex has two closely coupled V sensed events. Do these sensed events on the ventricular channel correlate with any physiologic electrical activity? The patient undergoes successful DFT testing at 14 J twice, and the events seen on the ventricular channel are no longer seen. The pocket is closed and the patient is returned to his room uneventfully. The next morning during the post implant device check, the following EGM is elicited during pacing (Fig. 94.2). The atrial output was 3.5 V at 0.5 ms. There is intermittent sensing on the ventricular channel of an event that appears to reproducibly follow the paced P wave. What are possible explanations for why the P wave is being sensed on the ventricular channel (far-field P wave oversensing)? What diagnostic study may aid in explaining this finding? Can the choice of ventricular lead affect the prevalence of this problem? Review of the patient’s morning chest x-ray shows that the ventricular lead has “come back” and has lost most of its “heel” in the right atrium. The decision is made to bring the patient back to the EP lab to reposition the ventricular lead.
K.A. Ellenbogen () Cardiac Electrophysiology and Pacing, Department of Cardiology, VCU School of Medicine, P.O. Box 980053, Richmond, VA 23298-0053, USA e-mail:
[email protected]
The ICD lead is positioned more distally out in the apex and a new true bipolar rate sense lead is inserted and positioned in the right ventricular septum. No further evidence of far-field P wave oversensing is seen. The patient underwent repeat DFT testing successfully at 14 J. While the pocket was being closed, the patient received two successive shocks while the cardiac monitor revealed sinus bradycardia in the 1940. Device interrogation shows surface lead I, Atrial egm, ventricular rate/sense egm, and marker channel (Fig. 94.3) just prior to delivery of the shocks. Does the rapid “sensed” ventricular event on the ventricular rate/sense EGM have a likely physiologic explanation?
Case Discussion The atrial, ventricular, and shock electrograms of one of the two episodes are shown (Fig. 94.4). The “rapid ventricular” event is seen only on the rate/sense lead. What should be the next step in diagnosing the cause of this problem? The pocket was opened and the set screws on the rate sense port examined. The screws were appropriately tightened but the artifact remained. A second generator was connected with resolution of the rapid, rhythmic activity previously seen only on the ventricular rate/sense lead. The original generator underwent evaluation by the manufacturer where a large tear in the rate/sense header sealing ring was seen. In Fig. 94.1, the VS event preceding the first and sixth QRS complexes have a sharp EGM and do not appear to correlate with any atrial activity or with any other part of the ventricular electrogram such as the T wave and should be considered “nonphysiologic.” The VS event preceding the third QRS complex could possibly be explained by far-field P wave oversensing. At that time no further evidence of inappropriate sensing on the ventricular channel was seen and DFTs were performed successfully. The next morning there is clear evidence of intermittent sensing of the P wave on the ventricular channel. In this case, lead position must be assessed, and this can be done with a post implant day chest x-ray. If the RV lead is dislodged, implanted
A. Natale et al. (eds.), Cardiac Electrophysiology, DOI: 10.1007/978-1-84996-390-9_94, © Springer-Verlag London Limited 2011
379
380
K.A. Ellenbogen and R. Bolanos
Fig. 94.1 Surface ECG, atrial and ventricular EGM prior to DFT testing
too proximally in the RV septum, or in the RV outflow tract then P wave oversensing can be seen. The P wave pacing output is not particularly high in this patient, but a high atrial pacing output would be more likely to result in far-field P wave oversensing. The ICD lead utilized in this case is an integrated bipolar lead and is more susceptible to this phenomenon. The chest film revealed that the ICD lead had pulled back some and this likely was the fundamental event that resulted in oversensing of the P wave on the ventricular rate/sense lead. The patient was appropriately returned to the lab and not only was the ICD lead positioned more distally, but a separate true bipolar rate sense lead was inserted to minimize the likelihood of future far-field P wave oversensing. Review of the EGMs associated with the two “inappropriate shocks” during pocket closure shows rapid, sharp, and rhythmic VS events that fell into the VF zone and resulted in two shocks. The VS EGMs do not correlate with any ventricular activity and there is no clear atrial activity that is being oversensed in this instance. When both the rate/sense EGM and Shock coil EGMs are compared, it is clear that only the rate/sense lead is being affected. Potential nonphysiologic causes of EMI typically effect both leads and EMI typically has a pulsed, high frequency appearance that is of varying duration seen
on all EGMs. Diaphragmatic myopotential oversensing is typically seen on the rate sense channel with integrated leads like the one used in this patient; but the sharp, well-demarcated mostly regular appearing EGMs are not typical of diaphragmatic myopotentials. The appearance of such nonphysiologic artifacts on only one lead particularly after a recent implant demands close inspection of that lead and the connection to the device header. The rate sense lead was inspected and was unremarkable. With insertion of the header torque wrench, the artifact could be reproduced and the generator was exchanged with elimination of the artifact. The original device underwent inspection by the manufacturer that revealed a large tear in the sealing ring of the rate/sense lead port. Compromise of the set-screw seal plug can lead to oversensing due to air escaping from the header connectivity cavity. This oversensing is typically seen on the ventricular rate sense lead and usually lasts only 1–2 days. This phenomenon can lead to inappropriate detection of ventricular tachycardia/ventricular fibrillation in approximately 20% of cases. To prevent damage to the seal plug during insertion of the torque wrench, the manufacturer recommended approaching the sealing ring at a 45° angle with the torque wrench prior to fully engaging the header screw.
Case 94
Fig. 94.2 Surface ECG, atrial and ventricular EGM on post-op day one
Fig. 94.3 Surface ECG, atrial and ventricular EGM after lead revision and DFT
381
382
K.A. Ellenbogen and R. Bolanos
Fig. 94.4 Atrial, ventricular and shock electrograms during episode that resulted in shock
Bibliography Cheung JW, Iwai S, Lerman BB, Mittal S. Shock-induced ventricular oversensing due to seal plug damage: a potential mechanism of inappropriate device therapies in implantable cardioverter-defibrillators. Heart Rhythm. 2005;2:1371-1375.
Lee BP, Wood MA, Ellenbogen KA. Oversensing in a newly implanted dual-chamber implantable cardioverter-defibrillator: what is the mechanism? Heart Rhythm. 2005;2:782-783. Weretka S, Michaelson J, Becker R, et al. Ventricular oversensing: A study of 101 patients implanted with dual chamber defibrillators and two different lead systems. PACE. 2003;26(Part 1):65-70.
Case 95 Byron K. Lee
Case Summary A 46-year-old man with a history of ventricular tachycardia and ICD implantation presented urgently to the Device Clinic 1 day after experiencing several shocks while having sex. After four painful and startling shocks, they suddenly stopped occurring. Several minutes later he was back to his baseline status and chose to come into the clinic the next day, rather than to go to the emergency department immediately. Interrogation of his Medtronic Marquis ICD showed that there were several episodes of tachycardia that crossed the lower rate cutoff of the VT zone and triggered therapies (Fig. 95.1). These episodes corresponded to when the patient felt his shocks the night before. The device settings are shown in Fig. 95.2. The intracardiac electrogram from one of the episodes of antitachycardic pacing (ATP) is shown in Fig. 95.3. Can we determine if the clinical arrhythmia is a supraventricular tachycardia or a ventricular tachycardia?
Case Discussion The beginning of the recording shows the clinical tachy cardia. There is AV association with one A electrogram for every V electrogram. This suggests that the clinical
arrhythmia is likely a supraventricular tachycardia but it does not rule out ventricular tachycardia. AV association can also occur with ventricular tachycardia when there is 1:1 retrograde conduction. Further examination of the intracardiac electrogram shows that during ATP there is clear capture of the ventricle. During ventricular capture, the atrial rate is unchanged and continues on at exactly the same rate as the clinical tachycardia. This indicates that the clinical tachycardia is driven by the atria, clinching the diagnosis of a supraventricular tachycardia rather than ventricular tachycardia. In this case, exertion preceded the shocks, strongly suggesting that the supraventricular tachycardia is simply sinus tachycardia. Therefore, the shocks were inappropriate. The shocks experienced by this patient could probably have been avoided if his therapy zones were programmed more appropriately for someone his age. 220 – age is a reasonable estimate of the maximum sinus rate for a patient. Therefore, you could expect this patient would reach sinus rates of around 174 bpm with heavy exertion. This rate is higher than the programmed lower rate cutoff for his VT zone which was set at 167 bpm. Therefore, it was not surprising that he had inappropriate shocks. The patient had the lower rate cutoff for his VT zone increased to 182 bpm and he has had no further inappropriate shocks since.
B.K. Lee Division of Cardiology, University of California, San Francisco, 500 Parnassus Avenue, San Francisco, CA 94143, USA e-mail:
[email protected] A. Natale et al. (eds.), Cardiac Electrophysiology, DOI: 10.1007/978-1-84996-390-9_95, © Springer-Verlag London Limited 2011
383
384
B.K. Lee
Fig. 95.1 Episode list ICD Model: Marquis DR 7274 Serial Number: PKC 138209H
Episode Lists Report
Jul 14, 2005 15:21:26 9966 Software Version 4.0 Copyright Medtronic, Inc. 2001
Last Interrogation: Jul 14, 2005 13:18:17 Episodes Last Cleared: Jun 27, 2005 12:30:27 VT/VF Episodes ID#
Date/Time
9 8 7
350 ms VT Rx 6 No Jun 30 15:36:04 VT Jun 30 15:35:07 VT 350 ms VT Rx 2 Yes Jun 30 15:23:02 VT 350 ms VT Rx 2 Yes Last Session (Jun 27, 2005) (Data prior to last session has not been interrogated.)
Type
V. Cycle
Last Rx
Success
Duration 2.8 min 47 s 39 s
SVT/NST Episodes ID#
Date/Time
A. Cycle
V. Cycle
Duration
Reason
(No data since last session.) Last Session (Jun 27, 2005) (Data prior to last session has not been interrogated.)
ICD Model: Marquis DR 7274 Serial Number: PKC 138209H Therapy 1 2 3 4 5 6
VT 167-188 bpm Burst Pacing Ramp Pacing CV 15 J CV 30 J CV 30 J CV 30 J
Parameter Summary Report
VF 188-500 bpm Defib 26 J Defib 30 J Defib 30 J Defib 30 J Defib 30 J Defib 30 J
Brady Pacing
Fig. 95.2 Device parameters
Mode Lower Rate Upper Tracking Rate Upper Sensor Rate
DDD 50 ppm 130 ppm 95 ppm
Jul 14, 2005 15:20:20 9966 Software Version 4.0 Copyright Medtronic, Inc. 2001
Case 95
Fig. 95.3 Intracardiac electrogram during ATP
385
Case 96 Amin Al-Ahmad and Paul J. Wang
Case Summary
Case Discussion
A 20-year-old man with a history of cardiomyopathy, long QT syndrome, ventricular tachycardia, and an implantable cardioverter defibrillator is admitted with multiple ICD shocks. The patient has a Medtronic Maximo DR 7278 that was implanted 2 years ago. Interrogation of the device reveals five episodes classified as nonsustained VF and three episodes classified as VF that are treated with shocks. A representative episode is seen in Fig. 96.1. Device parameters are as follows:
Examination of the episode electrogram reveals episodic bursts of high ventricular rates on the ventricular electrogram. The cycle length during these bursts is less than 200 ms and does not appear to be consistent with a physiologic signal. This electrogram is most consistent with noise related to lead fracture. Lead fracture is not uncommon in younger patients who are very active, and can be the cause of painful inappropriate shocks. An impedance rise is commonly seen with lead fracture, although it is worth noting that at times the lead impedance can be normal. Asking the patient to perform isometric contractions of the upper extremities while checking the lead impedance can unmask a high lead impedance when it is not immediately seen. Figure 96.2 illustrates the lead performance trends report; this shows the lead impedance fluctuating from a normal reading to a very high reading.
Mode
AAI
LRL/URL
70 bpm
VF zone
300 ms (200 bpm)
Ventricular sensitivity
0.3 mV
What is the cause of the shock? Should any programming changes be made?
Fig. 96.1 Episode electrogram prior to shock delivery
A. Al-Ahmad and P.J. Wang (*) School of Medicine, Stanford University, 300 Pasteur Drive, H-2146, Stanford, CA 94305, USA e-mail:
[email protected];
[email protected] A. Natale et al. (eds.), Cardiac Electrophysiology, DOI: 10.1007/978-1-84996-390-9_96, © Springer-Verlag London Limited 2011
387
388
A. Al-Ahmad and P.J. Wang
Ventricular pacing impendance At implant Last
584 Ω 584 Ω
Highest Lowest
>3,000 Ω 400 Ω
Ω >3,000 2,000 1,500 1,000 800 600 400 300 <200 08/11/06
10/20/06
12/29/06 03/09/07 05/18/07 Last 80 weeks (min/max per week)
07/27/07
10/05/07
12/13/07 12/26/07 Last 14 days
Fig. 96.2 Lead performance report obtained during interrogation of the device
There are no programming solutions that will reliably solve this problem. The fractured lead was extracted without difficulty and a new lead placed. Lead extraction can be
performed safely by experienced operators and would be appropriate to consider in a young individual who may require multiple leads over a lifetime.
Case 97 Kenneth A. Ellenbogen and Rod Bolanos
Case Summary A 67-year-old man with a history of syncope, an ischemic cardiomyopathy, and an ejection fraction of 25–30% is referred for electrophysiologic evaluation. Nonsustained VT is seen on telemetry after admission. The patient underwent electrophysiology studies and received a dual chamber Guidant Ventak PrizmTM DR HE ICD for inducible, sustained monomorphic ventricular tachycardia. The patient then presented to the emergency room after receiving a shock from his device while performing maintenance on his car engine. He denied any prior sensation of palpitations or lightheadedness before receiving the shock. The device was interrogated and the atrial, ventricular, and shock electrograms associated with the episode are shown (Fig. 97.1). What is the cause of the patient’s shock? What should be done about it?
Case Discussion On the top tracing at the far right hand side, rapid activity is seen on all three channels with both an atrial (AF) and a ventricular arrhythmia (VF) detected per the marker channel. What else occurs as a result of this rapid, sharp activity as seen on the ventricular channel? At the far left of the bottom tracing, a VF episode is declared. Therapy is then delivered with the remainder of the tracing at the bottom of the page showing the post-shock activity on all three channels. There appears to be persistence or recurrence of the rapid activity (180 ms) on both the atrial and ventricular channels despite
the shock. What is the larger amplitude EGMs seen on all three channels due to? The rapid, sharp electrograms associated with the shock episode are remarkable because they are seen on all three channels. The finding of rapid, high frequency activity of constant amplitude that is evident on all the channels occupying the entire cardiac cycle is highly characteristic of electromagnetic interference (EMI). When the noise is first evident on the top right of Fig. 97.1, not only is a tachyarrhythmia “detected,” but atrial and ventricular pacing is inhibited. Prior to the noise the patient was being paced in both the atria and ventricle at approximately 80 beats per minute. EMI thus can result in both oversensing leading to suppression of pacing and inappropriate shocks as seen in this case. Syncope may be due to inhibition of pacing in a pacing-dependent patient. After the shock is delivered (bottom of Fig. 97.1), the noise persists, and is interspersed with the larger EGMs seen on all the channels correlating with an underlying sinus rhythm at a lower rate of approximately 60 beats per minute. Another EGM characteristic seen here that is consistent with EMI is the larger amplitude of the signal on the farfield (Shock) electrogram compared to the nearfield (Ventricular) electrogram. This is in contrast to the reverse pattern seen typically with diaphragmatic myopotentials. In this case the diagnosis of inappropriate shocks due to EMI was made with the EMI source traced to the patient’s automobile alternator. As a result, the patient was instructed to get no closer than 2 ft from his car’s alternator as recommended by the manufacturer. Clearly, all patients receiving ICDs should be counseled about activities and equipment in the environment that may result in EMI.
K.A. Ellenbogen () Cardiac Electrophysiology and Pacing, Department of Cardiology, VCU School of Medicine, Richmond, VA 23298-0053, USA e-mail:
[email protected] A. Natale et al. (eds.), Cardiac Electrophysiology, DOI: 10.1007/978-1-84996-390-9_97, © Springer-Verlag London Limited 2011
389
390
K.A. Ellenbogen and R. Bolanos
Fig. 97.1 Atrial, ventricular and shock electrograms during stored episode
Bibliography Sweesy MW, Holland JL, Smith KW. Electromagnetic interference in cardiac rhythm management devices. AACN Clin Issues. 2004; 15(3):391-403.
Yerra L, Reddy PC. Effects of electromagnetic interference on implanted cardiac devices and their management. Cardiol Rev. 2007;15(6): 304-309.
Case 98 Kenneth A. Ellenbogen
Case Summary
Case Discussion
A 70-year-old man with a history of an ischemic cardiomyopathy and ventricular tachycardia was found to have “pacer spikes on the QRS” while on telemetry after elective hip replacement. The patient has a Medtronic GemDRTM dual chamber ICD. The telemetry strip is shown (Fig. 98.1). What is the underlying rhythm? There are two successive pacer spikes just after the initiation of the QRS on beats 8, 10, 12, and 14. Is this normal pacemaker function? Interrogation of the device revealed the following bradycardia settings (Fig. 98.2). Note the sensed and paced AV delays are 160 and 190 ms, respectively. Measurement of the interval separating the two spikes on the telemetry strip shows an interval of 120 ms. Next the surface lead II, atrial and ventricular marker channels, and ventricular EGMs are shown (Fig. 98.3). Here one sees that the two closely spaced spikes seen on telemetry correlate with an “A paced” event followed by a “V sensed” EGM followed by a second spike right after the QRS with an associated pacing artifact. Why does the second spike only appear after an “A paced” event? What is this called? What management options can avoid this phenomenon?
Review of the telemetry strip (Fig. 98.1) reveals sinus rhythm at approximately 80 bpm with all QRS complex morphologies appearing narrow and of normal duration. Beginning with the eighth QRS complex, there are two closely spaced spikes evident concurrent with the initial upslope of the QRS which do not result in a visible depolarization on the surface leads. The device settings reveal DDD programming with respective sensed and paced AV delays of 160 and 190 ms. The delay between the first and second spikes on the telemetry strip is 120 ms. This is normal pacemaker function as a result of undersensing of P waves on the atrial channel. The appearance of a second closely spaced pacer spike usually within 80–120 ms after an atrial paced depolarization is called Safety PacingTM and is designed to eliminate cross talk between the ventricle and the atrium with resultant oversensing or inhibition of ventricular pacing and asystole in a patient who has complete heart block. Cross talk can result from oversensing of atrial pacing stimuli on the ventricular lead due to the atrial pacing stimuli, a VPC, or other noise. Cross talk is more likely to occur
Fig. 98.1 Telemetry strip showing pacing spikes
K.A. Ellenbogen Cardiac Electrophysiology and Pacing, Department of Cardiology, VCU School of Medicine, Richmond, VA 23298-0053, USA e-mail:
[email protected]
A. Natale et al. (eds.), Cardiac Electrophysiology, DOI: 10.1007/978-1-84996-390-9_98, © Springer-Verlag London Limited 2011
391
392 Fig. 98.2 Pacemaker programmed parameters
K.A. Ellenbogen Modes/rates Mode Mode switch A. Detect rate Lower rate Upper tracking rate Upper sensor rate
Atrial lead DDD On 176 bpm 70 ppm 120 ppm 105 ppm
A-V Intervals Paced AV Sensed AV Rate adaptive AV Start rate Stop rate Minimum paced AV Minimum sensed AV
Fig. 98.3 Surface ECG, marker channel and Ventricular EGM
190 ms 160 ms On 60 bpm 150 bpm 140 ms 30 ms
Amplitude Pulse width Sensitivity Pace blanking
4V 0.6 ms 0.45 mV 240 ms
Ventricular lead Amplitude Pulse width Sensitivity Pace blanking Refractory PVARP PVAB
310 ms 150 ms
3V 0.4 ms 0.3 mV 200 ms
Case 98
393
Fig. 98.4 Surface ECG, marker channel and Ventricular EGM after programming changes made
with high programmed atrial outputs, unipolar atrial pacing (e.g., not possible in defibrillators), and with high programmed ventricular sensitivity (e.g., low values). Any ventricular sensed event after atrial pacing falling in the AV interval starting from the end of the ventricular blanking period out to 110 ms (“cross talk window”) will result in ventricular Safety PacingTM as this AV interval is deemed to be “nonphysiologic” by the device. In this case, the ventricular sensed event falling in the cross talk window is a conducted QRS that is preceded by an inappropriate atrial pacing spike as a result of undersensing of the P wave by the atrial lead. The phenomenon does not occur on every beat as the
patient’s rate is faster than the lower pacing rate by about 10 bpm. As a result, if the V–V interval is short enough, the device will not undersense the P wave initiating the series of events resulting in Safety PacingTM. To correct this problem, the atrial sensitivity can be increased so that P waves are sensed appropriately eliminating the inappropriate atrial pacing. In this case, the lower rate was increased to force atrial pacing and capture (Fig. 98.4) with the AV delays programmed to minimize the likelihood of the intrinsic QRS falling in the cross talk window and thus abolishing the Safety PacingTM.
Case 99 Amin Al-Ahmad and Paul J. Wang
Case Summary
Case Discussion
A 19-year-old man with a history of familial dilated cardiomyopathy, congestive heart failure, and a biventricular implantable cardioverter defibrillator has an episode of syncope followed by a shock. The device is a Medtronic InSync ICD 7272. The pacing lower rate limit is set to 60 beats/min and the upper rate limit is set to 140 beats/min. Pacing thresholds, sensing, and impedance were all unchanged from prior and within acceptable limits. The device is set to a fast VT zone via VF with a rate cutoff of 240 ms, the VF zone is set to 280 ms. The device is programmed to deliver a single sequence of ATP prior to shock in the FVT zone. The episode interval plot that resulted in syncope and shock is shown in Fig. 99.1. What is the cause of the syncope? And why does the patient receive a shock?
Evaluation of the interval plot (Fig. 99.1) reveals that the ventricular rate has been approximately 330 ms and is dissociated from the slower atrial rate indicating that ventricular tachycardia has been ongoing for at least 16 s prior to device detection. This ventricular tachycardia is the likely cause of the syncope in this patient with heart failure. The interval plot also shows an acceleration of the ventricular rate which results in detection and ultimate therapy with a 33-J shock. Evaluation of the stored electrograms (Fig. 99.2) of this episode show that the patient remains in ventricular tachycardia at 330 ms throughout the duration of the episode; however double counting of each ventricular tachycardia beat resulted in detection in the VF zone and VF therapy delivery. While it is unclear what causes the double counting, it is possible that a widening of the QRS during VT may result
VT/VF Episode #11 Report ICD Model: InSync ICD 7272 ID#
Date/Time
11
Jun 01 00:20:40
Type VF
V-V
Interval (ms) 1,800 1,500 1,200 900 600
Serial Number: PJP 234201S
Date of Visit: Aug 16, 2007
V. Cycle
Last Rx
Success
Duration
160 ms
VF Rx 1
Yes
22 s
A-A
VF = 280 ms
FVT = 240 ms
33.0 J
400 200
Fig. 99.1 Device interval plot showing the episode that resulted in syncope
−20
−15
−10
−5
0 5 Time (s) [0 = Detection]
10
15
20
25
A. Al-Ahmad (*) and P.J. Wang School of Medicine, Stanford University, 300 Pasteur Drive, H-2146, Stanford, CA 94305, USA e-mail:
[email protected];
[email protected] A. Natale et al. (eds.), Cardiac Electrophysiology, DOI: 10.1007/978-1-84996-390-9_99, © Springer-Verlag London Limited 2011
395
396
Fig. 99.2 Device electrograms showing VT with a cycle length of 330 ms that is double counted
A. Al-Ahmad and P.J. Wang
Case 99
in this phenomenon. In either case, it is worth noting that the VT was slower than the device rate cutoff and would not have resulted in therapy had it not been double counted.
397
Reprogramming the device to a lower rate cutoff or adding a VT zone would be appropriate. In addition, the addition of ATP may also be useful as it may reduce the likelihood of shock.
Case 100 Kenneth A. Ellenbogen
Case Summary
Case Discussion
A 56-year-old man with a history of ischemic cardiomyopathy presents to clinic for a routine 3-month evaluation of his ICD. He underwent implantation of a Guidant PrizmTM 2 DR Defibrillator after sustained monomorphic VT was elicited during electrophysiology study. On questioning, the patient denies receiving any shocks but does report a few episodes of dizziness “lasting several seconds.” His device is programmed DDD 60–120 beats per minute (bpm) with a VT zone >180 and VF >200 bpm. Interrogation of the device reveals several nonsustained episodes of rapid ventricular rates on the device arrhythmia logbook (Fig. 100.1) with one event categorized as VF with an associated diverted shock. The atrial, RV (nearfield), and shock (farfield) electrograms at the initiation of this event are shown (Fig. 100.2). At the top of the figure, one sees marked bradycardia with no ventricular pacing despite the programmed lower rate limit of 60 bpm. On the bottom of the figure, there are several ventricular sensed events falling within the VT and VF zones on the RV EGM with no correlating EGMs on the shock electrogram. The sharp, nearly constant activity being sensed on the RV channel is most likely due to what? What is the likely cause of the patient’s reported presyncope. What feature in this device guards against absolute inhibition of pacing in this case? As a result of the oversensing of the noise on the RV channel, an episode is declared by the device. At the top left hand side of Fig. 100.3, the capacitor begins to charge, but ultimately the shock is aborted. Why did this occur?
The nonsustained episodes seen on the arrhythmia logbook are all due to the same cause, device oversensing of noise on the ventricular channel. The noise is nearly constant across systole and diastole and is of regular amplitude. EMI can have this pattern, but its presence on only one channel makes EMI less likely the culprit. Diaphragmatic myopotentials are classically seen predominantly on the RV channel particularly if the lead is positioned in the inferior apex of the right ventricle. The EGMs due to oversensing of myopotentials tend to be more varied in amplitude and frequency with often respiratory variability. Further interrogation of the device revealed high impedance on the RV lead pointing to a lead fracture as the underlying problem. The patient’s symptoms were likely due to the profound bradycardia caused by inhibition of ventricular pacing from the oversensing of the noise on the RV lead. Pacing was intermittently seen as evidence by the VP-Ns seen on the marker channel due to a feature of the device where if there is “continuous noise” during the noise window of the device (VN markers on the marker channel), the device will pace at the lower rate limit to prevent asystole from oversensing noise. In this instance, the oversensing nearly resulted in delivery of a shock after the device declared a VF episode (bottom left, Fig. 100.2), but the therapy was diverted during redetection (Fig. 100.3). This particular device requires 6/10 beats in the tachycardia zone during redetection to proceed with therapy.
K.A. Ellenbogen Cardiac Electrophysiology and Pacing, Department of Cardiology, VCU School of Medicine, 980053, Richmond, VA 23298-0053, USA e-mail:
[email protected] A. Natale et al. (eds.), Cardiac Electrophysiology, DOI: 10.1007/978-1-84996-390-9_100, © Springer-Verlag London Limited 2011
399
400 Fig. 100.1 Device arrhythmia logbook
K.A. Ellenbogen Guidant
VENTAK PRIZM 2 DR Arrhythmia Logbook Report
Date/Time
Episode
Rate bpm
Type zone
34 33 32 31 30 29 28 27 26 25 24 23 22 21 20 19 18 17 16 15 14 13 12 11 10 9 8 7 6 5 4 3 2 1
05 - DEC - 03 04 - DEC - 03 04 - DEC - 03 04 - DEC - 03 27 - NOV - 03 10 - NOV - 03 01 - OCT - 03 09 - SEP - 03 27 - JUL - 03 15 - JUL - 03 14 - JUL - 03 13 - APR - 03 13 - JAN - 03 07 - JAN - 03 31 - DEC - 02 29 - DEC - 02 29 - DEC - 02 23 - DEC - 02 25 - NOV - 02 24 - NOV - 02 24 - NOV - 02 24 - NOV - 02 24 - NOV - 02 24 - NOV - 02 24 - NOV - 02 24 - NOV - 02 22 - NOV - 02 22 - NOV - 02 15 - OCT - 02 14 - MAY - 02 20 - JAN - 02 08 - AUG - 01 08 - AUG - 01 08 - AUG - 01
End of Report
09 : 58 19 : 45 19 : 45 16 : 51 20 : 11 19 : 32 20 : 19 15 : 03 19 : 05 21 : 25 20 : 02 13 : 39 20 : 54 19 : 54 19 : 44 20 : 34 08 : 18 22 : 44 00 : 22 23 : 20 22 : 28 22 : 18 20 : 46 07 : 07 06 : 45 03 : 15 04 : 35 04 : 01 19 : 45 11 : 06 14 : 58 10 : 49 10 : 41 10 : 35
ATR ATR ATR ATR Spont Spont Spont ATR Spont Spont Spont ATR Spont Spont Spont Spont ATR Spont PMT PMT PMT PMT PMT PMT PMT PMT PMT PMT Spont ATR ATR Induce Induce Induce
VF
VF VF VF
137 253 150 122 138 132 174 100 130 164 176 104 169 143 185 118 93 155 120 120 120 120 120 120 120 120 120 120 155 105 89 255 316 245
Therapy/ Duration 00 : 16 m : s 00 : 07 m : s 00 : 08 m : s 00 : 05 m : s Nonsustained Nonsustained Nonsustained 00 : 05 m : s Nonsustained Nonsustained Nonsustained 00 : 14 m : s Nonsustained Nonsustained Diverted Nonsustained 00 : 06 m : s Nonsustained
Nonsustained 00 : 06 m : s 00 : 07 m : s 17 J 11 J, 17J 31J
V > A
Stab ms
A F i b
Ons
F F F
N/R N/R N/R
O O O
50% 56% 59%
F F F
N/R N/R N/R
O O O
56% 59% 34%
F F T F
N/R N/R 219 N/R
O O O O
59% 66% 59% 47%
F
N/R
O
72%
F
N/R
O
72%
T T T
41 54 96
O O O
N/R N/R N/R
Case 100
Fig. 100.2 Atrial, ventricular and shock electrograms during onset of stored episode
401
402
Fig. 100.3 Atrial, ventricular and shock electrograms during stored episode. Note, therapy diverted
K.A. Ellenbogen
Case 101 Amin Al-Ahmad and Paul J. Wang
Case Summary A 68-year-old man with coronary artery disease, congestive heart failure, and ICD is admitted with multiple ICD shocks for an ablation procedure. The device, a Boston Scientific Vitality HE, is interrogated. Atrial and ventricular lead parameters are within acceptable limits and are unchanged from prior device testing. An example of a representative episode is shown in Fig. 101.1. His device is set with a VT zone at 165 bpm and a VF zone at 200 bpm. He is programmed to receive two ATP trains followed by shock in the VT zone, and to maximum energy (41 J) shocks in the VF zone. Would an atrial flutter ablation result in a reduction in the number of shocks?
Case Discussion Examination of the stored electrogram reveals atrial flutter with ventricular pacing followed by an acceleration of the ventricular rate. Is this acceleration of the ventricular rate
conducted atrial flutter, in which case an atrial flutter ablation would be potentially helpful. Or is this ventricular tachycardia? In this case we do not have an intrinsic electrogram in sinus rhythm (or atrial flutter) to compare the shock morphology with that of the rapid ventricular rate. While conducted atrial flutter is possible, it is very unlikely as we would not expect a patient who is in ventricular pacing during atrial flutter to suddenly begin to rapidly conduct. Indeed, further history reveals that the patient is pacemaker dependent. Thus, this episode represents VT. Programming the device to add more ATP or to change the ATP to be more aggressive may be helpful, although this may result in a higher risk of inducing VF. In addition, he has not been responding to ATP and had been receiving multiple shocks despite ATP. Antiarrhythmic medications may also play a role in the management of this condition. In this patient, a VT ablation resulted in a significant reduction of spontaneous VT. Atrial flutter ablation was also performed at that time.
A. Al-Ahmad (*) and P.J. Wang School of Medicine, Stanford University, 300 Pasteur Drive, H-2146, Stanford CA 94305, USA e-mail:
[email protected];
[email protected] A. Natale et al. (eds.), Cardiac Electrophysiology, DOI: 10.1007/978-1-84996-390-9_101, © Springer-Verlag London Limited 2011
403
404
Fig. 101.1 The shock electrogram showing a typical episode resulting in shock
A. Al-Ahmad and P.J. Wang
Case 102 Kenneth A. Ellenbogen
Case Summary A 72-year-old man with a history of congestive heart failure due to non-ischemic cardiomyopathy and an implantable defibrillator presents to clinic for evaluation of palpitations and presyncope. The patient denies having received any shocks since his device was implanted 9 months ago. Prior interrogation of his device showed normal function except for atrial lead “sensing issues.” The atrial sensitivity was programmed to 0.9 mV. Interrogation of his Medtronic GemTM DR 7271 reveals several VT/VF episodes and no shocks were delivered. Device settings are as follows: Mode
DDD
Lower rate limit/upper rate limit
60/120
VT zone (rate > 180)
ATP × 3 (burst), 21 J × 1, 31 J × 4
VF zone (rate > 210 bpm)
31 J × 5
The following atrial and ventricular EGMs with the corresponding marker channels are shown (Fig. 102.1) during detection of a tachyarrhythmia in the VT zone. What is the
differential diagnosis for the tachyarrhythmia falling in the VT zone? Why does the device characterize the arrhythmia as VT? Once the device commits to therapy, ATP is delivered with the third ATP resulting in termination of the tachycardia (Fig. 102.2).
Case Discussion In this case, the third ATP sequence resulted in termination of the tachyarrhythmia with return to normal sinus rhythm (NSR) with some PVCs. Review of the sensed atrial rate (marker channel) during detection (Fig. 102.1) shows no clear correlation with the ventricular rate with clearly more Vs than As. Yet on the atrial channel the atrial EGM shows an atrial rate concordant with the ventricular rate as sensed on the ventricular lead. Here there are rapid atrial EGMs that are at the same rate as the ventricular EGMs, but are not sensed by the device as evidenced by their absence on the marker channel along with slower atrial EGMS falling at the
Fig. 102.1 Stored EGM showing the atrial and ventricular EGM, the marker channel and intervals at the onset of tachycardia event
K.A. Ellenbogen Cardiac Electrophysiology and Pacing, Department of Cardiology, VCU School of Medicine, Richmond, VA 23298-0053, USA e-mail:
[email protected] A. Natale et al. (eds.), Cardiac Electrophysiology, DOI: 10.1007/978-1-84996-390-9_102, © Springer-Verlag London Limited 2011
405
406
K.A. Ellenbogen
Fig. 102.2 Stored EGM showing ATP terminating the tachycardia
sinus bradycardia rate which are sensed and evident on the marker channel. Based on these observations, the arrhythmia mechanism differential diagnoses include SVT with a rapid ventricular response with atrial undersensing, ventricular tachycardia with VA dissociation (appropriate A sensing), and less likely ventricular tachycardia with VA conduction and atrial undersensing of the retrograde As. VT with 1:1 VA conduction and atrial undersensing is less likely as the sensed atrial rate appears to march through at a regular rate independent of the ventricular rate. If there was 1:1 VA conduction, the atrial rate would be dictated solely by the ventricular rate. SVT with one to one conduction is less likely as the atrial sensed events march out regularly independent of the ventricular rate. This event is an example of VT with VA dissociation, appropriate atrial sensing during sinus rhythm,
and an atrial EGM displaying an atrial signal plus far-field R waves that are appropriately not sensed as evident on the marker channel (although far-field R waves are “confusingly” visible on the marker channel). The device correctly classified the arrhythmia as VT based on the ventricular EGM and V > A. Additionally, this is an example of “farfield” and “near-field” electrograms. On the atrial channel, the near-field EGM is the sharp signal and this is sensed correctly by the device as atrial activity. The other electrogram on the atrial channel is far field, and it represents ventricular activity sensed in the atrium. It is a “far-field” EGM as it has a low frequency, “non-sharp” appearance consistent with a sensed signal from a further away source. It is appropriately not sensed on the atrial channel, but certainly is confusing when one first looks at the recorded strip shown here.
Case 103 Amin Al-Ahmad and Paul J. Wang
Case Summary
in the shock? Why does the patient still receive a shock despite termination of the arrhythmia?
A 58-year-old male with a history of congestive heart failure and an ICD comes to clinic after receiving an ICD shock while mowing the lawn. The patient was not symptomatic during the episode. His left ventricular ejection fraction is 25% and he is being treated with appropriate medications including an angiotensin converting enzyme inhibitor and a beta-blocker. Evaluation of his Medtronic EnTrust ICD shows lead thresholds, sensing, and impedance in an acceptable range and similar to prior device testing. The device is programmed with a VT zone set to 400 ms (150 bpm) and a VF zone set to 320 ms (188 bpm). Interrogation of the episode that resulted in shock reveals a 1:1 tachycardia with a cycle length of 300 ms (200 bpm) that spontaneously terminates prior to shock (Figs. 103.1 and 103.2). What is the rhythm that results
Case Discussion Examination of the interval plot reveals that the patient is initially tachycardic with a heart rate of 133 bpm prior to detection (Fig. 103.1). The electrogram (Fig. 103.2) is consistent with sinus tachycardia. The heart rate then accelerates to approximately 300 ms (200 bpm). The electrogram during this 1:1 tachycardia is similar to that of the electrogram prior to rate acceleration. This suggests that this rhythm acceleration is more consistent with a supraventricular tachycardia (SVT) such as atrial tachycardia with 1:1 conduction, rather than ventricular tachycardia with retrograde 1:1 conduction.
ATP Shocks Success ID# Seq
Type VF
0
35J V−V
Yes
Date
Time Duration Avg bpm Max bpm Activity at hh:mm hh:mm:ss A/V A/V Onset
21 08-Dec-2006 23:53
:15 214/214
VF = 320 ms Detection
A−A
---/---
Rest
VT = 400 ms 34.3 J
Interval (ms) 1,500 1,200 900 600
Term.
400 200
Fig. 103.1 Interval plot showing a 1:1 tachycardia that results in shock
−20
−15
−10
−5
0 Time (s)
5
10
15
20
A. Al-Ahmad (*) and P.J. Wang School of Medicine, Stanford University, 300 Pasteur Drive, H-2146, Stanford, CA 94305, USA e-mail:
[email protected];
[email protected] A. Natale et al. (eds.), Cardiac Electrophysiology, DOI: 10.1007/978-1-84996-390-9_103, © Springer-Verlag London Limited 2011
407
408
Fig. 103.2 Episode electrograms and marker channels
A. Al-Ahmad and P.J. Wang
Case 103
Despite the fact that this SVT spontaneously terminates and the heart rate decreases to 133 bpm, the patient still receives a shock. To understand this we need to understand the reconfirmation algorithm in this device. As the device capacitor charge ends (CE on the marker channel), the device attempts to reconfirm prior to discharge. To “reconfirm” this device requires two intervals to be detected after the charge end that are faster than the VT interval + 60 ms, in this case 460 ms. In this case the intervals are 450 ms; thus although these intervals are lower than the VT rate cutoff, they still result in therapy delivery. Reprogramming of the device to increase the VT rate cutoff may eliminate this problem; however the risk of doing
409
this is possible under detection of VT. Another option may be to increase the number of intervals needed to detect VF from 12/16 to 18/24; however this may cause delay in therapy. In this case it was determined that the patient was not taking his beta-blockers. Resumption of beta-blocker therapy would potentially decrease the maximum rates during exertion and would decrease the likelihood of this type of inappropriate shock without any device programming changes. This case illustrates the importance of understanding the detection and reconfirmation algorithms of ICDs.
Case 104 Kenneth A. Ellenbogen
Case Summary A 65-year-old woman with a history of syncope and congestive heart failure due to nonischemic cardiomyopathy presents to the ER after reportedly receiving several shocks from her ICD. Telemetry in the ER shows normal sinus rhythm with some single premature ventricular contractions (PVCs). Interrogation of her Medtronic MaximoTM DR 7278 reveals that the device had delivered three shocks for a tachyarrhythmia in the VF zone. Device settings are as follows: Mode
DDD
Lower rate limit/upper rate limit
60/120
VT zone (rate > 166 bpm)
ATP × 1 (burst), 21 J × 1, 31 J × 4
VF zone (rate > 188 bpm)
31 J × 5
The atrial and ventricular EGMs along with the marker channel are shown (Fig. 104.1) leading up to the delivery of the first shock. The device categorizes the ventricular arrhythmia as VF and the atrial arrhythmia as an SVT (double tachycardia). Is this correct and why? The interval plot is shown (Fig. 104.2). What effect does the first 34.5-J shock have on the double tachycardia?
Case Discussion At the initiation of the event, a regular rapid arrhythmia of 190 ms is evident on the atrial channel with a coexisting irregular ventricular rhythm well below the VT zone cutoff of 166 bpm. The most likely initial diagnosis is atrial
K.A. Ellenbogen (*) Cardiac Electrophysiology and Pacing, Department of Cardiology, VCU School of Medicine, P.O. Box 980053, Richmond, VA, USA, 23298-0053 e-mail:
[email protected]
fibrillation with a rapid and irregular ventricular rhythm. This relationship is evident at the beginning of the interval plot (Fig. 104.2). Subsequently, the ventricular rate abruptly drops into the VF zone twice as self-terminating rapid VT/VF episodes followed by a third VF episode that is sustained long enough to not only be detected, but to result in delivery of a 34.5-J shock. An alternative but less likely explanation is that these are brief episodes of a rapid and regular ventricular response to the atrial arrhythmia. This is less likely because the patient’s atrial rate is extremely rapid (although monomorphic in appearance), and thus more consistent with atrial fibrillation, than an organized atrial tachycardia. Additionally, there are subtle changes in the near field, or ventricular sensed electrogram suggesting this arrhythmia is more likely a ventricular tachyarrhythmia (a rapid SVT with aberration cannot be excluded). This first shock terminates the atrial arrhythmia (Fig. 104.3) and transiently terminates ventricular tachycardia, as well. The ventricular tachycardia quickly reinitiates, with the rate almost identical to the rate of the rapid and regular tachycardia seen prior to the first shock, and requiring a second 34.5-J shock that is also unsuccessful. In Fig. 104.4, we see the successful termination of the ventricular tachyarrhythmia with the third 34.5 J shock. It is worth noting here, that the atrial rhythm is now sinus and we clearly have a ventricular tachycardia with AV dissociation prior to shock delivery. The ventricular electrogram recorded from the rate sensing electrodes here is also different from what the rate sensing electrogram recorded during the atrial tachyarrhythmia prior to the first shock during what we thought were short bursts of ventricular tachycardia, even though the rates are similar. We have no simple explanation for this discrepancy. This case is most likely an example of appropriate classification by a device of a double tachycardia with termination of the atrial arrhythmia by the first shock followed by subsequent termination of the ventricular tachycardia by the third shock. In patients with VT/VF, dual tachycardias are quite common with a prevalence as high as 20%. Additionally, when therapy delivered for the VT/VF fails to convert the atrial arrhythmia, the time to the next VT/VF therapy is significantly shorter than
A. Natale et al. (eds.), Cardiac Electrophysiology, DOI: 10.1007/978-1-84996-390-9_104, © Springer-Verlag London Limited 2011
411
412
Fig. 104.1 Stored EGM showing the atrial and ventricular EGM, the marker channel and intervals at the onset of the episode
K.A. Ellenbogen
Case 104
413
Fig. 104.2 Episode interval plot
AFL terminates
VT continues
Fig. 104.3 Stored EGM showing the first shock
414
K.A. Ellenbogen
Fig. 104.4 Stored EGM showing final successful shock
when both arrhythmias are terminated by the initial shock. It has been shown that implanting dual chamber devices with atrial therapy capabilities in those with standard ICD indications does not reduce the incidence of VT/VF episodes.
Bibliography Gradaus R, Seidl K, Korte T, et al. Reduction of ventricular tachyarrhythmia by treatment of atrial fibrillation in ICD patients with
dual-chamber implantable cardioverter/defibrillators capable of atrial therapy delivery: the REVERT-AF Study. Europace. July 2007;9(7):534-539. Stein KM, Euler DE, Mehra R, et al. Do atrial tachyarrhythmias beget ventricular tachyarrhythmias in defibrillator recipients? J Am Coll Cardiol. July 2002;40(2):335-340.
Case 105 Kenneth A. Ellenbogen and Rod Bolanos
Case Summary A 59-year-old man presented to a local Emergency Room after receiving six shocks from his defibrillator. He reported feeling palpitations and lightheadedness followed by six consecutive shocks. He denied frank syncope. The patient had a single chamber Medtronic ICD and a MedtronicTM 6936 defibrillator lead implanted approximately 2 years ago.
Interrogation of his device (Fig. 105.1) reveals an episode lasting 2 min during which six episodes of “VF” (rate < 330 ms) were detected resulting in a total of six shocks. RV lead impedance (tip to can) was stable and within normal limits. Review of the short interval counter showed multiple brief nonsustained episodes with intervals <140 ms. The rate/sense and marker channels are shown for the first episode (Fig. 105.2). Is there evidence of normal device
Fig. 105.1 Stored EGM showing the atrial and ventricular EGM, the marker channel and intervals at the onset of the episode
K.A. Ellenbogen () Department of Cardiology, VCU School of Medicine, 980053, Richmond, VA 23298-0053, USA e-mail:
[email protected] A. Natale et al. (eds.), Cardiac Electrophysiology, DOI: 10.1007/978-1-84996-390-9_105, © Springer-Verlag London Limited 2011
415
416
K.A. Ellenbogen and R. Bolanos
Fig. 105.2 Episode interval plot
function in this case? Immediately after delivery of the initial shock, the device detects a second episode also in the VF zone (Fig. 105.3). What is remarkable about the marker channel intervals in this episode compared to the first episode? The patient goes on to receive four more successive shocks for “events” falling in the VF zone. The RV EGM is shown (Fig. 105.4) along with the marker channel for one of these episodes. What is potentially being over sensed, and what are the steps that may be taken to avoid the problem? Why were shortly coupled intervals only seen after the first shock?
Case Discussion In this case, a patient received multiple shocks over a short period of time. The first episode (Fig. 105.2) shows appropriate device function with detection and termination of an event in the VF zone. This is an episode of rapid ventricular tachycardia. Appropriate therapy is delivered for this episode. However, this episode is quickly followed by five subsequent shocks. The second episode detected (Fig. 105.3) immediately after the delivery of the first shock is characterized by several short coupled sensed intervals that are nonphysiologic (<150 ms) and is representative of the sequence of events associated with the remaining shocks. Inspection of the RV EGM (Fig. 105.4) shows an intermittently fractionated electrogram correlating to the short coupled events on the marker channel. The presence of very short coupled intervals points to a nonphysiologic cause for the five shocks that followed the
initial shock. In this case, the etiology lies in an inherent flaw in the design of the ICD lead utilized. The MDTTM 6936 lead is a coaxial lead utilizing polyurethane polymers for insulation. This ICD leads with polyurethane insulation is prone to breakdown of the insulation as a result of metal ion oxidation (MIO). This appears to be especially likely to be noted after a high energy shock. Almost all ICD leads today are designed with a multilumen approach where the conductors and coils are no longer wrapped around each other, but placed individually inside the lumen of the lead. Polyurethane lead failure of the MDTTM 6936 is characterized by oversensing of ventricular events typically following a shock possibly due to a “noncontact defect between the pace/sense ring conductor and the right ventricular high voltage conductor.” This oversensing of “electrical noise” after a shock is a “signature” presentation for failure of the 6936 lead. Monitoring of the ring to coil impedance and the device short interval counter may allow early detection of MIOinduced damage prior to frank lead failure allowing for elective replacement of the ICD lead. Finally, management of a patient who is receiving multiple inappropriate shocks should be mentioned. The etiologies for multiple shocks can include SVT, lead fracture, or rarely recurrent EMI or even incessant, rapidly recurring VT or VF. Immediate deactivation of the device is necessary after multiple recurrent (inappropriate or appropriate) shocks, and this can be accomplished in most devices by placing a doughnut shaped magnet over the device. In the vast majority of devices, this will suspend VT/VF therapy delivery, but
Case 105
417
Fig. 105.3 Stored EGM showing the first shock
Fig. 105.4 Stored EGM showing the first shock
permit continued bradycardia pacing. The patient must remain on telemetry so appropriate therapy for VT/VF may be delivered when necessary, or other appropriate therapies initiated (e.g., treatment for cardiac ischemia, antiarrhythmic drug infusion, etc.).
Bibliography Ellenbogen KA, Wood MA, Shepard RK, et al. Detection and management of an implantable cardioverter defibrillator lead failure: incidence and clinical implications. J Am Coll Cardiol. 2003;41:73-80.
Case 106 Kenneth A. Ellenbogen
Case Summary A 73-year-old man presents to device clinic after receiving a shock from his defibrillator. He has a past medical history of an ischemic cardiomyopathy with LVEF of 20–25% and symptomatic bradycardia. He had recently undergone implantation of a dual chamber ICD with a CPITM 0154 active fixation defibrillator lead. On questioning, the patient denies any lightheadedness, chest pain, or palpitations around the time of the shock. Interrogation of his ICD (Fig. 106.1) reveals two episodes detected by the device in the VT zone with therapy delivered for the second episode but not the first. The atrial, right ventricular (RV), and shock electrograms (Fig. 106.2) are shown during the initiation of the first episode and during device charging (Fig. 106.3). Why are the EGMs on the RV lead most consistent with “nonphysiologic noise,” and why is therapy not delivered for the first episode? After therapy is diverted, the device detects a second episode for which therapy is delivered (Fig. 106.4). The post-shock electrograms are remarkable for what findings on the atrial and ventricular channels? What is one possible cause for the “nonphysiologic noise”?
Case Discussion
Episode 81 Elapsed Time
Date 04-JAN-00
Time 21:18
Initial Detection Pre-Attempt Avg A Rate Pre-Attempt Avg V Rate Measured Onset Measured Stability A Fib V Rate > A Rate 00:04
00:20
00:52
Type Spontaneous VT Zone 101 bpm 202 bpm 38 %. 240 ms 58 ms FALSE TRUE
Attempt 1 VT Shock 1 Therapy Delivered Diverted -Reconfirm Charge Time 2.9 sec Ω Shocking Impedance 101 bpm Post-Attempt Avg A Rate Post-Attempt Avg V Rate 101 bpm Redetection Pre-Attempt Avg A Rate Pre-Attempt Avg V Rate Measured Stability A Fib V Rate > A Rate
VT Zone 99 bpm 199 bpm 115 ms OFF OFF
Attempt 2 VT Shock 1 Therapy Delivered Charge Time Shocking Impedance Post-Attempt Avg A Rate Post-Attempt Avg V Rate
17 J. Biphasic 0.3 sec 49 Ω bpm 127 bpm
End of Episode
Fig. 106.1 Details of stored device therapy episode
Review of the atrial, RV, and shock electrograms in Fig. 106.2 shows sharp, well-demarcated sensed events only on the RV electrogram channel (“near field” channel) and not on the shock electrogram. The sensed EGMs are in addition coupled to the preceding surface QRS with varying intervals. Oversensing of diaphragmatic myopotentials can be seen on the RV lead due to its proximity to the diaphragm; but
K.A. Ellenbogen (*) Department of Cardiology, VCU School of Medicine, P.O. Box 980053, Richmond, VA 23298-0053, USA e-mail:
[email protected]
typically one sees an EGM characterized by multiple, closely coupled signals of varying amplitude. Noise due to diaphragmatic myopotentials typically cannot be reproduced with isometric exercise or with changes in breathing, etc. Further questioning did not reveal any clear exposure to potential sources of EMI. At this point some other type of nonphysiologic signal is the most likely culprit. The first shock was diverted during redetection as two out of three beats failed to fall in the VT zone (Fig. 106.3). Subsequently, oversensing is seen again only on the RV channel, but this time the shock is committed after the first therapy and delivered (Fig. 106.4).
A. Natale et al. (eds.), Cardiac Electrophysiology, DOI: 10.1007/978-1-84996-390-9_106, © Springer-Verlag London Limited 2011
419
420
Fig. 106.2 Atrial, ventricular and shock electrograms during initiation of episode
Fig. 106.3 Atrial, ventricular and shock electrograms during device charging
K.A. Ellenbogen
Case 106
421
Fig. 106.4 Atrial, ventricular and shock electrograms during redetected episode
The fact that the device was recently implanted is an important clue, particularly since the noise was seen on only one of the channels. In this case, one must consider a possible problem with the lead pin/device header connection. External manipulation of the device header did not elicit the noise. Lead impedance trends were stable and not elevated, and thus not suggestive of a possible loose set screw. The source of the problem in this case was an inherent defect in the ICD lead involving the helix stabilizing post. If the lead was not securely attached to the myocardium, the binding post could move resulting in the nonphysiologic “chatter” on the rate sense channel (RV). Sometimes this is evident at the time of implant and can be corrected by retracting the helix and repositioning the lead in another part of the myocardium. This defect in lead design did not affect the
ability of the lead to defibrillate. The lead was ultimately redesigned with elimination of the problem. This patient was brought back to the lab and a new rate sense lead was implanted.
Bibliography Doshi RN, Goodman J, Naik AM, Shivkumar K, Chen PS, Peter CT. Initial experience with an active-fixation defibrillation electrode and the presence of nonphysiological sensing. Pacing Clin Electrophysiol. Dec 2001;24(12):1713-1720. Mela T, Ngarmukos T, Rosenthal L, Mittleman R. Inappropriate ICD therapy due to lead-related noise in an active fixation ICD lead. J Invasive Cardiol. May 2001;13(5):406-408.
Case 107 Paul A. Friedman and Charles D. Swerdlow
Case Summary
Case Discussion
A 62-year-old woman had previously received an implantable defibrillator due to a history of ventricular tachycardia. She presented following a shock that was not preceded by symptoms. A fracture of the right ventricular 6936 Transvene™ lead was identified as causing noise that led to inappropriate shock. Due to this, the 6936 lead was abandoned, the proximal electrodes capped, and a new Sprint™ 6945 lead placed in the right ventricular apex, taking care to avoid mechanical contact with the abandoned lead. The morning after implantation of the new lead, a pacing threshold test is performed (Fig. 107.1). On the basis of this tracing, what is your diagnosis?
The tracing demonstrating ventricular pacing that results in atrial capture (note third and fourth complex from the left). The “VP” (ventricular pacing) on the marker channel identifies the delivery of pacing pulses. In a single chamber defibrillator placed for sudden death prevention, the pace/sense lead is always placed in the ventricle. The differential diagnosis for ventricular pacing leading to atrial capture includes reversal of leads in the header (in dual chamber devices) or dislodgement of the ventricular lead into the atrium. In this case, the ventricular lead has dislodged to the atrium, near the tricuspid annulus. Note that for the third complex, ventricular pacing results in atrial capture (seen on the surface
Fig. 107.1 Tracings recorded the morning after insertion of a new lead. See text for details
P.A. Friedman (*) Division of Cardiovascular Diseases and Department of Internal Medicine, Mayo Clinic, 200 First Street SW, Rochester, MN, USA 55905 e-mail:
[email protected] C.D. Swerdlow Division of Cardiology, David Geffen School of Medicine, Cedars-Sinai Medical Center, Los Angeles, CA, USA e-mail:
[email protected] A. Natale et al. (eds.), Cardiac Electrophysiology, DOI: 10.1007/978-1-84996-390-9_107, © Springer-Verlag London Limited 2011
423
424
P.A. Friedman and C.D. Swerdlow
Fig. 107.2 Chest X-ray following new lead implanation. Details in text
ECG). The captured P-wave conducts to the ventricle, and the resultant QRS complex is sensed by the ICD (note the “VS” marker beneath the third QRS complex). The chest x-ray (Fig. 107.2) confirms dislodgment of the new lead. The left panel show the anteroposterior view, and the right panel the lateral view. The arrow in the left and right panels indicate the new, dislodged lead. Note that in the lateral view it has retracted proximally (farther from the sternum), toward the tricuspid annulus. The position of the right ventricular apex in both views is approximated by the nondislodged, nonfunctioning chronic lead. The middle panel in a zoomed-up view of the distal potion of the two leads shown in the lateral x-ray. Two important lead characteristics are observed. First, note that the lead conductor proximal to the defibrillation coil (short arrow) is a thin longitudinal conductor in the new, dislodged lead Sprint™ lead. In contrast, the chronic, nonfunctioning lead is composed of coaxial conductors that appear thicker radiographically. Coaxial defibrillation leads with circumferential filers (i.e., the Medtronic Transvene lead) have an increased risk of fracture and warrant close follow up. Second, the new lead is an integrated bipolar lead. As shown in Fig. 107.3, sensing and pacing occur between a dedicated tip and ring electrode in true bipolar leads. In integrated bipolar leads, sensing and pacing occur between a tip electrode and the large distal coil, which also functions as a defibrillation
Fig. 107.3 True bipolar and integrated lead designs
electrode. Since in the integrated lead pacing and sensing occur between the distal tip and the large surface area defibrillation coil, a large bipole is creating permitting sensing of ventricular complexes by the ICD despite the lead tip’s position in the atrium (as confirmed by atrial capture during pacing). Prior to lead revision, the patient developed a tachycardia inappropriately detected in the VF zone due to inappropriate sensing of atrial and ventricular signals. Characteristic interval plots and electrograms are shown in Fig. 107.4. The interval plot demonstrates a “railroad track” pattern, caused by alternating short and long intervals, and leads to overdetection of a slow tachycardia as VF.
Case 107
425
VT/VF Episode #4 Report ID# 4
Date/Time Jun 14 09:14:33 V–V
Page 1
Type V. Cycle
Last Rx Success
Duration
VF
VF Rx 1 Yes
3.2 min
220 ms
VF = 320 ms
“Railroad track” alternating intervals seen in oversensing of cardiac signals
VT = 400 ms
V–V Interval (ms) 1,800 1,500 1,200 900 600
32.2 J
400 200 −20
−15
−10
−5
0 5 10 15 Time (s) [0 = Detection]
20
25
30
35
Far field
Near field
Fig. 107.4 Recordings of a tachycardia event following new lead placement. Tpo left panel shows the R-R intervals preceding and following detection. Bottom right panel shows electrograms recorded during tachycardia
Case 108 Paul A. Friedman and Charles D. Swerdlow
Case Summary A 73-year-old man with coronary artery disease, hypertension, and depressed ventricular function presents after receiving multiple shocks from his single chamber ICD. On physical examination, the blood pressure is 148/73, with heart rate 86.
The jugular venous distension measures 8 cm, lungs are clear, gallops are absent, the abdomen is unremarkable, and trace lower extremity edema is noted. Figure 108.1 shows the VT/ VF episode list. A representative episode is shown in Fig. 108.2. Based on this, what is your diagnosis? Do you recommend device reprogramming or surgical system revision?
P.A. Friedman (*) Division of Cardiovascular Diseases and Department of Internal Medicine, Mayo Clinic, 200 First Street SW, Rochester, MN, USA, 55905 e-mail:
[email protected] C.D. Swerdlow Division of Cardiology, David Geffen School of Medicine, Cedars-Sinai Medical Center, Los Angeles, CA, USA e-mail:
[email protected]
A. Natale et al. (eds.), Cardiac Electrophysiology, DOI: 10.1007/978-1-84996-390-9_108, © Springer-Verlag London Limited 2011
427
428
Fig. 108.1 VT/VF Episode List
P.A. Friedman and C.D. Swerdlow
Case 108 Fig. 108.2 Sample episode. See text for details
429 V-V V–V Interval (ms) 1,800 1,500 1,200 900 600
VF = 320 ms
400 200 −30
−25
−20
−15
Case Discussion The VT/VF episode list (Fig. 108.1) confirms the delivery of multiple ICD shocks, and suggests malfunction by the presence of untreated “VF” episodes with duration up to 3.3 min. The top panel in Fig. 108.2 illustrates the interval plot leading to detection. The V-V intervals fall into two groups: one at 600 ms, and a second at approximately 250–300 ms. The electrogram tracing in the bottom panel shows (from top to bottom) the far-field electrogram (labeled HVA to HVB), the near-field electrogram (Vtip to Vring) and the Marker Annotations and V-V intervals. The far-field elctrogram, which approximates the surface ECG, shows the presence of normal sinus rhythm. In the near-field recording, ventricular electrograms (sharp spikes that align with the far-field QRS) highly variable amplitudes and large T-wave electrograms are present. The T-wave electrograms at times exceed the amplitude of the R-wave electrograms. The changing patterns and amplitudes are suggestive of respiratory variation in lead function. The T waves are sufficiently large to be counted as sensed events, resulting in T-wave oversensing. Due to the close coupling interval between a QRS complex in its T-wave, oversensed T-waves augment the VF counter,
−10
−5 0 5 * Time (s) [0 = Detection]
*
*
*
*
*
as shown by the corresponding “FS” (for “fib sense”) markers. Sensed events that do not augment a tachycardia counter are labeled “VS.” Thus, the shorter cycle-length events depicted on the interval plot are due to T-wave oversensing. A sufficient number of such events are present to result in repetitive inappropriate detection and shock. As shown in Fig. 108.3, T-wave oversensing can be classified into one of three types. Post-pacing T-wave oversensing (left panel) results in pauses in a paced rhythm or ATP since the sensed event resets the bradycardia (or ATP) escape interval. It does not lead to inappropriate shock unless it occurs during reconfimation. When T-wave oversensing occurs during spontaneous rhythm, as in the present case, inappropriate therapies may be delivered. If the R-waves are small relative to the size of the T-wave (as in the present example), reprogramming is not likely to result in acceptable sensing, and lead revision is required. If the R-waves are larger than the oversensed T-waves, reprogramming may correct the problem. In this patient, a separate pace-sense lead (a Novus™ 4076 bipolar pacing lead) was placed in the right ventricle since the was no evidence of impaired lead integrity and the pre-existing lead resulted in a satisfactory DFT. Figure 108.4 shows how reprogramming can eliminate
430
T-wave oversensing with the R-wave is large relative to the T-wave. On the left is shown the maximum/minimum sensitivity settings. At a setting of 0.3 mV, T waves are oversensed. Decreasing sensitivity to 0.45 mV eliminates oversensing. A setting of 0.45 mV indicates that any signal with smaller amplitude will not be sensed. Ventricular
P.A. Friedman and C.D. Swerdlow
sensing is dynamic, and is minimized upon R wave sensing to followed by progressively increasing sensitivity. This minimized the risk of T-wave oversensing by decreasing sensitivity immediately follow an R-wave, but prevents undersensing the small amplitude electrograms of ventricular fibrillation by progressively increasing sensitivity.
Fig. 108.3 Classification of T wave oversensing
Reference 1. Swerdlow CD, Friedman PA. Advanced ICD troubleshooting: Part I PACE1. Dec 2005;28(12):1322–1346.
Fig. 108.4 Effect of reprogramming sensitivity on T wave oversensing in Medtronic defibrillators
Case 109 Paul A. Friedman and Charles D. Swerdlow
Case Summary The 73-year-old man of case 109 (who underwent addition of a separate pace-sense lead due to T-wave oversensing)
receives a shock 1 month after system revision. He transmits ICD system information from home via the Carelink™ remote monitoring network. The stored episode is shown in Fig. 109.1. Based on your review of this tracing, what is your
Fig. 109.1 Electrograms transmitted remotely via careLinkTM network. See text for details
P.A. Friedman (*) Division of Cardiovascular Diseases and Department of Internal Medicine, Mayo Clinic, 200 First Street SW, Rochester, MN 55905, USA e-mail:
[email protected] C.D. Swerdlow Division of Cardiology, David Geffen School of Medicine, Cedars-Sinai Medical Center, Los Angeles, CA, USA e-mail:
[email protected] A. Natale et al. (eds.), Cardiac Electrophysiology, DOI: 10.1007/978-1-84996-390-9_109, © Springer-Verlag London Limited 2011
431
432
diagnosis? How do you counsel the patient? Should he present immediately, follow up in 2–3 days, or in 2–3 months?
Case Discussion From top to bottom in Fig. 109.1 are shown the far-field electrogram (HVA-HVB), the near-field electrogram (VtipVring), the Marker Channel™, and the V-V intervals. The far-field electrograms are recorded between the distal defibrillation coil in the right ventricle and the ICD pulse generator shell. Due to the large surface area of the electrodes and their wide spacing, this approximates the surface ECG recording. The far-field tracings demonstrate an irrgular narrow complex tachycardia with cycle length approximately 400 ms (two boxes), and minor variations in the QRS morphology. This is consistent with atrial fibrillation, with the minor QRS morphology changes due to longitudinal dissociation. The near-field electrogram, which is recorded between the tip and ring of the separate pace-sense lead that had been placed one month before this event, shows
P.A. Friedman and C.D. Swerdlow
fibrillation. The baseline artifact suggests that signal is highly amplified, consistent with a small amplitude electrogram. Note that after the shock, the far-field electrogram shows a wide, idioventricular rhythm and the near-field tracing shows termination of fibrillation, and a rate that is slower than the far-field ventricular rate. The only explanation for these findings is dislodgement of the separate pace-sense lead from the ventricle into the atrium. The dislodgment may have caused the atrial fibrillation, resulting in the inappropriate shock. Since the far-field signals are recorded from the non- dislodged chronic defibrillation lead, they continue to accurately show ventricular activation; the near-field signal from the dislodged bipolar lead records atrial activity. Figure 109.2 shows a real-time electrogram post-shock, transmitted remotely. It is now clear that the pace-sense lead had dislodged to the atrium and is recording atrial electrograms. The baseline artifact again confirms the small amplitude of the signals (possibly due to poor contact) and the high amplifier gain. The patient was advised to present expeditiously for lead revision in order to restore appropriate device function.
Fig. 109.2 Real-time electrogram obtained at the time of CareLinkTM transmission. Details in the text
Case 110 Paul A. Friedman and Charles D. Swerdlow
Case Summary A 68-year-old man with ischemic cardiomyopathy (NYHA Class II, left-ventricular ejection fraction 26%) underwent implantation of a prophylactic dual-chamber ICD (Medtronic EnTrust™ D154ATG) 2 years ago. His medications include lisinopril 20 mg daily, carvedilol 12.5 mg daily, simvastatin 40 ms daily, and aspirin 325 mg daily. The ICD was programmed for single zone detection as shown in the upper panel of Fig. 110.1. Six months ago, he had presyncope followed by a shock. ICD interrogation showed ineffective antitachycardia pacing followed by a successful shock for VT with AV dissociation at cycle length 240 ms. The patient was in his usual state of good health until he received a shock while shaving with his electric razor on December 9, 2007. At interrogation on December 20, 2007, the baseline rhythm is shown in Fig. 110.1, lower panel (HVB = right ventricular coil). Figure 110.2 shows the stored electrograms from the episode, which was detected as VF. The upper panel in Fig. 110.3 shows the corresponding interval plot. The lower panel shows a portion of the Cardiac Compass™ trend. What is the diagnosis? Was this shock preventable? What steps should be taken now?
Case Discussion The stored electrogram in Fig. 110.2 shows a regular atrial rhythm at cycle length 230 ms consistent with atrial flutter. The ventricular cycle length abruptly decreases from a range P.A. Friedman (*) Division of Cardiovascular Diseases and Department of Internal Medicine, Mayo Clinic, 200 First Street SW, Rochester, MN 55905, USA e-mail:
[email protected] C.D. Swerdlow Division of Cardiology, David Geffen School of Medicine, Cedars-Sinai Medical Center, Los Angeles, CA, USA e-mail:
[email protected]
between 400 and 500 to 230 ms, consistent with a change from variable AV conduction near 2:1–1:1 AV conduction. This is seen in the interval plot in Fig. 110.3. The ventricular cycle length is shorter than the programmed SVT limit of 240 ms, so no SVT-VT discriminators apply. The amplitude of the ventricular electrogram is greater during tachycardia than at baseline (note truncated peaks of ventricular electrograms denoted by arrow). Allowing for the change in amplitude, the morphologies of the tachycardia and sinus ventricular electrograms are nearly identical. During ventricular antitachycardia pacing (during charging) the atrial rhythm persists unaltered. These observations all indicate that the 1:1 tachycardia is 1:1 AV conduction of atrial flutter rather than 1:1 VA conduction of VT. The shock terminates atrial flutter (Fig. 110.2). By way of comparison (Fig. 110.4) shows an actual VT episode. The Cardiac Compass trend (Fig. 110.3) shows that the patient developed persistent asymptomatic atrial fibrillation or flutter in mid October. The maximum ventricular rate exceeded the VF detection rate once in late October and approached it twice in November. Internet-based alerts triggered by persistent atrial fibrillation/flutter or rapid ventricular rate would have alerted the physician both to the presence of atrial fibrillation/ flutter and the need for better rate control. The physician would then have been able to initiate therapy with rate control and/or rhythm control as well as anticoagulation. This would have reduced the risk of cardioversion of persistent atrial fibrillation/ flutter without anticoagulation, as happened on December 9. At this time, anticoagulation with warfarin should be initiated. Therapeutic options include increasing the dose of beta blockers for rate control and consideration of pharmacological, or ablative strategies for rhythm control. In some dual-chamber ICDs, atrial antitachycardia pacing would also be an option. Reprogramming the ICD would likely be of minimal value because of the short cycle length of 230 ms during 1:1 conduction of atrial flutter. Programming ON the 1:1 SVT rejection rule, which rejects SVT with near simultaneous atrial and ventricular activation, might be of limited value. But the sinus tachycardia rejection rule would not prevent therapy of 1:1 atrial flutter because it requires a gradual change of ventricular cycle lengths within an expected range to diagnose sinus tachycardia.
A. Natale et al. (eds.), Cardiac Electrophysiology, DOI: 10.1007/978-1-84996-390-9_110, © Springer-Verlag London Limited 2011
433
434 Fig. 110.1 Upper panel: programmed settings. Lower panel: baseline rhythm
P.A. Friedman and C.D. Swerdlow
Case 110
Fig. 110.2 Stored electrograms from episode of ICD-detected VF
435
436
P.A. Friedman and C.D. Swerdlow
Fig. 110.3 Upper panel: interval plot. Lower panel: Cardiac Compass™ trend plot for the last 80 weeks showing the total number of hours per day of atrial fibrillation/flutter in the upper panel
Case 110
437
Fig. 110.4 Stored electrograms for an episode of actual VT. Not that the atria (top tracing) are in sinus rhythm during this episode, and that the ventricular morphology (second tracing) is markedly different than the sinus or atrial flutter morphology
Case 111 Paul A. Friedman and Charles D. Swerdlow
Case Summary A 62-year-old man with ischemic cardiomyopathy (NYHA Class II, left-ventricular ejection fraction 29%) underwent implantation of a prophylactic dual-chamber ICD (Atlas DR V-242) 2 years ago. During a febrile illness, he received six shocks without premonitory symptoms. The ICD is programmed for nominal two detection zones, Tach A (VT) at 400 ms and Fib (VF) at 300 ms, and nominal duration for detection (12 intervals). The ventricular electrogram morphology discriminator (MD™) is programmed ON. SVT is diagnosed if five of eight ventricular electrograms in a sliding window have a match threshold of or higher 60%. The stored electrogram is shown in Fig. 111.1. What is the diagnosis? What steps should be taken now?
Case Discussion The upper panel shows a tachycardia with 1:1 AV relationship and AV interval shorter than the VA interval with cycle length 360 ms. Almost all morphology match scores exceed 60%, and VT therapy is inhibited at the “I=” markers in left side of upper panel and near middle of lower panel. Toward the middle of the lower panel, the atrial cycle length shortens gradually from about 360 to 260 ms, associated with a corresponding decrease in ventricular cycle length, but no change in morphology match score. VF is detected at the “D” marker despite an excellent morphology match because
P.A. Friedman (*) Division of Cardiovascular Diseases and Department of Internal Medicine, Mayo Clinic, 200 First Street SW, Rochester, MN 55905, USA e-mail:
[email protected]
SVT-VT discrimination does not apply in the VF zone of St. Jude ICDs or those made by most other manufacturers. SVT conducted at shorter cycle length than the shortest one at which SVT discriminators apply accounted for about half of all inappropriate therapy of SVT.1 There are several diagnostic and therapeutic considerations: Increasing the beta blocker dose will reduce the likelihood of rapid and rapidly conducted atrial arrhythmia. The atrial cycle length of 260 ms is probably too fast for sinus tachycardia, and additional pharmacological or ablative therapy for the atrial arrhythmia may be considered. Atrial antitachycardia pacing therapy might be considered in other ICDs. However, since this arrhythmia occurred only once during a febrile illness, increasing beta blockers may be a reasonable first step. The ICD should also be reprogrammed: The Fib interval should be shortened to a short cycle length (e.g., 240 ms). A second VT zone (Tach B) may be programmed. But whether or not two or three zone programming is used, the morphology discriminator should be programmed throughout the VT zone(s). The potential risk is that VT with 1:1 VA conduction might occur at a short cycle length and match the sinus morphology. The risk in a multicenter study was 10%2 and is probably lower in this case because of the primary prevention indication. It may be reduced by programming a sustained-duration override feature (SVT discrimination timeout in St. Jude ICDs). These features deliver therapy if an arrhythmia satisfies the ventricular rate criterion for a long programmed duration even if discriminators indicate SVT. The premise is that VT will continue to satisfy the rate criterion for the programmed duration while the ventricular rate during transient sinus tachycardia or atrial fibrillation will decrease below the VT rate boundary. The limitation is delivery of inappropriate therapy when SVT exceeds the programmed duration, which occurs in ~10% of SVTs at at 3 min.1
C.D. Swerdlow Division of Cardiology, David Geffen School of Medicine, Cedars-Sinai Medical Center, Los Angeles, CA, USA e-mail:
[email protected] A. Natale et al. (eds.), Cardiac Electrophysiology, DOI: 10.1007/978-1-84996-390-9_111, © Springer-Verlag London Limited 2011
439
440
P.A. Friedman and C.D. Swerdlow
Fig. 111.1 Right atrial (RA) and right ventricular (RA) true bipolar electrograms are shown. The top row of numerical values between the electrograms shows the morphology algorithm percent match score. The middle and lower numbers show atrial and ventricular cycle length,
respectively. Check mark above match scores indicates match at or above the 60% threshold. X marks indicate match scores less than 60%. ICD rhythm diagnoses are highlighted with squares. I= marker denotes inhibition of therapy in the rate branch with equal atrial and ventricular rates
References
2. Glikson M, Swerdlow CD, Gurevitz OT, et al. Optimal combination of discriminators for differentiating ventricular from supraventricular tachycardia by dual-chamber defibrillators. J Cardiovasc Electr ophysiol. July 2005;16(7):732-739.
1. Klein G, Manolis A, Viskin S, et al. Clinical performance of wavelet™ morphology discrimination algorithm in a worldwide single chamber ICD population (abstract). Circulation. 2003;110:III-345.
Case 112 Paul A. Friedman and Charles D. Swerdlow
Case Summary A 57-year-old woman with ischemic cardiomyopathy and a dual-chamber St. Jude Atlas II DR V-265 ICD presented with inappropriate shocks for sinus tachycardia detected as VT (Fig. 112.1, upper panel). The ventricular electrogram morphology discriminator (MD™)1,2 was programmed ON. This discriminator diagnoses SVT and withholds VT therapy if five of eight ventricular electrograms in a sliding window have a match score of or higher 60% with the template electrogram. The stored electrogram is shown in Fig. 112.1. The real-time electrogram in sinus rhythm, shown in the lower left panel shows a morphology match score of 100% on sinus beats. The lower right panel shows a real-time electrogram recorded during rapid atrial pacing. What is the problem? What are the possible solutions?
Case Discussion This case illustrates two limitations of morphology templates acquired in slow sinus rhythm.3 Accurate alignment of sinus template and tachycardia electrogram in the St. Jude morphology algorithm is sensitive to the value of the sensing threshold at the onset of the ventricular electrogram, as determined by Automatic Sensitivity Control™. If a template electrogram is acquired at the most sensitive setting of Automatic Sensitivity Control (either because of a slow sinus rate or after a ventricular paced beat), a low amplitude peak at the onset of the ventricular electrogram may be used for
alignment. An identical tachycardia electrogram may be acquired at a sufficiently fast rate that Automatic Sensitivity Control does not reach its most sensitive value at the onset of the R-wave. If this occurs, the low-amplitude peak at the onset of the ventricular electrogram may not be used for alignment. If identical template and tachycardia electrograms are then compared, their representations in the morphology algorithm may not match. In this case, most electrograms during inappropriately detected VT (e.g., fourth electrogram denoted by arrowhead) have morphology similar to that recorded in sinus rhythm in the lower left panel (arrowhead). The low-amplitude peak at the onset is probably sensed and used for alignment in sinus rhythm. In sinus tachycardia, Automatic Sensitivity Control does not reach minimum. Thus the low-amplitude peak is not sensed and not used for alignment. The lower right panel identifies a second source of morphology mismatch, rate-related bundle branch aberrancy. The surface ECG shows a wider complex during atrial pacing than in sinus rhythm. The ventricular electrogram morphology is distinctly different from that in sinus rhythm, but similar to that of the 11th beat in the top panel (asterisk) and the final electrograms before the shock (asterisks, second row). In patients who have dual-chamber ICDs and intact AV conduction, this problem may be solved by acquiring or verifying the template during atrial pacing at a cycle length shorter than the VT detection interval. Automatic template updating should be disabled. In single-chamber ICDs, solutions to the first alignment problems may include altering the minimum sensitivity, Threshold Start, or Threshold Delay, or acquiring a template during exercise testing.
P.A. Friedman (*) Division of Cardiovascular Diseases and Department of Internal Medicine, Mayo Clinic, 200 First Street SW, Rochester, MN 55905, USA e-mail:
[email protected] C.D. Swerdlow Division of Cardiology, David Geffen School of Medicine, Cedars-Sinai Medical Center, Los Angeles, CA, USA e-mail:
[email protected] A. Natale et al. (eds.), Cardiac Electrophysiology, DOI: 10.1007/978-1-84996-390-9_112, © Springer-Verlag London Limited 2011
441
442
P.A. Friedman and C.D. Swerdlow
Fig. 112.1 Top panel shows stored electrogram from an episode of ICD-detected VT. Lower left panel shows real-time electrogram of sinus rhythm indicating template match in of 100% for sinus beats and 0% for premature ventricular complexes. Lower right panel shows realtime electrogram in atrial pacing. In each panel, the top row of numerical values between the electrograms shows the morphology algorithm
percent match score. The middle and lower numbers show atrial and ventricular cycle length, respectively. Check mark above match scores indicates match at or above the 60% threshold. X marks indicate match scores less than 60%. ICD rhythm diagnoses are highlighted with squares. D= marker denotes detection of VT in the rate branch with equal atrial and ventricular rates
References
2. Boriani G, Biffi M, Dall’Acqua A, et al. Rhythm discrimination by rate branch and QRS morphology in dual chamber implantable cardioverter defibrillators. Pacing Clin Electrophysiol. 2003;26(1 Pt 2): 466-470. 3. Swerdlow CD, Friedman PA. Advanced ICD troubleshooting: part I. Pacing Clin Electrophysiol. 2005;28(12):1322-1346.
1. Duru F, Bauersfeld U, Rahn-Schonbeck M, Candinas R. Morphology discriminator feature for enhanced ventricular tachycardia discrimination in implantable cardioverter defibrillators. Pacing Clin Electrophysiol. 2000;23(9):1365-1374.
Case 113 Paul A. Friedman and Charles D. Swerdlow
Case Summary A 64-year-old man with ischemic cardiomyopathy, refractory congestive heart failure (New York Heart Association Class III), and chronic atrial fibrillation undergoes implantion of an InSync Sentry™ 7299 CRT-D defibrillator. The morning after the implantation, he receives a shock while shaking his cardiologist’s hand. The hospital telemetry during the handshake is shown in Fig. 113.1 (top panel), and episode report is shown in Fig. 113.1 bottom panel. Figure. 113.2 shows lead impedance values (top panel) taken following the shock, and the stored electrograms of the event (bottom panel). In order to identify the cause of the problem, detection is turned off, and various electrograms are recording during a provocative maneuver (hand shake), as shown in Fig. 113.3. Based on this information, what is your diagnosis? What is your recommended plan of action?
Case Discussion Hospital telemetry at the time of the shock shows atrial fibrillation with a ventricular rate around 100 bpm, a shock artifact, followed by a slightly accelerated rhythm with a wider QRS complex (Fig. 113.1). The delivery of a shock during a “normal” rhythm with heart rate below a tachycardia detection zone indicates a system-related malfunction. The wider rhythm following the shock is likely a brief accelerated idioventricular rhythm, although post-shock aberrancy cannot be excluded. The VT/VF episode report (Fig. 113.1, bottom) P.A. Friedman (*) Division of Cardiovascular Diseases and Department of Internal Medicine, Mayo Clinic, 200 First Street SW, Rochester, MN 55905, USA e-mail:
[email protected] C.D. Swerdlow Division of Cardiology, David Geffen School of Medicine, Cedars-Sinai Medical Center, Los Angeles, CA, USA e-mail:
[email protected]
indicates that in the 5 seconds before detection, the device was sensing intervals shorter than the detection cutoff of 320 ms, while absence of rapid complexes is confirmed by the surface ECG. A loose set screw, lead dislodgement, electrode contact with an abandoned lead (if present), and lead fracture could all result in early post-implant inappropriate shock. The atrial impedance >2,500 W is not germane to the shock; as seen in the VT/VF report, the shock was delivered in the VF zone based solely on the rapid ventricular rates; detection enhancements did not play a role. In this case, the atrial impedance is high due to absence of an atrial lead; an atrial lead was not implanted due to chronic atrial fibrillation. The RV lead impedance is high and suggestive of loose set screw or fracture; typically, any value >2,000 W is abnormal. In the perioperative period, a loose set screw would be far more likely than lead fracture to give rise to these findings. The episode electrogram (Fig. 113.2, bottom panel) shows a near-field electrogram (Vtip-Vring) that is oversaturated with the type of noise characteristic of make-break contact artifact seen in lead fracture or loose set screw. The presence of V-V intervals in the nonphysiologic range below 150 ms also favor fracture/loose set-screw. These short intervals lead to detection of VF (“FD,” the fourth marker). Following detection, only “VS” (ventricular sensed event) markers are present, irrespective of the V-V interval. This is an idiosyncracy of the Medtronic system, which only displays “VS” markers once detection is met. The “CE” marker denotes charge end, and “CD” charge delivery after a brief confirmation interval. In Fig. 113.3, provocative maneuvers are performed with detection turned off to further identify the malfunctioning component. In each panel, electrograms using different sources are recorded during a hand shake. Note that the only electrogram that did not have artifact present during the handshake was the can-RV coil (top right panel). The RVtip-RVring (top left), RVtip-RV distal coil (bottom left), and RVtip-LVtip (bottom right panel) all contain artifact (sharp, nonphysiologic deflections). Thus, any electrogram that incorporates the RV tip has a noisy signal during handshaking, indicating a loose setscrew or lead fracture involving the RV tip conductor. At surgical revision, reinserting the RV distal electrode connector in the header and tightening the screw eliminated the malfunction.
A. Natale et al. (eds.), Cardiac Electrophysiology, DOI: 10.1007/978-1-84996-390-9_113, © Springer-Verlag London Limited 2011
443
444
P.A. Friedman and C.D. Swerdlow
May 02, 2006 09:50:41 9998 Software Version 2.0 Copyright Medtronic, Inc. 2004
ICD Model: InSync Sentry 7299 Serial Number: PRK 124359H
VT/VF Episode #12 Report
Page 1
ID#
Date/Time
Type
V. Cycle
Last Rx
Success
Duration
12
May 02 10:20:49
VF
140 ms
VF Rx 1
Yes
10 s
Interval (ms) 2,000 1,700 1,400 1,100 800
V−V
VF = 320 ms
A−A
FVT = 260 ms 34.8 J
600 400 200 −40
−35
−30
−25
−20
−15
−10
−5
0
5
10
15
Time (s) [0 = Detection]
Fig. 113.1 Top panel: surface telemetry recorded at the time of shock delivered during a handshake. Bottom panel: Episode report from the same episode Lead Performance EGM amplitude pacing impedance Defibrillation impedance SVC impedance
Atrial Not Taken >2,500 Ω
Note 1: 0−35 J charge time not available.
Atip Aring
Fig. 113.2 Interrogation following the shock. Top panel: lead impedance values. Bottom panel: stored electrograms from the event that triggered the shock
Vtip Vring
40 Ω
RV 6.8 mV 1,936 Ω 32 Ω
LV 324 Ω
Case 113
Fig. 113.3 Electrograms recorded during provactive maneuver (handshake) while detection is turned off
445
Case 114 Paul A. Friedman and Charles D. Swerdlow
Case Summary
interfence is characterized by normal lead impedance values, noise throughout the cardiac cycle, and a greater amplitude signal on the far-field than the near-field elecA 67-year-old man received a Guidant 1850 single chamber trogram. The latter occurs since the far-field electrogram defibrillator on an Endotak™ right ventricular lead for sudis recorded from larger electrodes with a greater interelecden death prevention in the setting of an ischemic cardiotorde spacing than the near-field electrogram, resulting in myopathy. He was using an auger when he received a shock a larger “antenna” that is more susceptible to external without antecedent symptoms. The ICD is interrogated and signals. Integrated bipolar leads (which includes the the episode electrograms shown in Fig. 114.1. Lead impedEndotak™) are more likely to record EMI than true bipoance values were normal. What is your diagnosis, and lar leads. See Case 108 for depiction of integrated and management plan? bipolar leads. Figure 114.2 shows examples of oversensing of extra-cardiac noise. The far-left panel shows a lead fracture, characterized by saturated high frequency elecCase Discussion trograms that account for less than 10% of the cardiac cycle. Artifact is seen on the near-field recording to a The tracing shows (from top to bottom) the near-field ven- greater extent than the far-field recording. With a lead tricular electrogram, far-field ventricular electrogram, and fracture, the lead impedance may be elevated; an impedmarkers and V-V intervals. Halfway through the top panel, ance >2,000 W is highly suggestive. The middle panel high frequency noise becomes apparent on both the near- shows myopotential oversensing. The electrogram affected field and far-field electrograms. The presence of multiple depends on the source; diaphragmatic oversensing affects high frequency events/cardiac cycle that are unrelated to the near-field channel and may lead to inappropriate the R-wave is characteristic of oversensing of extra- shocks. Pectoralis oversensing distorts morphology cardiac signals. Extra-cardiac signals may be due to lead templates in some systems, leading to SVT-VT misclasfracture/loose set screw, myopotential oversensing, or sification with use of morphology-based detection electromagnetic interference (EMI). Electormagnetic enhancements. The far right panel depicts EMI.
P.A. Friedman (*) Division of Cardiovascular Diseases and Department of Internal Medicine, Mayo Clinic, 200 First Street SW, Rochester, MN 55905, USA e-mail:
[email protected] C.D. Swerdlow Division of Cardiology, David Geffen School of Medicine, Cedars-Sinai Medical Center, Los Angeles, CA, USA e-mail:
[email protected] A. Natale et al. (eds.), Cardiac Electrophysiology, DOI: 10.1007/978-1-84996-390-9_114, © Springer-Verlag London Limited 2011
447
448
Fig. 114.1 Device interrogation following shock delivery
P.A. Friedman and C.D. Swerdlow
Case 114 Fig. 114.2 Characteristic electrograms recorded in the setting of lead fracture (left panel), myopotential oversensing (middle panel), and electromagnetic interference (EMI, right panel)
449
Case 115 Paul A. Friedman and Charles D. Swerdlow
Case Summary
What changes in bradycardia pacing parameters should be made?
A 78-year-old man with ischemic cardiomyopathy (NYHA Class IV, left-ventricular ejection fraction 14%) and renal failure underwent implantation of a prophylactic, cardiac-resynchronization ICD in 2005. He had sinus rhythm with first degree AV block and left bundle branch block. His heart failure improved to Class III and ejection fraction improved to 21%. In May 2007, he saw his cardiologist for a routine followup visit. ICD interrogation showed 96% ventricular pacing and no episodes of VT. ECG monitoring showed frequent ventricular tracking of atrial premature complexes, resulting in asymptomatic rhythm irregularities. To reduce this tracking, bradycardia programming was changed to the settings shown in left panel of Fig. 115.1. In August 2007, he returned for a routine visit, reporting an increase in exertional dyspnea and reduction in exercise capacity. ICD interrogation showed 41% ventricular pacing and asymptomatic, devicedetected VT, all terminated by antitachycardia pacing. The plasma natriuretic peptide concentration has increased from 165 pg/mL in May to 536 pg/mL. Figure 115.1 (right panel) shows a stored EGM of device-detected VT with onset identical to the seven other episodes. ICD programming for detection and therapy of VT/VF is shown below: Intervals to detect
Detection interval (ms)
Therapy
VF
18/24
320
35 J Shock × 6
Fast VT
18/24
240
ATP X1, 35 J Shock × 6
VT
16
400
ATP X3, 35 J Shock × 6
P.A. Friedman (*) Division of Cardiovascular Diseases and Department of Internal Medicine, Mayo Clinic, 200 First Street SW, Rochester, MN 55905, USA e-mail:
[email protected] C.D. Swerdlow Division of Cardiology, David Geffen School of Medicine, Cedars-Sinai Medical Center, Los Angeles, CA, USA e-mail:
[email protected]
Case Discussion Figure 115.1 shows an episode of monomorphic VT at cycle length 340 ms terminated by antitachycardia pacing. The post-therapy rhythm is a SVT with 1:1 AV association. The pattern of atrial and ventricular timing at the end of the antitachycardia pacing sequence indicates that the likely diagnosis is atrial tachycardia with first degree AV block. The rhythm preceding VT is sinus tachycardia at cycle length 560 ms without cardiac-resynchronization pacing. The patient’s heart failure has worsened because of reduction in cardiac resynchronization pacing, which generally needs to be applied to more than 90% of QRS complexes for therapeutic effect. The root cause of insufficient cardiac- resynchronization pacing is the long post-ventricular atrial refractory period (PVARP) of 500 ms, which was increased in May to reduce tracking of atrial premature complexes. This prevents tracking of the sinus P waves that fall within the PVARP (VS–AR interval 200–270 ms) as the sinus cycle length decreases. This episode and the other seven episodes of VT were preceded by an atrial-paced event, setting up safety pacing, which initiated VT. Assuming the activity sensor at 0, the device will pace at the lower rate (70 bpm), and thus the VS–AP interval = lower rate interval – PAV ~ 850 − 130 ms = 720 ms. After the VS–VS interval shortens to 510 ms, probably because of a premature ventricular complex, the measured VS–AP interval is 710 ms, indicating that the sensor was driving the pacing rate slightly faster than the lower rate of 70 bpm. Note also that AR–AP interval is 300 ms, which means that the noncompetitive atrial pacing feature (NCAP), designed to prevent pacing-induced atrial fibrillation, withheld the Ap event until the NCAP timer expired. The conducted VS beat from the preceding AR P wave times in the cross-talk window after the AP event. This initiates “Safety Pacing” with an AP–VP interval of
A. Natale et al. (eds.), Cardiac Electrophysiology, DOI: 10.1007/978-1-84996-390-9_115, © Springer-Verlag London Limited 2011
451
452
P.A. Friedman and C.D. Swerdlow
Fig. 115.1 Bradycardia pacing parameters and stored electrogram of VT terminated by antitachycardia pacing. See text for details
110 ms, and the resulting fusion beat. The fusion beat is followed by a VS beat with V-V interval of 430 ms. This completes a short-long-short sequence that initiates VT. Without electrograms, we determine from the intervals alone if this last beat is a premature ventricular beat or if it is conducted from the NCAP Ap beat with a Ap-–VS interval of 480 ms. The latter explanation suggests proarrhythmia related to specific features of bradycardia pacing (long PVARP and NCAP) and first degree AV block.1 The correct action to increase percent of cardiac resynchronization pacing is to shorten the PVARP. This may also reduce
the incidence of VT by preventing pacing-related proarrhythmia.
Reference 1. Sweeney MO, Ruetz LL, Belk P, Mullen TJ, Johnson JW, Sheldon T. Bradycardia pacing-induced short-long-short sequences at the onset of ventricular tachyarrhythmias: a possible mechanism of proarrhythmia? J Am Coll Cardiol. 2007;50(7):614-622.
Case 116 Paul A. Friedman and Charles D. Swerdlow
Case Summary The 78-year-old man described in previous case 116 is followed. No programming was performed at an August 2007 visit. Instead, amiodarone therapy was initiated to prevent asymptomatic VT requiring termination by antitachycardia pacing. Six weeks later, on September 28, 2007, the patient developed rapid palpitations after receiving bad news. Within a few minutes, he experienced dyspnea and his typical angina. He used nitroglycerin spray twice without relief. Twenty minutes later he collapsed and died. Figure 116.1 shows the data retrieved postmortem from an episode of ICD-detected VT that was recorded about 20 min after he collapsed. What is the diagnosis? What steps could have been taken to prevent the patient’s death?
Case Discussion The Flashback™ intervals in the top left panel show RR intervals for approximately 11 min preceding ICD detection of “VT.” The extreme RR variability is characteristic of
undersensing during ventricular fibrillation.1 The interval plot at lower left shows two unsuccessful attempts at antitachycardia pacing denoted by arrowheads. The stored EGM at right shows, relatively slow, late-stage VF, probably with substantial undersensing. It is difficult to assess the magnitude of undersensing because the true-bipolar sensing EGM was not recorded. The most likely explanation is that amiodarone slowed the rate of VT so that it remained undetected. The patient became ischemic, and the rhythm degenerated into VF. Antiarrhythmic drugs that slow the rate of VT below the programmed VT rate cut-off can prevent initial detection of VT2 or divert appropriate therapy during reconfirmation. In this case, VT could likely have been prevented or reduced in frequency by reprogramming the PVARP. Even in the absence of proarrhythmia, infrequent asymptomatic VT terminated by antitachycardia pacing may not require pharmacological prophylaxis. Amiodarone often slows the VT cycle length by 100 ms or more, and patients with advanced heart failure may not tolerate even relatively slow VT. 2 When therapy with amiodarone is initiated either for therapy of atrial or ventricular arrhythmias in a patient with known monomorphic VT, the VT detection interval should usually be increased by 50 ms.
P.A. Friedman (*) Division of Cardiovascular Diseases and Department of Internal Medicine, Mayo Clinic, 200 First Street SW, Rochester, MN 55905, USA e-mail:
[email protected] C.D. Swerdlow Division of Cardiology, David Geffen School of Medicine, Cedars-Sinai Medical Center, Los Angeles, CA, USA e-mail:
[email protected] A. Natale et al. (eds.), Cardiac Electrophysiology, DOI: 10.1007/978-1-84996-390-9_116, © Springer-Verlag London Limited 2011
453
454
P.A. Friedman and C.D. Swerdlow
Stored electrogram VT 400 ms FVT 320 ms VF 240 ms
RR Interval (ms)
Flashback intervals
1,000 800 600 400 200 0
−600
−400
−200
0
Time (s) [0 = Detection]
Interval plot A−A
V−V Interval (ms) 2,000 1,700 1,400 1,100 800
VF = 320 ms FVT = 240 ms
VT = 400 ms
Burst
600 400 200 −40
−30
−20
−10
0 10 20 Time (s) [0 = Detection]
30
*
*
*
Septermber 28, 2007
Fig. 116.1 Flashback intervals, interval plot, and stored electrogram from ICD-detected VT. See text for details
References 1. Swerdlow CD, Friedman PA. Advanced ICD troubleshooting: part II. Pacing Clin Electrophysiol. 2006;29(1):70-96.
2. Bansch D, Castrucci M, Bocker D, Breithardt G, Block M. Ventricular tachycardias above the initially programmed tachycardia detection interval in patients with implantable cardioverter- defibrillators: incidence, prediction and significance. J Am Coll Cardiol. 2000;36(2):557-565.
Case 117 Paul A. Friedman and Charles D. Swerdlow
Case Summary A 24-year-old woman with complex congenital heart disease, depressed systemic ventricular function, and congestive heart failure previously received an InSync II Marquis™ 7289 CRT-D utilizing a Medtronic Sprint Fidelis 6949 right ventricular defibrillation lead, 4193 left ventricular lead, and
a Novus 5076 right atrial lead. While in hospital on telemetry, she develops recurrent episodes of pacing failure, ventricular arrhythmias, and shocks (Fig. 117.1). The device status report is shown in Fig. 117.2. In order to further troubleshoot the device malfunction, a real-time electrogram and surface ECG are simultaneously recorded (Fig. 117.3). What is the diagnosis? How is the problem corrected?
Fig. 117.1 Telemetry recordings obtained during hospitalization. See text for details
P.A. Friedman (*) Division of Cardiovascular Diseases and Department of Internal Medicine, Mayo Clinic, 200 First Street SW, Rochester, MN 55905, USA e-mail:
[email protected] C.D. Swerdlow Division of Cardiology, David Geffen School of Medicine, Cedars-Sinai Medical Center, Los Angeles, CA, USA e-mail:
[email protected] A. Natale et al. (eds.), Cardiac Electrophysiology, DOI: 10.1007/978-1-84996-390-9_117, © Springer-Verlag London Limited 2011
455
456
Fig. 117.2 ICD status report. Note impedance values. Discussion in text
P.A. Friedman and C.D. Swerdlow Aug 08, 2007 09:43:11 9989 Software Version 2.0 Copyright Medtronic, Inc. 2002
ICD Model: InSync II Marquis DR 7289 Serial Number: PRJ 620851S
Status report
Page 1
Last Interrogation: Aug 08, 2007 07:13:28 Battery voltage
Lead impedance
(ERI = 2.62 V) Aug 08, 2007 07:13:26 Voltage Last capacitor formation Mar 30, 2007 19:54:26 Charge time Energy Last charge
7.96 s 0.0 - 30.0 J
Aug 08, 2007 03:00:03 A. Pacing RV Pacing LV Pacing V. Defib SVC Defib EGM amplitude
Aug 08, 2007 05:27:42 Charge time Energy Sensing integrity counter
4.51 s 0.0 − 24.0 J
Aug 08, 2007 05:26:38 P wave amplitude RVR-wave amplitude Last high voltage rherapy
2.88 V
(if>300 counts, check for sensing issues) Since Aug 08, 2007 03:25:21 120−130 ms V−V intervals 17
Case Discussion The patient has striking episodes of failure of ventricular output (seen in Fig. 117.1 and 117.3) leading to short–long–short intervals, which are pro–arrythmic and induce polymorphic ventricular arrhythmias (Fig. 117.1). The tracing in Fig. 117.3 is duplicated in Fig. 117.4 with additional labels added. The arrows indicate the paced atrial beats not followed by a QRS, which lead to alternating V-V intervals. Note that the marker channel for these beats displays “VS” with two bars of differing heights (circled in Fig. 117.4). This double-bar marker indicates safety pacing. Safety pacing prevents “cross talk,” which occurs when an atrial pacing event is sensed on the ventricular channel, inhibiting ventricular output. As shown in Fig. 117.5 an atrial pacing event is followed by a ventricular blanking period, then a cross-talk window. Any event sensed during the cross talk window will result in delivery of a ventricular pacing stimulus with an abbreviated atrioventricular interval (AVI). Thus, if an atrial event is sensed on the ventricular channel during the cross-talk window, a pacing stimulus is delivered after an abbreviated AVI that may be suboptimal hemodynamically, but preferable to asystole. If an intrinsic
Aug 08, 2007 05:08:31 Measure impedance Delivered energy Vaveform Pathway
432 Ω 560 Ω 368 Ω 49 Ω None
1.4 mV 5.0 mV
42 Ω 24.1 J Biphasic B>AX
ventricular event is sensed during the cross-talk window, the ventricular pacing stimulus follows shortly after its onset (due to the abbreviated AVI), avoiding pacing during repolarization and proarrhythmia. In the present case, by its design, the 7289 CRT-ICD delivers safety pacing pulses only to the right ventricular lead. Due to failure of the Fidelis™ lead in this patient, output failure occurs, awnd the ventricle is not paced. Note that the other (captured) pacing pulses in Fig. 117.4 are associated with the “BV” marker for biventricular pacing. It is likely that only the left ventricular lead is capturing during pacing, so that during safety pacing, which is delivered only to the nonfunction right ventricular lead, ironically, no ventricular capture occurs. Figure 117.6 further corroborates this. Note that after an external shock is delivered to terminate ventricular tachycardia, biventricular pacing occurs that captures only the LV (first arrow). Due to the width of the LV-only paced QRS, the same ventricular complex is sensed on the RV lead, giving rise to an “FS” marker (second arrow), for “fib sense” event. Thus, while the Fidelis™ fails to pace, it remains capable of sensing. Also note that the impedance of the failed lead is not elevated (Fig. 117.2). Fidelis lead malfuncntion without significant increases in impedance have been described.
Case 117 Fig. 117.3 Simultaneous surface telemetry (top panel) and device markers and electrograms (bottom panel)
Fig. 117.4 Simultaneous surface and device recordings, as in 118.3. Large arrows indicated paced p waves without subsequent QRS. Each such event has a characteristic marker (The first one is circled). Details in text
457
458 Fig. 117.5 Venricular sensing following a paced atrial event. Sensing during the cross talk window will trigger a paced ventricular event with an abbreviated AV interval. Figure courtesy of Dr. David Hayes
Fig. 117.6 Episode from same patient treated with shock. Note the last complex is biventricularly paced, but sensed on the RV channel. See text for discussion
P.A. Friedman and C.D. Swerdlow
Case 118 Paul A. Friedman and Charles D. Swerdlow
Case Summary A 59-year-old man with ischemic cardiomyopathy had a single-chamber Medtronic Entrust™ ICD and Sprint Fide lis™ Model 6949 dual-coil true bipolar lead implanted over 2 years ago after out-of-hospital VF. His NYHA Class is III and left ventricular ejection fraction is 34%. After an episode of monomorphic VT storm a year ago, he has been treated with amiodarone 200 mg per day to decrease the frequency of VT. He now presents for a routine follow-up visit. The pacing threshold is 0.5 V at 0.4 ms, and the pacing lead impedance is stable at 560 W. One episode of device-detected VT is stored in the ICD. The patient had no awareness of this episode, which occurred 6 weeks ago. Figure 118.1 shows the corresponding stored electrogram. Figure 118.2 shows 80-week trend plots for R wave amplitude and impedance measurements corresponding to each-high voltage lead which are displayed with each interrogation. What is the diagnosis? What steps should be taken now?
Case Discussion Figure 118.1 an episode of monomorphic VT with AV dissociation that was detected immediately and terminated by antitachycardia pacing. The high-voltage impedance trends show intermittent high values over the last 5 weeks. The defibrillation impedance for
P.A. Friedman (*) Division of Cardiovascular Diseases and Department of Internal Medicine, Mayo Clinic, 200 First Street SW, Rochester, MN 55905, USA e-mail:
[email protected]
the right ventricular coil is measured between that coil and the ICD can. The impedance for the superior vena cava electrode is measured between that electrode and the right ventricular coil, the high-voltage electrode of opposite polarity. Simultaneous fluctuations in both measurements indicate intermittent conduction failure in the common portion of the circuit that includes right ventricular coil. The differential diagnosis includes isolated high-voltage conductor failure in the lead or failure of conductive contact in the header (e.g., loose set screw). Recalled Fidelis leads have been implanted in more patients than any other lead family.1 About 95% of Fidelis lead failure involves one of the two pace-sense electrodes. Failures of pace-sense electrodes present with intermittent high pacing impedance measurements and evidence of oversensing, including inappropriate shocks.2 See Fig. 118.3. A modification of previous lead failure algorithm may assist in identifying lead failures before patients receive inappropriate shocks.3 Isolated high-voltage failures represent only about 5% of reported Fidelis failures. But the time course in this case is unusual for header-connector problems, which usually occur in the first year after implant. So the differential diagnosis includes two unlikely events, both of which require operative intervention despite the fact that the patient is asymptomatic and therapy of VT was successful; successful antitachycardia pacing provides no information about the present or future efficacy of defibrillation, and the likelihood of needing defibrillation is significant in a patient with a history of out-of-hospital VF. If no problem is identified within the header at surgery, the lead should be replaced regardless of intraoperative impedance measurements, which may be normal in cases of intermittent conductor failure. No other diagnostic testing is helpful. Chest radiography does not identify Fidelis lead failures, and noninvasive defibrillation testing may not identify intermittent high-voltage failures.
C.D. Swerdlow Division of Cardiology, David Geffen School of Medicine, Cedars-Sinai Medical Center, Los Angeles, CA, USA e-mail:
[email protected] A. Natale et al. (eds.), Cardiac Electrophysiology, DOI: 10.1007/978-1-84996-390-9_118, © Springer-Verlag London Limited 2011
459
460
Fig. 118.1 Stored electrogram of device-detected VT
P.A. Friedman and C.D. Swerdlow
Case 118
461
R wave amplitude At implant 15.8 mV Last 15.2 mV
Highest 16.6 mV Lowest 11.2 mV mV >20.0 10.0 5.0 3.0 2.0 1.0
09/08/06
11/17/06
01/26/07 04/06/07 06/15/07 Last 80 weeks (min/max per week)
08/24/07
11/02/07
0.0 <0.3 01/05/08 01/18/08 Last 14 days
V. Defibrillation impedance At implant 46 Ω Highest 144 Ω Last 50 Ω Lowest 38 Ω Ω >200 150 100 80 60 40 30 09/08/06
11/17/06
01/26/07 04/06/07 06/15/07 Last 80 weeks (min/max per week)
08/24/07
11/02/07
<20 01/05/08 01/18/08 Last 14 days
SVC (HVX) Deffib impendance At implant 62 Ω Highest 168 Ω Last 63 Ω Lowest 49 Ω Ω >200 150 100 80 60 40 30 09/08/06
11/17/06
01/26/07 04/06/07 06/15/07 Last 80 weeks (min/max per week)
08/24/07
11/02/07
<20 01/05/08 01/18/08 Last 14 days
Fig. 118.2 R wave and high-voltage impedance trends. Data points indicate daily measurements in last 2 weeks and the range of weekly values before then
462
P.A. Friedman and C.D. Swerdlow
Atrial EGM V Tip-Ring
RV pacing impendance At implant 728 Ω Last >3,000 Ω
CareAlert™ Highest Lowest
>3,000 Ω 552 Ω Ω >3,000
2,498 short V−V intervals (120 − 130 ms) in last week
2,000 1,500 1,000 800 600 400 300 <200 05/27/05
08/05/05 10/14/05 12/23/05 Last 80 weeks (min/max per week)
Fig. 118.3 Inappropriate shock caused by failure of the ring conductor in a Fidelis lead. The upper panel shows characteristic oversensing caused by lead or connector (header, adapter, or set-screw) problems.3,4 Oversensing is intermittent and usually occurs only during a small fraction (<10%) of the cardiac cycle. Nonphysiologically short intervals £ 130 ms are recorded, and make-break potentials saturate the sensing amplifier. The lower panel shows that 2,498 short intervals have been recorded by the Sensing Integrity Counter™ in the last week. It
03/03/06
05/12/06
07/20/06 08/02/06 Last 14 days
provides a cumulative count of nonphysiologic short intervals caused by oversensing normal values short intervals are less than 300 (over about a 3-month period),5,6 but values for true-bipolar leads rarely exceed 50. Intermittent high measurements of the pacing-lead began 12 days prior to the interrogation, indicating complete or partial interruption of the pace-sense circuit. These triggered an Internet-based alert in the Medtronic CareLink follow-up network. The patient was advised to have the lead replaced at that time, but refused because he was feeling well
Case 118
References 1. Hauser RG, Kallinen LM, Almquist AK, Gornick CC, Katsiyiannis WT. Early failure of a small-diameter high-voltage implantable cardioverterdefibrillator lead. Heart Rhythm. July 2007;4(7):892-896. 2. Swerdlow CD. Small-diameter defibrillation electrodes: can they take a licking and keep hearts ticking? Heart Rhythm. July 2007;4(7):900-903. 3. Gunderson B, Patel A, Bounds C. Automatic identification of implantable cardioverter–defibrillator lead problems using intracardiac electrograms. Comput Cardiol. 2002;29:121-124.
463 4. Swerdlow CD, Friedman PA. Advanced ICD troubleshooting: part II. Pacing Clin Electrophysiol. January 2006;29(1):70-96. 5. Gunderson BD, Patel AS, Bounds CA, Ellenbogen KA. Automatic identification of clinical lead dysfunctions. Pacing Clin Electrophysiol. January 2005;28(Suppl 1):S63-67. 6. Vollmann D, Erdogan A, Himmrich E, et al. Patient Alert to detect ICD lead failure: efficacy, limitations, and implications for future algorithms. Europace. May 2006;8(5):371-376.
Case 119 M. Eyman Mortada, Jasbir S. Sra, and Masood Akhtar
Case Summary A 62-year-old Caucasian male with history of cardiomyopathy and implantable cardioverter defibrillator (ICD) implanted 8 months ago came to the emergency room after experiencing two ICD shocks. Both occurred without prodromal symptoms and after he did heavy work in his backyard. The stored EGM from the time when the patient received the shock is shown in Fig. 119.1a and b (Panel B is a continuous recording from panel A). The current EGM recording is shown in Fig. 119.2. What is the most appropriate next step for managing this condition?
Case Discussion Interrogation of the device indicates a single-lead device. The EGM recording (V-tip to V-ring) prior to the shock showed fast rhythm with a rate of 460–480 ms. This fast rhythm (most likely sinus tachycardia) is due to the patient’s physical activity (heavy labor in his backyard). Yet, the device detected a faster rhythm with a rate of 220–270 ms, almost double the rate of the actual rhythm detected on the EGM. The extraventricular detection recorded by the device matches the location of the tall T-wave, leading to the diagnosis of T-wave oversensing. Interestingly, after the ICD shock, the T-wave becomes smaller and the oversensing resolves for several beats. This could be related to the effect of the electrical jolt on ventricular repolarization.
When comparing the current EGM to the one recorded at the time of ICD shock, the T-wave is smaller and the oversensing episodes are fewer, but still present in a few beats. Thus, tachycardia in this patient increased the amplitude of the T-wave and the risk for oversensing and ICD shock. Therefore, treatment must be aimed at slowing the heart rate by reducing physical activity and starting b-blocker therapy. Amiodarone therapy has no role in resolving T-wave oversensing. The device could be reprogrammed to reduce these events by lowering the sensitivity. After lowering the sensitivity from 0.3 to 0.45 mV, T-wave oversensing disappeared, as shown in the following real-time EGM (Fig. 119.3). Unfortunately, in this case, the R-waves are small to start with. When the sensitivity was reduced slightly further, down to 0.5 mV, some of the R-waves were not detected. Therefore, there is no safety margin to detect real ventricular fibrillation, and the patient would be at high risk for sudden cardiac death. If large R-waves are present and a good safety margin exists within the sensitivity range, one could reduce the sensitivity and perform a defibrillation test to confirm the efficacy of the device. If the device is a St. Jude ICD, oversensing of spontaneous T-waves can be reduced by programmable “threshold start” and “Decay Delay.” The best solution would be lead revision. Rarely, SVT-VT morphology discriminator and stability algorithm may help in reducing the inappropriate shock from T-wave oversensing. Finally, forcing ventricular pacing might eliminate the T-wave oversensing due to alteration of the repolarization sequence, but this may result in adverse, desynchronizing hemodynamic effects.
M.E. Mortada (*), J.S. Sra, and M. Akhtar Department of Electrophysiology, Aurora Cardiovascular Services, Aurora Sinai/Aurora St. Luke’s Medical Centers, University of Wisconsin School of Medicine and Public Health, 2801 W. Kinnickinnic River Parkway, #777, Milwaukee, WI 53215, USA e-mail:
[email protected] A. Natale et al. (eds.), Cardiac Electrophysiology, DOI: 10.1007/978-1-84996-390-9_119, © Springer-Verlag London Limited 2011
465
466
M.E. Mortada et al.
Fig. 119.1 Panel A illustrates the ICD reading from top to bottom: V-tip to V-ring electrocardiograms (EGM), leadless ECG, channel markers, and event markers. Panel B is a continuous recording from panel A, which demonstrated the shock with 34.4 Joules
Fig. 119.2 ICD reading at the time of the visit. From top to bottom: V-tip to V-ring EGM, channel markers, and then the leadless ECG. It shows one T-wave oversensing after the fifth ventricular beat
Case 119
467
Fig. 119.3 ICD reading after lowering the sensitivity from 0.3 to 0.45 mV causing the resolution of T-wave oversensing. From top to bottom: V-tip to V-ring EGM, channel markers, and then the leadless ECG
Case 120 Paul A. Friedman and Charles D. Swerdlow
Case Summary A 76 year-old man with prior inferior wall myocardial infarction (NYHA Class II, left-ventricular ejection fraction 41%, first degree AV block, and narrow QRS) underwent implantation of a dual-chamber ICD (Medtronic EnTrust™ D154ATG) for sustained monomorphic VT requiring cardioversion. He was treated with sotalol 120 mg q 12 h to reduce the frequency of VT. Figure 120.1 shows an episode retrieved from the ICD’s SVT log with programmed detection and bradycardia pacing parameters. Why is VT misdiagnosed as sinus tachycardia?
Case Discussion Present Medtronic ICDs discriminate 1:1 AV conduction of sinus tachycardia from 1:1 VA conduction of VT using a dual chamber onset algorithm (Sinus Tachycardia Rule™) that evaluates both PR and RR interval onset.1 If the predominant AV pattern at the onset of tachycardia is 1:1, the algorithm determines if the R–R and P–R intervals are within the “expected” range based on the range of variability for recent P–R and R–R intervals. Sinus tachycardia is diagnosed and VT therapy is withheld if both the R–R and P–R intervals are within the “expected” range. Premature ventricular complexes increase the expected range of R–R intervals. Figure 120.1 shows ICD programming, stored electrogram, and interval plot corresponding to the episode of VT,
which begins during atrial-sensed, ventricular-paced rhythm. VT begins with a non-physiologically short P–R interval. The next ventricular beat is not preceded by an atrial electrogram, but it and subsequent ventricular beats are followed by an atrial electrogram with a fixed VA interval. This pattern indicates the onset of VT with 1:1 VA conduction. The “ST” marker at the 16th tachycardia beat (lower right) indicates withholding of VT therapy because of the ICD’s diagnosis of sinus tachycardia. The Sinus Tachycardia Rule applies because this tachycardia has a 1:1 AV relationship and cycle length between the VT detection interval and SVT Limit. Since the tachycardia begins during ventricular-paced rhythm, the expected P–R window does not apply. The range of expected R–R intervals is wide because of frequent premature ventricular complexes indicated by short intervals followed by compensatory pauses on the interval plot (e.g. third ventricular electrogram in middle panel). The change in R–R interval between sinus tachycardia with cycle length of 500 ms and VT with cycle length 430 ms is insufficient to exceed the expected range of R–R intervals. Thus the 1:1 AV relationship and expected R–R interval are classified as consistent with sinus tachycardia, and the expected P–R interval criterion is not applied. This error in classification can be prevented by programming “Managed Ventricular Pacing™,”2,3 which permits AV conduction during sinus rhythm with first degree AV block. During Managed Ventricular Pacing, the expected P–R interval is required to withhold VT therapy and diagnose sinus tachycardia, so VT would probably have been diagnosed correctly.
P.A. Friedman (*) Division of Cardiovascular Diseases and Department of Internal Medicine, Mayo Clinic, 200 First Street SW, Rochester, MN 55905 e-mail:
[email protected] C.D. Swerdlow Division of Cardiology, David Geffen School of Medicine, Cedars-Sinai Medical Center e-mail:
[email protected] A. Natale et al. (eds.), Cardiac Electrophysiology, DOI: 10.1007/978-1-84996-390-9_120, © Springer-Verlag London Limited 2011
469
470
P.A. Friedman and C.D. Swerdlow
Fig. 120.1 Top panel shows programmed values for detection of VT/VF, rejection of SVT, and bradycardia pacing. Middle panel shows stored electrogram. Lower panel shows interval plot. Open squares denote atrial events. Closed circles denote ventricular events. See text for discussion
References 1. Stadler RW, Gunderson B, Pearson A, Gillberg J. Adaptive algorithm to withhold ICD therapy during sinus tachycardia (abstract). Pacing Clin Electrophysiol. 2000;23(4 Part II):677. 2. Sweeney MO, Bank AJ, Nsah E, et al. Minimizing ventricular pacing to reduce atrial fibrillation in sinus-node disease. N Engl J Med. Sep 6 2007;357(10):1000-1008.
3. Sweeney MO, Ellenbogen KA, Casavant D, et al. Multicenter, prospective, randomized safety and efficacy study of a new atrial-based managed ventricular pacing mode (MVP) in dual chamber ICDs. J Cardiovasc Electrophysiol. Aug 2005;16(8):811-817.
Case 121 Kenneth A. Ellenbogen
Case Summary
Case Discussion
A 44 year old woman with NYHA Class III congestive heart failure, non-ischemic cardiomyopathy and a left bundle block (QRS duration: 150 ms) underwent implantation of a biventricular ICD 6 months ago. The patient returned 1 month after the initial implant and noted a significant improvement in heart failure symptoms. She is seen now at her 6 month clinic visit and her heart failure symptoms have worsened. Her device is interrogated and she is pacing her ventricle less than 50% of the time. A rhythm strip is printed from the programmer (Fig. 121.1) and the programmed parameters are shown below:
This is a patient with loss of response to CRT (cardiac resynchronization pacing). The etiologies are numerous, but can be divided into SYSTEM RELATED and PATIENT RELA TED. System related causes include atrial undersensing, atrial oversensing, ventricular oversensing and loss of left ventricular capture. Patient related causes include atrial fibrillation, poor lead placement, lack of dyssynchrony, suboptimal AV and V–V coupling. In this patient, the problem is clearly ventricular oversensing. The cause appears to be due to T wave oversensing. The morphologies of the 2 “VS” events are very different and most likely not both PVCs. The second VS event occurs only 520 ms after the first VS (Fig. 121.2). The second VS is narrow and looks like an intrinsic depolarization. This brings into question whether the first VS is a true PVC. The first VS following the paced ventricular event is likely due to T wave oversensing. T wave oversensing is causing a loss of BiV pacing. The patient’s intrinsic R wave falls shortly after the T wave and resets the timer. By decreasing the ventricular sensitivity (increasing its value), the T wave is no longer sensed and normal biventricular pacing resumes (Fig. 121.3). In this case, the ventricular sensitivity was programmed from 0.3 to 0.45 mV with a restoration of BiV pacing, and the device was programmed to 0.6 mV as the R wave was >12 mV during sinus rhythm. Defibrillation thresholds (DFTs) should be done to confirm adequate sensing during ventricular tachyarrhythmias at the programmed sensitivity of 0.6 mV.
Atrial Lead Atrial lead amplitude
3.5 V
Pulse width
0.4 ms
Sensitivity
0.3 mV
Pace blanking
200 ms
RV Lead Amplitude
3 V
Pulse width
0.4 ms
Sensitivity
0.3 mV
Pace blanking
200 ms
LV lead Amplitude
6 V
Pulse width
1.1. ms
Pace polarity
LV tip to RV coil
Why is the ventricle paced only 50% of the time and what programming changes are recommended?
K.A. Ellenbogen Department of Cardiology, VCU School of Medicine, 980053, Richmond, VA 23298-0053 e-mail:
[email protected] A. Natale et al. (eds.), Cardiac Electrophysiology, DOI: 10.1007/978-1-84996-390-9_121, © Springer-Verlag London Limited 2011
471
472 Fig. 121.1 Rhythm strip printed from programmer showing atrial and ventricular EGMs and the marker channel
Fig. 121.2 Rhythm strip printed from programmer showing atrial and ventricular EGMs and the marker channel. Note TW oversensing
Fig. 121.3 No further TW oversensing after programing changes
K.A. Ellenbogen
Case 122 Anurag Gupta
Case Summary
Case Discussion
A 75 year-old male with permanent atrial fibrillation, a history of congestive of heart failure due to ischemic cardiomyopathy, and a biventricular implantable cardioverter defibrillator is admitted with medically refractory recurrent ventricular tachycardia resulting in disabling shock therapy. He undergoes successful endocardial catheter ablation, with no subsequent inducible ventricular tachycardia nor spontaneous sustained events for the remainder of his hospitalization. However, in the context of atrial fibrillation, he is noted on telemetry to have a persistently elevated paced ventricular rate of approximately 90 ppm, including overnight while sleeping comfortably (Fig. 122.1). Interrogation of his Guidant Contak Renewal 3 HE biventricular ICD shows adequate battery longevity and stable lead parameters. Device settings are as follows:
Notable reasons for ventricular pacing at rate exceeding the lower rate limit include but are not limited to: (a) sensor driven rate changes, unlikely in this patient as his accelerometer would not be predicted to lead to sustained elevations in heart rate while stationary and/or during sleep; (b) P-wave tracking modes, not possible in this patient whose pacing mode is VVIR; (c) programmed interventions such as rate smoothing, rate hysteresis, rate drop response, overdrive algorithms, conducted atrial fibrillation response, ventricular sense response, and ventricular rate regularization (discussed below); (d) reversion to alternate mode and/or rate, for example as with magnet application, electromagnetic interference, and battery depletion; and (e) device malfunction including component and/or circuit failures with runaway. Several features deserve special mention in individuals such as the one discussed in this case who have biventricular devices and experience frequent PVCs and/or conducted atrial tachyarrhythmias. This may lead to unwanted (a) competition with pacing and thus decrease in cardiac resynchronization therapy; and/or (b) irregularity in ventricular cycle length, which may lead to hemodynamic and symptomatic compromise. One intervention, ventricular sense response, allows triggering of ventricular pacing in response to sensed ventricular events, mitigating the potential loss of efficacy with conducted tachyarrhythmias or frequent PVCs. An additional potential therapy, and the one that accounts for this patient’s elevated ventricular rates, is ventricular rate regularization, or similar algorithms that represent modifications of rate smoothing. Ventricular rate regularization may reduce symptoms by minimizing V–V cycle length variation during atrial arrhythmias by adjusting the ventricular pacing rate (up to a programmable maximum rate) in accordance to a weighted sum of current ventricular cycle lengths.1 A telemetry strip in this patient demonstrates how ventricular rate regularization leads to greater maintenance of V–V regularity despite frequent PVCs and to elevated ventricular pacing rate above the lower rate limit (Fig. 122.1). Note that frequent PVCs, as opposed to rapid conduction of his
Mode
VVIR
Lower rate
70 ppm
Maximum sensor rate
120 ppm
Pacing chamber
Biventricular (LV offset 0 ms)
Sensor (accelerometer)
Activity threshold: Medium
Rate hysteresis
Off
Rate smoothing
Off
Ventricular rate regularization
Maximum (maximum pacing rate 120 ppm) Begin at 125 bpm
Ventricular tachyarrhythmia therapies
What are the possibilities accounting for his ventricular pacing at an elevated rate that exceeds his lower rate limit?
A. Gupta Cardiac Electrophysiology Service, Division of Cardiology, Department of Medicine, Stanford University Hospital and Clinics, 300 Pasteur Drive, Room H2146, Stanford, CA 94305-5233 e-mail:
[email protected]
A. Natale et al. (eds.), Cardiac Electrophysiology, DOI: 10.1007/978-1-84996-390-9_122, © Springer-Verlag London Limited 2011
473
474
A. Gupta
Fig. 122.1 Telemetry recording demonstrating ventricular pacing above the lower rate limit
Fig. 122.2 12 h overnight heart rate trend. Note the abrupt transition in ventricular rate after the ventricular rate regularization feature was turned off (arrow)
underlying atrial fibrillation, are present in this patient and are responsible for utilization of the ventricular rate regularization feature. The device was reprogrammed to turn off the ventricular rate regularization function. Subsequently, the ventricular rate promptly decreased from approximately 90–70 ppm, as illustrated during overnight 12 h telemetry trend (Fig.. 122.2). More specifically, a telemetry strip with the new settings
shows that following a PVC, the next ventricular beat occurs at the completion of the lower rate interval, keeping the overall ventricular rate close to 70 ppm though with greater V–V irregularity (Fig. 122.3). This case illustrates the importance of recognizing reasons for changes in ventricular rate, including programmed interventions used to mitigate irregularity in ventricular pacing such as ventricular rate regularization and/or to minimize loss of biventricular pacing.
Case 122 Fig. 122.3 Telemetry recording demonstrating ventricular pacing with ventricular rate regularization feature turned off. Note the irregularity in V–V cycle due to PVC and the overall slower paced ventricular rate
Reference 1. Tse HF, Newman D, Ellenbogen KA, Buhr T, Markowitz T, Lau C. Effects of ventricular rate regularization pacing on quality of life and symptoms in patients with atrial fibrillation (Atrial fibrillation symptoms mediated by pacing to mean rates [AF Symptoms Study]). Am J Cardiol. 2004;94:938-941.
475
Case 123 Ronald Lo, Amin Al-Ahmad, and Paul J. Wang
Case Summary A 48 year old male with a history of coronary artery disease, hypertension, ischemic cardiomyopathy with an ejection fraction of 10% with an biventricular ICD presented with an shock from his device. He has had increased social stressors at home which have resulted in him running out of his heart failure medications. Interrogation of his Medtronic InSync Sentry biventricular ICD revealed an episode of atrial fibrillation with a rapid ventricular response followed by an aborted discharge and subsequent 34.9 J shock with restoration of sinus rhythm (Fig. 123.1). What is the ventricular rhythm, and why does he receive a shock?
Case Discussion Examination of the interrogation report reveals an atrial and ventricular electrogram with rapid atrial fibrillation. The atrial activity is clearly disorganized with an irregular
ventricular rhythm. The ventricular rhythm is secondary to the variable conduction down the AV node from the rapid atrial fibrillation. It is unlikely to be a ventricular tachycardia given the irregularity of the rhythm. Careful examination of the episode reveals that the episode was detected and then aborted, followed by detection in the VF zone with an ICD discharge which fortuitously converts him to sinus rhythm. Treatment of atrial fibrillation and prevention of rapid conduction down the AV node with beta blockers or calcium channel blockers is the primary therapeutic treatment for this occurrence. Inappropriate ICD discharges remain frequent, with up to 20–30% of ICD discharges being inappropriate, with supraventricular arrhythmias being a frequent culprit.1 Programming the device to a higher VT or VF zone may alleviate the problem, but may risk the undertection of true VT or VF. SVT discriminators may have a role in the prevention of inappropriate discharges; however it may be very difficult distinguishing true VT from an SVT. This case illustrates that ICD discharges may occur in the setting of non-ventricular tachyarrhythmias.
R. Lo (*) Riverside Electrophysiology, 4000 14th Street, Suite 209, Riverside, CA 92501 e-mail:
[email protected] A. Al-Ahmad and P.J. Wang School of Medicine, Stanford University, 300 Pasteur Drive, H-2146, Stanford, CA 94305 e-mail:
[email protected];
[email protected] A. Natale et al. (eds.), Cardiac Electrophysiology, DOI: 10.1007/978-1-84996-390-9_123, © Springer-Verlag London Limited 2011
477
478
Fig. 123.1 Device interrogation showing an atrial fibrillation with an ICD discharge
R. Lo et al.
Case 123
Reference 1. Korte T, Köditz H, Niehaus M, Thomas PT, Tebbenjohanns J. High incidence of appropriate and inappropriate ICD therapies in children and adolescents with implantable cardioverter defibrillator. Pacing Clin Electrophysiol. 2004;27(7):924-932.
479
Case 124 Ronald Lo, Amin Al-Ahmad, and Paul J. Wang
Case Summary
be conducted to the ventricle and sensed appropriately in the ventricle. The differential diagnosis here remains ventricular tachycardia or a supraventricular tachycardia. A 55 year old female with idiopathic nonischemic dilated Upon closer inspection of the intervals, one notes that cardiomyopathy and complete left bundle branch was seen in there is not a constant VA interval that might suggest venthe clinic for significant dyspnea on exertion. She underwent tricular tachycardia with 1:1 retrograde conduction. Rather, implantation of a biventricular implantable cardioverter- the AV interval remains quite constant increasing the likelidefibrillator with significant improvement in her symptoms hood that this is an atrial tachycardia with 1:1 ventricular of heart failure. conduction. The device detects this episode in the VT zone More recently she has been having episodes of palpitaand delivers anti-tachycardia pacing (ATP) which is seen to tions due to medication noncompliance, and usually associterminate the tachycardia. The absence of VA conduction ated with significant weight gain. during the ATP pacing train also argues against the likeliInterrogation of her Medtronic InSync II Marquis device hood that this is VT with 1:1 retrograde conduction. revealed an episode of tachycardia as shown in (Fig. 124.1). While ATP is fairly successful in terminating some venThe device settings are as follows: tricular tachycardias, termination of a supraventricular tachycardia such as an atrial tachycardia can also occur. ATP may Mode: DDD with lower rate limit of 50 and upper rate limit of 120 offer the potential for painless termination of ventricular Mode Switch: On at 155 bpm tachycardia or other tachyarrhythmias. Reduction in painful VF Zone (rate > 207 bpm) : 30 Joule x 6 VT Zone (rate > 167 bpm): ATP x 3 (Burst), 15 Joule x 1 and 30 shocks may improve patient quality-of-life and extend the Joule x 4 longevity of the implanted device. In this case, ATP successWhat is the mechanism of the tachycardia and how did it fully terminated an atrial tachyarrhythmia and prevented an inappropriate shock. terminate?
Case Discussion Examination of the intracardiac electrograms demonstrates a 1:1 tachycardia that starts with an atrial beat that is in the post ventricular atrial refractory period. This beat appears to
R. Lo (*) Riverside Electrophysiology, 4000 14th Street, Suite 209, Riverside, CA 92501 e-mail:
[email protected] A. Al-Ahmad and P.J. Wang School of Medicine, Stanford University, 300 Pasteur Drive, H-2146, Stanford, CA 94305 e-mail:
[email protected];
[email protected] A. Natale et al. (eds.), Cardiac Electrophysiology, DOI: 10.1007/978-1-84996-390-9_124, © Springer-Verlag London Limited 2011
481
482
Fig. 124.1 Intracardiac electrogram of tachycardia with termination via ATP
R. Lo et al.
Case 125 Ronald Lo, Amin Al-Ahmad, and Paul J. Wang
Case Summary A 72 year old man with a history of nonischemic dilated cardiomyopathy with an ejection fraction of 20%, complete left bundle branch block and mechanical aortic and mitral valves. He has been having symptomatic Class III–IV heart failure symptoms and subsequently underwent implantation of a St Jude Medical Atlas + HF Model V-340 biventricular defibrillator. His symptoms of heart failure have improved significantly after implantation of his cardiac resynchronization device. However, he does note occasionally the sensation of palpitations with subjective worsening of dyspnea on exertion. His device interrogation revealed the event displayed in Fig. 125.1. What is the rhythm displayed? Why would this rhythm cause him to experience worsening heart failure symptoms?
Case Discussion Interrogation of his device revealed recording of two electrogram channels, the atrial and ventricular channels. Prior to the event, he is noted to be tracking his atrium with biventricular pacing. The fourth beat of the electrogram reveals
the start of an atrial arrhythmia, likely an atrial tachycardia with predominately 1:1 conduction to the ventricle. It is interesting to note that the PR interval during the atrial tachycardia is shorter than the PR interval during a paced rhythm of the first two beats. This may be due to the location of the atrial tachycardia being located closer to the AV node. Upon detection of an atrial arrhythmia, or atrial high rate episodes, the device mode switches from a tracking mode to a demand backup pacing mode, in this case DDI. In DDI mode, there is sensing in both the atrium and the ventricle, with the only action the device is taking is inhibition of pacing with a sensed complex. The benefit of DDI pacing is prevention of rapid atrial tracking, however with less optimal atrial and ventricular timings, especially in a cardiac resynchronization device. In this patient, there is loss of biventricular pacing as the ventricular complexes are due to the intrinsic conduction from the atrial tachycardia. The loss of beneficial biventricular pacing and atrial-ventricular synchrony contributes to worsening hemodynamic function and mechanical dyssynchrony. Treatment of the atrial tachycardia with beta blockers or antiarrhythmic medications may decrease the likelihood of this from recurring. This case illustrates the importance of maintaining biventricular pacing in patients who have atrial tachyarrhythmias.
R. Lo (*) Riverside Electrophysiology, 4000 14th Street, Suite 209, Riverside, CA 92501 e-mail:
[email protected] A. Al-Ahmad and P.J. Wang School of Medicine, Stanford University, 300 Pasteur Drive, H-2146, Stanford, CA 94305 e-mail:
[email protected];
[email protected] A. Natale et al. (eds.), Cardiac Electrophysiology, DOI: 10.1007/978-1-84996-390-9_125, © Springer-Verlag London Limited 2011
483
484 Fig. 125.1 Device interrogation
R. Lo et al.
Section Clinical Cases
III
Case 126 Mehmet K. Aktas, Abrar H. Shah, and James P. Daubert
Case Summary
Case Discussion
A 51-year-old man on chronic methadone therapy for a history of heroin abuse presented to the emergency room with cough and dyspnea. On exam he was found to be tachypneic and pulse oximetry showed a saturation of 88% on room air and a portable chest X-ray revealed a right lower lobe infiltrate. His serum potassium was 3.6 meq/L and his serum magnesium was 2.0 meq/L. An electrocardiogram performed upon arrival showed sinus arrhythmia with evident U waves and a prolonged QTc interval (Fig. 126.1) although this was not recognized at the time. He was given intravenous moxifloxacin while in the emergency room and was hospitalized for in-patient antibiotic therapy. Twelve hours later the patient reported feeling anxious and was found to be diaphoretic. Telemetry monitoring was initiated and showed sinus rhythm with frequent ventricular ectopy. An electrocardiogram showed ventricular bigeminy with significant QT interval prolongation (Fig. 126.2). Minutes later the patient became pulseless and apneic and was found to be in ventricular fibrillation (Fig. 126.3). Chest compressions were started and within a minute spontaneous return of sinus rhythm was noted. Serial electrocardiograms demonstrated progressive QT prolongation with rate corrected QT intervals (QTc) as high as 630 ms. What was the likely cause for this patient’s cardiac arrest?
The QT interval, the electrocardiographic gauge of ventricular repolarization, is often overlooked or misinterpreted. Alter ations to the timing and mechanism of ventricular repolari zation can lead to ventricular tachyarrhythmias particularly “short-long-short” sequences, which are often a trigger of torsade de pointes. The factors influencing the QT interval are complex and may include a variety of channelopathies, changes in autonomic innervation, and acquired factors such as drugs or electrolyte disturbances.1,2 The patient described was on methadone which is a known QT prolonging drug and hence prudence would require that the QT interval be closely monitored when other QT prolonging drugs are begun.3–5 Even with a prolonged baseline QTc (which was overlooked) he was started on a fluoroquinolone type antibiotic, moxifloxacin, which has been shown to consistently prolong the QT interval and has rarely been associated with torsade de pointes.6,7 The combination of methadone, and moxifloxacin in this patient led to significant QT prolongation and torsade de pointes. Once drug induced torsade de pointes (or even significant QT prolongation without torsade) is identified, immediate discontinuation of the offending drug or drugs is required. Temporary pacing may be considered to prevent pause related ventricular tachyarrhythmias and “short-long-short” sequences. Chronotropic agents, such as isoproterenol or atropine, may also be considered to increase the heart rate in attempt to shorten the QT interval and eliminate “short-longshort” sequences. Intravenous magnesium sulfate, a safe and effective adjunctive therapy, may also be given for the acute termination of torsade de pointes.8 Potassium should be maintained in the high normal range. The patient above continued to have salvos of ventricular fibrillation despite these measures and so low-dose intravenous dopamine for chronotropic support was started and the episodes of ventricular fibrillation stopped. Moxifloxacin was discontinued and over the ensuing days his QT interval returned to baseline. His methadone was gradually tapered with further normalization of his QTc. Since discharge has remained in sinus rhythm with no further arrhythmias or syncope.
M.K. Aktas (*) Department of Cardiology/Electrophysiology, University of Rochester Medical Center, 601 Elmwood Ave, 679C, Rochester, NY 14642 e-mail:
[email protected] A.H. Shah Department of Cardiology, University Cardiovascular Associates, 2365 South Clinton Ave., Suite 100, Rochester, NY 14618 e-mail:
[email protected] J.P. Daubert Cardiac Electrophysiology, Cardiology Division, Duke University Health System, DUMC Box 3174 Duke Hospital 7451H, Durham, NC 27710 e-mail:
[email protected]
A. Natale et al. (eds.), Cardiac Electrophysiology, DOI: 10.1007/978-1-84996-390-9_126, © Springer-Verlag London Limited 2011
487
488
M.K. Aktas et al.
Fig. 126.1 Electrocardiogram at presentation showing sinus arrhythmia. The QTc interval is prolonged at 495 ms
Fig. 126.2 Electrocardiogram shows a ventricular bigeminal rhythm which confounds calculation of the QTc. Nevertheless, the QT interval is severely prolonged extending out to the succeeding QRS complex. The QTc is estimated at about 630 ms
Case 126
489
Fig. 126.3 Telemetry strip showing ventricular bigeminy with a “short-long-short” sequence followed by torsade de pointes
References 1. Roden DM. Drug-induced prolongation of the QT interval. N Engl J Med. 2004;350:1013-1022. 2. Moss AJ. Drug-induced QT prolongation: an update. Ann Noninvasive Electrocardiol. 2006;11:1-2. 3. Wedam EF, Bigelow GE, Johnson RE, Nuzzo PA, Haigney MC. QT-interval effects of methadone, levomethadyl, and buprenorphine in a randomized trial. Arch Intern Med. 2007;167:2469-2475. 4. Krantz MJ, Lewkowiez L, Hays H, Woodroffe MA, Robertson AD, Mehler PS. Torsade de pointes associated with very-high-dose methadone. Ann Intern Med. 2002;137:501-504.
5. Chugh SS, Socoteanu C, Reinier K, Waltz J, Jui J, Gunson K. A community-based evaluation of sudden death associated with therapeutic levels of methadone. Am J Med. 2008;121:66-71. 6. Dale KM, Lertsburapa K, Kluger J, White CM. Moxifloxacin and torsade de pointes. Ann Pharmacother. 2007;41:336-340. 7. Sherazi S, DiSalle M, et al. (2008). Moxifloxacin-induced torsades de pointes. Cardiol J 15(1): 71-73. 8. Tzivoni D, Banai S, Schuger C, et al. Treatment of torsade de pointes with magnesium sulfate. Circulation. 1988;77:392-397.
Case 127 Loren P. Budge and John P. DiMarco
Case Summary A 55-year-old man was brought to the emergency department after an episode of palpitations and syncope. He had no prior cardiac history, and his medical history was significant only for hypertension, which he has been trying to manage with a salt-restricted diet. He has been quite active and denied previous symptoms of angina or heart failure. He has not been taking any medications. His symptoms started abruptly while sitting at his desk at work, and consisted of rapid palpitations with chest pain, shortness of breath, lightheadedness, and diaphoresis. A nurse was present and reported a heart rate near 200 bpm. After a few minutes, he briefly lost consciousness, then quickly awoke and felt well. He has no
known history of arrhythmias, although he has had several prior episodes of palpitations in the past which have resolved spontaneously for which he had not sought evaluation. The rescue squad was called. During transport several short runs of a wide complex tachycardia were noted on monitor but no strips were saved. Upon arrival at the emergency room, his heart rate was 80, with a blood pressure of 108/72. His electrocardiogram is shown in Fig. 127.1. An echocardiogram was obtained later that day and is shown in Fig. 127.2. This led to a cardiac magnetic resonance study as shown in Fig. 127.3. What is this patient’s diagnosis? What arrhythmias is this patient likely to have? What would you do next?
Fig. 127.1 ECG on arrival to the emergency department
L.P. Budge (*) Cardiovascular Division, University of Virginia Health System, 1215 Lee Street, Box 800662, Charlottesville, VA, 22908 USA e-mail:
[email protected] J.P. DiMarco Cardiovascular Division, University of Virginia Health System, 1215 Lee Street, Charlottesville, VA, 22908 USA e-mail:
[email protected] A. Natale et al. (eds.), Cardiac Electrophysiology, DOI: 10.1007/978-1-84996-390-9_127, © Springer-Verlag London Limited 2011
491
492
Fig. 127.2 Transthoracic echocardiogram apical four chamber view at end-systole. This echocardiogram is diagnostic for left ventricular noncompaction. Pertinent findings include prominent left ventricular trabeculae with a ratio of non-compacted to compacted myocardium >2:1 (double-sided arrows demarcate compacted and non-compacted segments). Intertrabecular recesses are also seen (one-sided arrow). Color Doppler (not shown) demonstrates flow within these recesses with communication to the left ventricular cavity
L.P. Budge and J.P. DiMarco
Fig. 127.3 Cardiac magnetic resonance (CMR) apical short axis view in diastole. This CMR demonstrates the hallmark features of left ventricular noncompaction. There is a greater than 2.3:1 ratio of non-compacted to compacted myocardium (demarcated by the doublesided arrows shown) with prominent inferolateral trabeculae. Cine (not shown) reveals hypokinetic contraction of the noncompacted segments with evidence of blood flow within the trabecular space with communication to the left ventricular cavity
Case Discussion This gentleman presented with an episode of syncope with non-sustained wide complex tachycardia noted by the rescue squad prior to his admission. His admission ECG is nonspecific since it shows only normal sinus rhythm with lateral T wave inversions. However, his echocardiogram shows prominent left ventricular trabeculae and is diagnostic for isolated left ventricular non-compaction (LVNC). LVNC is an uncommon congenital cardiomyopathy. Patients with LVNC will manifest on echocardiography prominent left ventricular (LV) trabeculae with deep intertrabecular recesses. This pattern is caused by intrauterine arrest of compaction, resulting in two layers of myocardium: compacted and non-compacted. There is continuity between the LV cavity and the intertrabecular recesses, without any communication to the epicardial vessels. LVNC most commonly affects the inferolateral portion of the LV apex, although other areas, including the RV can also be affected. There is usually a corresponding decrease in LV ejection fraction. The most common clinical presentations of symptomatic LVNC are heart failure, arrhythmia (atrial or ventricular), chest pain or systemic embolism. The sinus rhythm ECG usually shows only non-specific ST or T wave abnormalities. Associated conduction abnormalities such as bundle branch or fascicular blocks, or Wolf-Parkinson-White syndrome may be seen. When LVNC occurs in families, it has been linked to mutations in a number of cytoskeletal proteins and the
inheritance pattern is usually autosomal dominant. There is some phenotypic and genotypic overlap with other cardiomyopathies, especially hypertrophic cardiomyopathy. LVNC may also occur with several neuromuscular disorders such as Barth Syndrome, Charcot-Marie-Tooth 1a, MelnickNeedles Syndrome and Nail-patella Syndrome. It may rarely be seen in conjunction with other forms of congenital heart disease, including Ebstein’s anomaly, bicuspid aortic valve, L-TGA, left atrial appendage isomerism, and ventricular septal defects. The prevalence of LVNC was originally thought to be extremely rare, but as imaging modalities and awareness foster recognition of milder forms of the disorder, more cases with less prominent clinical and imaging findings are being diagnosed. The diagnostic criteria of isolated LVNC differ depending on the imaging modality used. Echocardiographic criteria include an absence of coexisting cardiac abnormalities, a ³ 2:1 ratio of non-compacted to compacted myocardium (NC/C ratio) at end-systole with thickening of the myocardial wall (as opposed to apical thrombus, which would not thicken) and documentation of flow within the intertrabecular recesses. Cardiac magnetic resonance imaging criteria for LVNC require a NC/C ratio of >2.3:1 at end-diastole which has a sensitivity of 86% and a specificity of 99%. Prognosis in LVNC is controversial. Most studies report no more than 3–4 year follow-up durations and are biased towards patients with the more severe forms of the disease. Those who present in childhood have a reported 14% 3 year mortality. LV function often recovers transiently if heart
Case 127
failure therapy is initiated, only to again worsen in early adulthood. Of adults who present with symptoms, 41% of patients had documented ventricular tachycardia (VT), 53% were hospitalized for heart failure, and 24% had a thromboembolic event over a mean 44 month follow-up period. Transplant-free survival in symptomatic patients was 58% at 5 years, with a 35% mortality rate, half of whom died suddenly. However among asymptomatic patients with an incidental finding of LVNC, 5 year transplant-free survival was 97%, with less than a 10% thromboembolic rate. Further studies on the long-term prognosis of asymptomatic patients with findings of LVNC during cardiac imaging are clearly needed. Treatment recommendations include standard medical therapy according to ejection fraction, heart failure symptoms, and atrial arrhythmias. Due to the propensity for thrombus formation, warfarin is recommended for patients with an EF £ 40% or with atrial fibrillation even if other risk factors are absent. Given that sudden cardiac death is common in this population, and there is currently no reliable way to determine who is at risk for life-threatening arrhythmia, recent guidelines support consideration of ICD therapy as a Class 2B indication. This patient presented with heart failure and what was most likely ventricular tachycardia. He was started on therapy with an ACE inhibitor and a beta blocker. A dual chamber ICD was inserted. Three months after insertion, he had an episode of lightheadedness and an electrogram stored by the ICD showed a burst of monomorphic ventricular tachycardia that was broken with antitachycardia pacing. No atrial arrhythmias have been detected.
493
Bibliography Engberding R, Yelbuz TM, Breithardt G. Isolated noncompaction of the left ventricular myocardium – a review of the literature two decades after the initial case description. Clin Res Cardiol. 2007;96:481-488. Epstein AE, DiMarco JP, Ellenbogen KA, et al. ACC/HRS 2008 guidelines for device-based therapy of cardiac rhythm abnormalities: a report of the American College of Cardiology/American Heart Association Task Force on Practice Guidelines (Writing Committee to Revise the ACC/AHA/NASPE 2002 Guideline Update for Implantation of Cardiac Pacemakers and Antiarrhythmia Devices): developed in collaboration with the American Association for Thoracic Surgery and Society of Thoracic Surgeons. Circulation. 2008;117:e350-e408. Frischknecht BS, Attenhoffer Jost CH, et al. Validation of noncompaction criteria in dilated cardiomyopathy, and valvular and hypertensive heart disease. J Am Soc Echo. 2005;18:865-872. Jenni R, Oechslin E, Schneider J, Attenhofer Jost C, Kaufmann PA. Echocardiographic and pathoanatomical characteristics of isolated left ventricular non-compaction: a step towards classification as a distinct cardiomyopathy. Heart. 2001;86:666-671. Kobza R, Jenni R, Erne P, Oechslin E, Duru F. Implantable cardioverster-defibrillators in patients with left ventricular noncompaction. PACE. 2008;31:461-467. Murphy RT, Thaman R, Blanes JG, et al. Natural history and familial characteristics of isolated left ventricular non-compaction. Eur Heart J. 2005;26:187-192. Oechslin EN, Attenhofer Jost CH, Rojas JR, Kaufmann PA, Jenni R. Long-term follow-up of 34 adults with isolated left ventricular noncompaction: a distinct cardiomyopathy with poor prognosis. J Am Coll Cardiol. 2000;36:493-500. Petersen SE, Selvanayagam JB, Wiesmann F, et al. Left ventricular non-compaction: insights from cardiovascular magnetic resonance imaging. J Am Coll Cardiol. 2005;46:101-105.
Case 128 David J. Callans
Case Summary
What is the likely mechanism responsible for these findings?
A 62-year-old man without previous cardiac history presented to the hospital with complaints of near syncope and exercise intolerance. The presenting electrocardiogram (Fig. 128.1) demonstrates sinus rhythm with a normal PR interval and narrow QRS on half of the conducted beats, intermittent AV block, and alternating left and right bundle aberrancy on the other half of the conducted beats.
Case Discussion An electrophysiologic study was performed to investigate the pathophysiology of his AV block (Fig. 128.2). A split His potential was recorded and intermittent AV block was 500 ms
I II III aVR aVL aVF V1 V2 V3 V4 V5 V6 4:54:12 PM
4:54:13 PM
4:54:14 PM
4:54:15 PM
4:54:16 PM
Fig. 128.1 Presenting electrocardiogram with intermittent AV block and alternating left and right bundle aberration
D.J. Callans Department of Cardiology, University of Pennsylvania, 3400 Spruce Street, Philadelphia, PA 19104, USA e-mail:
[email protected] A. Natale et al. (eds.), Cardiac Electrophysiology, DOI: 10.1007/978-1-84996-390-9_128, © Springer-Verlag London Limited 2011
495
496
D.J. Callans 200 ms
I
aVF
V1
S
V6
hRA
CT
HIS d
C
HIS p
C 4:16:04 PM
4:16:05 PM
Fig. 128.2 Surface and intracardiac recordings during sinus rhythm and atrial pacing. A split His potential is seen, and atrial pacing results in intermittent intra-His conduction block
A proposed mechanism for this phenomenon is presented in Fig. 128.3. Despite normal infra-His conduction, alternating aberrancy is favored during 3:2 conduction because of alternating retrograde concealment, resulting in long-short input to the other bundle branch on the following portion of the sequence. Alternating bundle branch block is recognized as a high risk situation and is considered a Class 1 indication for permanent pacing. The patient was treated with dual chamber pacing, indicated because of the finding of intra His block, which resolved his symptoms in follow up.
Fig. 128.3 Concealment into the distal left bundle branch on the first aberrant beat “protects” the left bundle from long-short stimulation (by making the pause shorter with reference to the left bundle than the right) following the sinus complex that blocks at the infra His level. The pattern reverses itself in the next part of the sequence, shielding the right bundle branch in the same manner
induced with atrial pacing, with an intra-His level of conduction block. The alternating bundle branch pattern was not reproduced during the EP study, because a steady state pattern of 3:2 block could not be demonstrated, despite pacing autonomic manipulations.
Bibliography Bharati S, Lev M, Wu D, Denes P, Dhingra R, Rosen KM. Pathophysiologic correlations in two cases of split His bundle potentials. Circulation. 1974;49:615-623. Lerman BB, Marchlinski FE, Kempf FC, Buxton AE, Waxman HL, Josephson ME. Prognosis in patients with intra-Hisian conduction disturbances. Int J Cardiol. 1984;5:449-460. McAnulty JH, Murphy E, Rahimtoola SH. Prospective evaluation of intrahisian conduction delay. Circulation. 1979;59:1035-1039. Wu D, Denes P, Dhingra RC, et al. Electrophysiological and clinical observations in patients with alternating bundle branch block. Circulation. 1976;53:456-464.
Case 129 Andrew E. Darby and John P. DiMarco
Case Summary A 51 year-old man is referred for evaluation of heart failure with AV block. He had been in his usual state of health until 6 months earlier when he began to noted easy fatigue. He then developed gradually progressive dyspnea with exertion. Two weeks prior to presentation, the patient’s dyspnea with exertion progressed to the point of shortness of breath with walking only fifty feet. The onset of orthopnea and paroxysmal nocturnal dyspnea finally prompted him to seek care in the emergency department. His medical history includes hypertension, dyslipidemia, and obesity. He had donated his left kidney to a brother with polycystic kidney disease 13 years earlier. The patient does not smoke or use illicit substances. He lives with a wife and one child, and he works as a long-haul commercial truck driver. The patient was afebrile on presentation. He was relatively hypotensive with a blood pressure of 85/60, and he had a heart rate of 90 bpm. He was noted to clinically have heart failure with elevated jugular venous pressure, bibasilar crackles, and lower extremity edema. Studies obtained included a chest x-ray demonstrating cardiomegaly with pulmonary vascular congestion. His presenting electrocardiogram is shown (Fig. 129.1). A chest CT scan revealed pulmonary edema with prominent mediastinal and paratracheal lymphadenopathy. A transthoracic echocardiogram indicated severe left ventricular dysfunction with an approximate ejection fraction of 25%. A cardiac catheterization demonstrated no significant coronary disease. The patient’s dyspnea improved with diuresis, but he developed further changes in his electrocardiogram. Initially
A.E. Darby (*) Department of Cardiology, University of Virginia, 1215 Lee Street, Charlottesville, VA 22908, USA e-mail:
[email protected] J.P. DiMarco Cardiovascular Division, University of Virginia Health System, 1215 Lee Street, Charlottesville, VA 22908, USA e-mail:
[email protected]
he was in sinus rhythm with first degree AV block and left bundle branch block (Fig. 129.1). He was later noted to have type II second degree AV block alternating with periods of complete heart block (Fig. 129.2). What disease process could account for this patient’s presentation of heart failure with advanced AV block?
Case Discussion The differential diagnosis of heart failure with advanced AV block should include infiltrative processes (sarcoidosis or myocarditis), lyme carditis, and certain genetic conditions which may present in the fourth or fifth decades of life (lamin A/C deficiency or alpha-myosin heavy chain gene mutation). This patient’s chest CT showed mediastinal lymphadenopathy which suggested sarcoidosis as the potential etiology. A cardiac MRI revealed patchy areas of delayed hyperenhancement in a non-coronary distribution suggestive of an infiltrative process (Fig. 129.3). He subsequently underwent a lymph node biopsy which revealed noncaseating granulomas diagnostic of sarcoidosis. The patient’s heart failure and advanced AV block were therefore deemed secondary to cardiac sarcoidosis. Sarcoidosis is a chronic, multisystem disorder of unknown etiology. It is characterized by the accumulation of T lymphocytes and macrophages in tissues leading to the formation of noncaseating granulomas which disrupt normal tissue architecture. The disorder may involve any organ system but most commonly affects the lungs, skin, eyes, liver, and lymphatics. Cardiac granulomas are found in nearly 25% of patients with sarcoidosis examined at autopsy. Importantly, it accounts for 13–25% of sarcoidosis-related deaths. Cardiac involvement may precede, follow, or occur concurrently with other organ involvement. The most common cardiac manifestations of sarcoidosis are conduction abnormalities, heart failure and ventricular arrhythmias. Sarcoid granulomas have an affinity for the conduction system. First degree AV block is common due to involvement of the AV node or bundle of His. Interventricular
A. Natale et al. (eds.), Cardiac Electrophysiology, DOI: 10.1007/978-1-84996-390-9_129, © Springer-Verlag London Limited 2011
497
498
A.E. Darby and J.P. DiMarco
Fig. 129.1 Presenting electrocardiogram demonstrating a first-degree AV block and left bundle branch block
Fig. 129.2 An electrocardiogram demonstrating high grade AV block. Since there is now a right bundle branch block pattern and the original ECG showed a left bundle branch block, this may represent either 2:1
AV block with alternating bundle branch aberration or complete AV block with a ventricular escape rate ½ the sinus rate
conduction defects (right or left bundle branch block) may also be apparent on the electrocardiogram. The most common rhythm abnormality among patients with clinicallyevident cardiac sarcoidosis is complete heart block, occurring in up to 30% of patients.
Ventricular dysrhythmias may also occur, and patients with cardiac sarcoidosis are at increased risk for sudden death. Infiltrating granulomas can cause inflammation with subsequent scar formation. This process may create a substrate for reentrant dysrhythmias. Ventricular tachyarrhythmias (VT)
Case 129
are the second most common mode of presentation of cardiac sarcoidosis. Sustained or nonsustained ventricular tachycardia are seen during Holter monitoring in about 23% of patients. Sudden death due to ventricular arrhythmias or complete heart block accounts for 25–65% of deaths due to cardiac sarcoidosis. Atrial arrhythmias are less common, occurring in approximately 19% of patients. Only limited data about the electrophysiologic findings in patients with sarcoid are available. Multiple VT morphologies are common in patients who present with VT. Low voltage areas of scar can be seen in either ventricle. Both epicardial and endocardial sites of VT origin are possible. Heart block and VT may occur separately or together. Heart failure is another common mode of presentation of cardiac sarcoidosis. Granulomatous infiltration and the subsequent inflammatory response may damage the myocardium resulting in systolic dysfunction. The infiltrative process may also cause abnormalities in diastolic function. Ventricular aneurysm formation has been noted to occur, and they may be a focus for ventricular dysrhythmias. Diagnostic criteria for cardiac sarcoidosis have been proposed (Table 129.1). The guidelines in the table do not incorporate updated imaging techniques, but they can serve as a reference point. Cardiac MRI has emerged as an extremely useful diagnostic tool as illustrated in Fig. 129.3. Published reports indicate an approximate 100% sensitivity for detecting myocardial infiltration suggestive of sarcoidosis. Therapy for cardiac sarcoidosis primarily consists of immunosuppression. Corticosteroids, such as prednisone dosed 1 mg/kg/day, are the standard of care. Steroid dosing should be gradually tapered based upon the clinical response. An inadequate response is managed by escalating
Table 129.1 Japanese Ministry of Health and Welfare Guidelines for the Diagnosis of Cardiac Sarcoidosis 1. Histologic Diagnosis Group: endomyocardial biopsy demonstrates epithelioid granulomata without caseating granulomata 2. Clinical Diagnosis Group: in patients with a histological diagnosis of extracardiac sarcoidosis, cardiac sarcoidosis is suspected when (a) and at least one of criteria (b) to (d) is present, and other etiologies such as hypertension and coronary artery disease have been excluded: a. Complete RBBB, LBBB, left axis deviation, AV block, VT, PVC, or pathological Q or ST-T change on resting or ambulatory electrocardiogram b. Abnormal wall motion, regional thinning, or dilatation of the left ventricle c. Perfusion defect by thallium-201 myocardial scintigraphy or abnormal accumulation of gallium-67 or technetium-99m myocardial scintigraphy d. Abnormal intracardiac pressure, low cardiac output, abnormal wall motion, or depressed ejection fraction of the left ventricle e. Interstitial fibrosis or cellular infiltration over moderate grade even if the findings are non-specific
499
Fig. 129.3 Cardiac magnetic resonance scan showing diffuse
immunosuppression with the addition of antimalarials, methotrexate, or azathioprine. Patients with left ventricular dysfunction should be treated with standard medications including ACE inhibitors, beta-blockers, and diuretics if needed. Patients with cardiac sarcoidosis often develop indications for permanent cardiac pacing. Strong consideration should be given to ICD placement given the high risk of ventricular arrhythmias and sudden cardiac death. Standard guidelines for primary and secondary prevention apply. ICD placement has been suggested for primary prevention, regardless of left ventricular function, for patients with frequent ventricular ectopy or nonsustained ventricular tachycardia. It is likely prudent to avoid amiodarone secondary to the potential confounding effects of pulmonary toxicity in patients with sarcoidosis. Our patient was started on 70 mg of prednisone daily. He underwent dual-chamber pacemaker implantation and was started on an ACE inhibitor and beta-blocker. The patient declined an ICD as implantation would preclude continuing his job as a commercial truck driver.
Bibliography Banba K, Kusano KF, Nakamura K, et al. Relationship between arrhythmogenesis and disease activity in cardiac sarcoidosis. Heart Rhythm. 2007;4:1292-1299. Dubrey SW, Bell A, Mittal T. Sarcoid heart disease. Postgrad Med J. 2007;83:618-623. Furushima H, Cinushi M, Sugiura H, Kasai H, Washizuka T, Aisawa Y. Ventricular tachyarrhythmia associated with cardiac sarcoidosis: its mechanisms and outcome. Clin Cardiol. 2004;27:217-222. Hiraga H, Yuwai K, Hiroe M. et al. Guideline for the diagnosis of cardiac sarcoidosis study report on diffuse pulmonary diseases. The Japanese Ministry of Health and Welfare 1993; 23–24.
500 Iannuzzi MC, Rybicki BA, Teirstein AS. Medical progress: sarcoidosis. N Engl J Med. 2007;357:2153-2165. Koplan BA, Soejima K, Baughman K, Epstein LM, Stevenson WG. Refractory ventricular tachycardia secondary to cardiac sarcoid: electrophysiologic characteristics, mapping and ablation. Heart Rhythm. 2006;3:924-929.
A.E. Darby and J.P. DiMarco Smedema JP, Snoep G, van Kroonenburgh MP, et al. Evaluation of the accuracy of gadolinium-enhanced cardiovascular magnetic resonance in the diagnosis of cardiac sarcoidosis. J Am Coll Cardiol. 2005;45:1683-1690. Syed J, Myers R. Sarcoid heart disease. Can J Cardiol. 2004;20: 89-93.
Case 130 Thomas J. Sawyer, Burr W. Hall, and James P. Daubert
Case Summary A 13-year-old boy is being evaluated for syncope. At the time of the syncopal event, he was a national, junior tennis champion, but had recently complained of increased fatigue and shortness of breath while playing tennis. One month
previously, his brother collapsed and died suddenly while running in his driveway at home. The patient’s paternal grandfather had also died suddenly just days before his brother. The 13-year-old boy’s ECG is shown below (Fig. 130.1). Representative echocardiographic images are also shown (Figs. 130.2 and 130.3). Of note, there was no LV outflow obstruction noted at rest or with provocative
Fig. 130.1 ECG at presentation
T.J. Sawyer (*) Cardiac Study Center, 1901 South Cedar St., Suite 301, Tacoma, WA 98405, USA e-mail:
[email protected] B.W. Hall Department of Cardiology, University of Rochester Medical Center, 601 Elmwood Avenue, Rochester, NY 14618, USA e-mail:
[email protected] J.P. Daubert Cardiology Division, Duke University Health System, DUMC Box 3174, Duke Hospital 7451H, Durham, NC 27710, USA e-mail:
[email protected] A. Natale et al. (eds.), Cardiac Electrophysiology, DOI: 10.1007/978-1-84996-390-9_130, © Springer-Verlag London Limited 2011
501
502
T.J. Sawyer et al.
at high risk for sudden death primarily as a result of ventricular arrhythmias. Treatment of the symptoms associated with HCM and identification of this high-risk subset of patients is the primary goal of the initial evaluation of these patients.
Findings in HCM
Fig. 130.2 Parasternal long axis image – late systolic frames. Note absence of SAM (systolic anterior motion of the mitral valve)
Classic physical findings include a systolic murmur that increases with maneuvers, which decrease either preload or afterload.2 Obstruction is not present in all patients (even with Valsalva or exercise) and thus a murmur may not be noted. Evidence of LVH is the hallmark of HCM. This is often suspected electrocardiographically and confirmed by 2D echocardiography. The hypertrophy is usually asymmetric with hypertrophy of the septum being greater than that of the free wall, but can be concentric or more predominant in other regions.1,2
Work Up for HCM Patients Once the diagnosis has been made, a detailed family history should be obtained. Special attention is given to a history of sudden death or unexplained syncope. All first degree relatives should undergo echocardiographic screening. Initial evaluation includes a 48 h Holter monitor and an exercise test. All patients should be told to avoid dehydration and strenuous exercise.
Fig. 130.3 Parasternal long axis image – early systolic frames. Note absence of SAM (systolic anterior motion of the mitral valve)
aneuvers What is your diagnosis? What is an appropriate m management strategy?
Case Discussion Hypertrophic cardiomyopathy (HCM) affects approximately 1 in 500 people (130.2). Thirty to fifty percent of patients with HCM have associated dynamic left ventricular outflow tract (LVOT) obstruction with even a higher percent having obstruction with exercise. Hemodynamically based symptoms consist primarily of shortness of breath and decreased exercise tolerance. Overall mortality in HCM is <1% per year.1,2 However, there is clearly a subset of patients
Risk Stratification in HCM Obviously, the most feared complication of HCM is sudden cardiac death. Despite intense work in this area, the complexity of the disease continues to make the identification of high risk markers for sudden death problematic. A prior cardiac arrest or sustained VT denotes very high risk for sudden death and clearly warrants an ICD. Stratification of risk is usually focused on the younger patient as the attainment of advanced age usually indicates a more benign form of the disease. Most experts agree that the factors listed below suggest particularly high risk for the patient without a prior arrest (primary prevention of sudden cardiac arrest)1: • Family history of sudden death • Syncope unlikely to be neurocardiogenic in etiology • Left ventricular wall thickness >30 mm
Case 130
• Abnormal blood pressure response to exercise (hypotension or failure to increase BP) • Non-sustained ventricular tachycardia Other less potent or agreed upon risk factors include LVOT obstruction, specific genotype, atrial fibrillation, and ischemia.
Treatment In the majority of patients, an implantable defibrillator is not necessary and therapy is focused on the relief of symptoms. Agents which block the effects of catecholamines and improve diastolic function are the mainstays of therapy. Increased diastolic filling time improves LVOT obstruction in patients with this variety of the disease. Beta-blockers are usually the initial therapeutic choice. Verapamil has also been used effectively and, like beta-blockers, has negative chronotropic and inotropic effects without significant alterations in afterload.1 If patients remain symptomatic despite optimal pharmacologic therapy, then several invasive options may be considered. Surgical septal myomectomy is the gold standard for the treatment of symptomatic, obstructive cardiomyopathy refractory to medical therapy. Successful operations can result in complete resolution of both mitral regurgitation and outflow gradient. Excellent long-term follow-up has been achieved with sustained improvement in exercise capacity and symptoms. Major complications typically occur in <3% and include complete heart block, aortic insufficiency, septal defects, and death.1,2 Alcohol induced ablation is performed when 100% alcohol is selectively perfused into a septal perforator supplying the proximal septum. This induces a controlled myocardial infarction ultimately resulting in thinning of the basal septal region. Initial results of this procedure demonstrate improved hemodynamics and exercise tolerance but not to the extent seen with surgery. Heart block remains a major complication seen in 15–20%, although lower in some series. The production of
503
scar in the basal septal region potentially puts this patients at high-risk for subsequent tachyarrhythmic events.1,2
Prevention of Sudden Cardiac Death For patients at high-risk for sudden cardiac death, an implantable defibrillator is the treatment of choice. There are no definitive criteria to unequivocally determine who will benefit from defibrillator implantation. A prior cardiac arrest or sustained VT is a clear indication for an ICD. The presence of two or more risk factors is widely agreed upon to be a good indication for an ICD.5 Recent data from Maron, et al. suggest that any patient with one or more of the five major risk factors listed above should be considered for an ICD, as the chance of experiencing ICD therapy in follow-up was approximately as high for patients with one risk factor as for those with two or more.6
References 1. Nishimura RA, Holmes DR. Hypertrophic Obstructive Cardio myopathy. NEJM. 2004;350:1320-1327. 2. Maron BJ. Hypertrophic cardiomyopathy. A systematic review. JAMA. 2002;287:1308-1320. 3. Elliott PM, Poloniecki J, Dickie S, et al. Sudden death in hypertrophic cardiomyopathy: identification of high risk patients. J Am Coll Cardiol. 2000;36:2212-2218. 4. Maron BJ, Spirito P, Shen W, et al. Implantable cardioverter- defibrillators and prevention of sudden cardiac death in hypertrophic cardiomyopathy. JAMA. 2007;298:405-412.
Bibliography Maron BJ, Roberts WC, McAllister HA, et al. Sudden death in young athletes. Circ. 1980;62:218-229. Maron BJ. Sudden death in young athletes. NEJM. 2003;349:1064-1075.
Case 131 John P. DiMarco
Case Summary A 21-year-old woman presents to the emergency room with a sustained tachycardia (Fig. 131.1). She has a 3 year history of intermittent palpitations but prior episodes have lasted only a few minutes and she has never consulted a physician for this problem. In the emergency department her initial
heart rate is 230 bpm and her blood pressure is 90 systolic. She is alert and oriented but aware of her rapid heart rate. During placement of an intravenous line she converts to sinus rhythm (Fig. 131.2). If she had failed to convert spontaneously, what would have been the most appropriate acute management? What would you recommend for chronic therapy?
Fig. 131.1 Supraventricular tachycardia on presentation to the emergency room
J.P. DiMarco Cardiovascular Division, University of Virginia Health System, 1215 Lee Street, Charlottesville, VA 22908 USA e-mail:
[email protected] A. Natale et al. (eds.), Cardiac Electrophysiology, DOI: 10.1007/978-1-84996-390-9_131, © Springer-Verlag London Limited 2011
505
506
J.P. DiMarco
Fig. 131.2 ECG showing sinus rhythm with preexcitation after conversion. The delta waves polarity suggests a right anterior pathway location
Case Discussion The initial ECG during tachycardia shows a regular narrow complex rhythm with a cycle length of 260 ms. The early ST segment is notched suggesting a short RP’ tachycardia. In this clinical situation, AV reentrant tachycardia (AVRT) is the most likely diagnosis and termination should be possible if transient AV nodal block can be achieved. Vagal maneuvers (e.g., carotid massage, breath holding, Valsalva) may be effective, particularly in the first few minutes of an episode. More sustained episodes typically require intravenous drug therapy. Adenozine, verapamil and diltiazem are the drugs of choice and all three agents are highly effective in AV nodal dependent arrhythmias. Most authors would prefer adenozine in a patient with moderately severe hypotension but, in most cases, the hypotension will resolve if the SVT is terminated by any agent. Although adenozine may transiently shorten the refractory period of accessory
pathways, this is not a contraindication to its use in orthodromic AVRT. After conversion, the ECG shows preexcitation. The delta wave polarity is consistent with an anterior or anteroseptal pathway. Although the anterograde ERP of the pathway, a major determinant of risk for sudden death, is unknown, we do know the pathway’s retrograde properties can support a rapid SVT. Catheter ablation of the accessory pathway therefore would represent optimal therapy if it can be safely performed. In this patient, the probable location of the accessory pathway raises the possibility of inadvertent damage to the normal conduction system during mapping and ablation. In this situation, many electrophysiologists would prefer to use a cryoablation technique rather than use radiofrequency since the initial response with the former is reversible. In this patient, electrophysiologic study confirmed a site adjacent to the His bundle and a successful cryoablation was carried out (Figs. 131.3 and 131.4).
Case 131 Fig. 131.3 Mapping of the accessory pathway location. The shortest AV interval was located just adjacent to the His bundle recording site
507 0
ABLATION SITE
1000
2000
I DELTA II aVF V1 A
HRA
A V
V
ABLATE−D
A V
HBED
RVA
0 Speed: 100 mm/sec Time: 00: 18: 31
1000
2000
Study #1 Protocol #7 Protocol Name: AAP
0
University of Virqinie Medical Center
1000
2000
− 30 Degrees Centigrade I II aVF V1 DCS
Fig. 131.4 Cryomapping of an anteroseptal accessory pathway. The cryo ablation catheter was used to map the shortest AV interval and then loss of preexcitation was seen when −30 ° C was reached. Completion of the lesion was then perfomed at −70 ° C for 4 min with continued monitoring of AV conduction
ABLATE−D
A
A
H
HBED RVA
0 Speed: 100 mm/sec Time: 00: 13: 37
1000 Study #1 Protocol #12 Protocol Name: Mapping
2000 University of Virqinie Medical Center
Case 132 John P. DiMarco
Case Summary
Case Discussion
A 17-year-old man presents to the emergency room complaining of 2 h of rapid palpitations and chest pressure. His ECG is shown in Fig. 132.1. He’s had several similar episodes in the past but those episodes had terminted spontaneously and he had not sought medical attention. His blood pressure during tachycardia is 100/70. He is given 12 mg of intravenous adenosine. His tachycardia converts to a different rhythm as shown in Fig. 132.2. What is the second rhythm and was it related to the adenosine?
The second tracing shows an irregular rhythm with both narrow and wide (preexcited) beats and is most consistent with atrial fibrillation with intermittent preexcitation. Although adenosine is a primary option for terminating PSVT, it has several electrophysiologic effects that may lead to proarrhythmia. Adenosine stimulates an outward potassium current in atrial cells that shortens action potential duration and this facilitates atrial fibrillation induction by spontaneous ectopy. Adenosine also transiently shortens the effective refractory periods of most accessory pathways and may
Fig. 132.1 Narrow complex tachycardia on presentation to the emergency room
J.P. DiMarco Cardiovascular Division, University of Virginia Health System, 1215 Lee Street, Charlottesville, VA 22908 USA e-mail:
[email protected] A. Natale et al. (eds.), Cardiac Electrophysiology, DOI: 10.1007/978-1-84996-390-9_132, © Springer-Verlag London Limited 2011
509
510
J.P. DiMarco
Fig. 132.2 Irregular tachycardia recorded after adenosine administration (simultaneous 12 lead ECG reformatted)
0
1000
2000
3000
4000
ADENOSINE INDUCED ATRIAL FIBRILLATION I aVF V1 HRA HIS−D DCS
Fig. 132.3 Effect of adenosine at EP Study. Adenosine had been administered during an episode of AV reentrant tachycardia. The tachycardia terminated just before this recording. After the fourth beat, a spontaneous atrial premature beat initiated atrial fibrillation which is conducted over both the AV node and the accessory pathway
MCS2 PCS RVA +140 Arterial 0 0
accelerate ventricular rates in preexcited atrial arrhythmias. For these reasons, adenosine should only be used with continuous ECG monitoring with facilities for resuscitation available. This patient also received an initial adenosine dose of 12 mg. Since many patients will respond to the lower recommended initial dose (i.e., 6 mg), the lower dose is preferred even if more patients will require two doses to terminate SVT. In this patient, the preexcited atrial fibrillation spontaneously converted to sinus rhythm after about 45 s and no intervention was required. During his electrophysiologic study, a 30 s episode of atrial fibrillation was again observed when adenosine was readministered (Fig. 132.3).
1000
2000
3000
4000
Bibliography Exner DV, Muzyka T, Gillis AM. Proarrhythmia in patients with the Wolff-Parkinson-White syndrome after standard doses of intravenous adenosine. Ann Intern Med. 1995;122:351-352. Gupta AK, Shah CP, Maheshwari A, Thakur RK, Hayes OW, Lokhandwala YY. Adenosine induced ventricular fibrillation in Wolff-Parkinson-White syndrome. Pacing Clin Electrophysiol. 2002;25:477-480.
Case 133 John P. DiMarco
Case Summary A 57-year-old man presented to his local cardiologist with a complaint of intermittent palpitations. He had a history of mild hypertension for which he was being treated with lisonopril. He had no symptoms of angina and an exercise test 1 year ago had been negative for signs of ischemia. He had a chronic left anterior fascicular block. An event recorder was scheduled and the patient transmitted several episodes of paroxysmal atrial fibrillation. An echocardiogram showed mild left atrial enlargement but was otherwise normal. Routine laboratory studies were normal. Anticoagulation was discussed but in view of a CHADS2 score of 1, he was started on aspirin. The cardiologist started him on flecainide 100 mg
twice daily but did not add any other medications because the patient was relatively bradycardic at rest. Three days later the patient became acutely short of breath and lightheaded. He called the rescue squad and was taken to the emergency room. His ECG on presentation is shown in Fig. 133.1. What is the mechanism of this arrhythmia? How might this have been prevented?
Case Discussion The tracing shows a regular wide complex tachycardia with a rate of 200 bpm. The QRS duration is about 140 ms. AV dissociation is not identifiable but there is extreme left axis
Fig. 133.1 Wide complex tachycardia on presentation to the emergency room
J.P. DiMarco Cardiovascular Division, University of Virginia Health System, 1215 Lee Street, Charlottesville, VA 22908, USA e-mail:
[email protected] A. Natale et al. (eds.), Cardiac Electrophysiology, DOI: 10.1007/978-1-84996-390-9_133, © Springer-Verlag London Limited 2011
511
512
J.P. DiMarco
Fig. 133.2 Tachycardia 2 h later after administration of metoprolol
deviation. The physicians in the emergency room took the history that the patient had just recently started flecainide for paroxysmal atrial fibrillation and suspected that this may be a proarrhythmic effect of the flecainide. Atrial proarrhythmia with flecainide is relatively uncommon but this setting is a typical situation in which it might be expected. Often patients who previously had atrial fibrillation will have organization of their arrhythmia to atrial flutter during therapy with either flecainide or propafenone. The flutter cycle length is often longer than normal because of the effects of the drug. In addition, with the rapid ventricular rate, bundle branch block is often observed. If the patient is exercising or is not also on an AV nodal blocking agent, 1:1 conduction with ventricular rates of 200 bpm or more may occur. Since Ic antiarrhythmic agents, like flecainide and propafenone show “use dependence”, their sodium channel blocking effects become more pronounced at high rates also setting the stage for ventricular proarrhythmia. This patient received intravenous metoprolol and a tracing obtained later that day, before he underwent elective cardioversion, is seen in Fig. 133.2. Note that the right bundle branch block pattern has resolved at the lower rate. In most
patients, flutter with 1:1 conduction can be prevented if AV nodal blocking agents are coadministered with either flecainide or propafenone. In some patients, as in the patient in this case, this may be difficult because of resting sinus bradycardia. When patients present with wide complex regular tachycardia on flecainide for paroxysmal atrial fibrillation, physicians should be aware of the possibility of proarrhythmia. In many cases, a trial of adenosine or another AV nodal blocking agent will help make the ECG diagnosis when the patient presents.
Bibliography Feld GK, Chen PS, Nicod P, Fleck P, Meyer D. Possible atrial proarrhythmic effects of class 1C antiarrhythmic drugs. Am J Cardiol. 1990;66:378-383. Kawabata M, Hirao K, Higuchi K, et al. Clinical and electrophysiological characteristics of patients having atrial flutter with 1:1 atrioventricular conduction. Euro Pace. 2008;10:284-288.
Case 134 John P. DiMarco
Case Summary A 41-year-old woman presents to the emergency room with palpitations and dyspnea. She had had intermittent palpitations for many years. In the past, the spells had lasted only a few minutes and then had terminated spontaneously. She had learned how to break most of the episodes by either bearing down or rubbing on her neck. Today, she had been involved in an argument at work and then developed sustained palpitations that have persisted for the past 3 h. She states that at first the palpitations were more severe but that since she has arrived in the hospital, she feels more comfortable. Her electrocardiogram is shown in Fig. 134.1. What is the most probable mechanism for the arrhythmia? How frequently is this observed?
Case Discussion The initial ECG shows a narrow complex tachycardia at a rate of approximately 110 bpm. A P wave is visible in the middle of the RR interval. The P wave morphology appears to be biphasic in the inferior leads. The differential diagnosis for this rhythm would include an atrial tachycardia, atypical atrial flutter with 2:1 conduction, AV reentry with a relatively long RP¢ interval, and a variant of AV node reentry. The patient was referred for electrophysiologic study. Initiation of tachycardia is shown in Fig. 134.2. An ablation
catheter had been placed in the coronary sinus at this time. The tracings show initiation of a narrow complex tachycardia with two atrial extrastimuli. There is a critical AH delay and the VA time during the tachycardia is less than 70 ms. This tachycardia was initiated on two more occasions and the VA relationship did not change. Ventricular pacing during tachycardia produced a VAV response (not shown). Later in the study, the patient spontaneously developed the rhythm shown in Fig. 134.3. The 12 lead ECG during this rhythm matched the ECG that had been seen in the emergency room. Block below the His spike is seen. The most likely diagnosis for these findings is 2:1 infra-Hisian block during AV node reentrant tachycardia. Spontaneous 2:1 AV block during AV node reentrant tachycardia is relatively uncommon in the emergency room but can be seen in up to 10% of patients at the time of electrophysiologic study. As reported by Mann et al., 2:1 AV block during AVNRT is usually due to functional infranodal block seen because of the rapid rate of the AV nodal reentry. One also must question whether catheter trauma to the conduction sytem or the common use of isoproterenol for induction contributes to the increased frequency during electrophysiologic studies compared to spontaneous presentations. Clinical clues to this diagnosis are the position and morphology of the P wave and the fact that patients often report sudden changes in the rate of the tachycardia as they switch from 1:1 to 2:1 conduction.
J.P. DiMarco Clinical Electrophysiology Lab, Cardiovascular Division, University of Virginia Health System, 1215 Lee Street, Charlottesville, VA 22908, USA e-mail:
[email protected] A. Natale et al. (eds.), Cardiac Electrophysiology, DOI: 10.1007/978-1-84996-390-9_134, © Springer-Verlag London Limited 2011
513
514
J.P. DiMarco
Fig. 134.1 Tachycardia on presentation in the emergency room
I II aVF V1 HRA DCS
ABLATE−D
ABLATE−P PCS HBED
Fig. 134.2 Initiation of typical AV nodal reentrant tachycardia with double atrial extrastimuli. The ablation catheter at this point had been placed in the coronary sinus
RVA
0
1000
2000
Case 134 Fig. 134.3 Spontaneous 2:1 AV block during an episode of tachycardia. Note that there is a 1:1 HA relationship with slow pathway anterograde conduction and fast pathway retrograde conduction, during the period of AV block
515
I II aVF V1 HRA DCS PCS HBED HBEP RVA
0
Bibliography Mann KC, Prinkman K, Bogun F, et al. 2:1 atrioventricular block during atrioventricular node reentrant tachycardia. J Am Coll Cardiol. 1996;28:1770-1774.
1000
2000
Case 135 John P. DiMarco
Case Summary
does well for 3 weeks, then reports several episodes of palpitations that occurred when she was running. Her ECG does not show recurrence of the prior pattern of preexcitation. What do you think is the most likely mechanism for the recurrent tachycardia?
A 23-year-old female, medical student is referred from the emergency room after an episode of supraventricular tachycardia. After conversion in the emergency room, her ECG had shown preexcitation (Fig. 135.1). She undergoes electrophysiologic study and initiation of tachycardia is shown in Figs. 135.2 and 135.3. A left sided accessory pathway is mapped and successfully ablated with a single lesion using a transseptal approach. There is no evidence of accessory pathway conduction at the end of the case and her ECG is now normal. She
Case Discussion The patient clearly initially has Wolff-Parkinson-White syndrome with a delta wave on her surface electrocardiogram consistent with a left-sided accessory pathway. However,
I
aVR
V1
V4
II
aVL
V2
V5
III
aVF
V3
V6
III
Fig. 135.1 Narrow complex tachycardia on presentation to the emergency room. A retrograde P wave is easily seen in the early ST segment in many leads
J.P. DiMarco Cardiovascular Division, University of Virginia Health System, 1215 Lee Street, Charlottesville, VA 22908 USA e-mail:
[email protected] A. Natale et al. (eds.), Cardiac Electrophysiology, DOI: 10.1007/978-1-84996-390-9_135, © Springer-Verlag London Limited 2011
517
518
Fig. 135.2 12 lead ECG after tachycardia termination. The delta wave polarity suggests a left free wall pathway
Fig. 135.3 Initiation of tachycardia during the first EP study
J.P. DiMarco
Case 135
when tachycardia was initiated during her electrophysiologic study, the first three beats have a different retrograde atrial activation sequence. The VA interval for the first three beats is most consistent with AV reentry and then the patient switches to eccentric retrograde activation over the accessory pathway. This phenomena occurred with only two of many SVT initiations during the electrophysiologic study and was not noted by the operators. The coexistence of dual AV nodal pathways and accessory pathways is relatively common in people with WolffParkinson-White syndrome. In a recent report by Sclapfer and Fromer, recuurent tachycardia after ablation was caused by recovery of accessory pathway conduction and by AV node reentry in approximately equal numbers of patients. Palpitations may also be reported after ablation even when monitoring shows only sinus rhythm of atrial or ventricular premature beats. It is often a difficult clinical decision as to whether to proceed to slow pathway ablation if this is an incidental finding during an electrophysiologic study. However, in this case, the fact that three AV nodal echo beats are seen at the beginning of the tachycardia suggests that this
519
is likely to be a future clinical problem and she probably should have undergone mapping and ablation of the slow pathway at her initial study. At a follow-up study, there was no evidence for recovery of conduction over the accessory pathway. Sustained AV nodal reentry could be initiated during low dose isoproterenol. After a slow pathway ablation, she has remained symptom free.
Bibliography Calkins H, Yong P, Miller JM, et al. Catheter ablation of accessory pathways, atrioventricular nodal reentrant tachycardia, and the atrioventricular junction: final results of a prospective, multicenter clinical trial. Atakr Multicenter Investig Group Circ. 1999;99:262-270. Schlapfer J, Fromer M. Late clinical outcome after successful radiofrequency catheter ablation of accessory pathways. Euro Heart J. 2001;22:605-609. Schluter M, Cappato R, Ouyang F, Antz M, Schluter CA, Kuck KH. Clinical recurrences after successful accessory pathway ablation: the role of “dormant” accessory pathways. J Cardiovascr Electro physiol. 1997;8:1366-1372.
Case 136 John P. DiMarco
Case Summary A 35-year-old man has a several year history of recurrent palpitations. He has no history of structural heart disease and a 2-D echocardiogram obtained 2 years ago was normal. Recently, he presented to his local emergency room and was found to be in a wide complex tachycardia (Fig. 136.1). He was electrically cardioverted and referred for evaluation. His ECG at baseline shows sinus rhythm and was felt to be within normal limits.
At electrophysiologic study, a tachycardia is induced with atrial pacing and the response to adenosine is assessed (Fig. 136.2). What is the likely mechanism of the tachycardia?
Case Discussion The initial tracing during tachycardia shows a regular wide complex tachycardia with a left bundle branch block morphology. AV dissociation is not evident. The QRS axis does
Fig. 136.1 Wide comple tachycardia with a left bundle bracnch block configuration on presentation to the emergency room J.P. DiMarco Clinical Electrophysiology Lab, Cardiovascular Division, University of Virginia Health System, 1215 Lee Street, Charlottesville, VA 22908, USA e-mail:
[email protected] A. Natale et al. (eds.), Cardiac Electrophysiology, DOI: 10.1007/978-1-84996-390-9_136, © Springer-Verlag London Limited 2011
521
522 Fig. 136.2 Findings at EP study. The first four beats show the wide complex, preexcited tachycardia with conduction over the atriofascicular pathway. Adenosine, 9 mg, was infused and the tachycardia stops when block during anterograde conduction over the atriofascicular pathway is observed. Note that the next sinus beat is normally conducted but that effects on the AV node are still present and the second sinus beats conducts over the atriofascicular tract
J.P. DiMarco
I
II
aVF
V1
HRA
HIS d
HIS p
RVA
not suggest a right ventricular outflow tract VT. During the EP study, there is a 1:1 VA relationship but the wide QRS complexes are not preceded by a His potential making aberrancy unlikely. Adenosine terminates the tachycardia by producing block during antegrade conduction. These findings, and the normal ECG in sinus rhythm, should suggest that the patient has an atriofascicular fiber, also known as a Mahaim tract. Atriofascicular pathways typically originate in the right atrium and the fiber traverses the tricuspid annulus and inserts into the distal right bundle. Atriofascicular fibers have relatively slow antegrade conduction and do not manifest retrograde conduction. Preexcitation is usually not evident during sinus rhythm but can be brought out by atrial, especially low right atrial, pacing. Due to the pathways distal insertion, the local electrogram at the RV apex precedes the ventricular electrogram on the His catheter during tachycardia. The classic “Mahaim tract” tachycardia has a left bundle branch
morphology, often with a left axis. Unlike most other accessory pathways, conduction over atriofascicular pathways is adenosine sensitive as shown in Fig. 136.2. During mapping the appropriate ablation site can be located by recording a discrete potential near the tricuspid annulus.
Bibliography Ellenbogen KA, Rogers R, Old W. Pharmacological characterization of conduction over a Mahaim fiber: evidence for adenosine sensitive conduction. Pacing Clin Electrophysiol. 1989;12:1396-1404. Prystowsky E, Yee R, Klein GJ. Wolff-Parkinson-White syndrome. In: Zipes DP, Jalife J, eds. Cardiac Electrophysiology: From Bench to Bedside. 4th ed. Philadelphia, PA: Saunders; 2004:868-878. Sternick EB, Cruz FES, Timmermans C, et al. Electrocardiogram during tachycardia in patients with anterograde conduction over a Mahaim fiber: old criteria revisited. Heart Rhythm. 2004;4:406-413.
Case 137 John P. DiMarco
Case Summary A 24-year-old African-American man is referred for evaluation of persistent tachycardia. The patient states that he was told as a child that his heart rate had “always been too high.” At age 17, he was evaluated by a pediatric cardiologist because of a persistent tachycardia detected during a high school physical. At that time, an echocardiogram was reported to be normal and he was cleared for a sports partici pation. Recently, he has developed moderate dyspnea on exertion. On physical examination, he was a muscular young man in no acute distress. His jugular venous pressure was elevated to 6–8 cm above the clavicle but no murmur or gallop was heard. His pulse was irregular with a heart rate of app roximately 115 bpm. His ECG is shown in Fig. 137.1. A twodimensional echocardiogram showed four chamber enlargement and severe left ventricular dysfunction. What is the probable mechanism for the tachycardia?
Case Discussion This case represents an incessant, probably lifelong, tachycardia presenting with congestive heart failure in a young individual. The ECG shows a narrow complex tachycardia with dissociated P waves. When the P wave is able to conduct, fusion beats are seen. A narrow complex tachycardia with VA block is a very rare phenomenon. The differential diagnosis includes junctional tachycardia with VA block, reentrant tachycardia using
a nodofascicular pathway, and AV node reentry with proximal common pathway retrograde block. The patient was started on carvedilol and his average heart rate over 24 h decreased to about 110 bpm. His symptoms of heart failure improved slightly, but after 4 weeks of therapy, his left ventricular dysfunction was still severely depressed. The patient was taken to the electrophysiology (EP) laboratory. In the EP lab, he was in the same tachycardia in an incessant fashion. Initiation of the tachycardia could not be tested because of the tachycardia’s continuous nature. Adenosine transiently slowed but did not terminate the tachycardia. No VA conduction was present. A His spike preceded each QRS complex (Fig. 137.2). His synchronous PVCs had no effect on the tachycardia. No evidence for preexcitation was seen with atrial stimulation. These findings are felt to be most consistent with a nonparoxysmal junctional tachycardia. Nonparoxysmal junctional tachycardia is a relatively uncommon arrhythmia. It is very rare for it to present in an incessant fashion and to manifest VA block as it did in this patient. In one series, only two of eleven patients had VA block during junctional tachycardia. In this patient, it was probably the cause of a tachycardia-related cardiomyopathy. Therapy for junctional tachycardia can include drug therapy and/or selective catheter ablation. The latter carries some risk of producing AV block. Mapping of the AV junctional area was performed using a cryoablation catheter. Cryoablation lesions were placed in the region of the His bundle with careful monitoring of AV conduction (Fig. 137.3). Eventually, the junctional tachycardia was eliminated and the patient remained in sinus rhythm (Fig. 137.4). Three months later his ejection fraction had returned to normal.
J.P. DiMarco (*) Clinical Electrophysiology Lab, Cardiovascular Division, University of Virginia Health System, 1215 Lee Street, Charlottesville, VA 22908, USA e-mail:
[email protected] A. Natale et al. (eds.), Cardiac Electrophysiology, DOI: 10.1007/978-1-84996-390-9_137, © Springer-Verlag London Limited 2011
523
524
J.P. DiMarco
Fig. 137.1 ECG at the time of referral. The patient was taking carvedilol. 12.5 mg bid, at this time
I
II
aVF
V1
HRA
HIS d
Fig. 137.2 Intracardiac recordings made during the patient’s EP study. Note the His spike before QRS and the AV dissociation except for fusion beats
HIS p
RVA
Case 137 Fig. 137.3 Cryoablation of junctional tachycardia. As the temperature on the ablation catheter falls to −30°C, the junctional tachycardia slows and the patient is in sinus rhythm. The lesion was then continued at a lower temperature for 4 min
525
I
II
aVF
V1
HRA
HIS d
ABL−D
RVA
I
aVR
V1
V4
II
aVL
V2
V5
III
aVF
V3
V6
Fig. 137.4 Final ECG at the end of the procedure
Bibliography Hamdan MH, Kalman JM, Lesh MD, et al. Narrow complex tachycardia with VA block: diagnostic and therapeutic implications. Pacing Clin Electrophysiol. 1998;21:1196-1206. Hamdan MH, Page RL, Scheinman MM. Diagnostic approach to narrow complex tachycardia with VA block. Pacing Clin Electrophysiol. 1997;20:2984-2988.
Scheinman MM, Gonzalez RP, Cooper MW, Lesh MD, Lee RJ, Epstein LM. Clinical and electrophysiologic features and role of catheter ablation techniques in adult patients with automatic atrioventricular junctional tachycardia. Am J Cardiol. 1994;74:565-572.
Case 138 John P. DiMarco
Case Summary A 79-year-old man is referred because of recent onset atrial fibrillation. He has a history of hypertension and had two bare metal stents placed in his right coronary artery after presenting with an acute coronary syndrome 3 years ago. Subsequently, he did well on a medical regimen of enalapril, hyrochlorthiazide, metoprolol, atorvastatin and aspirin. He had a brief episode of atrial fibrillation 6 months ago that was associated with a urinary tract infection. He also describes several additional self-terminating episodes of minor palpitations, which were not documented electrocardiographically, in the past several months. Last week he presented to his local physician for a routine office visit and the electrocardiogram shown in Fig. 138.1 was obtained. He was only aware of some minor irregularities in his heart rate and had noted a recent slight decrease in his exercise tolerance. These symptoms had not been severe enough for him to seek medical attention. A two-dimensional echocardiogram showed a preserved left ventricular ejection fraction with mild left ventricular hypertrophy and moderate left atrial enlargement. What would you recommend now?
Case Discussion This patient gives a typical history for an elderly patient who presents with atrial fibrillation. He has a history of hypertension, one of the most common risk factors for atrial fibrillation. He has had several episodes of palpitations and atrial fibrillation suggesting that this will be a recurrent problem implying that just a cardioversion without drug therapy would not be helpful. The options for therapy that should be considered include
J.P. DiMarco Clinical Electrophysiology Lab, Cardiovascular Division, University of Virginia Health System, 1215 Lee Street, Charlottesville, VA 22908, USA e-mail:
[email protected]
appropriate anticoagulation for stroke prevention and whether he wishes to pursue a rate control a rhythm control strategy. The patient’s age and his history of hypertension give him a CHADS2 score of two indicating that he should be placed on warfarin therapy unless he has contraindications. No contraindications to long-term anticoagulation were present and he was begun on warfarin. The decision on rate control versus rhythm control is somewhat more difficult. Due to his history of ischemic heart disease, he would not be a good candidate for a class 1C antiarrhythmic drug. Sotalol or dofetilide could be used but he does have mild LVH on his echo and LVH is a potential risk factor for druginduced torsades de pointes. Amiodarone or dronedarone would therefore be the likely choice for drug therapy and they are likely to be help maintain sinus rhythm in this situation. Catheter ablation would also be an option but the patient did not want to undergo any invasive procedures unless absolutely necessary. An important point is that the patient had only minimal symptoms at his initial presentation even though his rate wasn’t optimally controlled. Therefore, a rate control strategy is also an option. It would involve only relatively simple drugs and would have a high probability of keeping him symptom free. There have been seven published or reported randomized clinical trials that have compared rate control and rhythm control strategies in patients with persistent and/or paroxysmal atrial fibrillation. The largest trial, AFFIRM, showed that there was no difference in overall survival between the two strategies with a slight trend in favor of rate control. The other studies noted that sinus rhythm may be difficult to maintain and that a rhythm control strategy was associated with more therapeutic complications and more hospitalizations. Even in patients with congestive heart failure, data from the Atrial FibrillationCongestive Heart Failure Study (AF-CHF) show that there was no improvement in survival with a rhythm control strategy. This patient elected to follow a strategy of rate control and anticoagulation. He did well on warfarin with no bleeding complications. His metoprolol dose was increased and he reported that his exercise tolerance was back to his prior baseline. Several office visits showed resting heart rates of 60–80 beats per minute. A follow-up electrocardiogram is shown in Fig. 138.2. During monitored exercise at his rehab facility, his
A. Natale et al. (eds.), Cardiac Electrophysiology, DOI: 10.1007/978-1-84996-390-9_138, © Springer-Verlag London Limited 2011
527
528
J.P. DiMarco
Fig. 138.1 ECG at the time of his initial presentation
Fig. 138.2 Follow-up ECG obtained during treatment with metoprolol, 50 mg twice daily
peak heart rate during a symptom limited exercise test was 130 bpm. He completed stage 3 of a Bruce protocol. He has continued to be asymptomatic over several years follow-up.
Bibliography Camm AJ, Kirchof P, Lip GYK, et al. Guidelines for the management of atrial fibrillation. The Task for the Management of Atrial Fibrillation of the European Society of Cardiology. Euro Heart J 2010;31:2369-2429.
Crijns HJ. Rate versus control in patients with atrial fibrillation: what the trials really say. Drugs. 2005;65:1651-1667. Fuster V, Ryden LE, Cannom DS, et al. ACC/AHA/ESC 2006 guidelines for the management of patients with atrial fibrillation: a report of the American College of Cardiology/American Heart Association Task Force on practice guidelines and the European Society of Cardiology Committee for Practice Guidelines (Writing Committee to Revise the 2001 guidelines for the management of patients with atrial fibrillation) developed in collaboration with the European Heart Rhythm Association and the Heart Rhythm Society. Europace. 2006;8:651-745. Wyse DG, Waldo AL, DiMarco JP, et al. A comparison of rate control and rhythm control in patients with atrial fibrillation. N Engl J Med. 2002;347:1825-1833.
Case 139 John P. DiMarco
Case Summary An 85-year-old woman with a history of hypertension had presented with atrial fibrillation with a rapid ventricular response 6 months ago. She had been treated with amiodarone (600 mg daily for 7 days, then 200 mg daily) after an elective cardioversion. Her other medications included: lisinopril 20 mg daily, hydrochlorothiazide 12.5 mg daily, warfarin 2 mg daily, and metoprolol 25 mg twice daily. After her cardioversion, she had no symptomatic recurrences of atrial fibrillation and had returned to her baseline functional status. Today, she returns for a routine scheduled office visit. She reports 3 weeks of a nonproductive cough that was not associated with fever. She has also noted progressive dyspnea on exertion and had difficulty walking from the parking garage for her office visit. Her primary care physician had first stopped her lisonopril and then prescribed a 7 day course of azithromycin for a presumed bronchitis 10 days ago. She completed this without improvement in her symptoms. You order a chest x-ray which is shown in Fig. 139.1. What is the most likely diagnosis? What should you do now?
Case Discussion In this patient, amiodarone pulmonary toxicity must be considered early in the differential diagnosis. In surveys of patients treated with amiodarone for ventricular arrhythmias, often at doses of 400–600 mg daily, the incidence of amiodarone induced pulmonary toxicity was as high 5–10%. More recently, lower doses of amiodarone have become standard and the incidence of pulmonary toxicity has been
J.P. DiMarco Clinical Electrophysiology Lab, Cardiovascular Division, University of Virginia Health System, 1215 Lee Street, Charlottesville, VA 22908, USA e-mail:
[email protected]
Fig. 139.1 Chest x-ray at presentation
lower. Based on data from randomized trials, the current estimate for the incidence of pulmonary toxicity on a dose of 200–300 mg per day is about 2%. Truly long-term data about incidence of pulmonary toxicity over years of therapy with amiodarone are not available. Amiodarone pulmonary toxicity is thought to be multifactorial in its etiology. Amiodarone can damage lung tissue indirectly via immunologic reactions or directly via a cytotoxic process. Amiodarone induces CD8-positive cytotoxic T cells and can result in the production of oxygen free radicals. Phospholipid accumulation may occur in lung tissue and this may have an additional direct cytotoxic effect. A number of different clinical presentations of amiodarone toxicity have been described. Acute or subacute toxicity with respiratory failure and an adult respiratory distress syndrome (ARDS) has mostly been reported in patients with critical illnesses in intensive care units or after cardiothoracic surgery. In many of the reported cases, the relationship to amiodarone has been only circumstantial but there does seem to be real risk of acute toxicity. The more common
A. Natale et al. (eds.), Cardiac Electrophysiology, DOI: 10.1007/978-1-84996-390-9_139, © Springer-Verlag London Limited 2011
529
530
clinical presentation of amiodarone induced lung toxicity is similar to that shown here in this case. Patients will often present with the insidious onset of dyspnea and a chronic, nonproductive cough. The chest x-ray characteristically shows a diffuse interstitial process. Pulmonary function tests will show restrictive disease with a marked decrease in carbon monoxide diffusion capacity (DLCO). The diagnosis of amiodarone pulmonary toxicity is often one of exclusion. The chest x-ray picture can be confused with infection, pulmonary edema, the adult respiratory distress syndrome (ARDS), interstitial or chronic eosinophilic pneumonia, disseminated malignancy or bronchiolitis obliterans organizing pneumonia unrelated to amiodarone. In amiodarone toxicity, the onset is usually insidious and the patient’s symptoms may be much less severe than would be expected based on the abnormalities on chest x-ray, pulmonary function tests and arterial blood gasses. Chest computed tomography will show a diffuse ground glass appearance or reticular abnormalities. If amiodarone pulmonary toxicity is suspected, the wisest course of action is to immediately discontinue therapy. If amiodarone is continued, irreversible respiratory failure may occur. There is substantial anecdotal experience showing favorable responses to corticosteroid therapy but there are no
J.P. DiMarco
randomized clinical trials to support this. However, in the patient with severe toxicity or an ARDS-like pattern, a trial of steroids should be considered. The usual regimen would start with 40–60 mg of prednisone daily with a gradual decrease over several months. Long-term steroid therapy should not be required.
Bibliography Goldschlager N, Epstein AE, Naccarelli GV, et al. A practical guide for clinicians who teat patients with amiodarone: 2007. Heart Rhythm. 2007;4:1250-1259. Jessurun GAJ, Boersma WG, Crijns HJGM. Amiodarone-induced pulmonary toxicity: predisposing factors, clinical symptoms and treatment. Drug Saf. 1998;18:339-344. Olshansky B, Sami M, Rubin A, et al. Use of amiodarone for atrial fibrillation in patients with preexisting disease in the AFFIRM study. Am J Cardiol. 2005;95:404-405. Ott MC, Khoor A, Leventhal JP, Paterick TE, Burger CD. Pulmonary toxicity in patients receiving low-dose amiodarone. Chest. 2003; 123:646-651. Singh SN, Fisher SG, Deedwania PC, Rohatagi P, Singh BN, Fletcher RD. Pulmonary effect of amiodarone in patients with heart failure. J Am Coll Cardiol. 1997;30:514-517.
Case 140 John P. DiMarco
Case Summary A 43-year-old man was referred for electrophysiologic study because of a history of recurrent supraventricular tachycardia. He had experienced recurrent episodes of SVT since age 17. Several episodes had been terminated during emergency room visits with adenosine. His baseline ECG did not show preexcitation. Two examples of his tachycardia are shown in Fig. 140.1, panels A and B. During the electrophysiologic study, sustained SVT was initiated with a single atrial extrastimulus (Fig. 140.2). The earliest retrograde activation was seen on the proximal His bundle recording electrodes. Presence of a concealed septal accessory pathway was confirmed by advancement of the A with a His synchronous PVC (not shown). How would you proceed?
Case Discussion This patient has a concealed retrograde septal accessory pathway and highly symptomatic SVT. Concealed septal pathways are often difficult to treat safely with ablation since there is significant risk of damaging the AV conduction system. In particular, if one monitors the ablation during continuous ventricular pacing, one might affect antegrade conduction over the normal AV node His bundle with the
ablation lesion and not notice the effects because of the ventricular pacing. In this patient, we proceeded with mapping during ventricular pacing using a cryoablation catheter. The ablation site is seen in Fig. 140.3. Once a change in retrograde activation had been seen, we stopped ventricular pacing and monitored AV conduction. First degree AV block was seen but no higher grades of AV block. After completion of the ablation lesion, we repeated para-Hisian pacing (Fig. 140.4). Before ablation, the stimulus-to-A interval had stayed the same when the His bundle was captured (not shown). After ablation, there is a shortening of the VA time when the His bundle was captured suggesting that retrograde conduction is now over the AV node. Another method to differentiate between AV nodal versus accessory pathway retrograde conduction would pharmacologic administration of verapamil or adenosine. Both these agents should block or delay retrograde AV nodal conduction but should not affect conduction over an accessory pathway. Veparamil, in particular may be very useful during mapping is there is a fusion of conduction of the AV node and an accessory pathway during ventricular pacing. ParaHisian pacing is however, relatively easy to perform and straightforward. A catheter is positioned near the His bundle recording site and then pacing performed, As the stimulus output is increased or as the catheter shifts with respiration the His bundle may be captured and retrograde conduction assessed.
J.P. DiMarco Clinical Electrophysiology Lab, Cardiovascular Division, University of Virginia Health System, 1215 Lee Street, Charlottesville, VA 22908, USA e-mail:
[email protected] A. Natale et al. (eds.), Cardiac Electrophysiology, DOI: 10.1007/978-1-84996-390-9_140, © Springer-Verlag London Limited 2011
531
532 Fig. 140.1 ECGs obtained during two separate episodes of tachycardia. Panel A shows a regular tachycardia with a right bundle branch block pattern. Panel B shows another episode of tachycardia, this time with a left bundle branch block pattern. Retrograde atrial activity with a short RP' interval is seen best in leads V1 and V2
J.P. DiMarco I
aVR
V1
V4
II
aVL
V2
V5
III
aVF
V3
V6
I
aVR
V1
V4
II
aVL
V2
V5
III
aVF
V3
V6
Case 140 Fig. 140.2 Initiation of SVT with programmed atrial stimulation. A single atrial extrastimulus initiates tachycardia. The earliest atrial activation during SVT is recorded on the His bundle electrodes
533 I II aVF V1 HRA CS−D CS 3−4 CS 5−6 CS 7−8 CS 9−10 HIS d HIS p RVA STIM
600
270
I II aVF V1 HRA CS−D CS 3−4 CS 5−6 CS 7−8 CS 9−10 HIS d
Fig. 140.3 Mapping during ventricular pacing. The retrograde atrial electrogram on the ablation catheter is now earlier than seen on all other catheters. The ablation catheter was now immediately adjacent to the His bundle catheter on fluoroscopy
HIS p ABL−D ABL−P RVA STIM
400
400
400
400
534 Fig. 140.4 Para-Hisian pacing after successful ablation. The first and third beats are wider indicatng ventricular capture only while the narrower second, fourth and fifth beats show His bundle capture. The Stimulus-to-A interval is longer in the first and third beats than when the His is captured (narrower beats 2, 4, and 5)
J.P. DiMarco I II aVF V1 HRA CS−D CS 3−4 CS 5−6 CS 7−8 CS 9−10 HIS d HIS p ABL−D ABL−P RVA STIM
500
Bibliograhpy Hirao K, Otomo K, Wang X, et al. Para-hisian pacing: a new method for differentiating retrograde conduction over an accessory AV pathway from conduction over the AV node. Circulation. 1996;94: 1027-1035.
500
500
500
Nakagawa H, Jackman WM. Para-hisian pacing: useful clinical technique to differentiate retrograde conduction between accessory atrioventricular pathways and atrioventricular nodal pathways. Heart Rhythm. 2005;2:667-672.
Case 141 John P. DiMarco
Case Summary A 26-year-old woman is brought to the emergency room complaining of dizzy spells and palpitations. His ECG is shown in Fig. 141.1. Her physical exam was normal except for heart rate. An echocardiogram showed concentric hypertrophy without outflow tract obstruction. No valvular abnormalities were seen. Upon further questioning, she admitted that many of her family members had been told they had Wolff-Parkinson-White syndrome. Her 21-year-old brother’s ECG is shown in Fig. 141.2. He also had a history of palpitations and had been told they were due to paroxysmal atrial fibrillation. Several older members of the family had a history of palpitations as young adults but then required pacemakers later in life. What is the most likely diagnosis for the syndrome seen in this family?
Case Discussion The syndrome of early preexcitation, left ventricular hypertrophy, and late progressive conduction system disease has only recently been recognized as a discrete entity. Patients with the syndrome often have a preexcitation pattern noted on their ECG early in life and may have typical AV reentrant tachycardia as children and young adults. Over time, however, they often go on to develop progressive signs of left ventricular hypertrophy with diastolic dysfunction, sinus
bradycardia and progressive AV block. Skeletal muscle symptoms, also related to glycogen deposition, may occur. Many require pacemakers as young adults. Sudden death has been reported in some individuals but does not appear to be common. The inheritance pattern is autosomal dominant with a high degree of penetrance. The genetic locus for this syndrome was first identified on chromosome 7q34–q36 in 1995. Subsequently, the characteristic mutations have been identified as missense mutations in the gene (PRKAG2) that encodes the gamma-2 regulatory subunit of AMP activated protein kinase. This results in myocardial hypertrophy without myofibrillar disarray and only minimal fibrosis. Histologically, the myocytes can be shown to be enlarged by large cytosolic vacuoles that contain inhomogeneous granular material (glycogen) and also display contractile elements. These glycogen laden myocytes can both disrupt the annulus fibrosis leading to preexcitation as muscular connections between the atria and ventricles are created and also infiltrate the conduction system leading to AV block. A murine model of this syndrome has been developed which exhibits a very similar phenotype. From a clinical standpoint, it is important to recognize this syndrome. Although young patients with a PRKAG2 defect may present with what looks like preexcitation with AVRT, the associated findings of significant ventricular hypertrophy and the positive family history of both preexcitation and late conduction system disease should make one cautious before recommending ablation therapy. Patients who undergo ablation are likely to require permanent pacing, if not immediately, then in the near future.
J.P. DiMarco Clinical Electrophysiology Lab, Cardiovascular Division, University of Virginia Health System, 1215 Lee Street, Charlottesville, VA 22908, USA e-mail:
[email protected] A. Natale et al. (eds.), Cardiac Electrophysiology, DOI: 10.1007/978-1-84996-390-9_141, © Springer-Verlag London Limited 2011
535
536
J.P. DiMarco
Fig. 141.1 ECG on presentation. Note the short PR interval, the sinus bardycardia and the very unusual preexcited QRS morphology
Fig. 141.2 ECG in the patient’s 21 year old brother
Case 141
Bibliography Charron P, Genest M, Richard P, Komajda M, Pochmalicki G. A familial form of conduction defect related to a mutation in the PRKAG2 gene. Europace. 2007;9:597-600. Gollob MS, Green MS, Tang AS, et al. Identification of a gene responsible for familial Wolff-Parkinson-White syndrome. N Engl J Med. 2001;344:1823-1831. Murphy RT, Morgensen J, McGarry K, et al. Adenosine monophosphate-activated protein kinase disease mimicks hypertrophic cardiomyopathy and Wolff-Parkinson-White syndrome: natural history. J Am Coll Cardiol. 2005;45:922-930.
537 Sidhu JS, Rajawat YS, Rami TG, et al. Transgenic mouse model of ventricular preexcitation and atrioventricular reentrant tachycardia induced by an AMP-activated protein kinase loss-of-function mutation responsible for Wolff-Parkinson-White syndrome. Circulation. 2005;111:21-29. Sternick EB, Oliva A, Magalhaes LP, et al. Familial pseudo-WolffParkinson-White syndrome. J Cardiovasc Electrophysiol. 2006; 17:724-732. Wolf CM, Ahmad M, Arad F, et al. Reversibility of PRKAG2 glycogenstorage cardiomyopathy and electrophysiological manifestations. Circulation. 2008;117:144-154.
Case 142 John P. DiMarco
Case Summary
flecainide 50 mg bid and had only very rare spells of atrial fibrillation once her hyperthyroidism had been corrected. Recently, she had undergone back surgery to revise spinal hardware in place to correct scoliosis. After the procedure, she developed episodes of the tachycardia shown in Fig. 142.1. A trial of intravenous adenosine resulted in transient atrial fibrillation with a rapid ventricular rate which later converted spontaneously to sinus rhythm. A second episode of SVT was treated with intravenous verapamil (10 mg) with termination of the tachycardia (Fig. 142.2). She was then referred for electrophysiologic study. The patient underwent electrophysiologic study using standard techniques. Her baseline recording in sinus rhythm is shown in Fig. 142.3.
A 59 year old woman with severe rheumatoid arthritis developed recurrent episodes of a narrow complex tachycardia after an orthopedic surgical procedure. She had a history of recurrent supraventricular tachycardia that began at age 17 years. In 1991, at age 42, she had undergone an electrophysiologic study which showed dual AV nodal pathways and easily inducible typical (slow-fast) AV node reentrant tachycardia. She underwent a fast pathway ablation which resulted in a persistent first degree AV block (PR interval 240–340 ms) that was asymptomatic. Seven years after the first electrophysiologic study, she developed hyperthyroidism and in that setting had episodes of paroxysmal atrial fibrillation. She was treated with
I
aVR
V1
V4
II
aVL
V2
V5
III
aVF
V3
V6
Fig. 142.1 ECG of tachycardia that developed after orthopedic surgery
J.P. DiMarco Cardiovascular Division, University of Virginia Health System, 1215 Lee Street, Charlottesville, VA 22908, USA e-mail:
[email protected] A. Natale et al. (eds.), Cardiac Electrophysiology, DOI: 10.1007/978-1-84996-390-9_142, © Springer-Verlag London Limited 2011
539
540
J.P. DiMarco
Fig. 142.2 Rhythm strip shortly after intravenous verapamil
I
II
aVF
V1
HRA
HIS
d
HIS
p
RVA
Fig. 142.3 Baseline recordings at electrophysiologic study. The PR and AH intervals are prolonged consistent with the patient’s prior fast pathway ablation
At electrophysiologic study, the tachycardia could not be initiated with either rapid atrial pacing or atrial premature stimulation. The tachycardia was reliably initiated with a single ventricular extrastimulus over a wide range of coupling intervals (Fig. 142.4).
The ablation catheter was removed from the coronary sinus and replaced with a 14-pole catheter. Figure 142.5 shows the activation sequence during tachycardia and the response to ventricular pacing at a cycle length 40 ms faster than the tachycardia cycle length.
Case 142
541
I
II
aVF
V1
HRA
CS ABLd
CS ABLp
HIS
d
RVA
Fig. 142.4 Tachycardia initiation. A quadripolar ablation catheter had been positioned in the mid-coronary sinus. A single ventricular extrastimulus initiates tachycardia with a short RP¢ interval. Note that the
atrial electrograms in the coronary sinus recordings precede the right atrial signals
Figure 142.6 shows the response to a ventricular premature stimulus delivered during tachycardia. What do you think is the most likely mechanism of the tachycardia? What structure might you target for ablation?
available, slow pathway ablation became the preferred approach in patients with AVNRT since it carried a much lower risk for the inadvertent production of AV block. Slow pathway ablation was at least as effective as fast pathway ablation and the PR interval after ablation remained normal. Only a few papers have looked at the long-term effects of fast pathway ablation. During the relatively short term follow-up in those series, late progression to, as opposed to early development, of high grade AV block was not described. This patient underwent a fast pathway ablation and then had 17 years without recurrent regular tachycardia, even though she did have some intermittent atrial fibrillation after a bout of hyperthyroidism. It is interesting that she was able to tolerate flecainide and metoprolol for her paroxysmal atrial fibrillation with a very long baseline PR interval without developing higher grade AV block. However, during the stress of a surgical procedure, she developed recurrent tachycardias.
Case Discussion This case raises several interesting issues. Fast pathway ablation was an early technique used for catheter ablation in patients with AV node reentry. The technique used required that the tip of the ablation catheter be near the His bundle recording position. The catheter was then withdrawn slightly. During RF delivery, effects on the fast pathway were assessed by monitoring the PR interval. A 50% prolongation of the PR interval and an inability to reinitiate tachycardia were the usual endpoints for the ablation. After techniques for localization and ablation of the slow pathway became
542
J.P. DiMarco
I II aVF V1 HRA CS 1–2 CS 3–4 CS 5–6 CS 7–8 CS 9–10 CS 11–12 CS 13–14 HIS d HIS p RVA Stim
Fig. 142.5 Ventricular pacing during SVT. The coronary sinus catheter is positioned so that poles 11–12 are at the coronary sinus os
Working out the probable mechanism for the tachycardia requires a number of steps. The ECG in Fig. 142.1 shows a relatively short RP’ interval and is most consistent with either the “slow-slow” form of AV node reentry or an accessory pathway mediated tachycardia. An atrial tachycardia, however, must also be considered. Verapamil administration broke the tachycardia and a careful examination of the ECG strip at the time of termination shows that the last atrial complex conducted to the ventricle (Fig. 142.2). Either an atrial tachycardia or AVNRT with retrograde block would give this response. However, this observation makes it unlikely that the tachycardia used an accessory pathway for retrograde conduction since most accessory pathways, including those with long conduction times, are insensitive to verapamil. The tachycardia could not be initiated with atrial stimulation but ventricular premature beats reliably initiated tachycardia. This would be common with either the “slow-slow” or the
“fast-slow” forms of AV node reentry or an accessory pathway mediated tachycardia but would be quite rare for an atrial tachycardia. Interestingly, the atrial activation sequence demonstrated earliest atrial activity on the coronary sinus electrogram recordings. The response to ventricular pacing in Fig. 142.5 shows what has been called a “pseudo VAAV” response. During ventricular pacing, the VA interval prolongs and the two right atrial deflections after the last paced beat are both entrained beats so this is really a VAV response. It is also noted that the post-pacing interval on the RV electrogram minus the tachycardia cycle length is very long, a finding consistent with retrograde conduction an AV nodal fiber. A premature stimulus delivered when the His is refractory in Fig. 142.6 does not advance the atrial electrograms. This does not completely exclude an accessory pathway unless the stimulus is delivered near the AV groove and there is no decremental conduction in the pathway.
Case 142
543
I
II
aVF
V1
HRA
CS
1–2
CS
3–4
CS
5–6
CS
7–8
CS
9–10
CS
11–12
CS
13–14
HIS d
HIS p
RVA
Fig. 142.6 A single VPD is introduced during tachycardia
Left sided extensions of AV nodal fibers have been reported. Some authors have said this occurs in up to 6–8% of patients with AV node reentry and the phenomenon seems to be more common in patients with the “slowslow” or “fast-slow” variants. In some reports, retrograde atrial activation over what has been called an “AV nodal fiber” has been reported even when the earliest retrograde atrial recording is in the distal coronary sinus. Precise criteria to differentiate between an accessory pathway with decremental conduction properties and unusually positioned AV nodal fibers are difficult to define since the responses to either pharmacologic manipulations or stimulation maneuvers may not be entirely conclusive. In this
case, however, because of the history of previous AV node reentry, the response to verapamil and the long post-pacing interval, it is likely that left sided AV nodal fibers comprised the retrograde limb of the circuit. Since this patient had a history of a prior fast pathway ablation, we were reluctant to try to ablate in the typical slow pathway position in the triangle of Koch, even though other authors have used this approach safely and successfully. We therefore mapped earliest retrograde activation to a site 1.5 cm within the coronary sinus os (Fig. 142.7). Application of RF here terminated the tachycardia and prevented reinitiation. She has continued to be free of recurrent tachycardia during follow-up.
544
J.P. DiMarco
I II aVF V1 HRA CS 1-2 CS 3-4 CS 5-6 ABL d ABL p CS 7-8 CS 9-10 CS 11-12 CS 13-14 HIS d RVA Stim
Fig. 142.7 The ablation catheter tip is about 1.5 cm inside the coronary sinus os. This was the shortest VA time recorded in either atrium or in the coronary sinus
Bibliography Chen J, Anselme F, Smith TW, et al. Standard right atrial ablation is effective for atrioventricular nodal reentry with earliest activation in the coronary sinus. J Cardiovasc Electrophysiol. 2004;15:2-7. Greenberg LBB, Overholt ED M, et al. Differential electrophysiologic properties of decremental retrograde pathways in long RP’ tachycardia. Circulation. 1987;76:21-31. Hwang C, Martin DJ, Goodman JS, et al. Atypical atrioventriculart nodal reciprocating masquerading as tachycardia using a left-sided accessory pathway. J Am Coll Cardiol. 1997;30:218-225. Jais P, Haissaguerre M, Shah DC, et al. Successful radiofrequency ablation of a slow atrioventricular nodal pathway on the left posterior atrial septum. PACE. 1999;22:525-527. Kottkamp H, Hindricks G, Borgreffe M, Breithardt G. Radiofrequency catheter ablation of the anterosuperior and posteroinferior atrial
approaches to the AV node for treatment of AV nodal reentrant tachycardia. J Cardiovasc Electrophysiol. 1997;8:451-468. Lee MA, Morady F, Kadish A, et al. Catheter modification of the AV junction with radiofrequency energy for control of atrioventricular nodal reentry tachycardia. Circulation. 1991;83:827-835. Mehta D, Gomes JA. Long term results of fast pathway ablation in atrioventricular nodal reentry tachycardia using a modified technique. Br Heart J. 1995;74:671-675. Nam G-B, Rhee K-S, Kim J, Choi K-J, Kim Y-H. Left atrionodal comnnections in typical and atypical atrioventricular nodal reentrant tachycardias: activation sequence in the coronary sinus and results of radiofrequency catheter ablation. J Cardiovasc Electrophysiol. 2006;17:171-177. Vijayaraman P, Kok LC, Rhee B, Ellenbogen KA. Unusual variant of atrioventricular nodal reentrant tachycardia. Heart Rhythm. 2005;2: 100-102.
Case 143 John P. DiMarco
Case Summary A 61 year old woman is referred because of repeated episodes of narrow complex tachycardia. She had been well with no cardiac history until 2.5 years before referral. At that time, she began to notice episodes of palpitations. These would occur at various intervals from every few days to every few weeks. Most terminated on their own within an hour or two but several episodes had lasted long enough for
her to reach the emergency room. In the emergency room, she was found to have a narrow complex tachycardia at about 155 bpm (Fig. 143.1). The tachycardia was reliably terminated during each of her emergency room visits with 6 mg of intravenous adenosine. A trial of oral metoprolol 50 mg twice daily was ineffective and she was referred after several repeat emergency room visits. At electrophysiologic study, the tachycardia could be easily initiated with single atrial extrastimuli (Fig. 143.2).
I
aVR
V1
V4
II
aVL
V2
V5
III
aVF
V3
V6
Fig. 143.1 ECG of the patient’s presenting tachycardia in the emergency room
J.P. DiMarco Cardiovascular Division, University of Virginia Health System, 1215 Lee Street, Charlottesville, VA 22908, USA e-mail:
[email protected] A. Natale et al. (eds.), Cardiac Electrophysiology, DOI: 10.1007/978-1-84996-390-9_143, © Springer-Verlag London Limited 2011
545
546
J.P. DiMarco
I II aVF V1 HRA CS-D CS 3-4 CS 5-6 CS 7-8 CS 9-10 ABL-D HIS d HIS p RVA STIM
Fig. 143.2 Initiation of sustained tachycardia with a single atrial extrastimulus (500/320). The tachycardia could be reliably initiated over a wide range of coupling intervals
The response to ventricular overdrive pacing is seen in Fig. 143.3. Another tracing of the tachycardia is shown in Fig. 143.4. What is the mechanism of the tachycardia? What therapy would you recommend now?
Case Discussion This patient’s tachycardia has several interesting characteristics. The baseline electrocardiogram shows a narrow complex tachycardia without obvious P waves visible on the surface ECG. The initiation of the tachycardia shows prolongation of the AH interval and then what appears to be a midline retrograde activation sequence. This could easily be interpreted as AV node reentry. However, in Fig. 143.3, during overdrive ventricular pacing, there is no entrainment of the atrium due to VA block during the tachycardia, and in Fig. 143.4 you can see that there is actually AV dissociation during running tachycardia. These findings are most consistent with an atrial tachycardia arising from the region around the AV node.
These tachycardias, for unknown reasons, occur mostly in middle aged or elderly women. As in this case, they may have an ECG pattern that closely resembles AV node reentry, particularly when the PR interval is close to the cycle length of the tachycardia. They are almost always adenosine sensitive. For these reasons, in many cases, they are incorrectly thought to be AV nodal reentrant tachycardias. The findings of a AV dissociation and lack of VA conduction make atrial tachycardia the likely diagnosis. The atrial mechanism is uncertain. There tachycardias are easy to start with extrastimuli. Although they are sensitive to adenosine, they usually do not require isoproterenol for initiation. This suggests that they may be due to reentry involving portions of AV nodal tissue rather than being caused by triggered activity or abnormal automaticity in the atrium itself. Most can be ablated from the right atrium but there are occasional reports where ablation from the left side of the atrial septum was required. Mapping and ablation of these tachycardias require care. Although we have previously had success with radiofrequency ablation, the potential risk for producing inadvertent AV block has caused us to shift to cryoablation. With this technique, most of these tachycardias can be successfully mapped and ablated (Figs. 143.5 and 143.6).
Case 143
I
II
aVF
V1
HRA
CS-D
CS 3-4
CS 5-6
CS 7-8
HIS d
HIS p
RVA
STIM
Fig. 143.3 Ventricular pacing during an episode of tachycardia
547
548
J.P. DiMarco
I
II
aVF
V1
HRA
CS-D
CS
3-4
CS
5-6
CS
7-8
CS
9-10
ABL-D
HIS d
HIS p
RVA
Fig. 143.4 Additional recordings during tachycardia made later during the case
Case 143
549
Fig. 143.5 Ablation site. A cryoablation catheter was a 4 mm tip was used for mapping and ablation. The local electrogram, on the ablation catheter is seen to precede the atrial signal on the His electrogram by about 25 ms
Bibliography Chen CC, Tai CT, Chiang CE, et al. Atrial tachycardias originating from the atrial septum: Electrophysiologic characteristics and radiofrequency ablation. J Cardiovasc Electrophysiol. 2000;11: 744-749. Iesaka Y, Takahashi A, Goya M, et al. Adenosine-sensitive atrial reentrant tachycardia originating from the atrioventricular nodal transitional area. J Cardiovasc Electrophysiol. 1997;8:854-864. Lai LP, Lin JL, Chen TF, Ko WC, Lien WP. Clinical, electrophysiological characteristics, and radiofrequency catheter ablation of atrial tachycardia near the apex of Koch’s triangle. PACE. 1998;21: 367-374. Marrouche NF, SippensGroenewegen A, Yang Y, Dibs S, Scheinman MM. Clinical and electrophysiologic characteristics of left septal atrial tachycardia. J Am Coll Cardiol. 2002;40:1133-1139.
Fig. 143.6 Ablation site for the septal atrial tachycardia. The 4 mm tipped croablation catheter is positioned immediately next to the His bundle catheter
Case 144 John P. DiMarco
Case Summary A 20-year-old woman is referred for management of refractory heart failure and consideration of heart transplantation. She was first seen at another hospital with a complaint of dyspnea that began about 3 weeks prior to her seeking medical attention. Prior to that she had been completely well and had participated in high school sports. She denied any recent substance abuse. Several relatives do have a history of myocardial infarctions in late middle age. Physical examination at her initial presentation revealed a persistent tachycardia at
rates between 130 and 160 beats per min. Her ECG is shown in Fig. 144.1. She was afebrile. No murmurs were audible. She had bilateral pulmonary crackles. Her jugular venous pressure was elevated. Her chest x-ray showed cardiomegaly and pulmonary congestion. Her laboratory studies were not remarkable. Her rhythm was initially thought to be sinus tachycardia in the setting of decompensated heart failure of unknown cause. She failed to improve after initiation of an ACE inhibitor, a diuretic, and a beta blocker over 10 days at the outside hospital. She gradually became more short of breath and
Fig. 144.1 ECG at the time of initial presentation
J.P. DiMarco Clinical Electrophysiology Lab, Cardiovascular Division, University of Virginia Health System, 1215 Lee Street, Charlottesville, VA, 22908 USA e-mail:
[email protected] A. Natale et al. (eds.), Cardiac Electrophysiology, DOI: 10.1007/978-1-84996-390-9_144, © Springer-Verlag London Limited 2011
551
552
J.P. DiMarco
Fig. 144.2 Change in tachycardia during sleep
developed hypotension. These developments led to her transfer to a referral center. During the next 24 h of monitoring, she was noted to stay in this tachycardia almost continuously. However, on rare occasions during sleep, monitor strips showed that the rhythm could change spontaneously as shown in Fig. 144.2. A 2-dimensional echocardiogram was obtained showing an ejection fraction that was estimated to be 15–20% with four chamber dilatation. A cardiac magnetic resonance scan confirmed the depressed ventricular function noted on her echocardiogram but failed to show any evidence for acute myocarditis. What do you think the mechanism of the tachycardia is? Do you think this is related to the patient’s heart failure?
Case Discussion In young individuals, incessant or nearly incessant tachycardias are an important cause of congestive heart failure. In individuals without associated heart disease, atrial tachycardias and the permanent form of junctional reciprocating tachycardia (PJRT) are the most common arrhythmias identified. In older adults, atrial fibrillation with uncontrolled ventricular rates would be the most common cause for tachycardia-induced heart failure. It is important to recognize this syndrome in young patients who present with heart failure and tachycardia since treatment of the arrhythmia,
usually with catheter ablation for atrial tachycardias and PJRT, may be the only therapy required. At the outside hospital, an initial diagnosis of sinus tachycardia had been made in this patient since the P waves were strongly positive in II, III and aVF. Although she did not report any recent febrile illnesses and was afebrile, the referring physicians believed that she had myocarditis. After transfer, the ECG was looked at more closely since it was felt that the tachycardia could be the cause of her heart failure. It was now noted that the P waves were negative in lead I and aVL (Fig. 144.1), making sinus tachycardia unlikely. The site of origin most consistent with this ECG pattern would be the left atrial appendage. She later underwent electrophysiologic study. Recordings made with an ablation catheter positioned at the base of the left atrial appendage are shown in Fig. 144.3. Ablation at this site terminated the tachycardia and sinus rhythm was restored. After the ablation, the patient’s symptoms gradually improved and she was eventually discharged still on an ACE inhibitor, a beta blocker and a diuretic. Four weeks later, her symptoms had completely resolved and these medications were discontinued. A 3 month follow-up echocardiogram showed an ejection fraction of 60% and she was totally symptom-free. Resolution of a tachycardia-induced cardiomyopathy usually takes at least several weeks. Once the tachycardia has been eliminated or the ventricular rate controlled, the patient should be carefully reevaluated to make sure the tachycardia was the primary cause of the problem.
Case 144
553
I
II
ABL d
ABL−p
CS d
CS 3−4
CS 5−6
CS 7−8
CS p
RVA
Fig. 144.3 Activation map during SVT. The ablation catheter is now positioned at the base of the left atrial appendage. This was the earliest site of activation identified with the local electrogram preceding the P wave onset by 35–40 ms
Bibliography Calo L, De Ruvo E, Sette A, et al. Tachycardia-induced cardiomyo pathy: mechanisms of heart failure and clinical implications. J Cardiovasc Med. 2007;8:138-143. Kato M, Adachi M, Yano A, et al. Radiofrequency catheter ablation for atrial tachycardia originating from the left atrial appendage. J Interv Card Electrophysiol. 2007;19:45-48. Umana E, Solares CA, Alpert MA. Tachycardia-induced cardiomyopathy. Am J Med. 2003;114:51-55.
Wang YL, Li XB, Quan X, et al. Focal atrial tachycardia originating from the left atrial appendage: electrocardiographic and electrophysiologic characterization and long-term outcomes of radiofrequency ablation. J Cardiovasc Electrophysiol. 2007;18:459-464. Yamada T, Murakami Y, Yoshida Y, et al. Electrophysiologic and electrocardiographic characteristics and radiofrequency catheter ablation of focal atrial tachycardia originating from the left atrial appendage. Heart Rhythm. 2007;4:1284-1291.
Case 145 John P. DiMarco
Case Summary An 81-year-old man is brought to the emergency room complaining of palpitations and dyspnea. His cardiac history includes coronary artery bypass surgery performed at age 76 because of exertional angina. He had no prior history of any arrhythmia. Since bypass surgery, he has done extremely well and has been fully active. Last year he had a stress test which showed no inducible ischemia and no fixed perfusion defects. His left ventricular ejection fraction was estimated to be 60%. He is currently taking: aspirin, metoprolol, lisinopril for hypertension, and atorvastatin. Today, while working in his yard, he noted the sudden onset of palpitations and shortness of breath. The rescue squad was called and he was brought to the emergency room. On arrival to the emergency room, his heart rate was 180 bpm. His blood pressure was 80/50. He was complaining of moderately severe chest discomfort. His electrocardiogram is shown in Fig. 145.1. What is your diagnosis for his arrhythmia? What would you do next?
Case Discussion This elderly gentleman presents with a wide complex tachycardia. Several aspects of the clinical history are important. He has a history of coronary artery disease but does not report ever having a myocardial infarction. Recent testing showed normal ventricular function. This is his first episode of symptomatic arrhythmia. The ECG shows a regular wide complex tachycardia at 180 bpm. The QRS duration is about 140 ms. There is a fairly typical left bundle branch block pattern observed. The QRS axis is normal. Atrial activity cannot
be definitely identified. The mechanism of the arrhythmia cannot be definitely diagnosed on the electrocardiogram. However, given the patient’s hypotension and significant chest discomfort, the most appropriate initial therapy would be to terminate the arrhythmia with cardioversion. He was therefore anesthetized and cardioverted electrically. His chest pain immediately resolved. It is important to recall that the diagnosis of the arrhythmia is still uncertain and at this point in time an electrophysiologic study would certainly be appropriate. Given the patient’s history of coronary disease and his age, one might guess that this was either ventricular tachycardia or an atrial tachycardia with left bundle branch block but only an electrophysiologic study in which the clinical tachycardia was reproduced would enable one to make the diagnosis with certainty. He therefore underwent electrophysiologic study. During the study, AV node reentry was induced (Figs. 145.2 and 145.3) on multiple occasions. Most of the episodes had a narrow QRS complex but several showed the left bundle branch block pattern observed clinically. He underwent a slow pathway ablation and has done well since. Although the clinical history of coronary artery disease in this patient could make one suspect either ventricular tachycardia or an atrial tachycardia, AV node reentry can appear for the first time in an elderly patient. In some series, up to 10–15% of patients with AV node reentrant tachycardia will first present with their arrhythmia after age 65. As in younger patients, slow pathway ablation in elderly patients has been safe and highly effective. In this case, the clinical history and the ECG were not sufficient to allow a diagnosis to be made with certainty when the patient presented. The clinical situation determined the need for cardioversion and the diagnosis was finally made during an elective electrophysiologic study.
J.P. DiMarco Clinical Electrophysiology Lab, Cardiovascular Division, University of Virginia Health System, 1215 Lee Street, Charlottesville, VA, 22908 USA e-mail:
[email protected] A. Natale et al. (eds.), Cardiac Electrophysiology, DOI: 10.1007/978-1-84996-390-9_145, © Springer-Verlag London Limited 2011
555
556
J.P. DiMarco I
aVR
V1
V4
II
aVL
V2
V5
III
aVF
V3
V6
V1
Fig. 145.1 ECG and rhythm strip on arrival in the emergency room
Case 145
557
I
II
V1
HRA
HIS D
RVA
Fig. 145.2 Tracings ashowing AV node reentrant tachycardia with a left bundle branch block. Bundle branch block was noted intermittently d uring episodes of tachycardia
558 WIANT, WILLIAM
J.P. DiMarco 1251911
04/09/08
16 : 11 : 00
Baseline Intervals
150mm/sec
0.400 mv
I
II
V1
HRA
HIS D
RVA
Fig. 145.3 AV node reentrant tachycardia with a normal QRS
Bibliography Akhtar M, Shenasa M, Jazayeri M, Caceres J, Tchou PJ. Wide QRS complex tachycardia. Reappraisal of a common clinical problem. Ann Intern Med. 1988;109:905-912. Channamsetty V, Aronow WS, Sorberra C, Butt A, Cohen M. Efficacy of radiofrequency catheter ablation in treatment of elderly patients with supraventricular tachyarrhythmias and ventricular tachycardia. Am J Therapeutics. 2006;13:513-515. Chen SA, Chiang CE, Yang CJ, et al. Accessory pathway and atrioventricular node reentrant tachycardia in elderly patients: clinical
f eatures, electrophysiologic characteristics and results of radio frequency ablation. J Am Coll Cardiol. 1994;23:702-708. Epstein LM, Chiesa N, Wong MN, Lee RJ, Griffin JC, Scheinman MM. Radiofrequency catheter ablation in the treatment of supraventricular tachycardia in the elderly. J Am Coll Cardiol. 1994;23:13561362. Kalusche D, Ott P, Arentz T, et al. AV nodal reentrant tachycardia in elderly patients: clinical presentation and results of radiofrequency catheter ablation therapy. Coron Artery Dis. 1998;9:359-363. Tchou P, Young P, Mahmud R, Denker S, Jazayeri M, Akhtar M. Useful clinical criteria for the diagnosis of ventricular tachycardia. Am J Med. 1988;84:53-56.
Case 146 Brett A. Faulknier, David T. Huang, and James P. Daubert
Case Summary A 19-year-old college swimmer suffered syncope during an intercollegiate race. She described losing consciousness “for two seconds,” and coming to the surface gasping for air and choking. Over the prior month she had also had 2–3 episodes of palpitations, one associated with lightheadedness all while training or competing. During a stress test she exercised 14 min, but during recovery developed a wide complex tachycardia at 300 beats/min lasting 15–20 s without syncope (Fig. 146.1). The past medical history, family history and physical examination were unremarkable. A urine drug screen was negative. An ECG taken in the EP clinic is shown (Fig. 146.2). What is the differential diagnosis and what further evaluation and management should be considered for this patient?
Case Discussion The differential diagnosis of this left bundle type wide QRS tachycardia (Fig. 146.1) includes: (1) supraventricular tachycardia (SVT) with aberrant conduction, (2) a preexcited tachycardia, especially atrial fibrillation with a bypass tract or a Mahaim pathway, (3) idiopathic “normal heart” VT,
B.A. Faulknier (*) Department of Cardiology and Electrophysiology, West Virginia University Physicians of Charleston, 3100 MacCorkle Avenue SE, Suite 700, Charleston, WV 25304, USA e-mail:
[email protected] D.T. Huang Department of Cardiology, University of Rochester, 601 Elmwood Avenue, Box 679, Rochester, NY 14642, USA e-mail:
[email protected] J.P. Daubert Cardiology Division, Duke University Health System, DUMC Box 3174, Duke Hospital 7451H, Durham, NC 27710, USA e-mail:
[email protected]
such as RV outflow tract VT, (4) polymorphic VT due to long QT syndrome (LQTS), (5) catecholaminergic polymorphic VT (CPVT), (6) hypertrophic obstructive cardiomyopathy (HCM) with ventricular tachycardia (VT), (7) an anomalous coronary artery (or coronary stenosis, unlikely in this demographic) resulting in ischemically mediated VT, or (8) myocardial scar-related reentrant VT due to myocardial infarction, dilated cardiomyopathy, an infiltrative process, non-compaction, arrhythmogenic RV cardiomyopathy/dysplasia (ARVC/D) or sarcoidosis. Aberrant SVT is essentially excluded since negative complexes in leads I and avL do not fit with LBBB aberrancy. Similarly, the morphology of the QRS complexes rule out an accessory pathway since negativity in leads I and avL points to a left-lateral pathway in which case leads V1–V3 should show positive delta waves; a Mahaim pathway would show a LBBB appearance but with a normal to leftward axis. Against idiopathic RVOT VT is the abnormal resting ECG, the irregularity (and also extreme rapidity) of the VT; palpitations are common but syncope is not a hallmark of this condition. Normal RV function and structure should be present for this condition to be confidently diagnosed. Regarding LQTS, repolarization is abnormal with inverted T waves in leads V1–V4, but the QT is only borderline prolonged, and VT due to LQTS is polymorphic with a torsade de pointes (“twisting of the points”) appearance. In CPVT, another ion channelopathy, frequent multiform PVC’s are usually seen during exercise (or stress), but were not seen in her stress test, and VT if it occurs is polymorphic, and often bidirectional. The VT observed (Fig. 146.1) was relatively monomorphic although somewhat irregular. A coronary angiogram showed normal coronary origin and anatomy, excluding differential number 6. An LV angiogram showed an LVEF of 50% with mild anterobasal and anterolateral hypokinesis. An RV angiogram was performed (Figs. 146.3 and 146.4) and showed a dilated RV with global hypokinesis and more severe hypo-to-akineis of the RV apex and RVOT. A very focal portion of the RV free wall was mildly dyskinetic. This leaves us in the last category, scar related reentry, and points to ARVC/D or possibly sarcoidosis. An RV endomyocardial biopsy showed moderate to marked interstitial fibrosis (70%), fatty replacement
A. Natale et al. (eds.), Cardiac Electrophysiology, DOI: 10.1007/978-1-84996-390-9_146, © Springer-Verlag London Limited 2011
559
560
B.A. Faulknier et al.
Fig. 146.1 Ventricular tachycardia during exercise stress test
Fig. 146.2 Baseline ECG showing possible epsilon wave (V2–V3), prolongation of the QRS in lead V1 consistent with delayed RV activation, and inversion of T-waves in the right precordial leads
Case 146
(5–10%), moderate myocyte hypertrophy, and minimal chronic inflammation. At electrophysiology study the patient had inducible left bundle morphology monomorphic VT with double extrastimuli at a cycle length of 250 ms (Fig. 146.5). The data was felt to be consistent with ARVC/D. Ultimately, the patient had a successful abdominal, submuscular implantation (rectus sheath pocket) of a dual-chamber ICD. Six months following implant the patient received an appropriate shock for ventricular tachycardia. She had also received two treatments with burst pacing within the prior month. Over the following 3 years the patient has received four appropriate ICD shocks for VT and required occasional
Fig. 146.3 2-D echocardiogram at end-systole
561
burst pacing. Her LV and RV function remain unchanged. She is treated with Sotalol 160 mg twice daily. Arrhyhthmogenic right ventricular cardiomyopathy is characterized macroscopically by a fatty appearance of the RV free wall. The fibrofatty replacement of the RV myocardium initially produces typical regional wall motion abnormalities that later become global, producing RV dilation. The tissue replacement can also involve areas of the left ventricle with relative sparing of the septum.1 Pathologically, two distinct types of ARVD with differing histologic features have been described. The fibrolipomatous (fibrofatty) pattern is characterized by myocardial atrophy and thinning with replacement of myocardium by both fibrous and fatty tissue, as well as patchy inflammatory infiltrates. RV aneurysms and LV involvement are each found in about three-quarters of patients with fibrofatty disease.2 The lipomatous (fatty) pattern is characterized by normal or increased myocardial thickness with exclusively fatty replacement and infrequent inflammation.2 The fatty type is controversial as to whether this is the same as ARVD. Approximately 30–50% of cases (or more) are familial. There are two patterns of inheritance: an autosomal dominant form, which is most common, and an autosomal recessive form called Naxos disease, in which ARVD is part of a syndrome including hyperkeratosis of the palms and soles and woolly hair.3 There are at least five established disease-causing genes that encode desmosomal proteins in the autosomal dominant form of the disease: plakoglobin, desmoplakin,
Fig. 146.4 RV angiogram in LAO projection at end-diastole (left) and end-systole (right)
562
B.A. Faulknier et al.
Fig. 146.5 Monomorphic VT at electrophysiology study, with left bundle branch block type morphology, and cycle length 250 ms
plakophilin-2, desmoglein, and desmocollin. Plakoglobin mutations have been found in Naxos disease. Sen-Chowdhry et al. have proposed that impaired desmosome function when subjected to mechanical stress causes myocyte detachment and cell death. The myocardial injury may be accompanied by inflammation as the initial phase of the repair process, which ultimately results in fibrofatty replacement of damaged myocytes.4 The prevalence of ARVD in the general population is estimated to be as high as 1:1,000 and is an important cause of sudden cardiac death.5,6 Presentation is most common between the ages of 10 and 50, with a mean age of diagnosis of approximately 30 years.7 The most common symptoms are palpitations, syncope, atypical chest pain, and dyspnea.7 Approximately 50% of patients present with symptomatic ventricular arrhythmias, ranging from PVCs to sustained tachycardia.8 The rate of sudden cardiac death reported varies from approximately 4–10%.9,10 There is an associated risk of VT and sudden cardiac death with exercise and patients should not participate in competitive sports or any activity that causes symptoms of palpitations, presyncope, or syncope.11 Diagnostic criteria are proposed based upon the presence of two major criteria, one major plus two minor, or four minor criteria from six categories: (1) family history, (2) ECG depolarization/counduction abnormalities, (3) repolarization abnormalties, (4) global and/or regional dysfunction and structural alterations, (5) arryhtymias, and (6) fatty or fibrofatty replacement of the RV free wall.12 Modifications to
the critearia have also been proposed.13 Beneficial information may be obtained from several different diagnostic modalities including ECG, echocardiography, radionuclide ventriculography, magnetic resonance imaging, right ventricular angiography, electroanatomic mapping, myocardial biopsy, and other tests. Current guidelines recommend ICD implantation for secondary prevention of sudden cardiac death in patients with sustained VT or VF and for primary prevention in selected high risk patients.14 Appropriate device interventions are estimated to occur in approximately 48–62% of patients.15,16 In one study which followed 60 patients with ARVD who received an ICD for a mean of 80 months, the survival at 1, 5, and 10 years was 100%, 94%, and 76%, respectively.17 Although thin areas of the RV myocardium are rarely perforated during placement of the RV leads, the fibrofatty changes in the RV often interfere with sensing of arrhythmias. Moreover, many ARVD patients are young and may require multiple ICD and lead replacements over their lifetime.18 Antiarrhythmic drugs have not been shown to reduce the risk of sudden cardiac death in ARVD. Guidelines suggest that sotalol or amiodarone may be effective therapies for the treatment of sustained VT or VF in patients with ARVD in whom ICD implantation is not feasible.14 Amiodarone or sotalol may also be used as an adjunctive therapy for ARVD patients with an ICD who have frequent ventricular arrhythmias or shocks, although data from the Northa American ARVD Registry suggested that amiodarone is much more effective than beta-blockers or sotalol.19
Case 146
References 1. Richardson P, McKenna WJ, Bristow M, et al. Repert of the 1995 WHO/ISFC Task Force on the definition and classification of the cardiomyopathies. Circulation. 1996;93:841. 2. Basso C, Thiene G, Corrado D, et al. Arrhythmogenic right ventricular cardiomyopathy. Dysplasia, dystrophy, or myocarditis? Circulation. 1996;94:983. 3. Protonotarios N, Tsatsopoulou A, Patsourakos P, et al. Cardiac abnormalities in familial palmoplantar keratosis. Br Heart J. 1986; 56:321. 4. Sen-Chowdhry S, Syrris P, McKenna WJ. Genetics of right ventricular cardiomyopathy. J Cardiovasc Electrophysiol. 2005;16:927. 5. Corrado D, Fontaine G, Marcus FI, Study Group on Arrhythmogenic Right Ventricular Dysplasia/Cardiomyopathy of the Working Groups on Myocardial and Pericardial Disease and Arrhythmias of the European Society of Cardiology and of the Scientific Council on Cardiomyopathies of the World Heart Federation, et al. Arrhythmogenic right ventricular dysplasia/cardiomyopathy: need for an international registry. Circulation. 2000;101:E101. 6. Corrado D, Basso C, Schiavon M, et al. Screening for hypertrophic cardiomyopathy in young athletes. N Engl J Med. 1998;339:364. 7. Hulot JS, Jouven X, Empana JP, et al. Natural history and risk stratification of arrhythmogenic right ventricular dysplasia/cardiomyopathy. Circulation. 2004;110:1879. 8. Dalal D, Nasir K, Bomma C, et al. Clinical profile and long-term follow-up of 37 families with arrhythmogenic right ventricular cardiomyopathy. J Am Coll Cardiol. 2000;36:2226. 9. Tabib a, Loire R, Chalabreysse L, et al. Circumstances of death and gross and microscopic obsevations in a series of 200 cases of sudden death associated with arrhythmogenic right ventricular cardiomyopathy and/or dysplasia. Circulation. 2003;108:3000. 10. Maron BJ, Carney KP, Lever HM, et al. Relationship of race to sudden cardiac deathin competitive athletes with hypertrophic cardiomyopathy. J Am Coll Cardiol. 2003;41:974. 11. Maron BJ, Ackerman MJ, Nishimura RA, et al. Task Force 4: HCM and other cardiomyopathies, mitral valve prolapse, myocarditis, and Marfan syndrome. J Am Coll Cardiol. 2005;45:1340.
563 12. McKenna WJ, Thiene G, Nava A, Task Force of the Working Group Myocardial and Pericardial Disease of the European Society of Cardiology and of the Scientific Council on Cardiomyopathies of the International Society and Federation of Cardiology, et al. Diagnosis of arrhythmogenic right ventricular dysplasia/cardiomyopathy. Br Heart J. 1994;71:215. 13. Marcus FI, McKenna WJ, et al. (2010). Diagnosis of arrhythmogenic right ventricular cardiomyopathy/dysplasia: proposed modification of the task force criteria. Circulation 121(13): 1533-1541. 14. Zipes DP, Camm AJ, Borggrefe M, et al. ACC/AHA/ESC 2006 guidelines for management of patients with ventricular arrhythmias and the prevention of sudden cardiac death: a report of the American College of Cardiology/American Heart Association Task Force and the European Society of Cardiology committee for Practice Guidelines (Writing Committee to Develop Guidelines for Management of Patients With Ventricular Arrhythmias and the Prevention of Sudden Cardiac Death). J Am Coll Cardiol. 2006;48:e247. 15. Corrado D, Leoni L, Link MS, et al. Implantable cardioverter- defibrillator therapy for prevention of sudden death in patients with arrhythmogenic right ventricular cardiomyopathy/dysplasia. Circulation. 2003;108:3084. 16. Dalal D, Nasir K, Bomma C, et al. Arrhythmogenic right ventricular dysplasia: a United States experience. Circulation. 2005;112: 3823. 17. Wichter T, Paul M, Wollmann C, et al. Implantable cardioverter- defibrillator therapy in arrhythmogenic right ventricular cardiomyopathy: single-center experience of long-term follow-up and complication in 60 patients. Circulation. 2004;109:1503. 18. Corrado D, Calkins H, et al. (2010). Prophylactic Implantable Defibrillator in Patients With Arrhythmogenic Right Ventricular Cardiomyopathy/Dysplasia and No Prior Ventricular Fibrillation or Sustained Ventricular Tachycardia. Circulation 122(12): 1144-1152. 19. Marcus GM, Glidden DV, et al. (2009). Efficacy of Antiarrhythmic Drugs in Arrhythmogenic Right Ventricular Cardiomyopathy: A Report From the North American ARVC Registry. J Am Coll Cardiol 54(7): 609-615.
Case 147 Stefan H. Hohnloser and Joachim R. Ehrlich
Case Summary
Case Discussion
A 60-year-old business man presented with complaints of a rapid irregular heart beat for the first time in 2002. He was found to have atrial fibrillation (AF) with a ventricular rate between 170 and 180 bpm. His prior history was completely unremarkable with no previous evidence for cardiovascular abnormalities. Shortly before scheduled DC cardioversion, spontaneous restoration of sinus rhythm occurred. He underwent a thorough clinical examination including a resting ECG, exercise stress testing, and 2-D transthoracic echocardiography. No evidence for structural heart disease was found but the patient suffered from mild arterial hypertension for which he was started on ramipril 5 mg once daily. Approximately 3 months later, the patient was admitted via the emergency room for a new episode of AF which had lasted for approximately 4 h. While in hospital, the patient was continuously monitored and pharmacological cardioversion by means of intravenous administration of 1 mg/kg flecainide was attempted. Approximately 50 min after the injection, sinus rhythm was restored. There were no abnormal findings on subsequent standard ECG recordings and on 24 h Holter monitoring, particularly no widening of the QRS complex or prolongation of the QT interval as a result of antiarrhythmic drug administration. What long-term therapy should be considered in this patient?
The patient was instructed to use a single oral dose of flecainide, 200–300 mg, in case of new-onset AF. During a follow-up period of 5 years, the patient experienced on average 3–4 episodes of AF per year. All but one episode were stopped by orally administered flecainide. The episode that did not respond to oral medication required a DC cardioversion, which was performed in the outpatient clinic without complications. At present, the patient continues to do well on the described therapeutic regimen. In many patients afflicted with paroxysmal AF, no evidence for structural heart disease can be detected on clinical examination. Many of these patients may suffer from some degree of arterial hypertension. The frequency of AF attacks varies widely from only a few per year up to almost daily episodes. Patients in whom only few but symptomatic episodes occur are candidates for intermittent antiarrhythmic drug therapy, the so-called “pill-in-the-pocket approach (Table 147.1).” The main rationale of this form of treatment is to avoid side effects associated with long-term exposure to antiarrhythmic drugs and to apply the medication only to stop an ongoing AF episode rather than preventing recurrent AF attacks. Over the last decade, several clinical trials have evaluated this type of intermittent therapy (Table 147.1).1–4 As a common feature of these and other similar trials, patients with significant structural heart disease, abnormalities in impulse formation or propagation (i.e., bardycardia <60 bpm, evidence for preexcitation), acute coronary syndromes or congestive heart failure were excluded. Generally, AF was present for no more than 7 days. These studies evaluated the efficacy and safety of administration of a single dose of class IC antiarrhythmic drugs such as propafenone or flecainide. When compared to placebo,1,2 both drugs restored sinus rhythm in a significantly shorter period of time than placebo. Importantly, patients were treated in-hospital with continuous ECG monitoring in most studies. In the study by Alboni et al., patients were exposed to the drugs for the first time in-hospital; responders (i.e., patients with successful pharmacological cardioversion) were then instructed how to
S.H. Hohnloser (*) Department of Cardiology, Section of Clnical Electrophysiology, J.W. Goethe University, Theodor-Stern-Kai 7, Frankfurt 60590, Germany e-mail:
[email protected] J.R. Ehrlich Division of Clinical Electrophysiology, J.W. Goethe University, Theodor-Stern-Kai 7, 60590 Frankfurt, Germany e-mail:
[email protected]
A. Natale et al. (eds.), Cardiac Electrophysiology, DOI: 10.1007/978-1-84996-390-9_147, © Springer-Verlag London Limited 2011
565
566
S.H. Hohnloser and J.R. Ehrlich
Table 147.1 Summary of clinical trial examining the use of anti-arrhythmic medications intermittently for acute episodes Study (reference) Patients Setting Therapy Main outcome Capucci et al.1
181
In-hospital
Propafenone 600 mg Flecainide 300 mg Placebo
Boriani et al.2
240
In-hospital
Propafenone 600 mg
Blanc et al.3
86
In-hospital
Propafenone 600 mg
Alboni et al.4
268
In-hospital followed by self-administration over 15 months as needed
Placebo Amiodarone 30 mg/kg
self-administer the previously successful antiarrhythmic drug.4 This approach worked in almost all patients resulting in significantly less emergency room visits and hospitalizations during follow-up compared to the year before the target AF episode.4 The so-called “pill-in-the-pocket approach” for treatment of paroxysmal AF should be considered in patients with rare to infrequent episodes of AF which are, however, promptly recognized by the patient upon their onset. Before recommending the self-administration of a specific antiarrhythmic drug, this medication should be administered in a monitored environment to ensure that no unwanted effects of the antiarrhythmic drug occur. Since clinical studies on this form of intermittent therapy have predominantly used class IC antiarrhythmic drugs such as flecainide or propafenone, only patients with no or – at the most – minimally structural heart disease should be considered for this approach.
Propafenone 600 mg Flecainide 300 mg
Propafenone and flecainide superior to placebo in restoring SR with 72%, 78%, and 39% at 8 h Propafenone superior to placebo in restoring SR with 76% and 37% at 8 h Mean time to restoration of SR shorter for propafenone, at 24 h no difference Therapy successful in 94% of patients over 15 ± 5 month follow-up
References 1. Capucci A, Boriani G, Botto GL, et al. Conversion of recent-onset atrial fibrillation by a single oral loading dose of propafenone or flecainide. Am J Cardiol. 1994;74:503-506. 2. Boriani G, Biffi A, Capucci A, et al. Oral propafenone to convert recent-onset atrial fibrillation in patients with and without underlying heart disease. Ann Inter Med. 1997;126:621-625. 3. Blanc JJ, Voinov C, Maarek M, on behalf of the PARSIFAL Study group. Comparison of oral loading dose of propafenone and amiodarone for converting recent-onset atrial fibrillation. Am J Cardiol. 1999;84:1029-1032. 4. Alboni P, Botto GL, Baldi N, et al. Outpatient treatment of recent-onset atrial fibrillation with the “pill-in-the-pocket” approach. N Engl J Med. 2004;351:2384-2391.
Case 148 Joachim Ehrlich and Stefan H. Hohnloser
Case Summary A 62-year-old male patient with a history unremarkable for cardiac arrhythmias was admitted to the hospital for a first episode of symptomatic atrial fibrillation with rapid ventricular rate. There was no evidence for structural heart disease by means of history, physical exam and ECG and the patient was started on intravenous therapy with amiodarone for pharmacological conversion. There were no signs of abnormal repolarization on the resting ECG - the QTc (averaged over 3 cycles) measured 430 ms. Sinus rhythm was restored after 48 h of drug infusion with a total dose of 3 g of amiodarone. At this point, the patient developed cardiac arrest with ventricular fibrillation and had to be resuscitated. What is the likely cause of the cardiac arrest and what further testing should be considered?
Case Discussion The surface ECG revealed pronounced QTc prolongation (592 ms) along with a prominent macroscopic T-wave alternans (Figs. 148.1a, b). Development of this T-wave alternans Coincided with the development of polymorphic ventricular tachycardia of the torsades de pointes type degenerating into ventricular fibrillation (Fig. 148.1c). Amiodarone plasma concentration at that time was in the low therapeutic range (0.87 mg/L; therapeutic range 0.59–2.5 mg/L). The patient was successfully resuscitated, taken off amiodarone and
recovered fully without neurological sequelae. Prior to hospital discharge, ECG had returned to normal with a QTc interval of 416 ms (Fig. 148.2a). Microvolt T-wave alternans testing by means of submaximal exercise testing revealed negative findings (Fig. 148.2b). Subsequent genetic testing revealed a previously unpublished mutation at the KCNE2-gene (exon 1, position A269G) leading to a glycine substitution for glutamine within the c-terminus of this potassium channel b-subunit corresponding to the long QT syndrome type VI entity. The present case nicely illustrates several important clinical, electrocardiographic and molecular findings in patients with drug-induced long QT syndrome and torsades de pointes(Table 148.1). The term “reduced repolarization reserve” has been coined as a unifying concept to explain the variable risk for this syndrome.1,2 This framework suggests that the physiological mechanisms that maintain normal cardiac repolarization vary among patients but are not apparent in the basal state. However, exposure to a substance that prolongs the QT interval, or the development of a risk factor such as bradycardia after restoration of sinus rhythm in a patient presenting with atrial fibrillation (see Table 148.2), is more likely to cause exaggerated QT prolongation in a susceptible patient than in a nonsusceptible one. Perhaps the most intriguing finding in this patient was that of a novel mutation corresponding to the long QT syndrome type VI entity. Before exposure to amiodarone which was administered for pharmacological conversion of recent onset atrial fibrillation, the patient had no cardiovascular history. Similarly, his family history was unremarkable. These findings emphasize the fact that the congenital long QT syndrome has been recognized as a syndrome with incomplete Table 148.1 Key Points
J. Ehrlich (*) Division of Clinical Electrophysiology, J.W. Goethe University, Theodor-Stern-Kai 7, 60590, Frankfurt, Germany e-mail:
[email protected] S.H. Hohnloser Department of Cardiology, Section of Clnical Electrophysiology, J.W. Goethe University, Theodor-Stern-Kai 7, Frankfurt 60590, Germany e-mail:
[email protected]
• Although it is a rare phenomenon, amiodarone (particularly at low plasma concentrations) can induce proarrhythmia (e.g., torsades de pointes) • The concept of reduced repolarization reserve has been used to explain the variable risk of patients to develop drug-induced proarrhythmia • There are several clinical risk indicators for drug-associated proarrhythmia which need to be checked in all patients prior to initiation of antiarrhythmic drug therapy
A. Natale et al. (eds.), Cardiac Electrophysiology, DOI: 10.1007/978-1-84996-390-9_148, © Springer-Verlag London Limited 2011
567
568
J. Ehrlich and S.H. Hohnloser
Fig. 148.1 (a) Normal baseline QT interval during rapidly conducted atrial fibrillation. (b) Prominent QT prolongation and macroscopic TWA (arrows) after restoration of sinus rhythm during amiodarone
therapy. (c) Continuous monitoring showing the emergence of polymorphic tachycardia. Horizontal arrows indicate the typical “shortlong-short” initiating sequence of torsades de pointes
penetrance; that is, family members with near-normal QT intervals may carry nevertheless the same mutations in genes associated with the congenital long QT syndrome.3,4 Current evidence suggests that 5–10% of patients in whom torsades de pointes develops upon exposure to QT-prolonging drugs harbor mutations associated with the long QT syndrome and can thus be viewed as having a subclinical form of the congenital syndrome.2,5 This clinical observation nicely fits in the concept of reduced repolarization reserve arising from a mutation in an ion-channel gene which predisposes the carrier to drug-induced proarrhythmia. In the present case, several electrocardiographic hallmarks of drug-induced proarrhythmia became evident. Besides marked prolongation of the QT interval, macroscopic T-wave alternans became evident. This very rare phenomenon has been first described shortly after the invention of electrocardiography. In 1948, Kalter and Schwartz examined the ECGs from 6,059 patients and described an association between macroscopic TWA (observed in 5 patients) and an increased mortality of the affected patients.6 Subsequent case reports described the occurrence of visible TWA in various clinical situations such as myocardial ischemia, coronary artery spasm, electrolyte disturbances, and particularly in the setting of the congenital long QT syndrome.7 Obviously, this
grossly abnormal repolarization process gave then rise to the occurrence of torsades de pointes. The arrhythmia started with the typical initiation sequence shown in (Fig. 148.1). Of note, after stopping the offending drug amiodarone and subsequent normalization of the QT interval, there was no evidence for microscopic T-wave alternans in our patient (Fig. 148.2). Whether this observation indicates complete reversibility of the proarrhythmic changes induced by amiodarone, remains speculative at the present time. It appears important to note that amiodarone treatment is associated with an exceptionally low rate of proarrhythmic events compared to other antiarrhythmic drugs.8 If, however, amiodarone-induced proarrhythmia occurs, it does typically so at lower plasma concentration.9,10 This is in line with recent experimental studies providing evidence that acute amiodarone application on the background of reduced repolarization reserve (induced by augmentation of late INa) exhibits a biphasic effect with proarrhythmic actions occurring at lower and anti-arrhythmic effects at higher concentrations.11 Low nanomolar concentrations of amiodarone that alone caused no significant action potential duration prolongation caused significant prolongation and torsades de pointes tachycardia when administered in combination with enhanced late INa. Acutely administered amiodarone inhibits
Case 148 Fig. 148.2 (a) Resting ECG prior to discharge. (b) Negative microvolt T-wave alternans test performed after discontinuation of miodarone treatment
569
a
b 120
HR (bpm)
Max. 113
100 80
% bad
20
noise (µV)
0 2 1 0
VM
X
Y
Z
2 uV
0
2
4
6
8
10
12
Min.
Phase 1
HERG currents at low concentrations (proarrhythmic) while its effect on sodium current (antiarrhythmic) occurs at several orders of magnitude higher concentrations.11 Finally, our patient is an exceptional example of how it may be impossible to detect patients at risk for proarrhythmic events with classical clinical tools (Table 148.2). The diagnosis of congenital long QT syndrome may in some cases only be made on the basis of genetic testing. Whether broader availability of reliable and fast methodology to detect genetic alterations may improve safety of antiarrhythmic drug therapy remains to be seen.
Table 148.2 Risk factors for drug-induced torsades de pointes • Female sex • Hypokalemia, hypomagnesemia • Bradycardia • Recent conversion from atrial fibrillation, particularly with a QT-prolonging drug • Congestive heart failure • Digitalis therapy high drug concentration (exceptions: quinidine, amiodarone) • Rapid rate of infusion with a QT-prolonging drug • Subclinical long QT syndrome • Ion-channel polymorphisms
570
References 1. Roden DM. Drug-induced prolongation of the QT interval. N Engl J Med. 2004;350:1013-1022. 2. Roden DM. Taking the “idio” out of “idiosyncratic”: predicting torsades de pointes. Pacing Clin Electrophysiol. 1998;21:10291034. 3. Keating MT, Sanguinetti MC. Molecular and cellular mechanisms of cardiac arrhythmias. Cell. 2001;104:569-580. 4. Mohler PJ, Schott JJ, Gramolini AO, et al. Ankyrin-B mutation causes type 4 long-QT cardiac arrhythmia and sudden cardiac death. Nature. 2003;421:634-639. 5. Yang P, Kanki H, Drolet B, et al. Allelic variants in long-QT disease genes in patients with drug-associated torsades de pointes. Circulation. 2002;105:1943-1948. 6. Kalter HH, Schwartz ML. Electrical alternans. NY State J Med. 1948;1:1164-1166.
J. Ehrlich and S.H. Hohnloser 7. Schwartz PJ, Malliani A. Electrical alternation of the T wave: clinical and experimental evidence of its relationship with the sympathetic nervous system and with the long-QT syndrome. Am Heart J. 1975;89:45-50. 8. Hohnloser SH, Singh BN. Proarrhythmia with class III antiarrhythmic drugs: definition, electrophysiologic mechanisms, incidence, predisposing factors, and clinical implications. J Cardiovasc Electrophysiol. 1995;6:920-936. 9. Tomcsanyi J, Merkely B, Tenczer J, et al. Early proarrhythmia during intravenous amiodarone treatment. Pacing Clin Electrophysiol. 1999;22:968-970. 10. Bertholet M, Dubois C, Materne P, et al. Sudden marked QT prolongation after intravenous administration of amiodarone. Am J Cardiol. 1983;52:1361. 11. Wu L, Rajamani S, Shyrode JC, Li H, Ruskin J, Antzelevitch C, Belardinelli L. Augmentation of late sodium current unmasks the proarrhythmic effects of amiodarone. Cardiovasc Res. 2008; 77(3):481-488.
Case 149 Bradley P. Knight
Case Summary A 46-year-old woman, who was previously healthy, presented with cardiogenic shock and a heart rate of 180 bpm. An electrocardiogram is shown in Fig. 149.1. The rhythm is a long RP’ tachycardia with P-waves that have an inferior axis and are deeply inverted in lead V1. An echocardiogram revealed severe, global left ventricular dysfunction with no significant valvular abnormalities. She was treated with inotropic support initially and then with a beta blocker (carvedilol) and an angiotensin converting enzyme inhibitor (enalapril).
She recovered clinically and was referred to a heart failure clinic a few weeks later, where she was found to have a heart rate was just over 100 bpm. Her electrocardiogram showed a P-wave morphology that was similar to that when she presented initially, and there was mild first degree AV block (Fig. 149.2). Rhythm strips from when the patient was on telemetry during her initial hospitalization were obtained (Fig. 149.3). Does this patient have sinus tachycardia as a result of an idiopathic dilated cardiomyopathy, or a tachycardia-mediated cardiomyopathy?
Fig. 149.1 12-lead electrocardiogram obtained at the time of initial presentation
B.P. Knight Division of Cardiology, Northwestern Medical Center, 676 N. St. Clair, Suite 600, Chicago, IL 60611, USA e-mail:
[email protected] A. Natale et al. (eds.), Cardiac Electrophysiology, DOI: 10.1007/978-1-84996-390-9_149, © Springer-Verlag London Limited 2011
571
572
Fig. 149.2 12-lead electrocardiogram obtained a few weeks after the initial presentation
Fig. 149.3 Rhythm strip obtained during initial hospitalization
B.P. Knight
Case 149
Case Discussion Rapid, sustained ventricular rates can lead to a tachycardiainduced cardiomyopathy. This has been well documented in patients as well as in pacing-induced heart failure animal models.1 However, in a patient with ventricular dysfunction, it can be occasionally difficult to determine whether or not a supraventricular tachycardia is the primary problem. This is particularly true when the P-wave morphology is similar to a sinus P-wave morphology. A useful clue in this case can be seen in the monitor strips in Fig. 149.3, which show Wenckebach type, 2° AV block. This clue is useful, because it would be unlikely for a patient with heart failure to demonstrate Wenckebach type, 2° AV block during sinus tachycardia. A study by Gelb and Garson in 19902 found three observations that differentiated a right atrial ectopic tachycardia from sinus tachycardia: an atrial rate
573
greater than 150% of that predicted for age, an inverted or notched P-wave in lead V1, and 2° AV block. This patient satisfied had all three criteria. Successful catheter ablation of an ectopic atrial tachycardia arising from the right atrium was performed three months after her initial presentation. An echocardiogram three months later showed normal ventricular function.
References 1. Shinbane JS, Wood MA, Jensen DN, Ellenbogen KA, Fitzpatrick AP, Scheinman MM. Tachycardia-induced cardiomyopathy: a review of animal models and clinical studies. J Am Coll Cardiol. 1997;29:709-715. 2. Gelb BD, Garson A Jr. Noninvasive discrimination of right atrial ectopic tachycardia from sinus tachycardia in “dilated cardiomyopathy”. Am Heart J. 1990;120:886-891.
Case 150 Bradley P. Knight
Case Summary A 21-year-old woman presented to her obstetrician with an incessant supraventricular tachycardia at 160 bpm during her 27th week of pregnancy. She was asymptomatic and unaware of her tachycardia. Left ventricular function was normal. There were times when the tachycardia persisted despite transient AV block. Transesophageal pacing failed to restore sinus rhythm. She was treated with quinidine and a metoprolol. The tachycardia did not terminate, but the atrial rate slowed to 130 bpm. An electrocardiogram is shown in Fig. 150.1 after the initiation of antiarrhythmic drug therapy. She underwent uneventful delivery of her child and was subsequently referred for further management. The tachycardia was still present, but the heart rate was around 100 bpm. What is the best approach to this patient at this stage? Based on the electrocardiogram, what is the mechanism of the tachycardia and from where is it arising?
Case Discussion This is a patient with an incessant tachycardia who was treated medically at the time of presentation, because she was pregnant and asymptomatic. Although catheter ablation can be performed safely during pregnancy, medical therapy was reasonable as the initial treatment in this patient. After the patient has delivered her child, the question is whether to continue medical therapy or recommend ablative therapy.
Given her young age and the possibility of a tachycardiamediated cardiomyopathy, an ablation procedure should be considered as first-line therapy for this patient. The observation that the tachycardia persisted despite transient AV block excludes AV re-entry using an accessory pathway as the mechanism, and the absence of deeply inverted P-waves in the inferior leads excludes AV nodal reentry. Therefore, the mechanism of the tachycardia must be atrial. The P-wave morphology is clearly inverted in lead I and upright in lead aVR. This suggests a left atrial origin, away from the septum. In addition, the P-wave is mostly upright in the inferior leads, but the axis is not inferior enough to be consistent with an upper pulmonary vein origin. It is most likely that the rhythm is arising from the mitral annulus or left atrial appendage. A catheter ablation procedure was performed. Activation mapping was performed in the left atrium and an electroanatomic map was created (Fig. 150.2). The view is right anterior oblique, with the right-sided pulmonary veins to the left and the mitral annulus to the right. The color coding is from red to purple and represents the relative timing at each recorded site during an activation map. The earliest activation is depicted as red and is located near the base of the left atrial appendage. Delivery of radiofrequency energy at this site terminated the tachycardia and rendered it noninducible. Although focal atrial tachycardias can arise anywhere in the atria, there is a characteristic anatomic distribution.1 Common sites include the crista terminalis, the pulmonary veins, and the left atrial appendage.2 Catheter ablation for focal atrial tachycardias is associated with a high success rate.
B.P. Knight Division of Cardiology, Northwestern Medical Center, 676 N. St. Clair, Suite 600, Chicago, IL 60611, USA e-mail:
[email protected] A. Natale et al. (eds.), Cardiac Electrophysiology, DOI: 10.1007/978-1-84996-390-9_150, © Springer-Verlag London Limited 2011
575
576 Fig. 150.1 Electrocardiogram obtained during tachycardia
Fig. 150.2 Electroanatomic map of the left atrium during an atrial tachycardia
References 1. Roberts-Thomson KC, Kistler PM, Kalman JM. Focal atrial tachycardia I: clinical features, diagnosis, mechanisms, and anatomic location. Pacing Clin Electrophysiol. 2006;29:643-652. 2. Kato M, Adachi M, Yano A, et al. Radiofrequency catheter ablation for atrial tachycardia originating from the left atrial appendage. J Int Card Electrophys. 2007;19:45-48.
B.P. Knight
Case 151 Bradley P. Knight
Case Summary A 37-year-old man is referred for evaluation of atrial fibrillation. He presented with his first episode of atrial fibrillation 6 months earlier. A nuclear perfusion stress test was reportedly normal. He was treated with atenolol and warfarin, and underwent electrical cardioversion. He had a second episode of atrial fibrillation 2 months ago and was started on oral amiodarone after another electrical cardioversion. A recent Holter monitor showed sinus rhythm
without atrial fibrillation and a 5-beat run of nonsustained monomorphic VT. He was doing well without complaints and his physical examination was unremarkable. He denied syncope. He had not history of hypertension or other medical problems. However, he did recall that when he was a child, his mother was found dead at home unexpectedly at age 41. An electrocardiogram was obtained and is shown in Fig. 151.1. What is the most likely diagnosis? How should he be managed?
Fig. 151.1 A 12-lead electrocardiogram of a young patient with recurrent atrial fibrillation
B.P. Knight Division of Cardiology, Northwestern Medical Center, 676 N. St. Clair, Suite 600, Chicago, IL 60611, USA e-mail:
[email protected] A. Natale et al. (eds.), Cardiac Electrophysiology, DOI: 10.1007/978-1-84996-390-9_151, © Springer-Verlag London Limited 2011
577
578
Case Discussion This case is a young patient with atrial fibrillation and a family history of premature sudden death. In such cases, it is important to thoroughly evaluate the patient for possible heritable diseases, including structural heart diseases and channelopathies. The electrocardiogram shows sinus rhythm with biatrial enlargement, left ventricular hypertrophy with marked repolarization abnormalities. Given the absence of hypertension and his malignant family history, the most likely diagnosis is hypertrophic cardiomyopathy. The diagnosis was confirmed by echocardiography. He had severe concentric left ventricular hypertrophy, but no outflow tract gradient. This case emphasizes the importance of a careful search for secondary causes of atrial fibrillation, even in patients who are referred several months after their initial presentation. Patients with hypertrophic cardiomyopathy are at risk for cardiac arrest. Various clinical and echocardiographic markers have been identified that appear to increase the risk of sudden death (Table 151.1).1 Given this patient’s family history and the presence of nonsustained ventricular tachycardia, he underwent defibrillator implantation.2,3 Because he had recurrent atrial fibrillation, a dual-chamber device with algorithms that help manage his atrial fibrillation was implanted.4
B.P. Knight Table 151.1 Potential risk factors for sudden cardiac death in patients with hypertrophic cardiomyopathy • History of cardiac arrest • Family history of premature death • History of syncope • Presence of nonsustained ventricular tachycardia Holter • Early onset of disease • Magnitude of left ventricular hypertrophy • Abnormal blood pressure response to exercise • Myocardial ischemia on perfusion tomography • Extent of myocyte disarray or interstitial fibrosis • “Malignant” causal mutations or modifier genes Adapted from1
References 1. Marian AJ. On predictors of sudden cardiac death in hypertrophic cardiomyopathy. J Am Coll Cardiol. 2003;41:994-996. 2. Maron BJ, Shen WK, Link MS, et al. Efficacy of implantable cardioverter-defibrillators for the prevention of sudden death in patients with hypertrophic cardiomyopathy. N Engl J Med. 2000;342:365-373. 3. Maron BJ, Spirito P, Shen WK, et al. Implantable cardioverter- defibrillators and prevention of sudden cardiac death in hypertrophic cardiomyopathy. JAMA. 2007;298:405-412. 4. Knight BP, Gersh BJ, Carlson MD, et al. Role of permanent pacing to prevent atrial fibrillation: science advisory from the American Heart Association Council on Clinical Cardiology. Circulation. 2005;111:240-243.
Case 152 Andrew D. Krahn
Case Summary
Case Discussion
A 39-year-old athletic male presented to the emergency room after an episode of syncope that occurred within a minute of completing a 90-minute training session that included sprints and distance running. He described relatively abrupt onset of lightheadedness with imminent loss of consciousness, after which he lay down. He lost consciousness briefly, and awakened fatigued but oriented. Because of the concern of his fellow running club members, he was brought to the emergency room where an ECG was performed (Fig. 152.1). He was monitored in the emergency room for several hours, and discharged with arrangements made to obtain an urgent out patient echocardiogram, external loop recorder and Cardiology follow-up. Cardiology review found that he had “fainted” once at age 14 at his cousin’s wedding on a hot day. His uncle died suddenly of a “heart attack” at age 42. His echocardiogram showed mild concentric left ventricular hypertrophy in keeping with an athletic heart. An external loop recorder worn for 2 weeks captured the cardiac rhythm during presyncope, which was different in character than the index syncopal episode (Fig. 152.2). Because of concern regarding the circumstances of syncope and his family history, an implanted loop recorder (ILR) was inserted. Syncope of similar character recurred 2 months later while standing awaiting a ride several minutes after a 90-minute training run (Fig. 152.3). What is the interpretation of the findings seen in Figs. 152.2 and 152.3? Will this patient benefit from pacemaker implantation?
Syncope affects 40% of the population at one point in their life, and is most often explained by vasovagal syncope in the absence of underlying heart disease. A careful history and exclusion of heart disease by physical examination, resting ECG and an echocardiogram is often sufficient to arrive at a clinical diagnosis and exclude more sinister causes. In the current case, syncope occurred in the context of exercise, which should always raise concern that cardiac obstruction or an arrhythmia may have been induced by exercise. In this patient, a careful history indicated that the episode was immediately after exercise and not during exertion, a much less sinister presentation. Venous return is dependent on venous pressure and the skeletal muscle pump. When exercise is stopped abruptly, venous return may fall precipitously, leading to a fall in blood pressure and presyncope or syncope. This is seen frequently when patients undergo exercise testing, and are unsteady when the treadmill or bicycle is stopped. Cool down exercise typically attenuates symptoms. Follow-up assessment in the current case demonstrated no evidence of obstruction or tachyarrhythmia. The resting ECG shows sinus arrhythmia, which is typical in athletes. Importantly, the ECG did not show evidence of a repolarization abnormality such as a prolonged QT interval. An exercise test to screen for catecholaminergic ventricular tachycardia (CPVT) or other benign exercise induced arrhythmia would have been reasonable. The external loop recorder again demonstrated sinus arrhythmia during presyncope, suggesting high vagal tone supporting the clinical diagnosis of probable vasovagal syncope. Finally, the ILR captured an episode of post exercise syncope that demonstrated sinus rhythm with premature atrial beats, followed by dramatic sinus slowing and junctional rhythm, followed by a 21-second pause. Note the baseline artifact during the pause that likely represents the motion artifact from striking the ground during syncope. There is also sinus acceleration and tachycardia after the episode. The gradual slowing preceding the pause suggests a profound cardioinhibitory form of vasovagal syncope. Interrogation of implantable loop recorders after a recurrence of syncope will often demonstrate rhythm findings
A.D. Krahn Division of Cardiology, University of Western Ontario, 339 Windermere Road, London, ON, N6A 5A5, Canada e-mail:
[email protected]
A. Natale et al. (eds.), Cardiac Electrophysiology, DOI: 10.1007/978-1-84996-390-9_152, © Springer-Verlag London Limited 2011
579
580
Fig. 152.1 Resting ECG
Fig. 152.2 External loop recorder rhythm strip during presyncope
A.D. Krahn
Case 152
581
Fig. 152.3 Interrogation of the implantable loop recorder after an episode of loss of consiousness. See text for discussion
that require clinical correlation and considerable judgment to interpret. Even with documented asystole, the need for permanent pacemaker therapy in presumed vasobagal syncope is controversial. The ISSUE investigators have proposed a useful classification system for symptomatic events in loop recorders applied to patients with syncope.1 This is often helpful in assigning a probable mechanism of syncope, and is likely to be adopted for research purposes to create consistency in end points. Despite the dramatic documented pause, pacemaker implantation for what is presumed to be vasovagal syncope is a IIa indication for pacing. Since the guidelines were published in 2002, two additional randomized double blinded studies have not found benefit from pacing, so that pacing in this population is seldom performed.2, 3 The currently enrolling ISSUE3 study is enrolling patients with vasovagal syncope that undergo ILR implantation, and pacemaker implantation in those with documented asystole such as the current patient. The pacemaker is then randomized to on or
off, and outcome will be assessed. This study promises to be an important trial that uses ILR based bradycardia instead of tilt testing to identify vasovagal syncope patients that may benefit from pacing.
References 1. Brignole M, Moya A, Menozzi C, Garcia-Civera R, Sutton R. Proposed electrocardiographic classification of spontaneous syncope documented by an implantable loop recorder. Europace. 2005;7:14-18. 2. Connolly SJ, Sheldon R, Thorpe KE, et al. Pacemaker therapy for prevention of syncope in patients with recurrent severe vasovagal syncope: second Vasovagal Pacemaker Study (VPS II): a randomized trial. JAMA. 2003;289:2224-2229. 3. Raviele A, Giada F, Menozzi C, et al. A randomized, double-blind, placebo-controlled study of permanent cardiac pacing for the treatment of recurrent tilt-induced vasovagal syncope. The vasovagal syncope and pacing trial (SYNPACE). Eur Heart J. 2004;25: 1741-1748.
Case 153 Byron K. Lee, Melvin M. Scheinman, and Zian H. Tseng
Case Summary A 71-year-old woman with a history of chronic obstructive pulmonary disease was admitted with worsening dyspnea on exertion. On evaluation, she was found to be bradycardic with a ventricular rate of 35 bpm. Telemetry showed that she was in sinus rhythm with 2:1 conduction to the ventricle (Fig. 153.1). Her 12 lead ECG revealed right bundle branch block and left anterior fascicular block. It was suspected that the patient’s dyspnea on exertion was at least partially due to AV nodal conduction disease. However, she was unable get out of bed and exercise to allow assessment of AV node conduction at higher heart rates. Therefore, carotid sinus massage (CSM) was performed (Fig. 153.2). What does the reaction to CSM tell you about AV nodal function?
Case Discussion Second degree AV block is often classified as either Mobitz Type 1 or Mobitz Type 2. In Type 1 block there is gradual PR interval prolongation in successive beats before block occurs. In Type 2 block, the PR interval in the preceding beats is constant before a P wave is dropped. Type 1 block is thought to be
B.K. Lee (*) Division of Cardiology, University of California, 500 Parnassus Avenue, San Francisco, CA 94143, USA e-mail:
[email protected] M.M. Scheinman Department of Cardiac Electrophysiology, University of California, 500 Parnassus Avenue, MUE 436, San Francisco, CA 94143-1354, USA e-mail:
[email protected] Z.H. Tseng Cardiac Electrophysiology Section, Cardiology Division, University of California, San Francisco, 500 Parnassus Avenue, MUE, RM 433, San Francisco, CA 94143, USA e-mail:
[email protected]
a characteristic of block in the AV node while Type 2 block is more common with block below or within the bundle of His. In stable second degree AV block with 2:1 conduction, the Mobitz classification scheme cannot be applied since every other P wave is blocked. PR prolongation can’t be measured. Therefore, the site of block cannot be inferred. Determination of the site of block, however, is an important factor for clinical decision making and several maneuvers may be helpful. Patients with AV nodal block are often strongly affected by changes in vagal tone. These patients typically do not frequently progress to complete heart block and therefore will rarely require a pacemaker. Patients with block in the His– Purkinje system typically have structural degeneration of the conduction system that is strongly rate dependent and is not affected by changes in vagal tone. Block in the His–Purkinje system does frequently progress to complete heart block, and a pacemaker is usually necessary.1–2 We can often determine the site of second degree AV block by simply exercising patients to increase their sinus rate. When second degree AV block occurs in the AV node, you should see improvement of conduction when vagal tone is withdrawn and worsening if vagal tone is augmented. If the site of the second degree AV block is below the node, you should see worsening of conduction with a higher ratio of blocked beats at faster sinus rates and improvement if the sinus rate slows. In this patient, exercise could not be performed because she was initially unable to get out of bed. Therefore, CSM was performed, CSM typically causes a temporary increase in centrally mediated vagal tone which causes the sinus rate to slow. Since second degree AV block in the AV node is mainly caused by excess vagal tone, you may see the AV block worsen with CSM in these patients. In contrast, increasing vagal tone with CSM in patients with second degree AV block in the His–Purkinje system may paradoxically improve conduction. As the sinus rate slows there is more time between beats for recovery in the infranodal conduction system and 1:1 conduction may be restored. In this patient, CSM improved AV conduction to 1:1 as the sinus rate slowed from 72 to 50 beats per min (Fig. 153.2). Therefore, block in the His–Purkinje system is the likely diagnosis. The patient had a dual chamber PPM implanted. Her dyspnea on exertion subsequently improved.
A. Natale et al. (eds.), Cardiac Electrophysiology, DOI: 10.1007/978-1-84996-390-9_153, © Springer-Verlag London Limited 2011
583
584
B.K. Lee et al.
Fig. 153.1 Baseline telemetry strip shows 2:1 AV conduction with an underlying sinus cycle length of 840 ms
Fig. 153.2 Telemetry during and after carotid sinus massage. The sinus cycle length has prolonged to 1,200 ms
References 1. Wallick DW, Stuesse SL, Levy MN. Mechanisms of vagosympathetic control of the atrial-AV nodal conduction. In: Mazgalev TN, Tchou PJ, eds. Electrophysiology: A View from the Millenium. Armonk, NY: Futura; 2000:133-135.
2. Saoudi N, Rinaldfi JP, Imianitoff M, et al. Atrioventricular and intraventricular conduction disorders. In: Crawford MH, DiMarco JP, Paulus WJ, eds. Cardiology. 2nd ed. London: Mosby; 2004: 645-658.
Case 154 Srijoy Mahapatra
Case Summary A 55-year-old man with a history of an inferior myocardial infarction treated with primary angioplasty with stenting 4 h after the onset of pain presented to an outside hospital with lightheadness. His blood pressure was 90/60 and his heart rate was 190 bpm. His ECG revealed a wide complex tachycardia (Fig. 154.1). He was cardioverted and started on amiodarone.
His echocardiogram the next morning revealed an EF of 45% with an inferior wall motion abnormality. Coronary angiography showed a patent right coronary stent and no disease in the left coronary system. Despite oral loading with 10 g of amiodarone over the next several days he continue to have symptomatic runs of the same wide complex tachycardia. He underwent an EP study which confirmed the diagnosis of ventricular tachycardia (VT) and an attempted
Fig. 154.1 ECG obtained in the emergency room showing a wide QRS tachycardia at 200 bpm
S. Mahapatra Department of Biomedical Engineering, University of Virginia, 1215 Lee Street, Charlottesville, VA 22908, USA e-mail:
[email protected] A. Natale et al. (eds.), Cardiac Electrophysiology, DOI: 10.1007/978-1-84996-390-9_154, © Springer-Verlag London Limited 2011
585
586
endocardial ablation at the other hospital. The ablation attempt was unsuccessful and the operator reported he did not find any early endocardial points. He was then transferred for further evaluation. What do you think is the mechanism for the VT? Can you predict the site of origin? What should be your next step?
Case Discussion This patient’s ECG and the clinical history of a failed endocardial ablation attempt suggest that this VT has an epicardial site of origin or exit. In sinus rhythm and many VTs, depolarization occurs from the endocardium to the epicardium. If there is an epicardial VT focus (or exit), this sequence is reversed. Since the focus is now further from the conduction system, the QRS width is relatively wide and the first part of the QRS often has a slower upstroke than the second part. Although there are no large, prospective series, clinical and ECG criteria proposed that have been associated with an epicardial focus or exit are listed below (Table 154.1). The diagnosis of an epicardial VT can be confirmed at EP study with mapping. In an endocardial VT, the EP study will show an earliest spot (focal VT) or an area where “earlymeets-late” (circuit). The lack of either suggests an epicardial focus. With epicardial VT the endocardial activation map will show a zone of earliest activation but there is often no endocardial point activated before the onset of the surface QRS. Another clue is that the endocardial voltage at that point may be normal, although this is rare.
Fig. 154.2 ECG findings that suggested an epicardial VT
S. Mahapatra Table 154.1 Clinical and ECG findings associated with epicardial VT Clinical factors 1. Inferior, as opposed to anterior, myocardial scar 2. Non-ischemic cardiomyopathies 3. Prior failed endocardial ablation ECG (Fig. 154.2) findings 1. VT QRS >198 ms 2. A slurred QRS onset with a time from start of Q to peak of R > 0.55 of the total QRS duration using the precordial lead with the lowest ratio 3. Intrinsicoid time >85 ms (time from earliest Q or R deflection to nadir of S in V1–V6) 4. Pseduodelta >34 ms (time from first V deflection to first rapid V deflection in one lead)
This patient underwent both an endocardial and epicardial EP study. The latter was via a percutanous subxiphoid puncture as described by Sosa and collegues. VT was easily inducible but, was not hemodynamically stable. Thus, a voltage map was created. (Figs. 154.3a, b) It showed multiple scars both endocardially and epicaridially but one area in the epicardium showed a narrow isthmus that could support VT. Furthermore, pacing in this zone yielded a 12/12 pace map. Thus, a substrate based ablation was undertaken to produce a line of block in this isthmus after a coronary angiogram showed that there were no major epicardial vessels in the area. At the end of the study, no VT was inducible despite isoproterenol and epinephrine infusions and programmed stimulation. After 9 months of follow-up he has not experienced any recurrent episodes of VT.
Case 154
A
587
B
Fig. 154.3 Endocardial (Panel A) and epicardial (Panel B) voltage maps. The narrow isthmus that was the critical part of the epicardial circuit is seen at about 3 o’clock in panel B
Bibliography Berruezo A, Mont L, Nava S, et al. Electrocardiographic recognition of the epicardial origin of ventricular tachycardias. Circulation. 2004; 109:1842-1847. Daniels D, Lu YY, Morton JB, et al. Idiopathic epicardial left ventricular tachycardia originiating remote from the sinus of valsalva. Circulation. 2006;113:1659-1666.
Marchlinski FE, Callans DJ, Gottlieb CD, et al. Linear Ablation lesions for control of unmappable ventricular tachycardia in patients with ischemic and nonischemic cardiomyopathy. Circulation. 2000;101: 1268-1296. Sosa E, Scanavacca M, d’Avilla A, Pillegi F. A new technique to perform epicardial mapping in the electrophysiology laboratory. J Cardiovasc Electrophys. 1996;7:531-536.
Case 155 Pamela K. Mason
Case Summary A 65-year-old man presents to the emergency department complaining of palpitations and dizziness. The patient was found to be in a wide-complex tachycardia (Fig. 155.1) and hypotensive. He was electrically cardioverted with return to normal sinus rhythm (Fig. 155.2). His past medical history was significant for an aortic valve replacement with a single vessel right coronary bypass 8 weeks prior to this presentation and chronic left bundle branch block. After cardioversion, an echocardiogram showed a normally functioning aortic prosthetic valve, moderate left ventricular dyssynchrony, and a left ventricular ejection fraction of 40%. He was referred for electrophysiology study. Figure 155.3 shows the intracardiac electrograms obtained during the tachycardia. What is the mechanism of the tachycardia? How should it be treated?
Case Discussion Reentry in the His–Purkinje system is a well known form of ventricular tachycardia (VT) that accounts for 6% of cases of VT with a left bundle branch block morphology in patients with ischemic heart disease but up to 40% of cases in patients with non-ischemic cardiomyopathies. It has also been found in patients without structural heart disease, with the universal commonality between all patients who develop bundle branch reentry the presence of an abnormal conduction in the His–Purkinje system. This is demonstrated by an intraventricular conduction delay on the baseline ECG, most commonly an incomplete or complete left bundle branch block, and a prolonged HV interval at electrophysiologic stuidy. Because the mechanism of the tachycardia is a macroreentrant circuit using the right bundle for anterograde
P.K. Mason Department of Internal Medicine, University of Virginia, 800158, Charlottesville, VA 22908, USA e-mail:
[email protected]
conduction and the left bundle for retrograde conduction, the QRS morphology in the tachycardia is similar, though often not identical, to that in sinus rhythm. The patient described here demonstrates many of the usual electrophysiologic characteristics of patients with bundle branch reentry. The QRS morphology of the tachycardia is similar to that in the baseline ECG. A stable His electrogram is present before each QRS complex during the tachycardia, and the HV interval in tachycardia is prolonged over baseline. In addition, with further study it was noted that changes in the R–R interval were preceded by changes in the H–H interval, and the tachycardia terminated with block in the His–Purkinje system. Ablation of the right bundle should eliminate this form of VT since the right bundle forms the anterograde limb of the circuit. Most patients, however, will have delayed anterograde conduction over the left bundle rather than complete block, so the ECG if the right bundle is ablated will show a right bundle branch block configuration (Fig. 155.4). Ablation of the left bundle using a retrograde aortic approach is also possible and effective. Many patients with bundle branch reentry will have other clinical or stimulationinduced arrhythmias, so an ICD is often indicated in patients with ventricular dysfunction even if they have a successful catheter ablation of the right bundle. Late AV block and worsened heart failure are potential complications of a right bundle ablation for bundle branch reentry and these constitute additional reasons to consider an ICD and resynchronization therapy. However, an ICD alone with ablation may not be an optimal approach. Even if the patient is going to be scheduled for an ICD, a presentation with VT with a left bundle branch block morphology, should make one consider an electrophysiologic study since elimination of this arrhythmia by ablation may decrease the frequency of shocks in the future. This is particularly likely to be true in patient with valvular disease, nonischemic cardiomyopathies and ECG evidence of delayed conduction in the left bundle at baseline. This patient had no other inducible arrhythmias and did not want either a pacemaker of ICD unless absolutely necessary. Even though his post ablation HV interval was 86 ms, he has not developed heart block or ventricular arrhythmias during follow-up.
A. Natale et al. (eds.), Cardiac Electrophysiology, DOI: 10.1007/978-1-84996-390-9_155, © Springer-Verlag London Limited 2011
589
590
Fig. 155.1 12-Lead ECG of the patient’s presenting rhythm
Fig. 155.2 12-Lead ECG after electrical cardioversion
P.K. Mason
Case 155 Fig. 155.3 Intracardiac electrograms during tachycardia. At this point in time there is variable VA block. A His potential can be seen before each QRS complex
591
I
II
AVF
V1 HRA
HBED
RVA
0 d: 100 mm/sec
1000 Time: 00: 04: 38
Study #1
Protocol #9
2000
Protocol Name: RRP
Fig. 155.4 Sinus rhythm with a right bundle branch block pattern QRS complex after ablation
Bibliography Becker R, Melkumov M, Senges-Becker JC, et al. Are electrophysiologic studies needed prior to defibrillator implantation? PACE. 2003;26:1715-1721. Caceres J, Jazayeri M, MacKinnie J, et al. Sustained bundle branch reentry as a mechanism of clinical tachycardia. Circulation. 1989;79:256-270. Cohen TJ, Chien WW, Lurie KG, et al. Radiofrequency catheter ablation for treatment of bundle branch reentrant ventricular tachycardia:
results and long-term follow-up. J Am Coll Cardiol. 1991;18: 1767-1773. Eckart RE, Hruczkowski TW, Tedrow UB, Koplan BA, Epstein LM, Stevenson WG. Sustained ventricular tachycardia associated with valve surgery. Circulation. 2007;116:2005-2011. Narasimhan C, Jazayeri MR, Sra J, et al. Ventricular tachycardia in valvular heart disease. Facilitation of sustained bundle-branch reentry by valve surgery. Circulation. 1997;96:4307-4313.
Case 156 Pamela K. Mason
Case Summary A 71-year-old man was admitted to the cardiology service after experiencing a syncopal spell while he was driving. He sustained only minor injuries after his motor vehicle accident. This was the fifth episode of frank syncope that he had experienced within the past 6 months, several of which had resulted in injury and hospitalization. His past medical history included mild hypertension and hyperlipidemia but he had no history of angina or prior myocardial infarction. There was no family history of fainting or arrhythmias. Because of his history of multiple syncopal spells, he had already had an extensive cardiac work-up.
A recent echocardiogram showed normal left ventricular function, normal chamber sizes, and no valvular abnormalities. During stress testing, he achieved ten METS, reached his target heart rate, and developed no angina or electrocardiographic abnormalities. Nuclear imaging was normal. He had also worn an ambulatory ECG monitor several times. These were all non-diagnostic but he had remained free of symptoms during the recordings. Physical examination performed at the time of this admission was normal as was his ECG. The monitor strip in Fig. 156.1 was obtained during a bedside diagnostic maneuver. What was this diagnostic maneuver? What treatment does this patient require?
Fig. 156.1 Rhythm strip during right carotid sinus massage in the emergency room
P.K. Mason Department of Internal Medicine, University of Virginia, 800158, Charlottesville, VA 22908, USA e-mail:
[email protected] A. Natale et al. (eds.), Cardiac Electrophysiology, DOI: 10.1007/978-1-84996-390-9_156, © Springer-Verlag London Limited 2011
593
594
Case Discussion The tracings shows a 6.2 s period of asystole associated with right carotid sinus massage. Despite multiple admissions and evaluations for syncope prior to this episode, carotid sinus massage had not previously been performed in this patient. Inclusion of carotid sinus massage as a routine part of the evaluation of elderly patients with syncope can make their evaluation much more efficient. In some series, up to 1/3 of elderly patient who present with syncope or falls will have a positive response to carotid sinus massage. Carotid sinus hypersensitivity is a form of neurally mediated syncope. In this syndrome, the carotid sinus baroreceptor is excessively responsive to application of extrinsic pressure. This may be produced by tight clothing, adjacent tumors or scarring or manual pressure but many patients will not report any obvious precipitating factor. Performing carotid sinus massage requires ECG and blood pressure monitoring. The carotid arteries should be checked for bruits, which are a contraindication, prior to applying pressure. Pressure should then be applied to the fusiform portion of the carotid sinus, the site of the greatest arterial pulsation, for 5–10 s. A positive response is defined as a pause of greater than 3 s or a systolic blood pressure drop of greater than 50 mmHg. If carotid message is negative on one side, the contralateral side can be checked after waiting for 30–120 s. Sensitivity is enhanced if the maneuver is performed in both supine and upright positions. Syncope due to CSH is a disease of the elderly, virtually never being seen in patients less than 50 years of age. However, it must be remembered that a hypersensitive carotid sinus massage response may be only a nonspecific finding since it can be observed in 10–30% of asymp tomatic, elderly individuals. The postulated mechanism is an up-regulation of the alpha-2 adrenoreceptors in the baroreceptor pathway such that stimulation results in an overshoot baroreflex efferent response that results in bradycardia and hypotension. Why some patients will
P.K. Mason
manifest an abnormal respone yet never develop symptoms is unknown. Diagnosing carotid sinus hypersensitivity over other forms of neurocardiogenic syncope is important in that it can alter management. Patients with recurrent, unexplained syncope with a cardioinhibitory response to carotid sinus massage have been shown to benefit from pacemaker placement. This is particularly true in elderly patients in whom no other cause of syncope is suggested after their initial evaluation. Benefits from pacing are less likely to be found in patients with tilt-induced neurocardiac syncope. The latter syndome often has both marked cardioinhibitory and vasodepressor components and the vasodepressr component often offsets any benefits from cardiac pacing. Indeed, carotid sinus hypersensitivity is currently the only form of neurally- mediated syncope that is a class I indication for pacemaker placement. This patient had undergone a thorough evaluation and no other cause for his recurrent syncope could be identified. He received a dual chamber pacemaker during this hospitalization. He has not had any recurrent syncope or presyncope since his pacemaker was placed.
Bibliography Hamill SC. Value and limitations of noninvasive assessment of syncope. Cardiol Clin. 1997;15:196-218. Kenny RAM, Richardson DA, Steen N, Bexton RS, Shaw FE, Bond J. Carotid sinus syndrome: a modifiable risk factor for nonaccidental falls in older adults (SAFE PACE). J Am Coll Cardiol. 2001;38:1491-1496. Kerr SR, Pearce MS, Braye C, Davis RJ, Kenny RA. Carotid sinus hypersensitivity in older persons: implicationsfor diagnosis of syncope and falls. Archiv Intern Med. 2006;166:515-520. Maggi R, Menozzi C, Brignole M, et al. Cardioinhibitory carotid sinus hypersensitivity predicts an asystolic mechanism of spontaneous neurally mediated syncope. Europace. 2007;9:563-567. Mallet M. Carotid sinus syndrome. Hosp Med. 2003;64:92-95. Shen WK, Decker WW, Smars PA, et al. Syncope Evaluation in the Emergency Department Study (SEEDS): a multidisciplinary approach to syncope management. Circulation. 2004;110:3636-3645.
Case 157 Pamela K. Mason
Case Summary A 21-year-old woman is referred to cardiology clinic for evaluation. She was a passenger in a motor vehicle accident, and when she was evaluated in the emergency department, she was incidentally noted to have a loud heart murmur on exam and mild cardiomegaly on a chest radiograph. An ECG was obtained during her clinic visit and is shown in Fig. 157.1. Physical exam was notable for splitting of the first and second heart sounds and a holosystolic murmur at the left sternal border that increased with inspiration. The patient denied any
history of chest pain or shortness of breath. She did report one previous episode of palpitations associated with near syncope. Her family history is notable for a cousin with a “heart rhythm problem.” What further testing does this patient require?
Case Discussion This patient demonstrates Wolff–Parkinson–White syndrome (WPW) on ECG and has a murmur consistent with tricuspid regurgitation. With this combination of findings, Ebstein’s
Fig. 157.1 Baseline 12-lead ECG obtained in clinic
P.K. Mason Department of Internal Medicine, University of Virginia, 800158, Charlottesville, VA 22908, USA e-mail:
[email protected] A. Natale et al. (eds.), Cardiac Electrophysiology, DOI: 10.1007/978-1-84996-390-9_157, © Springer-Verlag London Limited 2011
595
596
anomaly is a likely diagnosis. In Ebstein’s anomaly, the posterior and septal tricuspid leaflets and their chordal attachments are abnormal and displaced downward into the right ventricle. The proximal portion of the right ventricle is “atrialized.” Twenty to thirty-three percent of patients with Ebstein’s anomaly have accessory pathways (APs). Patients with Ebstein’s anomaly may present at any age. Those with marked hemodynamic abnormalities usually present as infants or young children, but, if the valve displacement is not severe, they may first present with arrhythmias. This patient needs to have an echocardiogram to evaluate her tricuspid valve. There is a great deal of variability in the degree of tricuspid valve incompetence among patients with this condition, and other abnormalities, such as atrial septal defects, are common. Some patients require only medical management, while others will need surgical treatment. While this patient has no symptoms of heart failure, the cardiomegaly present on her chest radiograph is concerning. She will also require electrophysiologic evaluation. She has evidence of preexcitation on her ECG and a history
P.K. Mason
of symptomatic palpitations. Most patients with Ebstein’s anomaly and WPW have right-sided or posteroseptal APs and multiple different APs are often found. Associated anatomic abnormalities make it difficult to use standard algorithms to locate the AP based upon the surface ECG. In addition, while WPW is the most common supraventricular arrhythmia found in Ebstein patients, other supraventricular arrhythmias are also more frequently seen than in the general population. Catheter ablation of the right-sided APs may be challenging due to the antatomic abnormalities. Surgical ablation can be considered in patients who require valve surgery.
Bibliography HareVan GF. Radiofrequency ablation of accessory pathways associated with congenital heart disease. Pacing Clin Electrophysiol. 1997;20:2077-2081.
Case 158 Pamela K. Mason
Case Summary A 33-year-old woman presented to her doctor complaining of fatigue and exercise intolerance. Her baseline ECG is shown in Fig. 158.1. She underwent ambulatory ECG monitoring and was found to have occasional episodes of second and third degree heart block scattered throughout the day. An echocardiogram was normal. She was referred for a dual chamber pacemaker. This was implanted without complications and her symptoms improved. However, 3 years later she returned to the emergency department after a witnessed
syncopal spell. Interrogation of her device showed several brief episodes of rapid ventricular rate. She was admitted for further monitoring. A repeat echocardiogram now showed moderate left ventricular dysfunction. She also related that two members of her family had developed cardiac problems. Her 40-year-old brother had recently died suddenly and without explanation. An autopsy revealed only cardiomegaly. Her 31-year-old sister was undergoing an evaluation in another city because of palpitations and frequents premature ventricular beats. The patient was referred for upgrade of her device to a dual chamber defibrillator. Several months after
Fig. 158.1 Baseline 12-lead ECG
P.K. Mason Department of Internal Medicine, University of Virginia, 800158, Charlottesville, VA 22908, USA e-mail:
[email protected] A. Natale et al. (eds.), Cardiac Electrophysiology, DOI: 10.1007/978-1-84996-390-9_158, © Springer-Verlag London Limited 2011
597
598
P.K. Mason
Fig. 158.2 ICD Interrogation after the patient received a shock
this, she again returned after receiving a shock from her defibrillator for ventricular tachycardia (Fig. 158.2). Subsequently, the patient has continued to have worsening left ventricular function, increasing heart failure symptoms, and has received several additional shocks from her defibrillator for ventricular tachycardia despite antiarrhythmic therapy. A second device upgrade with addition of a left ventricular lead was performed with some improvement in her symptoms. What genetic condition would be a likely cause of the syndrome observed in this patient?
arrhythmias. Her brother died suddenly at age 40 with no previous cardiac symptoms or evaluation. Her younger sister has ventricular ectopy but her ejection fraction is still normal. Although the onset of symptomatic disease may be delayed in patients with lamin A/C defects, disease progression can be rapid. Patients with lamin A/C deficiency have a worse prognosis compared to patients with non-inherited forms of cardiomyopathy. These patients are more likely to experience sudden cardiac death, severe congestive heart failure, and require heart transplantaion. Recognizing this condition is important for prognostic reasons, as well as for family counseling. Case Discussion The mechanism by which lamin A/C mutation causes DCM and conduction system disease is unclear. Lamin Approximately one-third of cases of dilated cardiomyopathy A and lamin C are both transcribed from the LMNA gene, (DCM) are familial. Different inheritance patterns have been and both are usually affected by a gene mutation. They are found, including X-linked, mitochondrial, and autosomal filamentous proteins found in the nuclear envelope, and mulrecessive, but the majority of congenital cases are inherited in tiple different mutations have been described. Four different an autosomal dominant manner. The autosomal dominant diseases have been associated with lamin A/C mutations: (1) cases can be separated into those that cause conduction sys- DCM with conduction system disease; (2) limb girdle mustem disease as well as DCM, and those that only cause DCM. cular dystrophy; (3) autosomal dominant variant of Emery– There is genetic heterogeneity among these groups. In patients Dreifuss muscular dystrophy; (4)autosomal dominant partial with pure autosomal dominant DCM, a number of genes have lipodystrophy. While these are generally considered to be been implicated, including actin, desmin, b-sarcoglycan, d- separate entities, elevations in creatinine phosphokinase have sarcoglycan, troponin T, a-tropomyosin, and b-myosin heavy been found in patients with DCM suggestive of skeletal muschain. Four genetic loci have been implicated in cases of cle involvement, and cardiac rhythm abnormalities have been DCM associated with conduction system disease, but the found in patients with muscular dystrophy demonstrating only identified gene, thus far, is lamin A/C, which accounts that these are likely a spectrum of illnesses. for approximately one-third of cases of DCM with conduction system disease. This patient has a history that is strongly suggestive of lamin A/C mutation. Patients with lamin A/C related cardio- Bibliography myopathies usually present with arrhythmias as young adults and then later develop ventricular dysfunction. She fits this Arbustini E, Pilotto A, Repetto A, et al. Autosomal dominant dilated cardiomyopathy with atrioventricular block: a lamin A/C defectpattern in that she presented initially with high-grade conrelated disease. J Am Coll Cardiol. 2002;39:981-990. duction system disease requiring pacing at a point where her Grunig E, Tasman JA, Kucherer H, Franz W, Kubler W, Katusw HA. ventricular function was normal. It was several years later Frequency and phenotypes of familial dilated cardiomyopathy. J Am Coll Cardiol. 1998;31:186-194. that she progressed to a severe DCM, with ventricular
Case 159 Lisa G. Umphrey and John Paul Mounsey
Case Summary
Case Discussion
A 50-year-old female with a past medical history of severe type 1 myotonic muscular dystrophy (MD1), was referred from neuromuscular clinic for evaluation. She was diagnosed with this condition 13 years previously and had experienced progression of her neuromuscular weakness. Most notably she stated that she had bilateral hand weakness, loss of dexterity, difficulty in communication with slurring of speech and lower extremity weakness when standing. She denied syncope or pre-syncope, palpitations, chest pain exertional dyspnea, lower extremity edema, orthopnea or paroxysmal nocturnal dyspnea. She had no family history of sudden cardiac death or premature coronary artery disease. She had a child who also suffered from myotonic muscular dystrophy who passed away at the age of 12 due to the severe juvenile form of the disease. An EKG performed demonstrated first degree AV conduction delay with a PR interval measuring 208 ms and a nonspecific intraventricular conduction delay with a QRS duration of 124 ms. T wave inversions were present in leads III, III, aVF, and V2 through V6 (see Fig. 159.1). Adenosine SESTAMIBI perfusion imaging showed no evidence of ischemia or prior infarction. A transthoracic echocardiogram demonstrated normal LV size and systolic function. Holter monitoring demonstrated previously identified first degree AV conduction delay and nonspecific intraventricular conduction delay. There were >4000 ventricular ectopic beats with some short bursts of nonsustained ventricular tachycardia (see Fig. 159.2a). There was one episode of 2:1 sinus exit block (see Fig. 159.2b). Within 1 year’s time frame she had worsening neuromuscular weakness and evidence of progression of her sinoatrial node dysfunction with a resting pulse of 44 bpm that had dropped from 56 on previous evaluation, and left bundle branch block. What treatment would you recommend?
Myotonic muscular dystrophy also known as Steinert’s disease is the most common muscular dystrophy in Caucasian adults occurring with a prevalence of 35 per 100,000 adults. It is inherited in an autosomal dominant disorder localized to the 3¢-untranslated region of the dystrophica myotonica protein kinase (DMPK) gene on chromosome 19q13.3. The genetic abnormality is a CTG repeat sequence, the length of which has been previously shown to correlate with the extent and severity of disease.1,2 Individuals without disease have 5–30 repeats where individuals with DM have 50–2,000 CTG repeats.3 The disease exhibits anticipation. This means that in successive generations the size of the repeat increases, resulting in earlier onset and severity of clinical manifestations as was seen in this case. Noncardiac manifestations of the disease include myotonia affecting the face and distal limbs, progressive muscle wasting, gonadal atrophy, frontal balding, cataracts, endocrinopathies and cognitive impairment. Cardiac manifestations include sinus node dysfunction, conduction defects, atrial and ventricular tachyarrhythmias, and cardiomyopathy. Patients with larger expansions of CTG repeats have been shown to be at increased risk of intraventricular conduction delay at baseline and rapid progression of conduction defects.4 Conduction abnormalities are common in patients with DM. First degree AV conduction delay is the most common ECG abnormality found in 20–40% of patients. Left anterior fascicular block as presented here, right and left bundle branch block manifesting in a widened QRS complex occur in 5–25% of patients. Prolongation of the HV interval has been demonstrated in half of patients. The progression of conduction system disease is typically slow but can be unpredictable. Thus in this patient population it is recommended to repeat the ECG at least yearly as the progression of conduction defects can progress faster than the neuromuscular manifestations.5 This patient received a pacemaker due to bradycardia and signs of progressive conduction delay. Supraventricular tachyarrhythmia’s consisting of atrial fibrillation and flutter are seen in 25% of patients with DM1
L.G. Umphrey and J.P. Mounsey (*) Division of Cardiology, University of North Carolina, 160 Dental Circle, CB 7075, Chapel Hill, NC 27599, USA e-mail:
[email protected];
[email protected]
A. Natale et al. (eds.), Cardiac Electrophysiology, DOI: 10.1007/978-1-84996-390-9_159, © Springer-Verlag London Limited 2011
599
600
L.G. Umphrey and J.P. Mounsey
Fig 159.1 Baseline 12 Lead ECG demonstrating first-degree and nonspecific intraventricular conduction delay
Fig 159.2 (a) Holter monitor strip demonstrating a short burst of non-sustained ventricular tachycardia. (b) Holter monitor strip demonstrating an episode of 2:1 sinus exit block
Case 159
and have been shown to be inducible on EPS study in the absence of a previously documented arrhythmia.6 Stroke prevention with systemic anticoagulation can be a challenging proposition in this patient population that is predisposed to falls due to neuromuscular weakness and must be evaluated on a case by case basis. Ventricular arrhythmias have also been documented in this patient population. Both sustained and non-sustained VT, as seen here, have been reported. On EPS study of patients referred with conduction abnormalities 18% had inducible VT.7 Monomorphic and polymorphic VT has been reported. Bundle branch reentry ventricular tachycardia has been shown to be one of the mechanisms for sustained monomorphic ventricular tachycardia.8 The conduction defects present in this disease provide the necessary substrate for reentry to occur. A premature ventricular impulse can find one bundle branch refractory but travel in a retrograde fashion up the other bundle to the His and then travel antegrade over the recovered branch to the ventricles creating a macroreentry circuit. Identification of this reentry circuit is essential as it can be cured with radiofrequency ablation in one of the limbs, usually the right bundle of the circuit negating the need for implantable cardiac defibrillator placement. Clinical symptoms of cardiomyopathy, systolic and diastolic occur in approximately 2% of patients. It can be difficult to detect as the neuromuscular disease often limits exertion. Cardiac imaging in 96 patients with the disease demonstrated left ventricular hypertrophy in 19.8%, left ventricular dilation 18.6% and systolic dysfunction in 14%.9 Echocardiographic studies have also shown diastolic dysfunction.10 Recommendations for evaluation and monitoring of DM 1 patients have been previously published.3 A baseline ECG with repeat monitoring every 6–12 months is recommended. Patients with a first-degree conduction delay should be closely monitored for progression of conduction abnormalities. Patients with additional ECG abnormalities or symptoms should undergo EPS. It has been previously recommended that patients with an HV interval of >70 ms should undergo pacemaker placement.11 Individuals with symptoms suggesting arrhythmia such as syncope, near syncope or increasing dyspnea without ECG changes should undergo further evaluation with Holter montoring, SAECG or EPS. If monomorphic VT is reproduced and is
601
consistent with a bundle branch reentrant tachycardia ablation of one bundle branch is curative and implantation of an ICD is not warranted. Symptoms of cardiomyopathy in DM patients can be masked by limited exertion. A low threshold for echocardiographic study should be maintained in this patient population. In conclusion, we present a case of myotonic muscular dystrophy with a classic presentation of sinus node dysfunction and conduction system defects. This patient received a dual chamber pacemaker and did not require an internal cardiac defibrillator due to the absence of syncope, near syncope or sustained VT.
References 1. Jaspert A, Fahsold R, Grehl H, Claus D. Myotonic dystrophy: correlation of clinical symptoms with the size of the CTG trinucleotide repeat. J Neurol. 1995;242:99-104. 2. Groh WJ, Lowe MR, Zipes DP. Severity of cardiac conduction involvement and arrhythmias in myotonic dystrophy type 1 correlates with age and CTG repeat length. J Cardiovasc Electrophysiol. 2002;13:444-448. 3. Sovari AA, Bodine CK, Farokhi F. Cardiovascular manifestations of myotonic dystrophy. Cardiol Rev. 2007;15:191-193. 4. Clarke NR, Kelion AD, Nixon J, Hilton-Rase D, Forfar JC. Does cytosine-thymine-guanine (CTG) expansion size predict cardiac events and electrocardiographic progression in myotonic dystrophy? Heart. 2001;86:411-416. 5. Mammarella A, Paradiso M, Basili S, DeMatteis A, Cardarello LM, et al. Natural history of cardiac involvement in myotonic dystrophy (Steinert’s disease): a 13 year follow-up study. Adv Therapy. 2000;17:238-251. 6. Phillips MF, Harper PS. Cardiac disease in myotonic dystrophy. Cardiovasc Res. 1997;33:13-22. 7. Lazarus A, Varin J, Ounnoughene Z, Radvanyi H, Junien C, et al. Relationships among electrophysiological findings and clinical status, heart function, and extent of DNA mutation in myotonic dystrophy. Circulation. 1999;99:1041-1046. 8. Merino JL, Carmona JR, Fernandez-Lozano I, Peinado R, Basterra N, Sobrino JA. Mechanisms of sustained ventricular tachycardia in myotonic dystrophy. Circulation. 1998;98:541-546. 9. Bhakta D, Lowe MR, Groh WJ. Prevalence of structural cardiac abnormalities in patients with myotonic dystrophy type 1. Am Heart J. 2004;147:224-227. 10. Fragola PV, Luzi M, Calo L, Antonini G, Borzi M, et al. Doppler echocardioghraphic assessment of left ventricular diastolic dysfunction in myotonic dystrophy. Cardiology. 1997;88:498-502. 11. Lazarus A, Varin J, Duboc D. Final results of the French diagnostic pacemaker study in myotonic dystrophy. PACE. 2002;25:599.
Case 160 James A. Reiffel
Case Summary A 57-year-old caucasian male has a history of tobacco abuse (stopped 1 year ago), hyperlipidemia (under good control with atorvastatin and omega-3 fish oil), hypertension (managed with trandolapril and indepamide), and angina (treated with carvedilol, isosorbide, and aspirin). His anginal history dates to the onset of exertional angina walking up hill to work approximately 8 months ago. A stress echocardiogram study suggested single vessel right coronary artery (RCA) disease and his symptoms were managed medically very successfully for 7 months with carvedilol. Over the past 3 weeks his exertional angina increased despite the addition of isosorbide. The patient was admitted to the hospital following one episode of rest pain that awakened him from sleep, associated with new T wave inversions anteriorly without any troponin or CK enzyme rise. Coronary angiography was performed and revealed a long left anterior descending (LAD) stenosis with an irregular filling defect plus an 85% RCA stenosis. Both stenotic regions were heavily calcified. The patient underwent coronary artery bypass surgery with a left internal mammary artery graft to the LAD and a saphenous vein graft to the RCA. Six hours postoperatively, while still intubated, off pressors, frequent atrial ectopy began to appear. Over the next 2 h, the patient awoke fully and was extubated. Carvedilol and aspirin were reinitiated. Four hours later the patient developed atrial fibrillation (AF). His beta blocker dose was adjusted to optimize rate control. He had no complaint except palpitations. Blood pressure remained normal. Arterial blood gases and serum chemistries were unremarkable. A new prominent pericardial friction rub was present on examination. There was no history of AF at any time previously in his history. When the AF persisted 2 h, it was converted with
J.A. Reiffel Department of Medicine, Division of Cardiology, Columbia University, 161 Fort Washington Ave, New York, NY 10032, USA e-mail:
[email protected]
intravenous ibutilide. Figure 160.1 shows the ECG 30 s before conversion; note the prolonged QT interval that developed with ibutilide. The following day, recurrent episodes of paroxysmal AF developed, lasting 20–90 min. Blood pressure was stable and the pericardial rub was softer. What change(s) in therapy should be initiated, if any?
Case Discussion Post operative AF is very common, occurring in more than 30% of untreated patients and multiple agents have been used to suppress AF in the post-op setting.1-6 Beta blockers are very useful in reducing the incidence of AF following cardiac surgery and are the standard of care in many institutions. When they cannot be used due to hemodynamic compromise, amiodarone is commonly given. In some centers amiodarone is the first line prophylactic agent chosen, but it is fraught with so many drug interactions, and, when given IV, with hypotension, that it is usually better considered as a second-line alternative. Also useful is changing the standard beta blocker to sotalol, which, in numerous comparative trials in the post-op setting, has been more effective. Careful attention to electrolyte status, renal function, and QT interval response is needed when sotalol is used. Dofetilide is a much less well studied alternative in this circumstance. Propafenone has also been used effectively when the LV is normal and no residual ischemic risk exists. Importantly, the incidence of AF after cardiac surgery has been reported to be increased in patients who were taking beta blockers and/or ACE-inhibitors prior to surgery when these agents were not restarted post operatively. In addition, AF after cardiac surgery has also been reduced by the administration of statin therapy and by omega-3 fish oil. In this patient, trandolapril, atorvastatin, and omega-3 fish oil were restarted when PAF recurred on post-op day number 2, and his carvedilol was changed to sotalol, 120 mg bid on day 3 after two more episodes. Figure 160.2 shows the time course of his post-op AF and the therapies used. Figure 160.3 reviews this patient’s medications and others in respect to suppressing post-op AF. On this regimen, no further AF occurred. The patient was discharged home 5 days post
A. Natale et al. (eds.), Cardiac Electrophysiology, DOI: 10.1007/978-1-84996-390-9_160, © Springer-Verlag London Limited 2011
603
604
J.A. Reiffel
Fig. 160.1 ECG demonstrating AF, as well as QT prolongation that developed on ibutilide
Fig. 160.2 The time course of this patient’s AF post-op and the therapies employed
operatively, His pericardial rub was gone. Since he had a CHADS2 score of only 1, warfarin was not instituted. At outpatient visits 3 and 6 weeks later, the patient reported no palpitations and his vital signs were normal. ECGs at these visits showed sinus rhythm. Sotalol was then tapered off. Now, 23 months later, no AF has recurred and the patient is doing well, although indepamide was restarted for recurrent mild hypertension. The absence of recurrent AF late after cardiac surgery (more than 2–3 months post-op) is the rule rather than the exception when the AF is only a perioperative phenomenon and was not part of the preoperative history.
Fig. 160.3 A review of this patient’s medications with respect to suppression of postoperative AF as well as other agents that could have been utilized
References 1. Echahidi N, Pibarot P, O’Hara G, Mathieu P. Mechanisms, prevention, and treatment of atrial fibrillation after cardiac surgery. J Am Coll Cardiol. 2008;51(8):793-801. 2. Tselentakis, Woodford E, Chandy J, Gaudette GR, Saltman AE. Inflammation effects on the electrical properties of atrial tissue and inducibility of post-operative atrial fibrillation. J Surg Res. 2006; 135:68-75.
Case 160 3. Fuster V, Ryden LE, Cannom DS, et al. ACC/AHA/ESC 2006 guidelines for the management of patients with atrial fibrillation – executive summary: a report of the American College of Cardiology/ American Heart Association Task Force on Practice Guidelines and the European Society of Cardiology Committee for Practice Guidelines (writing committee to revise the 2001 Guidelines for the Management of Patients with Atrial Fibrillation). J Am Coll Cardiol. 2006;48:854-906. 4. Burgess DC, Kilborn MJ, Keech AC. Interventions for prevention of post-operative atrial fibrillation and its complications after cardiac surgery: a meta-analysis. Eur Heart J. 2006;27:2846-2857.
605 5. Patti G, Chello M, Candura D, et al. Randomized trial of atorvastatin for reduction of post-operative atrial fibrillation in patients undergoing cardiac surgery: results of the ARMYDA-3 (Atorvastatin for Reduction of MYocardial Dysrhythmia After cardiac surgery) study. Circulation. 2006;114:1455-1461. 6. Calo L, Bianconi L, Colivicchi F, et al. N-3 Fatty acids for the prevention of atrial fibrillation after coronary artery bypass surgery: a randomized, controlled trial. J Am Coll Cardiol. 2005;45: 1723-1728.
Case 161 James A. Reiffel
Case Summary A 60-year-old Caucasian male with a background of hypertension, rheumatic mitral insufficiency, and occasional paroxysmal atrial fibrillation (PAF) develops angina and congestive symptoms in 1996 and undergoes triple vessel coronary artery bypass grafting and mitral valve replacement.
MAZE surgery was not performed at that time at his hospital. His angina and congestive symptoms resolve, but his PAF continues. Between 1997 and 2003 he suffers symptomatic AF 6–20 times/year despite trials of metoprolol, verapamil, and sotalol, given by his cardiologist. In addition, a new RBBB is noted on his ECG. Figure 161.1 shows his ECG with sinus rhythm in 2001 prior to RBBB; Fig. 161.2
Fig. 161.1 ECG during sinus rhythm in 2001
J.A. Reiffel Department of Medicine, Division of Cardiology, Columbia University, 161 Fort Washington Ave, New York, NY 10032, USA e-mail:
[email protected] A. Natale et al. (eds.), Cardiac Electrophysiology, DOI: 10.1007/978-1-84996-390-9_161, © Springer-Verlag London Limited 2011
607
608
J.A. Reiffel
Fig. 161.2 ECG during AF in 2003
shows his ECG in 2003 in AF with RBBB. In June 2003 amiodarone (Pacerone) 200 mg/day was begun in addition to the beta blocker being used for his hypertension and over the next 5 months he has 15 more episodes – all with good rate control and less awareness of palpitations. An amiodarone serum level was 0.6. In November 2003, his amiodarone is changed to another amiodarone product, Cordarone, 200 mg/day. The serum concentration rises to 1.0. The dose is increased to 300 mg/day. Between 11/03 and 3/04, only two episodes are detected; however, from 3/04 to 11/06, 31 additional episodes of PAF are noted by the patient – each of which leaves him feeling a little fatigued. He seeks consultation with an electrophysiologist and is offered ablation, but he refuses as a neighbor of his had a stroke during an ablative procedure. Purified omega-3 fish oil is added following reports that it may reduce AF. Between 11/06 and 3/07 the patient suffers 12 more episodes and revisits the electrophysiologist. He is again offered ablation, and again refuses. What else might be tried?
Case Discussion This patient appears to have recurrent, resistant PAF on a background of structural heart disease, in the form of prior rheumatic mitral insufficiency, prior angina, and
hypertension. Postoperative AF in this patient was not just a perioperative phenomenon but rather, it was part of a longer term AF story. Little has been written about antiarrhythmic drug combinations for the suppression of AF (in contrast to frequent drug combinations for ventricular rate control and occasional antiarrhythmic drug combinations for refractory ventricular tachycardia or fibrillation).1-7 However, in this patient, at this point in time, having refused non-pharmacological treatment, it was felt prudent to try to combine another agent with his amiodarone. Amiodarone has significant class III antiarrhythmic drug actions; it also has class IV actions and weak sodium channel blocking and sympatholytic actions. Increasing the effects of these less potent actions may improve antiarrhythmic efficacy. Beta blockers have been known to improve efficacy with amiodarone for years. Adding sodium channel blockade to increase refractoriness by an additional mechanism could serve to further impair macro- or micro-reentrant mechanisms, and sodium channel blockade could also reduce automatic triggers. Over 80% of the patient’s PAF episodes were nocturnal and disopyramide was added. Despite no prior history of symptoms of prostatism, disopyramide had to be discontinued in 2 weeks because of urinary obstruction. It was replaced with sustained release propafenone (Rythmol SR) 225 mg/bid. Over the next 2 months, only one short episode occurred and there were no adverse additional conduction changes on the ECG. However, the patient complained of
Case 161
vivid dreams. The propafenone was changed to flecainide (Tambocor) 100 mg bid. Figure 161.3 shows his ECG on this combination. The PR interval and RBBB have lengthened minimally. On this combination (amiodarone, flecainide,
609
carvedilol, omega-3 fish oil) the patient had one brief episode in 8/07 and none from 9/07 to 4/08 at the time of this report. Figure 161.4 summarizes his events and therapies. This case demonstrates that in resistant AF, antiarrhythmic
Fig. 161.3 ECG during sinus rhythm on amiodarone, flecainide, beta blocker, and fish oil
Fig. 161.4 Plot of average event rates per 3 months along with summary of therapy
610
drug combination therapy may be effective. However, when adding any agent to amiodarone, because of its proclivity for drug interactions with all antiarrhythmic drugs studied to date, the maximal dose should be limited to no more than half of the usual maximal dose.
References 1. Ross DL, Sze DY, Keefe DL, et al. Antiarrhythmic drug combinations in the treatment of ventricular tachycardia. Circulation. 1982;66:1205-1210. 2. Kanna B, Gopalam M. Prospective randomized study comparing amiodarone vs. amiodarone plus losartan vs. amiodarone plus perindopril for the prevention of atrial fibrillation recurrence in patients with lone paroxysmal atrial fibrillation. Eur Heart J. 2007;28:381.
J.A. Reiffel 3. Korantzopoulos P, Kolettis TM, Papathanasiou A, et al. Propafenone added to ibutilide increases conversion rates of persistent atrial fibrillation. Heart. 2006;92:631-634. 4. Wadhani N, Singh BN. Prolongation of repolarization as antifibrillatory action revisited: drug combination therapy in atrial fibrillation. J Cardiovasc Pharmacol Ther. 2005;10:149-152. 5. Fujiki A, Tsuneda T, Sakabe M, et al. Maintenance of sinus rhythm and recovery of atrial mechanical function after cardioversion with bepridil or in combination with aprindine in long-lasting persistent atrial fibrillation. Circulation. 2004;68:834-839. 6. Auer J, Weber T, Berent R, et al. Study of prevention of postoperative atrial fibrillation. A comparison between oral antiarrhythmic drugs in the prevention of atrial fibrillation after cardiac surgery: the pilot study of prevention of postoperative atrial fibrillation (SPPAF), a randomized, placebo-controlled trial. Am Heart J. 2004; 147:636-643. 7. Nagai S, Takeishi Y, Kubota I. Low-dose flecainide infusion followed by oral pilsicainide is highly effective and safe for paroxysmal atrial fibrillation. Cardiovasc Drugs Ther. 2003;17:95-97.
Case 162 Jens Seiler and William G. Stevenson
1. Diagnosis: Arrhythmogenic Right Ventricular Dysplasia A 47-year-old female presented with exercise wide QRS tachycardia at the age of 32 years. Coronary angiography and echocardiography were reportedly normal. An ICD was implanted. Ventricular tachycardia (VT) has become recurrent despite therapy with sotalol. The 12-lead ECG is shown in Fig. 162.1.1. At electrophysiology study, four morphologies of sustained VT were induced (Fig. 162.1.2). A voltage
map of the right ventricle was also obtained (Fig. 162.1.3). Based on these findings what is the most likely diagnosis? The findings suggest scar-related right ventricular tachycardia. Arrhythmogenic right ventricular dysplasia/cardiomyopathy (ARVC), cardiac sarcoidosis, and idiopathic cardiomyopathy with predominant right ventricular involvement are the major diagnostic considerations. Figure 162.1.1 shows the electrocardiogram during sinus rhythm. Several findings that are consistent with ARVC are present. Incomplete right bundle branch block (RBBB),
200 ms I II III aVR aVL aVF V1 V2
Fig. 162.1.1 Sinus rhythm ECG is shown. Sinus bradycardia of 54 bpm, incomplete right bundle branch block, localized QRS prolongation of 110 ms and epsilon waves (arrows) in V1 and V2, delayed S-wave upstroke in V2, and T-wave inversion in V1–V3
V3 V4 V5 V6
J. Seiler and W.G. Stevenson (*) The Arrhythmia Service, Cardiovascular Division, Brigham and Women’s Hospital, Boston, MA e-mail:
[email protected] A. Natale et al. (eds.), Cardiac Electrophysiology, DOI: 10.1007/978-1-84996-390-9_162, © Springer-Verlag London Limited 2011
611
612 Fig. 162.1.2 Surface ECG of four different sustained monomorphic VTs. All VTs have a left bundle branch block configuration, suggesting the origin in the right ventricle: (a) VT #1 cycle length (CL) of 290 ms, inferior QRS axis; (b) VT #2, CL 230 ms, inferior QRS axis; (c) VT #3, CL 300 ms, inferior QRS axis; (d) VT #4, CL 230 ms, superior QRS axis. VTs #1–3 with an inferior QRS axis have their origin in the right ventricular outflow tract; VT #4 has its origin in the base of the right ventricle
J. Seiler and W.G. Stevenson
a
b
c
d
200 ms
I II III aVR aVL aVF V1 V2 V3 V4 V5 V6
10.42 mV
Bipolar 1-SR endo > 102 Points
1.51 mV 0.49 mV 0.09 mV
Fig. 162.1.3 Electroanatomic endocardial voltage map of the right ventricle in sinus rhythm. Purple denotes normal amplitude bipolar electrograms (> 1.5 mV) with progressive decrease in amplitude with progression from blue to green to yellow to red. It is important to exclude poor contact as a cause of low-voltage regions. Gray indicates unexcitable scarred tissue that has a pacing threshold > 10 mA. Blue dots indicate sites with double, fractionated potentials
R
1.48 cm
Case 162
localized widening of the QRS complex of 110 ms and epsilon waves (fractionated terminal QRS deflections) in V1 and V2, delayed S-wave upstroke in V2 and T-wave inversions in V1–V3 are present. Inverted T-waves in leads V1–V3 are seen in more than 80% of patients with ARVC. Localized widening of the QRS duration to >110 ms in V1–V3 is seen in >60% of patients. A prolonged S-wave upstroke of >55 ms in V1–V3 is seen in >90% of patients, and epsilon waves are visible in approximately a third of patients. Figure 162.1.2 shows four different monomorphic VTs. All of these VTs had a dominant S-wave in V1, giving V1 a left bundle branch block (LBBB)-like configuration. This indicates that the exit for the VT is likely to be in the right ventricle (RV) or the septum. VT #1 had an inferior axis raising the possibility of an RV outflow tract location that is common for idiopathic VT. The transition from dominant S- to R-wave in the precordial leads occurs relatively late, however, at V3–V4. In addition, the QRS complexes are somewhat slurred, particularly in the inferior leads (II, III, aVF) that usually show tall peaked complexes in idiopathic RV outflow tract VT. The presence of multiple morphologies of monomorphic VT makes idiopathic VT without structural heart disease, extremely unlikely. Multiple morphologies of VT are common, however, in scar-related VTs. Figure 162.1.3 shows the bipolar right ventricular voltage map. Areas of normal voltage > 1.5 mV are purple. Electrogram amplitude progressively decreases from blue to green to yellow to red. Areas of low amplitude, consistent with scar are present in the free wall of the RV outflow tract, and at the inferior aspect of the RV, extending along the tricuspid annulus as well. These findings are consistent with scar-related RV tachycardia.
613
repolarization abnormalities in V1–V3 in absence of RBBB, late potentials, sustained LBBB type VT, frequent PVCs, and family history of arrhythmias without confirmation of ARVC. This patient has LBBB VT and a sinus rhythm ECG consistent with the diagnosis. It is well recognized, however, that these criteria are imperfect and are not met by some patients with the disease, while other causes of RV cardiomyopathy, notably sarcoidosis, can cause similar findings. This patient did not have a family history of the disease or of sudden death, but this absence does not exclude the diagnosis, as spontaneous mutations occur. Magnetic resonance (MR) imaging may show RV wall motion abnormalities and intramyocardial fat, which by itself is not sufficiently specific for the disease. Areas of delayed gadolinium enhancement, consistent with scar are more specific, particularly if present in the RV. MR imaging was not done in this patient due to presence of an ICD. Cardiac sarcoidosis can mimic the clinical features of ARVC. Absence of evidence of pulmonary sarcoid does not exclude the diagnosis. Endomyocardial biopsy that reveals non-caseating granulomas without other pathogens confirms the diagnosis, but biopsies may miss areas of involvement due to the patchy nature of the disease. Endomyocardial biopsy of the RV septum showed only nonspecific hypertrophy in this patient. Biopsy of the septum often misses abnormalities in ARVC that are more common in the free wall of the ventricles. Cardiac positron emission tomography can be helpful showing areas of FDG uptake indicating inflammation in sarcoidosis. Other causes of RV scar include prior surgery, such as repair of Tetralogy of Fallot, right ventricular infarction, and idiopathic cardiomyopathies. The latter remains a possibility in this patient.
Discussion Arrhythmogenic right ventricular dysplasia is the most likely diagnosis. This is a heterogeneous genetic disease with more than eight causative mutations identified. Most, but not all, mutations involve proteins coding for parts of desmosomes that are involved in cell-to-cell adhesion, including plakophilin, plakoglobin, desmoglein, and desmocollin. It is hypothesized sufficient wall stress to compromise cell adhesion in certain areas, such as along the tricuspid and pulmonary valve annulus leads to cell death and fibrofatty replacement. LV involvement is present in more than 30% of patients and predominates in some patients. The 1994 task force document defining the disease listed major diagnostic criteria as severely abnormal RV wall motion or localized definite aneurysms, epsilon waves or localized delayed depolarization in V1–V3, or confirmed familial disease. Minor criteria included less severe RV wall motion abnormalities,
Bibliography Calkins H. Arrhythmogenic right-ventricular dysplasia/cardiomyopathy. Curr Opin Cardiol. 2006;21:55–63. Sen-Chowdhry S, Syrris P, Ward D, et al. Clinical and genetic characterization of families with arrhythmogenic right ventricular dysplasia/cardiomyopathy provides novel insights into patterns of disease expression. Circulation. 2007;115:1710–1720. Sen-Chowdhry S, Syrris P, McKenna WJ. Role of genetic analysis in the management of patients with arrhythmogenic right ventricular dysplasia/cardiomyopathy. J Am Coll Cardiol. 2007;50: 1813–1821. Tandri H, Macedo R, Calkins H, et al. Role of magnetic resonance imaging in arrhythmogenic right ventricular dysplasia: insights from the North American arrhythmogenic right ventricular dysplasia (ARVD/C) study. Am Heart J. 2008;155:147–153. Corrado D, Basso C, Leoni L, et al. Three-dimensional electroanatomic voltage mapping increases accuracy of diagnosing arrhythmogenic right ventricular cardiomyopathy/dysplasia. Circulation. 2005;111: 3042–3050.
614
J. Seiler and W.G. Stevenson
Koplan BA, Soejima K, Baughman K, et al. Refractory ventricular tachycardia secondary to cardiac sarcoid: electrophysiologic characteristics, mapping, and ablation. Heart Rhythm. 2006;3: 924–929. Marchlinski FE, Zado E, Dixit S, et al. Electroanatomic substrate and outcome of catheter ablative therapy for ventricular tachycardia in setting of right ventricular cardiomyopathy. Circulation. 2004;110: 2293–2298. Tadamura E, Yamamuro M, Kubo S, et al. Images in cardiovascular medicine. Multimodality imaging of cardiac sarcoidosis before and after steroid therapy. Circulation. 2006;113:e771–773.
200 ms I II III aVR aVL aVF
2. Diagnosis: Tetralogy of Fallot Presenting with Wide QRS Tachycardia This 49-year-old male with Tetralogy of Fallot (TOF) repaired 40 years ago including placement of a patch repair of the ventricular septal defect (VSD), presented with hemodynamically stable wide QRS tachycardia that was terminated by cardioversion. The 12-lead ECG during sinus rhythm is shown in Fig. 162.2.1. At electrophysiology study, the tachycardia shown in Fig. 162.2.2 was induced. What is the most likely mechanism of tachycardia? Atrial tachycardias are the most common arrhythmia
V1 V2 V3 V4 V5 V6
Fig. 162.2.2 Surface ECG of monomorphic VT induced with programmed ventricular stimulation (VT #1), CL 300 ms, left bundle branch block morphology, QRS axis +10°, R/S transition V5
800 ms I II III aVR aVL aVF V1 160 ms
encountered late after repair of TOF. Ventricular tachycardia is also an important cause of morbidity and sudden death. During sinus rhythm (Fig. 162.2.1) right bundle branch block (RBBB) is present, as is usually the case after repair of TOF. The tachycardia, however, has a left bundle branch block (LBBB) configuration with a markedly negative deflection in V1, consistent with early depolarization of the RV or septum. Thus, the most likely diagnosis is ventricular tachycardia. This was confirmed at electrophysiology study and catheter ablation attempted (Case 163.2). The QRS morphology with an inferiorly directed frontal plane axis suggests origin in the superior aspect of the RV, potentially involving the outflow tract repair.
V2 V3 V4 V5 V6
Fig. 162.2.1 Baseline 12-lead ECG. The patient presented at baseline in sinus rhythm. Right bundle branch block with QRS width of 160 ms was present
Discussion VT is an important cause of sudden death late after repair of TOF. In a multicenter cohort study of 793 patients with TOF after surgical repair, 33 patients developed sustained VT and 16 died suddenly within a 10-year observation period. In addition to clinical variables (higher age at repair and need for transannular patch repair were predictors of sudden death), QRS duration of 180 ms or more and a faster increase
Case 162
of QRS duration were identified as predictors of VT and sudden death in a multivariate analysis. The most common wide QRS tachycardia encountered, however, is macroreentrant atrial flutter. Since the vast majority of patients have RBBB after repair, atrial arrhythmias present with RBBB. In contrast, because the VT is due to reentry through the RV areas of scar, VT often has an LBBB configuration. Occasionally, VT with an exit toward the septum will have an RBBB-type configuration in V1. ICD implantation is recommended. Programmed electrical stimulation does not reliably predict risk.
615
Bibliography Gatzoulis MA, Balaji S, Webber SA, et al. Risk factors for arrhythmia and sudden cardiac death late after repair of tetralogy of Fallot: a multicentre study. Lancet. 2000;356:975–981. Walsh EP. Interventional electrophysiology in patients with congenital heart disease. Circulation. 2007;115:3224–3234. Zeppenfeld K, Schalij MJ, Bartelings MM, et al. Catheter ablation of ventricular tachycardia after repair of congenital heart disease: electroanatomic identification of the critical right ventricular isthmus. Circulation. 2007;116:2241–2252.
Case 163 Jens Seiler and William G. Stevenson
1. Ablation: Arrhythmogenic Right Ventricular Dysplasia This 47-year-old female (see diagnostic 162.1) with arrhythmogenic right ventricular cardiomyopathy (ARVC) is referred for recurrent episodes of ventricular tachycardia (VT) causing ICD shocks despite therapy with sotalol. In addition to the history noted in Case 162.1, this patient had a prior epicardial ICD placed that became infected and was removed via a median sternotomy several years previously. An ablation procedure had been performed 10 months prior to referral, but VT recurred 3 months later. The electrophysiology study proceeded in steps. After insertion of catheters in the right atrium, His bundle position, and right ventricular (RV) apex, programmed ventricular stimulation was performed, confirming inducible sustained VTs. The initial VT (Fig. 162.1.2 (a), panel a in Case 162.1) had a left bundle branch block (LBBB) configuration and inferiorly directed frontal plane axis, consistent with an origin in the right ventricle. VT was rapid, necessitating termination for further mapping. Repeated programmed stimulation and pacing during VT for attempted termination revealed three more monomorphic VTs, with cycle lengths (CL) ranging from 230 to 300 ms. All VTs had an a LBBB morphology, consistent with an origin in the right ventricle. Three had an axis directed inferiorly, consistent with an origin in the outflow tract region; one had an axis directed superiorly indicating an exit located in the inferior portion of the ventricle. These VTs were not hemodynamically tolerated, allowing only limited mapping during VT. Therefore, sinus rhythm mapping was begun in the RV using an electroanatomic mapping system (Carto™) to create an endocardial voltage map (displaying peak-to-peak bipolar electrogram amplitudes) in the right ventricle. The
J. Seiler and W.G. Stevenson (*) The Arrhythmia Service, Cardiovascular Division, Brigham and Women’s Hospital, Boston, MA e-mail:
[email protected]
voltage map (Fig. 163.1.1, panel a) showed areas of abnormal low-amplitude signals (<1.5 mV), consistent with scar in the free wall of the right ventricular outflow tract (RVOT), and at the base of the right ventricle, consistent with ARVC. Pace mapping at the inferior aspect of the RVOT free wall low-voltage area produced a QRS morphology similar to VT #1, suggesting that this was a possible exit region. The mapping catheter was positioned at this site and programmed stimulation performed to induce VT. An LBBB inferior axis VT was induced that had slightly presystolic activation of – 10 ms at the inferior margin of the RVOT scar. Pacing at that site showed entrainment with QRS fusion and a post-pacing interval that was 30 ms longer than the VT cycle length (Fig. 163.1.2), suggesting that this was in or near an outer loop site, close to the circuit, but not in an isthmus. No earlier sites were identified and pacing at adjacent sites failed to identify endocardial isthmus sites. RF ablation using a saline-irrigated catheter at this region was performed (brown tags in Fig. 163.1.1), but VT remained inducible. Another VT that had a superior frontal plane axis demonstrated diastolic activity and isthmus sites at the inferior aspect of the tricuspid annulus (Fig. 163.1.3). RF lesions from the tricuspid annulus through that region abolished that VT, but the inferior axis VT remained inducible.
What Should Be Done Next? Failure of ablation can be due to inadequate mapping, inability to create an adequate ablation lesion, or inability to reach the arrhythmogenic substrate. Although mapping was limited by hemodynamic intolerance, limited mapping during VT demonstrated that moving away from the region initially identified moved further from the reentry circuit, evident from longer post-pacing intervals and later activation relative to the QRS onset during VT. These observations suggest that the reentry circuit was not endocardial. Although the right ventricle is generally viewed as thin-walled, reentry circuits that cannot be ablated from the endocardium, even with large tip or irrigated electrodes do occur.
A. Natale et al. (eds.), Cardiac Electrophysiology, DOI: 10.1007/978-1-84996-390-9_163, © Springer-Verlag London Limited 2011
617
618
J. Seiler and W.G. Stevenson
a
10.42 mV
Bipolar 1-SR endo > 127 Points
b 11.81 mV
Bipolar 3-SR epi > 100 Points
1.52 mV 0.49 mV
1.52 mV 0.49 mV 0.08 mV
0.09 mV R R
1.71 cm
1.49 cm
Fig. 163.1.1 Electroanatomic endocardial and epicardial voltage map of the right ventricle in sinus rhythm: (a) endocardial voltage map, RAO view, voltage color-coded; (b) epicardial voltage map, AP view, voltage color-coded. Purple denotes electrogram voltage >1.5 mV that
a
is likely to be normal tissue; red <0.49 mV; gray, unexcitable scarred tissue; respectively. Brown dots represent ablation lesions in the RVOT, right ventricular free wall, and right ventricular base. Blue dots indicate sites with double, fractionated potentials
b
200 ms
I II III V1 V5 Abl d Abl m 240 ms
Abl p
290 ms
260 ms
RVa S
S
S
Fig. 163.1.2 Endocardial entrainment and activation mapping of VT #2, CL 260 ms. (a) During entrainment pacing that is performed at a CL of 240 ms from the tip of the ablation catheter (Abl d), the post-pacing interval is 290 ms and minimal QRS fusion is evident, particularly with the change in QRS configuration in V5. (b) The electrograms at this site
are low-amplitude fragmented potentials preceding the QRS complex by 10 ms (arrows) recorded at the distal and middle electrode pairs of the ablation catheter (Abl d and Abl m). The findings are consistent with a pacing location close to an outer loop in the tachycardia circuit. Abl p proximal ablation catheter, RVa right ventricular apex, S stimulus
Case 163 Fig. 163.1.3 Surface ECG and intracardiac recordings during VT #4, CL 230 ms, superior QRS axis. (a) A fractionated diastolic potential preceding the QRS onset by 60 ms (arrow) is clearly visible at the ablation catheter (Abl), at the inferior aspect of the tricuspid annulus. (b) RF energy application at that site converted VT #4 (superior axis) to VT #2 (inferior axis). Note the abrupt shift of the QRS axis with a slight increase of CL from 230 to 250 ms. Abl d distal ablation catheter, Abl m middle ablation catheter, Abl p proximal ablation catheter, RVa right ventricular apex
619
a
b
200 ms
I II
800 ms
I II III
III
V1
aVR aVL aVF
V5 V1 Abl d Abl m
V2 V3 V4
Abl p V5 RVa
Prior cardiac surgery often results in pericardial adhesions that are a barrier to percutaneous epicardial access. In this case, the pericardial space was approached with a micropuncture needle and a few milliliters of contrast injected, which was observed to disperse in the pericardial space. Pericardial access was then successfully obtained with the method described by Sosa and colleagues. Epicardial mapping was possible with the exception of the region of the right ventricle immediately beneath the sternum, caudal to the low-voltage endocardial region. The epicardial voltage map also revealed a free wall lowvoltage region with late potentials (Fig. 163.1.1, panel b) and pace mapping consistent with the inferior axis VTs (Fig. 163.1.4). Following mapping, VTs were no longer reliably inducible, suggesting a mechanical, traumatic effect. Coronary angiography was performed prior to epicardial RF ablation to assure a safe distance between ablation lesions and epicardial vessels (the left anterior descending coronary artery) (Fig. 163.1.5). A series of RF lesions were then placed through the region guided by pace mapping and abnormal fractionated electrograms. Repeat programmed stimulation during isoproterenol infusion induced VT with an inferior axis that was slower than prior VTs and often terminated spontaneously. Additional epicardial and endocardial lesions were placed through this region rendering the pacing threshold at these sites >10 mA. No VTs were then inducible with programmed stimulation during isoproterenol infusion.
V6
Discussion In ARVC, RV regions of scar are characterized by significantly lower amplitudes than in unaffected RV regions. In a study of seven patients, affected RV regions had an endocardial bipolar voltage of 0.5 ± 0.1 mV compared to unaffected regions with a voltage of 4.6 ± 0.2 (mean ± SEM). An endocardial voltage of <1.5 mV for bipolar electrograms is generally accepted to indicate abnormal tissue. It is extremely important to consider the possibility that low amplitude at a site is due to poor catheter contact. The presence of normal fat in the epicardium is also a potential source of low-voltage regions. We would not expect fat to cause fractionated or delayed potentials, or long S-QRS delays during pacing (Fig. 163.1.4) and rely on these observations to help identify abnormal areas. We use assessment of capture during pacing at the site as well as assessment of catheter stability and position relative to the plane of the tissue surface to assess this possibility. These low-voltage areas often extend from the pulmonic or tricuspid annulus involving the RV free wall. VTs originate from these abnormal areas. The approach to mapping and ablation is as for other scar-related VTs. Techniques include activation, entrainment, and pace mapping, depending on hemodynamic stability. In our experience, epicardial mapping and ablation is not uncommonly required. VTs in an ARVC are amenable to RF catheter ablation, but recurrences during long-term follow-up are observed in 19–85% of patients when follow-up exceeds 2 years. In this
620 Fig. 163.1.4 Epicardial pace map at the RVOT low-voltage area. Unipolar pacing (panel a) produces a variable QRS morphology similar to one of the sustained VTs with inferior QRS axis (panel b). The delay from the stimulus (S) to QRS and the changing QRS morphology suggests pacing in an area of abnormal, slow conduction. The local epicardial electrogram at that site in sinus rhythm is abnormal fractionated (arrow, panel c). Abl d distal electrode pair of the ablation catheter
J. Seiler and W.G. Stevenson
a
b
I II III aVR aVL aVF V1 V2 V3 V4 V5 V6 S
c
S
100 ms
V5
Abl d
a
Fig. 163.1.5 Angiography of the left coronary system during epicardial mapping of the RVOT: (a) LAO 60°; (b) RAO 30° projection. The ablation catheter (1) is in the pericardial space overlying the RVOT. The left main coronary artery (2) and left anterior descending artery (3) are sufficiently distant to allow ablation at this site and regions to the right of the ablation catheter
b
200 ms
Case 163
621
patient, acute success was achieved by a combined endo- and epicardial RF ablation approach. After a follow-up of 8 months, there have been no recurrences of VT.
Bibliography Borger van der Burg AE, de Groot NM, van Erven L, et al. Long-term follow-up after radiofrequency catheter ablation of ventricular tachycardia: a successful approach? J Cardiovasc Electrophysiol. 2002;13:417–423. Boulos M, Lashevsky I, Reisner S, et al. Electroanatomic mapping of arrhythmogenic right ventricular dysplasia. J Am Coll Cardiol. 2001;38:2020–2027. Dalal D, Jain R, Tandri H, et al. Long-term efficacy of catheter ablation of ventricular tachycardia in patients with arrhythmogenic right ventricular dysplasia/cardiomyopathy. J Am Coll Cardiol. 2007;50: 432–440. Marchlinski FE, Zado E, Dixit S, et al. Electroanatomic substrate and outcome of catheter ablative therapy for ventricular tachycardia in setting of right ventricular cardiomyopathy. Circulation. 2001;110: 2293–2298. Sosa E, Scanavacca M, d’Avila A. A new technique to perform epicardial mapping in the electrophysiology laboratory. J Cardiovasc Electrophysiol. 1996;7:531–536.
a I II III aVR aVL aVF V1 V2 V3
Fig. 163.2.1 Surface ECG during sinus rhythm and ventricular tachycardia at electrophysiologic study (VT #1). (a) sinus rhythm and right bundle branch block at baseline, (b) VT #1, CL 300 ms, left bundle branch block morphology, QRS axis +10°
V4 V5 V6
2. Ablation: Tetralogy of Fallot Presenting with Ventricular Tachycardia Catheter ablation of VT is attempted in a 49-year-old male patient with Tetralogy of Fallot (TOF) and patch repair of the ventricular septal defect (VSD) (patient discussed in Case 162.2). At electrophysiologic study, sinus rhythm was present with a cycle length (CL) of 760 ms, right bundle branch block with a QRS width of 160 ms, and a slightly prolonged HV interval of 60 ms. Programmed ventricular stimulation easily induced wide QRS tachycardia with a CL of 300 ms, left bundle branch block morphology and a QRS axis of +10°. AV dissociation was present and His bundle deflections were not identifiable during the tachycardia, consistent with VT as the diagnosis (Fig. 163.2.1). The QRS morphology of this tachycardia (VT #1) was different from the tachycardia that had occurred spontaneously, which had a more vertical QRS axis. The tachycardia was hemodynamically tolerated, allowing mapping. Based on the QRS morphology that suggested a superiorly located right ventricular exit site, and the known association of the region between the tricuspid annulus and b
400 ms
622
J. Seiler and W.G. Stevenson
right ventricular outflow tract (RVOT) patch to be involved in these tachycardias, that area was explored first. Activation at the first site in that region was diastolic. Pacing at that site accelerated all electrograms and QRS complexes to the pacing rate with no change in the QRS morphology, consistent with entrainment with concealed fusion. The post-pacing interval (PPI) from the last stimulus to the next activation at the site was 310 ms, consistent with a reentry circuit site. These findings suggest the region between RVOT patch and tricuspid annulus is a protected isthmus site and critical part of the tachycardia circuit (Fig. 163.2.2). Pacing at the septal side of the RVOT accelerated all electrograms to the pacing CL, and there was a change in the QRS morphology consistent with constant fusion. The PPI was 25 ms longer than the tachycardia CL, again consistent with a proximity to the reentry circuit, but in an outer loop rather than an isthmus. VT terminated during catheter manipulation for mapping.
Mapping during sinus rhythm showed low-voltage/scarred areas in the free wall of the outflow tract at the site of the RVOT patch and in the region of the VSD patch. An isthmus between the RVOT patch and tricuspid annulus was discernible. Pace mapping at that isthmus site matched VT #1. Pace mapping at the septal side of the RVOT patch matched the spontaneous VT (Fig. 163.2.3). The findings are consistent with two different VT circuits that share a common isthmus but have different directions and exits (Fig. 163.2.4): one at the RVOT free wall, producing VT #1 that was induced at electrophysiologic study and the other at the septal side, producing the VT that had been documented to occur spontaneously. RF lesions were applied between the RVOT patch and the tricuspid annulus and at the free wall side of the RVOT. Energy was deployed until the targeted regions were unexcitable to pacing stimuli at 10 mA. At the end of the procedure, no VT was inducible with up to three extrastimuli and burst pacing from two sites in the RV.
200 ms
I
II
III
V1
V5 280 ms
Fig. 163.2.2 Entrainment of VT #1, CL 300 ms, from the region between tricuspid annulus and RVOT patch. Stimulation with the mapping and ablation catheter (Abl d) at that site with a CL of 280 ms produces no change in QRS morphology and results in a PPI of 310 ms, indicating this site being within the VT circuit in a protected isthmus. Local activation at that site precedes the onset of the QRS complex by 15 ms. Abl d distal ablation catheter, Abl m middle ablation catheter, Abl p proximal ablation catheter, HRA high right atrium, RVa right ventricular apex, S stimulus
310 ms
Abl d
Abl m
Abl p
HRA
RVa S
S
300 ms
15 ms
Case 163
623
Fig. 163.2.3 Pace mapping. (a) surface ECG during VT #1, (b) ventricular stimulation with a CL of 600 ms at the isthmus between tricuspid annulus and RVOT patch. The paced QRS complexes match that of VT #1 in all 12 leads. (c) Ventricular stimulation with a CL of 600 ms at the septal side of the RVOT produces QRS complexes with left bundle branch block morphology and inferior axis matching the QRS morphology of the spontaneous VT. S denotes stimulus
a
b
c
200 ms
I II III aVR aVL aVF V1 V2 V3 V4 V5 V6 S
S
Discussion
PV
P 2
1 TV
Fig. 163.2.4 Schematic drawing of the right ventricle (RAO view) depicting the presumed tachycardia mechanisms. The RVOT patch (P) and tricuspid valve (TV) annulus create a protected isthmus of slow conduction (five-point star). Pace mapping at that site produces QRS complexes matching VT #1, entrainment mapping at that site reveals a protected isthmus site of the reentry circuit. Pacing mapping at the septal side of the RVOT patch (four-point star) produces QRS complexes matching the clinical VT, entrainment mapping at that site reveals an outer loop site of VT #1. Deployment of ablation lesions at the described isthmus (dotted line) and at the right ventricular free wall rendered the patient non-inducible. The findings are consistent with two VT circuits with different exits and directions, sharing a common isthmus (arrow #1, VT #1; arrow #2, spontaneous VT). PV pulmonary valve
VT after repair of tetralogy of Fallot is usually due to reentry involving areas with scar in the right ventricular outflow tract and/or septum. The potential isthmuses available for reentry are related to the type of surgical repair. The most common isthmus location is between the tricuspid valve annulus and the area of dense scar or patch in the free wall of the RVOT. Other isthmuses can be present between the pulmonary annulus and RV free wall scar, or between the septal patch and tricuspid annulus or free wall scar. Mapping and ablation is done as for other scar-related VTs, using activation mapping and entrainment when possible. When VT is unstable, potential isthmuses for ablation can be identified from voltage maps combined with pace mapping. Ablation is most commonly used to reduce symptomatic VT episodes in patients with ICDs. Ablation has been used either as a primary treatment strategy in selected patients with hemodynamically stable arrhythmias.
Bibliography Walsh EP. Interventional electrophysiology in patients with congenital heart disease. Circulation. 2007;115:3224–3234. Zeppenfeld K, Schalij MJ, Bartelings MM, et al. Catheter ablation of ventricular tachycardia after repair of congenital heart disease: electroanatomic identification of the critical right ventricular isthmus. Circulation. 2007;116:2241–2252.
Case 164 Jens Seiler and William G. Stevenson
Case Summary A 59-year-old male with recurrent monomorphic ventricular tachycardia (VT) was referred for electrophysiologic study. He has an old inferior posterior myocardial infarction and a left ventricular ejection fraction of 25%. Sustained VT had occurred during exercise testing leading to coronary artery bypass surgery and mitral valve repair 7 days ago.
a
Postoperatively, recurrent episodes of monomorphic VT failed to respond to intravenous amiodarone. At electrophysiology study he is in sinus rhythm with firstdegree AV block (PR interval 255 ms) and premature ventricular contractions. The HV interval is prolonged to 75 ms (Fig. 164.1). Programmed ventricular stimulation induced a wide QRS tachycardia with a left bundle branch block (LBBB) configuration (Fig. 164.2). A His bundle deflection
800 ms
I II III
100 ms
b I II III
aVR aVL
Fig. 164.1 Baseline surface ECG and EGM. (a) Surface 12-lead ECG showing sinus rhythm, first-degree AV block (PQ 255 ms) and one PVC with right bundle branch block morphology. (b) Intracardiac readings during sinus rhythm show a significant HV prolongation to 75 ms. HRA denotes high right atrium, HIS d, HIS m, and HIS p distal middle and proximal His catheter, RVa right ventricular apex, respectively
V1
aVF
V5
V1
HRA
V2
HIS d
V3
75 ms
HIS m
V4 HIS p V5 V6
RVa
J. Seiler (*) Department of Cardiology, University Hospital, Freiburgstrasse, Bern 3010, Switzerland e-mail:
[email protected] W.G. Stevenson Cardiovascular Division, Harvard Medical School, Brigham and Women’s Hospital, 75 Francis Street, Boston, MA 02115, USA e-mail:
[email protected] A. Natale et al. (eds.), Cardiac Electrophysiology, DOI: 10.1007/978-1-84996-390-9_164, © Springer-Verlag London Limited 2011
625
626
J. Seiler and W.G. Stevenson 200 ms
100 ms I II
I
III
II
V1
III
V5
V1
HRA
V5 HIS p 360 ms
345 ms
360 ms
345 ms
HRA
RVa
HIS m
Fig. 164.2 Bundle branch reentry tachycardia (VT #1). Oscillations of the His to His intervals precede identical changes in ventricular cycle length, which is consistent with bundle branch reentry tachycardia. Arrowheads indicate His deflections. HRA high right atrium, HIS p His proximal, RVa right ventricular apex, respectively
340 ms
RVa S
is present preceding the QRS onset. What is the mechanism of the tachycardia?
Case Discussion The differential diagnosis of this tachycardia is supraventricular tachycardia with aberrancy, ventricular tachycardia with passive retrograde activation of the His–Purkinje system, bundle branch macroreentry, and participation of a portion of the Purkinje system in a scar-related VT. AV dissociation is present excluding atrial tachycardia or AV reentry using an accessory pathway. AV nodal reentry with VA block or junctional ectopic tachycardia are possible, but are rare. The latter is generally not induced by programmed stimulation. Analysis of oscillations in the H–H and V–V interval indicate that the Purkinje system is closely linked to the tachycardia. As shown in Fig. 164.2, although the His bundle deflection is low amplitude, the decrease in H–H from 360 to 345 ms anticipates the same decrease in V–V interval, supporting bundle branch reentry rather than passive retrograde activation of the Purkinje system during VT. Entrainment of the tachycardia from the right ventricular apex revealed that this pacing site is close to the circuit, which is also consistent with bundle branch reentry tachycardia (Fig. 164.3). The VT cycle length is 360 ms. Pacing at a cycle length of 340 ms changes the QRS morphology, indicating fusion, but the PPI is 380 ms, indicating close proximity to the reentry circuit, as would be expected if the right bundle branch
380 ms
360 ms
S
Fig. 164.3 Entrainment of bundle branch reentry tachycardia (VT #1) from the right ventricular apex. The CL of VT #1 is 360 ms. Entrainment from the right ventricular apex is performed at a cycle length of 340 ms and shows QRS fusion. The post-pacing interval (PPI) of the return cycle is 380 ms. The difference of PPI – VT CL = 20 ms reveals the right ventricular apex is in or near the circuit. QRS fusion indicates pacing in an outer loop type of site rather than a protected isthmus. These findings are consistent with bundle branch reentry tachycardia. HRA denotes high right atrium, HIS m middle pair of electrodes on the His bundle catheter, RVa right ventricular apex, respectively
was part of the circuit, although this finding would also be expected if VT originated from the apical septum or RV. In addition, placement of an ablation catheter along the His bundle and right bundle shows that the right bundle is activated in the anterograde direction (Fig. 164.4), as expected for bundle branch reentry that has a LBBB configuration. Thus, a diagnosis of bundle branch reentry using the left bundle in the retrograde direction and the right bundle in the anterograde direction was made. The right bundle branch was targeted for ablation by placing the ablation catheter along the right bundle, at the location where the RB-V was shorter than the His-V by approximately 15 ms. RF ablation at that site during ongoing bundle branch reentry tachycardia terminated the arrhythmia and produced right bundle branch block (RBBB), (Fig. 164.4). The HV interval increased further to 85 ms. Many patients with bundle branch reentry will have other forms of inducible VT. In this patient, following the initial ablation, VTs #2 and #3 were induced. These had a RBBB configuration with superior or inferior axis (Fig. 164.5). In both, mapping along the left side of the LV septum
Case 164
627
a
200 ms I
I
II
II
200 ms
III
III
V1
V1
V5
V5
HRA
Abl d
Abl d Abl m
Abl m
Abl p
Abl p HIS m
b
HIS d HIS p
15 ms
RVa
RVa
Fig. 164.4 Ablation of bundle branch reentry tachycardia (VT #1). The ablation catheter (Abl) is positioned at the right bundle. Ongoing bundle branch reentry tachycardia (first two QRS complexes) with right bundle branch potentials (arrowheads, visible on the ablation catheter, activation from proximal to distal) are preceded by a His potential (arrow, visible at His middle (HIS m)) by 15 ms. Termination of the tachycardia after 4 s of RF energy delivery. Note the RBBB after termination of the tachycardia and the increase of the PQ interval to 300 ms. The increase of the HV interval to 85 ms is not visible on this tracing. Abl d denotes distal ablation catheter, Abl m middle ablation catheter, Abl p proximal ablation catheter, RVa right ventricular apex, respectively
Fig. 164.6 Interfascicular reentry tachycardia (VT #2) after RF ablation of the right bundle. The ablation catheter (Abl) is positioned at the left fascicle, and catheter position is unchanged during VT (a) and sinus rhythm (b). (a) A sharp high frequency left fascicle potential (arrows) precedes the ventricular activation during VT. (b) A His potential (arrowhead) and left fascicle potential (arrows) are visible during sinus rhythm, indicating the potentials in (a) being true Purkinje/left fascicle potentials. A clearly discernable His potential is only visible during sinus rhythm. Note the similar QRS morphology during VT and sinus rhythm after right bundle ablation. HRA denotes high right atrium, Abl d, Abl m, Abl p distal, middle, and proximal ablation catheter, HIS d and HIS p distal and proximal His catheter, RVa right ventricular apex, respectively
demonstrated Purkinje potentials preceding the QRS. AV dissociation was present, making VT most likely. Furthermore, proximal to distal activation of the left bundle was demonstrable during VT (Fig. 164.6), making passive
retrograde activation of the Purkinje system less likely. These findings are consistent with interfascicular reentry with retrograde activation of the left posterior fascicle and antegrade activation of the left anterior fascicle in VT #2 and vice versa in VT #3. Passive retrograde activation of the left bundle branch from a scar-related LV tachycardia is also possible. Left ventricular RF applications at the inferior septum including portions of the posterior fascicle abolished these VTs and produced intermittent 2:1 infranodal block (Fig. 164.7). Frequent PVCs with early activation in the inferior septum originating from the distal Purkinje system were also ablated. At the end of the procedure, no VT was inducible with programmed ventricular stimulation. The following day VT #3 which had a RBBB superior axis recurred and became incessant. A repeat electrophysiologic study was performed. AV conduction was present in sinus rhythm with prolonged AH and HV intervals, RBBB, and frequent salvos of nonsustained VT. An ablation catheter was advanced into the left ventricle via a retrograde aortic approach and placed in the vicinity of the left anterior fascicle guided by electrogram and anatomical landmarks. Intracardiac electrograms during VT demonstrated left anterior fascicle potentials preceding the His deflection, consistent with retrograde conduction up the left anterior fascicle and antegrade conduction down the left posterior fascicle
a
b
c
200 ms
I II III aVR aVL aVF V1 V2 V3 V4 V5 V6
Fig. 164.5 Three different VTs with observed His–Purkinje potentials preceding ventricular activation. (a) VT #1, bundle branch reentry, cycle length (CL) 365 ms, LBBB morphology, QRS axis +60°, precordial R/S transition V5. (b) VT #2, interfascicular reentry, CL 395 ms, RBBB morphology, QRS axis +120°. (c) VT #3, interfascicular reentry, CL 320 ms, RBBB morphology, QRS axis −100°
628
J. Seiler and W.G. Stevenson 200 ms
400 ms I II III V1 V5 HRA
I II III V1 V5 Abl d Abl m
HIS m
Abl p HIS
Fig. 164.7 Transient infranodal block after ablation of the right bundle and the left posterior fascicle. Arrowheads indicate His deflections. HRA denotes high right atrium, HIS m His middle, RVa right ventricular apex, respectively
Fig. 164.8 Interfascicular reentry tachycardia (VT #3). The surface ECG leads show RBBB morphology and superior QRS axis. The ablation catheter is positioned at the left bundle. This portion of the left bundle is activated from proximal to distal (arrows) and precedes activation of the His bundle (arrowhead). These characteristics are consistent with interfascicular reentry tachycardia with retrograde activation up the left anterior fascicle and antegrade activation down the left posterior fascicle. Abl d denotes distal ablation catheter, Abl m middle ablation catheter, Abl p proximal ablation catheter, HIS His catheter, respectively
(Fig. 164.8). RF energy was applied in the region of the left anterior fascicle and a septal line was created. Because of the persistence of interfascicular reentry beats after left anterior fascicle ablation, RF energy was applied in at the left posterior fascicle position, guided by electrogram and anatomical landmarks. A line of ablation was created in the mid septal region which abolished ectopy. The HV interval prolonged to 128 ms. Following administration of isoproterenol, programmed stimulation induced sustained monomorphic VT with no His–Purkinje potentials preceding ventricular activation and a cycle length of 330 ms that was attributed to scar-related reentry. A line of ablation lesions was created through the low voltage inferior wall scar connecting the mid inferior left ventricle with the septum. During follow-up the patient has been free of VT. Of 234 consecutive patients with structural heart disease referred to our laboratory for VT ablation, involvement of the His–Purkinje system in one or more VTs was observed in 20 patients (8.5%). The frequency of Purkinje system involvement in VT was similar in ischemic and nonischemic heart disease. Bundle branch macroreentry was the most common form, but interfascicular reentry and automaticity in the distal fascicles of the Purkinje system were also
observed. Other scar-related reentrant tachycardias were present in 60% of patients who had VTs involving the Purkinje system. A recent study also emphasizes the potential involvement of the Purkinje system in forming one limb of a reentry circuit involving regions of scar. Previous studies have shown bundle branch reentry as an important cause of sustained monomorphic VT in patients with cardiomyopathy and valvular heart disease. When the Purkinje system is involved in causing VT, evidence of infranodal conduction delay is usually present during sinus rhythm. The VTs often have a typical LBBB, and less frequently RBBB configuration, that can mimic the sinus rhythm QRS when intraventricular conduction delay is present. Distinction from supraventricular tachycardias is usually possible by dissociating atrial activity. AV nodal reentry with persistent AV dissociation is rare. Automatic junctional tachycardias with AV dissociation are not generally paroxysmal or inducible with programmed stimulation. Distinguishing bundle branch and interfascicular reentry from a scar-related VT with passive retrograde activation of the Purkinje system relies on demonstrating linking of the Purkinje system to the VT circuit and appropriate activation of the bundle branches for the QRS morphology observed.
RVa
Case 164
Bibliography Bogun F, Good E, Reich S, et al. Role of Purkinje fibers in post-infarction ventricular tachycardia. J Am Coll Cardiol. 2006;48:2500-2507. Caceres J, Jazayeri M, McKinnie J, et al. Sustained bundle branch reentry as a mechanism of clinical tachycardia. Circulation. 1989;79:256-270.
629 Eckart RE, Hruczkowski TW, Tedrow UB. et al. Sustained ventricular tachycardia associated with corrective valve surgery. Circulation. 2007;116:2005-2011. Lopera G, Stevenson WG, Soejima K, et al. Identification and ablation of three types of ventricular tachycardia involving the His-Purkinje system in patients with heart disease. J Cadiovasc Electrophysiol. 2004;15:52-58.
Case 165 J. Jason West and John Paul Mounsey
Case Summary An 18-year-old man presented to the electrophysiology clinic for evaluation of an abnormal electrocardiogram. He was a freshman participating in the varsity wrestling program at a major university. He denied ever experiencing palpitations, syncope/pre-syncope, chest pain, or lightheadedness. As per the standard screening protocol of athletes at this institution, a pre-participation electrocardiogram had been obtained upon his arrival at school, and demonstrated a short PR interval and a delta wave (see Fig. 165.1). His work-up included a normal echocardiogram without evidence of Ebstein’s anomaly. He was referred for electrophysiologic study for risk stratification of his accessory pathway and possible catheter ablation. During the course of his preoperative work-up, he was noted to have lost preexcitation on his electrocardiogram, a finding thought to suggest a low risk for sudden death (see Fig. 165.2). At the time of electrophysiologic study under conscious sedation, he was noted to have regained preexcitation. Baseline electrophysiologic intervals revealed PR = 100, QRS = 116, QT = 302, AH = 83, HV = –3 ms. With atrial pacing, he demonstrated further preexcitation with earliest ventricular activation consistent with a left lateral pathway. The pathway lost antegrade conduction with atrial pacing at a cycle length of 520 ms, and retrograde conduction with ventricular pacing at a cycle length of 500 ms. With the
J.J. West Bend Memorial Clinic, 1501 N.E Medical Center Drive, Bend, OR 97701, e-mail:
[email protected] J.P. Mounsey (*) Department of Cardiology, University of North Carolina, 160 Dental Circle, CB 7075, Chapel Hill, NC 27599, USA e-mail:
[email protected]
infusion of isoproterenol at 1 mcg/m2/min, the characteristics of the accessory pathway changed significantly. The accessory pathway then supported 1:1 antegrade conduction during atrial pacing to 230 ms (see Fig. 165.3). Retrograde conduction properties also improved. Atrial fibrillation was not induced but short runs of AV reentrant tachycardia could be initiated with atrial stimulation. What would you do now?
Case Discussion In this young man who participates regularly in vigorous athletics, we were concerned by the change in the properties of the accessory pathway with isoproterenol and decided to proceed with ablation. Using a transseptal approach, radiofrequency energy was applied to the lateral wall at the site of continuous atrial and ventricular signals. Pathway conduction was terminated within two beats (see Fig. 165.4). Asymptomatic ventricular preexcitation is an uncommon yet vexing finding in competitive athletes. In the general adult population, it is generally accepted that the risk of sudden death in a patient with a previously asymptomatic accessory pathway is quite low. However, athletes are potentially at higher risk for sudden death due to their propensity for higher adrenergic states and a higher incidence of atrial fibrillation. To date, there have been no major trials examining this issue in athletes. We must, therefore, extrapolate from the literature focused upon the general population. Multiple population-based studies have shown a low risk of arrhythmia in individuals with asymptomatic preexcitation, and an even lower risk of sudden death.3,5 These risks are potentially underestimated by the older age of many individuals in these cohorts. Studies involving younger subjects suggest a somewhat higher risk.6,7 Athletes have two features that potentially increase their risk from bypass pathways relative to the general population. First, athletes exhibit a high resting vagal tone. This vagal influence results in an increased incidence of atrial
A. Natale et al. (eds.), Cardiac Electrophysiology, DOI: 10.1007/978-1-84996-390-9_165, © Springer-Verlag London Limited 2011
631
632
J.J. West and J.P. Mounsey
Fig. 165.1 Baseline 12-lead electrocardiogram demonstrating a short PR interval and delta wave consistent with a left lateral accessory pathway
Fig. 165.2 Repeat 12-lead electrocardiogram obtained on hospital admission demonstrating the absence of ventricular preexcitation
Case 165
Fig. 165.3 Intracardiac electrograms demonstrating rapid atrial pacing during infusion of isoproterenol resulting in maximal ventricular preexcitation. The earliest ventricular signal is in lead CSd consistent with a left lateral pathway. The pathway blocks at a pacing rate of 230 ms
fibrillation.4 Rapid antegrade conduction during atrial fibrillation is hypothesized as the mediator for sudden death in individuals with an accessory pathway. Second, despite this relatively high resting vagal tone, athletes are often producing high-adrenergic states during the course of training and competition. This adrenergic surge can alter the conduction of an accessory pathway, rendering an otherwise low-risk pathway into a more dangerous one.2 While the specific role of isoproterenol in the evaluation of accessory pathways is controversial, the characteristics of the adrenergically stimulated pathway are potentially much more in line with what athletes regularly experience. These risks must then be balanced by the risks associated with evaluation and treatment of the accessory pathway. While noninvasive testing probably still has some role in the identification of low-risk accessory pathways, the electrophysiologic study has become the gold standard in the evaluation of this condition. The safety of these studies and radiofrequency catheter ablation for accessory pathways has been repeatedly supported in large-scale ablation registries.1,8 Thus, the identification of athletes with asymptomatic preexcitation should prompt further work-up and potential curative therapy.
633
Fig. 165.4 Intracardiac electrograms demonstrating radiofrequency ablation of the left lateral pathway. Within 2 s, the pathway is eliminated and ventricular preexcitation resolves
References 1. Calkins H, Yong P, Miller JM, et al. Catheter ablation of accessory pathways, atrioventricular nodal reentrant tachycardia, and the atrioventricular junction: final results of a prospective, multicenter clinical trial. The Atakr Multicenter Investigators Group. Circulation. 1999;99:262-270. 2. German LD, Gallagher JJ, Broughton A, Guarnieri T, Trantham JL. Effects of exercise and isoproterenol during atrial fibrillation in patients with Wolff-Parkinson-White syndrome. Am J Cardiol. 1983;51:1203-1206. 3. Goudevenos JA, Katsouras CS, Graekas G, Argiri O, Giogiakas V, Sideris DA. Ventricular pre-excitation in the general population: a study on the mode of presentation and clinical course. Heart. 2000;83:29-34. 4. Mont L, Sambola A, Brugada J, et al. Long-lasting sport practice and lone atrial fibrillation. Eur Heart J. 2002;23:477-482. 5. Munger TM, Packer DL, Hammill SC, et al. A population study of the natural history of Wolff-Parkinson-White syndrome in Olmsted County, Minnesota, 1953-1989. Circulation. 1993;87:866-873. 6. Pappone C, Manguso F, Santinelli R, et al. Radiofrequency ablation in children with asymptomatic Wolff-Parkinson-White syndrome. N Engl J Med. 2004;351:1197-1205. 7. Sarubbi B, Scognamiglio G, Limongelli G, et al. Asymptomatic ventricular pre-excitation in children and adolescents: a 15 year follow up study. Heart. 2003;89:215-217. 8. Scheinman MM, Huang S. The 1998 NASPE prospective catheter ablation registry. Pacing Clin Electrophysiol. 2000;23:1020-1028.
Case 166 Darren Traub, James P. Daubert, and Spencer Rosero
Case Summary A 6-ft 3-in., 16-year-old male who was a high-school varsity soccer and basketball player was referred to pediatric cardiology to rule out Marfan’s syndrome based on his tall slim body habitus and occasional joint aches. The patient denied any history of palpitations, shortness of breath, or chest discomfort with exertion. He did mention that occasionally he became slightly lightheaded or dizzy at the end of a game, but would always feel better after a few minutes of rest. These occasional episodes of lightheadedness had not caused him or his family enough concern to seek medical attention. The patient’s family history was unremarkable for sudden cardiac death, with the exception of his maternal greatgrandmother who had a “seizure disorder” and died at the age of 36 of unknown causes. Physical exam was normal. An echocardiogram showed no structural heart disease and no anomalous coronary origin. The patient’s electrocardiogram (ECG) is shown below (Fig. 166.1). What is the likely diagnosis and what is the best management option for this patient?
Case Discussion This ECG demonstrates sinus bradycardia at a rate of 51 beats per minute. The T waves are broad and prolonged. The QT interval in lead II measures 560 ms. The QT interval
D. Traub (*) St. Luke’s Hospital and Health Network, 801 Ostrum Street, Bethlehem, PA 18015, USA e-mail:
[email protected] J.P. Daubert and S. Rosero Cardiac Electrophysiology, Cardiology Division, Duke University Health System, DUMC Box 3174, Duke Hospital 7451H, Durham, NC 27710, USA e-mail:
[email protected]
corrected for heart rate measures 520 ms.1 Multiple other ECGs from this patient demonstrated a prolonged QTc greater than 500 ms. The patient was started on atenolol 50 mg daily and referred to the electrophysiology service for evaluation. During outpatient clinical evaluation, the patient’s symptoms of lightheadedness were reproduced with jumping jacks. Telemetry monitoring while performing jumping jacks is displayed in Fig. 166.2. The underlying rhythm is sinus with ventricular bigeminy and a short burst of polymorphic ventricular tachycardia. Short bursts of polymorphic ventricular tachycardia occurred multiple times while performing exercise commensurate with the patient’s lightheadedness. He was admitted to the hospital for further testing and possible placement of an implantable cardioverter defibrillator. The evening of his admission he was awakened by a CCU resident. (Fig. 166.3) demonstrates telemetry monitoring when he was startled by this unexpected interruption of his sleep. This patient’s clinical presentation and electrocardiogram are consistent with the long-QT syndrome (LQTS). In the Pediatric Electrophysiology Society’s International Registry of 287 patients with the long QT syndrome, 61% of patients were symptomatic at presentation. Serious symptoms were present in 45% of patients including cardiac arrest (9%), syncope (26%), and seizures (10%). Six percent of patients presented with pre-syncope or palpitations. Among the symptomatic patients, 67% had symptoms related to exercise, 18% with exercise and emotion, and 7% with emotion alone.2 LQTS is thought to affect approximately 1 in 2,500 people and be responsible for up to 2,000–3,000 sudden deaths in children to young adults each year.3,4 Currently, more than 150 different mutations have been identified in ten genes linked to the long-QT syndrome. LQT1, LQT2, and LQT3 account for 95% of the identified mutations.4-7 An abnormal electrocardiogram demonstrating a prolonged QT interval corrected for heart rate (QTc) is the key to establishing the diagnosis. The QTc is prolonged when it is greater than 440 ms for men and 460 ms for women and children.5,6 The QTc interval can be variable, necessitating frequent
A. Natale et al. (eds.), Cardiac Electrophysiology, DOI: 10.1007/978-1-84996-390-9_166, © Springer-Verlag London Limited 2011
635
636
D. Traub et al.
Fig. 166.1 Baseline ECG showing sinus bradycardia at 51 beats per minute, a QT interval of 640 ms, and a QTc of 592 ms
Fig. 166.2 Telemetry monitoring while performing jumping jacks. The rhythm is sinus with ventricular bigeminy and short burst of polymorphic ventricular tachycardia
Case 166
637
Fig. 166.3 Telemetry monitoring when patient was startled by abrupt awakening
follow-up ECGs when long-QT syndrome is suspected but not confirmed during initial evaluation or an ECG demonstrates borderline prolongation.5 Exercise testing is frequently performed in the evaluation of patients with suspected long QT syndrome. The exercise test is particularly useful if it precipitates a diagnostic arrhythmia as occurred in our patient or if the QTc interval becomes excessively prolonged during the recovery phase.5–9 As the heart rate increases during exercise, the normal response of the QT interval is to shorten. In patients with long QT syndrome, the QT interval may fail to shorten or may paradoxically prolong with exercise or during the recovery phase. Exercise testing does have limitations. The QTc may be difficult to measure accurately during exercise due to motion artifact and fusion of the T and P waves.8,9 Variations in the response of different types of ion channel defects also limit the diagnostic utility of exercise testing for establishing a LQTS diagnosis. LQT1 in particular shows a failure of the QTC to shorten with exercise, while in LQT2 and LQT3 the response of the QTc to exercise is normal and supranormal, respectively.6,9 Thus exercise testing may give a suggestion to the LQTS subtype. Once the diagnosis of LQTS is established in a patient, ECGs should be obtained on all first-degree family members to determine whether others are affected. Unexplained sudden death in a young individual should also prompt an evaluation for the presence of LQTS in the family. Occasionally an asymptomatic individual may be noted to have QTc prolongation based on an ECG obtained for another reason. While the absence of symptoms does not rule out familial LQTS, a careful medication history is mandated.5,6 Genetic testing can identify a mutation in up to 75% of probands in whom the diagnosis of LQTS is relatively certain based on ECG and clinical presentation. Establishing the particular genetic mutation of a proband is useful for ruling out the diagnosis in family members. Additionally, the prognosis of LQTS patients and response to medical therapy appear to be gene specific.4-7,10,11 The QTc duration is a powerful predictor of risk for syncope, cardiac arrest, or sudden death in patients with LQTS.6,10,11 A QTc >500 ms has been associated with increased risk for cardiac events in two separate analysis of
patients with mutation confirmed LQTS.10,11 Female gender, LQT2 channel mutations, and a history of a syncopal event have also been associated with increased risk for aborted cardiac arrest or sudden cardiac death. Beta-blockers provide an approximately 60% reduction in the risk cardiac events and life threatening cardiac events.11 Patients with LQT1 obtain the greatest benefit from beta-blockade, with less of a therapeutic response in patients with LQT2 and unproven benefit among those with LQT3. Implantation of an ICD should be considered among those patients considered to be at high risk for sudden cardiac death including survivors of cardiac arrest, patients with syncope while receiving beta-blockers, and possibly those with an extremely prolonged QT interval.6,12 Because our patient demonstrated high-risk features consisting of a QTc interval of over 500 ms and symptoms with demonstrated polymorphic VT despite beta-blocker therapy, an ICD was placed. Screening electrocardiograms of the patient’s family revealed that his mother and one of his two sisters had QTc intervals of approximately 460–465 ms. Genetic testing is still in progress.
References 1. Bazett HC. An analysis of the time relations of electrocardiograms. Heart. 1918;7:353-370. 2. Garson A Jr, Dick M II, Fournier A, et al. The long QT syndrome in children: an international study of 287 patients. Circulation. 1993;87:1866-1872. 3. Etheridge SP, Sanatani S, Cohen MI, et al. Long QT Syndrome in Children in the Era of Implantable Defibrillators. J Am Coll Cardiol. 2007;50:1335-1340. 4. Vincent MG. The long QT syndrome. Indian Pacing Electrophysiol J. 2002;2(4):127. 5. Moss AJ. Long QT syndrome. JAMA. 2003;289:2041-2044. 6. Roden DM. Long-QT syndrome. N Engl J Med. 2008;358:169-176. 7. Morita H, Wu J, Zipes DP. The QT syndromes: long and short. Lancet. 2008;372(9640):750-763. 8. Dillenburg RF, Hamilton RM. Is exercise testing useful in identifying congenital long QT syndrome? Am J Cardiol. 2002;89:233-236. 9. Connuck DM. The role of exercise stress testing in pediatric patients with heart disease. Progr Pediatr Cardiol. 2005;20:45-52. 10. Priori SG, Schwartz PJ, Napolitano C, et al. Risk stratification in the long-QT syndrome. N Engl J Med. 2003;348:1866-1874.
638 11. Sauer AJ, Moss AJ, McNitt S, et al. Long QT syndrome in adults. J Am Coll Cardiol. 2007;49:327-337. 12. Zipes DP, Camm AJ, Borggrefe M, et al. ACC/AHA/ESC 2006 guidelines for management of patients with ventricular arrhythmias and the prevention of sudden cardiac death: a report of the American College of Cardiology/American Heart Association Task Force and
D. Traub et al. the European Society of Cardiology Committee for Practice Guidelines (writing committee to develop Guidelines for Management of Patients with Ventricular Arrhythmias and the Prevention of Sudden Cardiac Death): developed in collaboration with the European Heart Rhythm Association and the Heart Rhythm Society. Circulation. 2006;114:e385-e484.
Case 167 Vikas P. Kuriachan and George D. Veenhuyzen
Case Summary A 12-year-old girl with Wolff–Parkinson–White Syndrome underwent catheter ablation. (Fig. 167.1) shows programmed atrial stimulation from the proximal electrode pair of a multipolar catheter, approximately 2 cm inside the coronary sinus (CS). The last two beats of the pacing train (A1) are preexcited (short PR interval, delta wave with QRS widening, no visible His potential). The extra stimulus (A2) blocks in the accessory pathway (AP) and conducts to the ventricles exclusively over the normal AV conduction system (normal PR
I
interval, QRS duration, and HV interval), initiating a supraventricular tachycardia (SVT) with a 1:1 AV and HV relationship. What structures are responsible for this girl’s tachycardia?
Case Discussion At first glance, this resembles a usual induction of orthodromic atrioventricular reciprocating tachycardia (AVRT) by programmed atrial stimulation: The premature beat blocks in the AP and while traveling to the ventricle via the normal AV
S1: 500 S2: 310
200 ms
II aVF aVL V1 V3 V6 HRA HIS CS proximal
Fig. 167.1 Initiation of tachycardia with a single atrial extrastimulus. (A1A2 = 500/310 ms)
CS mid CS distal RV
V.P. Kuriachan (*) Department of Cardiac Electrophysiology, University of Calgary, Foothills Medical Centre, 1403-29th St. NW, Calgary, Alberta T2N 2T9, Canada e-mail:
[email protected] G.D. Veenhuyzen Libin Cardiovascular Institute of Alberta, University of Calgary, Foothills Hospital, 1403-29th Street NW, Calgary, Alberta T2N 2T9, Canada e-mail:
[email protected] A. Natale et al. (eds.), Cardiac Electrophysiology, DOI: 10.1007/978-1-84996-390-9_167, © Springer-Verlag London Limited 2011
639
640
V.P. Kuriachan and G.D. Veenhuyzen
conduction system, there is sufficient AV delay for the AP to recover and conduct the impulse retrogradely, inducing circus movement tachycardia. Indeed, a His-synchronous ventricular premature beat terminated this tachycardia without conduction to the atrium, proving that it was AVRT. The earliest atrial electrogram during the SVT was recorded by the distal CS electrode pair, identifying the location of the operative AP as along the free wall of the mitral annulus. However, the delta wave vector based on the first few preexcited beats is consistent with a right sided AP. Indeed, the earliest ventricular electrogram during ventricular preexcitation is recorded by the electrode at the His bundle region. Could these findings could be explained by a single AP? While the earliest atrial electrogram in AVRT is recorded at the distal CS, the ventricular electrogram recorded by this electrode pair during atrial pacing with ventricular preexcitation is inscribed at the end of those QRS complexes, indicating that the left free wall AP could not possibly have meaningfully contributed to ventricular preexcitation: There must be two AP’s.
V1
200 ms
S1: 500 S2: 320
V6 HRA HIS
C
CS proximal CS mid
Fig. 167.2 Fusion of retrograde conduction over different pathways after a ventricular extrastimulus (V1V2 = 500/320 ms)
CS distal RV
Figure 167.2 shows programmed ventricular stimulation. At a coupling interval of 320 ms, the retrograde atrial activation sequence is crescentic (with collision at the mid CS electrode pair) and appears non-decremental. This is consistent with retrograde conduction over both the right- and left-sided AP’s (a component of conduction also via the normal AV conduction system cannot be excluded). At a coupling interval of 310 ms, retrograde conduction over the right sided AP blocks, revealing conduction over the left free wall AP (Fig. 167.3). The induction of orthodromic AVRT employing the left free wall AP was facilitated by programmed atrial stimulation from the left atrium, and induction of orthodromic AVRT was facilitated by programmed atrial stimulation from the right atrium. Multiple accessory pathways have been reported in 5–20% of patients with preexcitation.1 This case demonstrates the importance of matching the delta wave vector during preexcitation with the earliest atrial electrogram during AVRT, and studying atrial activation during programmed ventricular stimulation carefully.
Case 167 Fig. 167.3 Change in atrial activation sequence in the coronary sinus with a shorter interval (V1V2 = 500/310 ms)
641
V1
200 ms
S1: 500 S2: 310
V6 HRA HIS
C
CS proximal CS mid CS distal RV
Reference 1. Zipes DP, Jaliffe J. Cardiac Electrophysiology: From Cell to Bedside. 4th ed. Philadelphia, PA: Saunders; 2004:870.
Case 168 Vikas P. Kuriachan and George D. Veenhuyzen
Case Summary A previously healthy 40-year-old male called paramedics after experiencing palpitations, weakness, and presyncope. In the emergency room, the 12-lead ECG shown in Fig. 168.1 was recorded. What rhythm is shown on the ECG? What is the best approach for acute therapy?
Case Discussion This wide complex, irregularly irregular tachycardia has an average ventricular rate of 250 bpm, with R–R intervals varying from 160 to 360 ms. Despite the marked beat to beat variation in rate, the QRS complex morphologies are all similar (but not the same). The irregularity should prompt consideration of atrial fibrillation (AF) as the underlying rhythm. Atrioventricular (AV) conduction intervals <250 ms are usually not possible through the normal AV conduction system, and should prompt consideration of AV conduction over an accessory AV pathway (AP). The QRS complexes result from variable fusion of conduction over the normal AV conduction system and the AP, resulting in subtle beat to beat variation in QRS complex morphology. The dominant feature of the QRS complexes is that they are wide, slurred, and bizarre, indicating that AV
V.P. Kuriachan (*) Department of Cardiac Electrophysiology, University of Calgary, Foothills Medical Centre, 1403-29 St. NW, Calgary, Alberta T2N 2T9, Canada e-mail:
[email protected] G.D. Veenhuyzen Libin Cardiovascular Institute of Alberta, University of Calgary, Foothills Hospital, 1403-29th Street NW, Calgary, Alberta T2N 2T9, Canada e-mail:
[email protected]
conduction is occurring mostly over the AP. Thus, this patient’s ventricular response to AF is determined by the electrophysiologic properties of his AP, not those of his normal AV conduction system. In Fig. 168.2, a flurry of extremely rapid preexcited beats (*) results in repeated ventricular stimulation at very short cycle lengths (CL) and is followed by the development of ventricular fibrillation (VF; cycle length = 100 ms). This is analogous to performing programmed ventricular stimulation with multiple extrastimuli at very short coupling intervals, which is capable of inducing VF in healthy hearts. Thus, Fig. 168.2 illustrates the mechanism by which Wolff–Parkinson–White (WPW) syndrome is associated with a small risk of sudden death. Two “hits” are required: (1) the development of an extremely rapid atrial tachyarrhythmia (usually AF) and (2) an AP with a very short antegrade refractory period. Therapy should be directed accordingly: (1) cardioversion and (2) slowing the ventricular response by prolonging the refractory period of the AP. It should be no surprise that agents that only slow conduction over the normal AV conduction system can precipitate sudden death (and are absolutely contraindicated) since they do nothing to help the patient (and thereby allow a medical emergency to continue unabated) and may cause hypotension or myocardial depression. While which of the two therapies described above should be applied is often debated, we advocate a simple approach, particularly in patients with R–R intervals <250 ms (a risk factor for sudden death in individuals with WPW syndrome), which is to perform both as quickly as possible. Because electrical cardioversion requires sedation and other preparation that can be time consuming, it is logical to concurrently and quickly provide an intravenous antiarrhythmic agent that will slow the ventricular response in the interim. Current guidelines recommend the use of flecainide, ibutilide, and procainamide.1 This may improve hemodynamics and facilitate the administration of adequate doses of sedation. Furthermore, the drug could result in pharmacologic cardioversion and/or help to prevent early recurrences of AF after electrical cardioversion.
A. Natale et al. (eds.), Cardiac Electrophysiology, DOI: 10.1007/978-1-84996-390-9_168, © Springer-Verlag London Limited 2011
643
644
V.P. Kuriachan and G.D. Veenhuyzen
Fig. 168.1 Tachycardia on presentation to the emergency room
Fig. 168.2 Degeneration of preexcited atrial fibrillation to ventricular fibrillation in another patient
Figure 168.3 shows a 12-lead ECG recorded after electrical cardioversion. The features of ventricular preexcitation are subtle (indicating that in sinus rhythm, ventricular activation is mostly over the normal AV conduction system) and become more clear after comparison with the ECG recorded after successful catheter ablation of a left free wall AP
(Fig. 168.4). These features include: (1) a short PR interval (120 ms in lead V5), (2) a delta wave (best appreciated in lead V5), and (3) an early R/S precordial transition in lead V2. Because the clues to the presence of a potentially life threatening AP are so subtle in sinus rhythm, this truly is “a Wolff in sheep’s clothing”!
Case 168
Fig. 168.3 ECG post-cardioversion: a “Wolff in sheep’s clothing.” Delta waves are visible but are quite subtle
Fig. 168.4 ECG post-catheter ablation of the left-sided accessory pathway. Note the absence of delta waves now in V1–V3
645
646
Reference 1. Blomstrom-Lundqvist C, Scheinman MM, Aliot EM, et al. ACC/ AHA/ESC Guidelines for the Management of Patients with Supraventricular Arrhythmias. A Report of the American College of
V.P. Kuriachan and G.D. Veenhuyzen Cardiology/American Heart Association Task Force and the European Society of Cardiology Committee for Practice Guidelines. Bethesda, MD: American College of Cardiology Foundation; 2003.
Case 169 Jeffrey D. Booker and George D. Veenhuyzen
Case Summary
Case Discussion
A 25-year-old female runner who had recently started training for a competitive marathon was admitted to hospital after complaining of recurrent episodes consisting of the sudden onset of extreme fatigue, lightheadedness, and transient loss of vision lasting a few seconds. As a result of these symptoms, she stopped training, and her symptoms abated. Upon this improvement, she began training again, only to have her symptoms return. She kept a training diary that revealed a clear repeating pattern over the weeks preceding her admission to hospital: intense running for several days followed by several days where she was unable to train at all because of the above mentioned symptoms. This was followed by improvement and several more days of training. Her symptoms were most pronounced at rest and were improved by mild activity (like walking), which led her to try exercising again. With continued exercise, her symptom frequency progressed to daily episodes that occasionally included recurrent night sweats and one episode of nocturnal enuresis. She decided to stop driving her car. Twenty-four hour ambulatory ECG monitoring during symptoms and during waking hours (Fig. 169.1) revealed high grade atrioventricular (AV) block with ventricular asystole up to 4 s in duration. In Panel B, lengthening of the PR interval prior to the first nonconducted P-wave (AV Wenckebach) is most consistent with block at the level of the AV node. Physical examination was unremarkable. Her resting heart rate was 50 bpm. A 12-lead ECG showed only sinus bradycardia. An echocardiogram was normal. What would you recommend now?
Since her symptoms seemed to be progressing rapidly, she was admitted to the hospital for observation. While undergoing continuous telemetry monitoring in hospital, initial intermittent 2:1 AV block and AV Wenckebach resolved over several days of observation. She completed 24 METS on an accelerated Bruce protocol with no evidence of AV block and appropriate shortening of her PR and QT intervals. She achieved a heart rate of 181 beats per minute with a physiologic blood pressure rise. No AV block or dysrhythmias were observed. Cardiac MRI revealed no evidence of myocarditis or regional fibrosis that may involve the conduction system. Lyme disease serology was negative. The final diagnosis was inappropriately high resting vagal tone secondary to high intensity physical training resulting in symptomatic AV block. She was discharged home with instructions to discontinue high intensity training. Her symptoms have subsequently resolved and she has been well during over 1 year of follow up. Resting bradycardia in athletes has been traditionally attributed to enhanced vagal tone, but there may also be intrinsic changes in physiology of the SA and AV nodes. Athletes commonly have sinus pauses, first degree AV block, and AV Wenckebach, but more advanced AV block is uncommon.1–4 Because this patient had marked high grade AV block, numerous investigations directed at identifying underlying structural or conduction disease were performed. In a series of athletes with both symptomatic and asymptomatic AV block, both symptoms and AV block resolved with cessation of high intensity exercise training. There were no adverse outcomes with long-term follow-up.4–6 This condition is important to identify because the inappropriate implantation of a permanent pacemaker in a young otherwise healthy person can have devastating complications over the course of a lifetime.
J.D. Booker (*) and G.D. Veenhuyzen Libin Cardiovascular Institute, University of Calgary, 1403 29th St. NW, Calgary, Alberta T2N2T9, Canada e-mail:
[email protected];
[email protected]
A. Natale et al. (eds.), Cardiac Electrophysiology, DOI: 10.1007/978-1-84996-390-9_169, © Springer-Verlag London Limited 2011
647
648
J.D. Booker and G.D. Veenhuyzen
Fig. 169.1 (a, b) ECG from 24-h ambulatory monitoring
References 1. Bjørnstad H, Storstein L, Meen HD, Hals O. Ambulatory electrocardiographic findings in top athletes, athletic students and control subjects. Cardiology. 1994;84:42-50. 2. Link MS, Homoud MK, Wang PJ, Estes NA 3rd. Cardiac arrhythmias in the athlete. Cardiol Rev. 2001;9:21-30. 3. Talan DA, Bauernfeind RA, Ashley WW, Kanakis C Jr, Rosen KM. Twenty-four hour continuous ECG recordings in long-distance runners. Chest. 1982;82:19-24.
4. Zehender M, Meinertz T, Keul J, Just H. ECG variants and cardiac arrhythmias in athletes: clinical relevance and prognostic importance. Am Heart J. 1990;119:1378-1391. 5. DiNardo-Ekery D, Abedin Z. High degree atrioventricular block in a marathoner with 5-year follow-up. Am Heart J. 1987;113: 834-837. 6. Meytes I, Kaplinsky E, Yahini JH, Hanne-Paparo N, Neufeld HN. Wenckebach A-V block: a frequent feature following heavy physical training. Am Heart J. 1975;90:426-430.
Case 170 M. Eyman Mortada, Jasbir S. Sra, and Masood Akhtar
Case Summary
Case Discussion
A 35-year-old male had a history of end-stage renal disease. Three months prior to presentation, he received a renal transplant. Other medical problems include: diabetes mellitus, hypertension, and bipolar disorder. He had normal coronaries and left ventricular function by cardiac catheterization. The patient was admitted to the hospital due to acute renal failure from graft rejection, respiratory failure with intubation, and systemic infection. During the course of his hospitalization he was on fluconazole, levofloxacin, azithromycin, haloperidol, valganciclovir, nephro-Vite, insulin, calcium carbonate, para calcitrol, and epoetin alfa. After a few days, he suddenly became hemodynamically unstable and presented with the following rhythm, which was terminated by cardioversion (Fig. 170.1). His baseline 12-lead ECG a few days prior to the incident is shown in Fig. 170.2. His 12-lead ECG post cardioversion is shown in Fig. 170.3. At that time, his electrolytes were normal (potassium, 4.5; sodium, 140; magnesium, 2.1), his cardiac markers were normal, and his head CT scan was normal. What was the rhythm when he was hemodynamically unstable? What is the etiology?
The first rhythm strip shows initiation of the tachycardia. On the left, the patient’s rhythm is sinus rhythm with first-degree AV block and long QT (>500 ms). During sinus rhythm, the patient was hemodynamically stable (illustrated at the bottom of Fig. 170.1). The third beat is a premature ventricular beat, which appears at the end of the T-wave without causing dysarrhythmia. Another premature ventricular beat occurs after the ninth beat. It occurs at the end of a long QT interval and initiates the polymorphic ventricular tachycardia. Note the change of blood pressure during the tachycardia. Baseline 12-lead ECG demonstrated sinus tachycardia with normal QT interval (QTc 450 ms). The later 12-lead ECG confirms the new onset of prolonged QT (QTc 616 ms). The causes of polymorphic ventricular tachycardia (PMVT) include: myocardial ischemia, bradyarrhythmia, or long QT. This patient was young, with normal coronaries by cardiac catheterization done a few months prior to this hospitalization. His 12-lead ECG is not suggestive of acute coronary artery closure by thrombus or embolus (no ST elevation was observed). Moreover, cardiac markers were normal, ruling out myocardial infarction. The patient is in tachycardia rather than bradycardia, both on the rhythm strip and his 12-lead ECG. Therefore, the only possible cause for the PMVT is long QT. The causes of long QT include: congenital abnormalities, metabolic disorders, bradyarrhythmia, drugs, and intracranial disease. The only possible etiology for the long QT interval in this scenario is drugs, specifically, the combination of fluconazole, levofloxacin, and haloperidol. Once the drugs were discontinued, the QT prolongation disappeared and the symptoms relating to the PMVT subsided.1
M.E. Mortada (*), J.S. Sra, and M. Akhtar Department of Electrophysiology, Aurora Cardiovascular Services, Aurora Sinai/Aurora St. Luke’s Medical Centers, University of Wisconsin School of Medicine and Public Health, 2801 W. Kinnickinnic River Parkway, #777, Milwaukee, WI 53215, USA e-mail:
[email protected];
[email protected];
[email protected]
A. Natale et al. (eds.), Cardiac Electrophysiology, DOI: 10.1007/978-1-84996-390-9_170, © Springer-Verlag London Limited 2011
649
650
M.E. Mortada et al.
Fig. 170.1 After becoming hemodynamically unstable, the patient presented with this rhythm, which was terminated by cardioversion. (Continuous rhythm strip of [from top to bottom]: Lead II, Lead V1, pulse oximetry “SPO2,” and arterial blood pressure)
Fig. 170.2 Twelve-lead ECG a few days prior to the incident
Case 170
Fig. 170.3 Twelve-lead ECG after cardioversion
Reference 1. A list of drugs causing long QT is available at: http://www.azcert.org/ medical-pros/drug-lists/pubMed-drug-list.cfm (Accessed November, 2008).
651
Case 171 M. Eyman Mortada, Jasbir S. Sra, and Masood Akhtar
Case Summary A 62-year-old female was admitted to the hospital for elective cardiac catheterization secondary to abnormal stress test and preserved left ventricular ejection fraction. The catheterization revealed 80% stenosis of the mid left anterior descending (LAD) coronary artery, where an angioplasty and stenting was performed. The patient did well until 5 h postprocedure, at which time she had asymptomatic intermittent runs of abnormal rhythm. The following rhythm was seen on the rhythm strips lasting 20 s, the terminal part is shown (Fig. 171.1).
Additional rhythm strips are shown in Fig. 171.2. What is the most likely diagnosis?
Case Discussion The possibility exists that any patient receiving coronary artery revascularization could experience dysrhythmia within several hours postprocedure. This is due either to reperfusion of the ischemic cardiac cells or dislodgement of small particles from the atherosclerotic plaque into the microvasculature, causing distal ischemia. Therefore, it is
Fig. 171.1 The patient had asymptomatic intermittent runs of abnormal rhythm at five hours post procedure. This rhythm, as seen in Panels A and B, lasted 20 seconds; the terminal part is shown. (Continuous rhythm strip of three leads: [from top to bottom] Leads II, III, and V1.)
M.E. Mortada (*), J.S. Sra, and M. Akhtar Department of Electrophysiology, Aurora Cardiovascular Services, Aurora Sinai/Aurora St. Luke’s Medical Centers,University of Wisconsin School of Medicine and Public Health, 2801 W. Kinnickinnic River Parkway, #777, Milwaukee, WI 53215, USA e-mail:
[email protected] A. Natale et al. (eds.), Cardiac Electrophysiology, DOI: 10.1007/978-1-84996-390-9_171, © Springer-Verlag London Limited 2011
653
654 Fig. 171.2 The dotted lines (Panel A) and arrows (Panel B) demonstrate the sharp spikes that represent the sinus QRS. The rest are artifact
M.E. Mortada et al.
A
B
essential to monitor all patients who undergo revascularization closely. In this patient, wide complex rhythm developed 5 h postcoronary angioplasty. Once the patient is stabilized, careful analysis of the rhythm strips is essential prior to proceeding with management options. The second rhythm strip reveals
irregular polymorphic wide complexes with regular marching narrow complexes or spikes. These narrow complexes march well with the sinus QRS complexes (marked by dotted lines in the top and arrows in the bottom tracing). They are the clue for the diagnosis – an artifact.
Index
A Ablation catheter atrial activation, 69, 70 circular mapping catheter, 69 3D mapping system, 69 posterior LA, 36, 37 Ablation site AT mapping, 29 conduction around mitral annulus, 29 early recurrence, AT, 29 entrainment, arrhythmia, 29 extensive LA, 29 NavX activation map, 29, 32 PV isolation, 29 restored sinus rhythm, 29, 32 RFAC and decapolar catheter insertion, CS anterior LA, 29, 30 complex fractionated electrograms site, 29, 31 Accessory pathway (AP), 57 Acute coronary syndrome, revascularization, 337, 339 Alternating bundle branch block, 496 Amiodarone-induced proarrhythmia, 568–569. See also Long QT syndrome Amiodarone pulmonary toxicity chest x-ray, 529 clinical presentations, 529–530 corticosteroid therapy, 530 diagnosis, 530 incidence, 529 prednisone, 530 Anterior interventricular vein (AIV), 267–269 Anteroseptal accessory pathway, 21 Antiarrhythmic medications, 51, 403 Antidromic AVRT, 258 Antitachycardia pacing (ATP), 403 atrial arrhythmia (see Atrial arrhythmia) intracardiac electrogram, 383, 385 termination, 481, 482 Aortic sinuses of valsalva (ASOV), 267 Arrhythmia, in post-cardiac transplant patients, 63 Arrhythmogenic right ventricular cardiomyopathy/ dysplasia (ARVC/D) ablation catheter contact, 619 combined endo-and epicardial RF ablation approach, 619, 621 coronary angiography, left coronary system, 619, 620 electroanatomic voltage map, 617–619 endocardial entrainment and activation mapping, 617, 618 epicardial ICD infection, 617 ICD shocks, 617 isoproterenol infusion induced VT, 619
left bundle branch block, 617 pace mapping, 619, 620 pericardial access, 619 repeated programmed stimulation and pacing, 617 sotalol, 617 tricuspid annulus, 617, 619 amiodarone, 562 baseline ECG, 560 diagnosis bipolar right ventricular voltage map, 612–613 cardiac sarcoidosis, 613 ECG during sinus rhythm, 611, 613 endomyocardial biopsy, 613 heterogeneous genetic disease, 613 LBBB VT, 613 monomorphic VT, 612–613 sotalol, 611 diagnostic criteria, 562 differential diagnosis, 559 disease-causing genes, 561–562 exercise stress test, 559, 560 histologic features, 561 ICD burst pacing, 561 dual-chamber, 561 recommended guidelines, 562 inheritance pattern, 561–562 monomorphic VT with left bundle morphology, 561, 562 prevalence, 562 RV angiogram, 559, 561 RV endomyocardial biopsy, 559, 561 Sotalol 160 mg, 561, 562 Aspirin, 3 Asymptomatic atrial fibrillation anticoagulation, 353 antitachycardia pacing, 353 pacemaker interrogation, 353, 354 radiofrequency catheter ablation, 353, 355 sotalol therapy, 353 “successfully” treated episode, 353–355 Athletic heart with left ventricular hypertrophy, 579 Atrial arrhythmia ATP therapy atrial and ventricular marker channels, 357, 359 cycle lengths, 357 interval plot, 357, 358 symptomatic atrial arrhythmia, 357 tachyarrhythmia biventricular pacing, 483 DDI pacing, 482 device interrogation, 482, 483
A. Natale et al. (eds.), Cardiac Electrophysiology, DOI: 10.1007/978-1-84996-390-9, © Springer-Verlag London Limited 2011
655
656 PR interval, 482 treatment, 482 Atrial capture, 331. See also Ventricular pacing Atrial depolarization, 53 Atrial ectopy endocardial tracing, LSPV, 8 P-wave morphology, 8 reverse activation pattern, 8 Atrial fibrillation (AF), 256 ablation, 39 ablation catheter positioning anterior and posterior, LA, 23, 25 anterior and septal, 23, 26 amiodarone pulmonary toxicity, 529–530 area scanning, 24, 26 arterial hypertension AF attacks, 565 clinical examination, 565 DC cardioversion, 565 flecainide, 565, 566 intermittent antiarrhythmic drug therapy, 565–566 pill-in-the-pocket approach, 565–566 propafenone, 565, 566 ramipril 5 mg, 565 athletes, accessory pathway adrenergic surge, 633 1:1 antegrade conduction, 631 asymptomatic ventricular preexcitation, 631 baseline ECG, short PR interval and delta wave, 631, 632 isoproterenol infusion, 631, 633 lost antegrade conduction with atrial pacing, 631 lost ventricular preexcitation, 631, 632 radiofrequency ablation, left lateral pathway, 631, 633 resting vagal tone, 631 risks associated, 633 atrial arrhythmia, 325 catheter positioning, 39 conduction block, roofline, 24 conventional quadripolar ablation catheter mapping, 24 coronary artery bypass surgery AF time course post-op, 604 amiodarone, 603 atorvastatin, 603 beta blockers, 603 carvedilol, 603 incidence, 603 indepamide, 604 omega-3 fish oil, 603 patient’s medications vs. postoperative AF suppression, 604 propafenone, 603 QT prolongation, 604 trandolapril, 603 focal re-entrant AT, 25 hypertension, 527–528 hypertrophic cardiomyopathy amiodarone, 577 atenolol and warfarin, 577 defibrillator implantation, 578 echocardiography, 578 electrical cardioversion, 577 electrocardiogram, 577 risk factors, 578 ICD discharges, 477 lasso catheter, LSPV and RSPV, 39, 40 12-lead EKG, 321, 322 localized source, discrete point, 24
Index macro re-entrant AT, 25 mapping area, 25 organized atrial activity, V1, 23 patient’s history, 39 permanent pacemaker electrocautery, recommendations, 343, 344 lead/connector problems, 343 oversensing (Vs), 343 ventricular electrograms, 344 quadripolar catheter, CS, 23 R-R intervals, 321 septal-to-lateral activation sequence, 23, 24 suppression, tachycardia atrium pacing, 375 pacemaker-repetitive nonreentrant ventriculoatrial synchronous rhythm, 375 premature ventricular captures, 375–377 programmable algorithm, 375, 377 PVARP shortening, 375–377 VIP mode, 377 SVT with severe rheumatoid arthritis, 539, 541 symptomatic paroxysmal ablation, left inferior pulmonary vein and mitral annulus, 185, 188 atrial flutter, 185–186 catheter-based radiofrequency ablation, 185 intracardiac recordings, 185, 186 pulmonary vein isolation, 185 surgical maze procedure, 185 three-dimensional voltage map, 185, 187 symptomatic persistent atrial flutter, 157 persistent left SVC ablation (see Persistent left superior vena cava) pulmonary vein isolation, 157–158 quinidine and diltiazem, 157 repeat ablation, 157 ventricular capture, 321 ventricular pacing, exceeding rate limit (see Ventricular pacing) ventricular rhythm, 477 Atrial Fibrillation-Congestive Heart Failure Study (AF-CHF), 527 Atrial fibrillation cycle length (AFCL) AF ablation active vs. passive pattern, 16 fractionated electrical activity, anterseptal LA, 16, 18 impact, 16 inferior left atrium, 16, 17 posterior part, LAA, 16, 17 prolongation of, 16 roof, LA, 16, 17 custom analysis software, 16 electrogram annotation, 15, 16 measurement, 15 PV isolation, 15, 16 Atrial flutter (AFL) ablation ATP, 403 catheter, 230 conducted atrial flutter, 403 postcardiac surgery patients, 4 shock, 403–404 ablation catheter left atrium, 12, 13 mitral valve, 12 atrial EGM, 347, 348 atrial electrical signals, 325, 326 canine model, 11
Index cardiac tracing, 51 cavo-caval anastomosis, 63 cavotricuspid isthmus, 230, 231 conduction, 4 dextrocardia catheter ablation, 123–124 counterclockwise AFL, 123–124 duodecapolar catheter, 123 duodecapolar recording donor atrium, 83–84 SVT, OHT patients, 83 earliest atrial activation, 51 electroanatomic mapping atrio-atrial anastamosis, 84 recurrent cavotricuspid isthmus dependent atrial flutter, 63, 64 right atrium, 51–52 entrainment mapping, 230 entrainment pacing, 51 left atrium, 12, 13 right atrium, 12 fluoroscopic RAO view, 83 intracardiac activation sequence, 83 isthmus-dependent, 3–4 12-lead ECG atrial arrhythmia, 3 tachycardia, 229 mode switch function, 325 orthotopic heart transplantation activation map, 102 intracardiac recording, 101 pacemaker function manufacturer-specific algorithms, 348 mode switch failure, 348 programmed parameters, 347, 348 PVAB and PVARP, 348 patient’s complaint, 11 patient’s history, 3 postpacing interval, 230 primary pulmonary hypertension counterclockwise and clockwise AFL, 217–218 dyspnea, 217 organized atrial activity with variable flutter wave morphology, 217 positive and negative F waves in inferior leads, 219 radiofrequency ablation, 218–219 surface and intracardiac recordings, 218 right atrial dilation, 11–12 right side vs. left side, 12 SVT initiation, 236 symptomatic persistent atrial fibrillation CT scan, 158 recurrence prevention, left SVC ablation, 157–158 TV-IVC isthmus ablation catheter positioning, 4 RF application, 4, 5 twelve-lead ECG, 11 typical counterclockwise, 199–200 Atrial high rate episode DDI mode, 483 stored electrograms EGM and marker intervals, 319, 320 initiation, 320 Atrial pacing event atrioventricular interval, 456 biventricular pacing, 456, 457
657 bradycardia settings, 392 cross talk Safety Pacing™, 393 ventricular lead, 391, 393 ventricular sensed event filling, 393 cross-talk window, 456 device settings, 351 ICD status report, 456 integrated dual sensors, 351 lower rate limit 12-lead ECG, 351, 352 verification, intracardiac electrogram, 352 minute ventilation sensor, 351 telemetry recordings, 455 ventricular sensing, 456, 457 Atrial premature depolarization (APD), 115, 119 Atrial programmed stimulation, tachycardia, 57, 59 Atrial stimulus delivery, 317 differential diagnosis, 361, 362 lead misconnection diagnosis, 361 QRS complex, 361 ventricular depolarization, 361 ventricular pacing, 361 Atrial tachycardia (AT). See also Ectopic atrial tachycardia AF ablation, 36 CARTO map electrical information, 127–129 inter-atrial connections, 127, 130 intra-cardiac electrograms, 125–127 and propagation maps, 127–128 catheter ablation, AF focal point tachycardia, 36 macro re-entry, 36 context of AF ablation, 36 cycle length, 35–36 early recurrence, AF, 35 EKG emergency room, 125 pre-vs. post-ablation p waves morphology, 131–132 electrogram-based ablation, RA1, 35 localized re-entrant, 36 mapped activity spanning, entire CL, 36 past medical history, 125 PV isolation, 35 radiofrequency ablation, 131 second ablation, 35 spontaneous changes, electrical rhythm, 125, 126 three close posterior spot mapping, 36, 37 Atrial Therapy Efficacy and Safety Trial (ATTEST), 353 Atriofascicular fiber, 522 Atrioventricular nodal reentrant tachycardia (AVNRT), 233, 234, 246, 371–372 ablation site, 71, 77 atrial overdrive pacing, 189–190 atrial premature depolarization cycle length, 182–183 RF energy delivery, 181 atypical clinical presentation, 111, 112 differential diagnoses, 111 intracardiac tracing, 111, 113 spontaneous premature ventricular capture, 111, 113 surface ECG, 111, 112 ventricular entrainment, 111, 113 1:1 AV conduction, 189–190 2:1 AV conduction
658 adenosine IV bolus, 159 2:1 AV conduction to 1:1 AV conduction, 162 baseline 12-lead ECG, 160 b-blockers/calcium channel blockers, 159 electrophysiology evaluation, 161 hemodynamic stability, 161 His-Purkinje system block, 159 mechanism, 162 continuous atrial stimulation (see Continuous atrial stimulation) coronary artery disease bypass surgery, clinical history, 555 cardioversion, 555 ECG and rhythm strip, 555, 556 left bundle branch block, 555, 557 normal QRS, 555, 558 slow pathway ablation, 555 distal coronary sinus, 81, 82 four surface ECG leads, 71, 72 ischemic heart disease with reduced LVEF first-degree AV block, 191, 192 intracardiac recording, programmed atrial stimulation, 192 junctional rhythm, ablation, 191, 193 retrograde fast pathway, Koch’s triangle, 191 left-sided conducting accessory pathway distal mapping catheter, 89 intracardiac recording at baseline, 88 normal PQ interval, 92 programmed atrial stimulation, 92 programmed ventricular stimulation, 89, 91 RF energy delivery, 90–91 short PQ interval with ventricular pre-exitation, 87 sinus rhythm, 90 SVT induced with catheter manipulation, 88 mapping catheter, RAO and LAO, 71, 79 pace mapping, Koch’s triangle anteroseptal region, 71, 76 midseptal region, 71, 76 posteroseptal region, 71, 77 palpitations and dyspnea initial presentation, emergency room, 513, 514 initiation of tachycardia, 513, 514 isoproterenol, 513 spontaneous 2:1 AV block, 513, 515 programmed atrial stimulation (see Programmed atrial stimulation) RF delivery, slow junctional beat induction, 71, 78 slow-fast type, 81 accelerated junctional rhythm, 222 atrial electrogram, 222 left-sided slow AV nodal pathway ablation, 221–222 retrograde aortic approach, 221 right-sided vs. left-sided approach, 221 typical atrial premature depolarization, 115, 119 diagnosis, 115, 116 diagnostic maneuver, 118 intracardiac recording, 115, 120 left bundle branch block morphology, 115, 117 RV pacing, 121 Wolff-Parkinson-White syndrome (WPW), 517–518 Atrioventricular reciprocating tachycardia (AVRT) orthodromic accessory pathways, 640 atrial activation sequence, 640, 641 delta wave vector, 640 fusion of retrograde conduction, 639–640 initiation of tachycardia, 639
Index programmed atrial stimulation, 639–640 programmed ventricular stimulation, 640 preexcited tachycardia, 253 Atrioventricular reentry tachycardia (AVRT) ablation catheter positioning, 47, 49 ablation of mitral annulus, 103, 106 accessory pathway and SVT, 47 accessory pathway location, 506, 507 adenozine, verapamil and diltiazem, 506 antidromic, 61 AV nodal block, 506 cryoablation, 506, 507 eccentric atrial activation, 61 ECG, induced tachycardia, 47, 48 initial presentation, emergency room, 505 left-sided accessory pathway ECG, 147, 148 RF energy application, 147, 149 VA block, 147, 150 long RP tachycardia, 144–145 orthodromic (see Orthodromic AV reentry tachycardia) QRS morphology, 103, 105 right-sided AP, 47 sinus rhythm with preexcitation, 505, 506 slowly conducting anterograde accessory pathway, 103 AV block dual chamber pacing system implantation pacemaker interrogation, 341 PVARP, 342 real time electrogram and marker channel, 341, 342 telemetry AAI/DDD pacing mode, 335 device settings, 335 ventricular oversensing, 335, 336 AV nodal conduction body surface ECG, 349, 350 disease/block carotid sinus massage, 583–584 2:1 conduction, 583, 584 dyspnea, 583 His-Purkinje system, 583 Mobitz classification, 583 pacemaker, 583 vagal tone, 583 pacing system implantation atrial pacing output, 349 DOO mode, 349 V pace event, 349 B Bachmann’s bundle, 52 Balloon angioplasty, pulmonary vein stenosis/occlusion, 135–136 Beta blockade therapy, 327 Bradycardia pacing antitachycardia pacing, 451, 452 ICD programming, 451 noncompetitive atrial pacing feature, 451, 452 post-ventricular atrial refractory period, 451, 452 sinus tachycardia, 451 Bundle branch block, 81, 247 Bundle branch reentry ventricular tachycardia, 589–591 ablation, right bundle branch, 626, 627 baseline ECG, 625 differential diagnosis, 626 entrainment, 626 exercise testing, 625
659
Index first-degree AV block and premature ventricular contraction, 625 His–Purkinje potentials, 626–627 interfascicular reentry tachycardia, 627–628 scar-related reentry, 628 transient infranodal block, 627, 628 C Cardiac arrest QTc interval prolongation chronotropic agents, 487 drug induced torsade de pointes, 487 intravenous magnesium sulfate, 487 methadone and moxifloxacin, 487 sinus arrhythmia, 487, 488 ventricular bigeminy, 487, 488 ventricular fibrillation, 489 Cardiac catheterization, 63, 83 Cardiac CompassT trend, 433, 436 Cardiac resynchronization pacing, 472 patient related causes, 471 programmed parameters, 471 system related causes, 471 T wave oversensing, 471, 472 ventricular oversensing, 471 Cardiac sarcoidosis diagnostic criteria, 499 differential diagnosis, 497 granuloma, 497–498 granulomatous infiltration, 499 therapy, 499 ventricular dysrhythmias, 498–499 ventricular tachyarrhythmias, 498–499 Cardiac tarcing, 51 Cardiomyopathy ischemic, 51 R-waves, 465 stored EGM, 465, 466 T-wave oversensing fifth ventricular beat, 466 real-time EGM, 465, 466 sensitivity, 465, 467 viral, 83 CareLink™ remote monitoring network baseline artifact, 432 dislodgement, 432 electrograms transmitted remotely, 431 far-field tracings, 432 real-time electrogram post-shock, 432 Carotid sinus hypersensitivity, 593–594 CARTO activation map electrical information, 127–129 inter-atrial connections, 127, 130 and intra-cardiac electrograms, 127 and propagation maps, 127, 128 Cavotricuspid isthmus, 63, 230, 231 Complex fractionated atrial electrogram (CFAE), 15 Concealed retrograde septal accessory pathway vs. AV nodal pathway, 531 ventricular pacing, highly symptomatic SVT ablation site, 531, 533 ECGs, 531, 533 initiation, 533 Para-Hisian pacing, 531, 534 ventricular pacing, 531 Continuous atrial stimulation
isoproterenol IV infusion, 71, 79 non-sustained AVNRT induction, 71, 75 Conventional mapping technique, 69 Coronary artery vasculopathy, 63 Coronary sinus (CS) activation, 47 Counterclockwise atrial flutter, 84 Cryoablation anteroseptal accessory pathway, 506–507 junctional tachycardia, 523, 525 vs. radiofrequency ablation, 153 septal atrial tachycardia, 546, 549 Wolff–Parkinson–White syndrome, 153–155 Cumulative stenosis index (CSI), 136 Custom analysis software, 16 Cycle length (CL), 81 D Decapolar catheter, CS, 7, 29, 35 Defibrillation threshold test, 373 Delta waves, 19 Dextrocardia, 123–124 Diaphragmatic myopotential oversensing, 380 Dilated cardiomyopathy (DCM) autosomal dominant DCM, 598 baseline ECG, 597 ICD dual chamber defibrillator, 597 interrogation, 597–598 lamin A/C mutation, 598 ventricular dysfunction, 597 Distal and proximal bipoles, conduction delay, 23 DOO mode, 349 Double tachycardia arrhythmia termination, 411, 414 atrial and ventricular EGMs, 411, 412 device settings, 411 first shock, 411 interval plot, 411, 413 Dual chamber pacing system implantation, 341–342 Duodecapolar catheter, 83 E Ebstein’s anomaly, 595–596 Ectopic atrial tachycardia carvedilol and enalapril, 571 catheter ablation, 573 first degree AV block, 571, 572 initial hospitalization, 572 initial presentation, 571 vs. sinus tachycardia, 573 Electrocautery, recommendations, 343, 344 Electromagnetic interference (EMI), 389–390 characteristics, 447 extra-cardiac noise oversensing, 447, 449 integrated bipolar leads, 447 myopotential oversensing, 447, 449 pectoralis oversensing, 447, 449 shock delivery, ICD, 448 Epicardial ventricular tachycardia, 586 F FlashbackT intervals, 453, 454 Flecainide, 39 Focal atrial tachycardia, 36 12-lead ECG, 3 pregnancy
660
Index catheter ablation, left atrium, 575 electroanatomic map, 575–576 electrocardiogram, 575, 576 mechanism of tachycardia, 575 quinidine and metoprolol, 575
H Heart failure AV block cardiac sarcoidosis (see Cardiac sarcoidosis) differential diagnosis, 497 first-degree AV block and left bundle branch block, 497, 498 high grade AV block, 497, 498 refractory heart failure (see Refractory heart failure) His bundle potential, 44 Hyperkeratosis, 561 Hypertension, 39, 71 metoprolol, 527–528 patients age and history, 527 rate vs. rhythm control, 527 warfarin therapy, 527 Hyperthyroidism with severe rheumatoid arthritis, 539, 542 Hypertrophic cardiomyopathy (HCM) amiodarone, 577 atenolol and warfarin, 577 clinical presentation, 502 defibrillator implantation, 578 diagnostic work up, 502 ECG, 501 echocardiography, 578 electrical cardioversion, 577 electrocardiogram, 577 parasternel long axis image, 502 physical findings, 502 risk stratification, 502–503, 578 treatment, 503 I ICD-detected VT amiodarone, 453 flashbackT intervals, 453, 454 interval plot, 453, 454 stored electrogram, 453, 454 Implantable cardioverter-defibrillator therapy, 275 Induced tachycardia, 47, 48 Inferior vena cava (IVC) isthmus-dependent atrial flutter, 3–4, 63 tricuspid isthmus ablation, 84 InSync SentryT 7299 CRT-D defibrillator implantation provocative maneuvers, 443, 445 shock lead impedance values, 444 telemetry, 443, 444 VT/VF episode report, 443, 444 Inter-atrial connections Bachmann Bundle, 130 fossa ovalis, 130 passive activation, 127, 130 Interfascicular reentry tachycardia, 627–628 Intermittent AV block, 495–496 Intra-His conduction block, 496 Intrinsic atrial activation, 317 Ischemic cardiomyopathy, 51 Isoproterenol, 39
J Junctional tachycardia cryoablation, 523, 525 nonparoxysmal, 523 VA block, 523 L Lasso catheter, 39 Lead fracture, 387–388 Left anterior descending (LAD) coronary artery angioplasty and stenting, 653–654 asymptomatic intermittent runs, abnormal rhythm, 653 dysrhythmia, 653 sinus QRS complex, 654 Left atrial myxoma, 151–152 Left atrial tachycardia ablation of the cavotricuspid isthmus, 199–200 entrainment mapping, 200 organized atrial rhythm with irregular ventricular rhythm, 199 typical counterclockwise activation of right atrium, 199–200 Left coronary cusp (LCC), 267, 268 Left-sided septal slow pathway ablation mitral annulus, 221 RF current application, junctional rhythm acceleration, 221, 222 site identification, 221, 222 Left ventricular ejection fraction (LVEF), 39, 51 Left ventricular hypertrophy, 39 Left ventricular non-compaction (LVNC) cardiac magnetic resonance, diastole, 492 clinical presentations, 492 ECG, 491 inheritance pattern, 492 neuromuscular disorders, 492 prevalence, 492 prognosis, 492–493 survival rate, 493 transthoracic echocardiogram, 491, 492 treatment recommendations, 493 Linear lesions, 29 Long-QT syndrome (LQTS) atenolol 50 mg, 635 baseline ECG, sinus bradycardia, 635, 636 beta-blocker therapy and ICD implantation, 637 drug induced torsades de pointes amiodarone-induced proarrhythmia, 568–569 baseline ECG, 568 gene mutation associated with, 567–568 microvolt T-wave alternans test, 567–569 reduced repolarization reserve, 567, 568 resting ECG, 567, 569 risk factors for, 569 tachycardia initiation, 568 visible TWA, 568 exercise testing, 636 gene mutations, 635 genetic testing, 637 QTc interval, 635, 637 risk for, 637 symptoms, 635 telemetry monitoring abrupt awakening, 635, 637 short bursts of polymorphic ventricular tachycardia, 635–636 sinus with ventricular bigeminy, 635–636
661
Index M “Mahaim tract” tachycardia, 522 Medtronic Kappa 400 pacemaker, 351, 352 Metal ion oxidation (MIO) induced damage, 416 Minute ventilation (MV) impedance measurements, 363, 367 sensor-driven tachycardia, 363 Mitral annulus, conduction, 29 Mitral regurgitation pacemaker syndrome, 327 ventricular pacing, reprogramming abnormal cardiac activation, 370 biventricular pacing, 370 echocardiogram, 369 heart failure, 369–370 Monomorphic PVCs, 295, 296 Monomorphic ventricular tachycardia (MMVT) Fidelis failures, 459, 462 high-voltage impedance, 459 PVCs activation and pacemapping, 295, 297 bipolar voltage map, 295, 297 monomorphic PVCs, 295, 296 VT initiation, 295, 296 R wave amplitude and impedance measurements, 461 stored electrogram, 459, 460 Multielectrode circular mapping catheter, 69 Multiple shocks etiologies, 416 first episode, 416 RV EGM, 417, 418 second episode, 416, 417 VT/VF therapy delivery, 416–417 Myotonic muscular dystrophy autosomal dominant disorder, 599 bundle branch reentry ventricular tachycardia, 601 first degree AV conduction delay, 599 baseline ECG, 600 dual chamber pacemaker, 601 evaluation and monitoring, 601 Holter monitor strip, 600 monomorphic ventricular tachycardia, 601 noncardiac and cardiac manifestations, 599 prevalence, 599 supraventricular tachyarrhythmia, 599, 601 N Narrow complex tachycardia, 41, 57, 58, 71 emergency room ablation, septal atrial tachycardia, 546, 549 adenosine, 545–546 AV dissociation, 546, 548 baseline electrocardiogram, 546 cryoablation, 546, 549 ECG, 545 initiation with single atrial extrastimulus, 545–546 ventricular pacing, 546, 547 QRS ablation of AP, 96, 99 left bundle branch block, 96, 98 para-Hisian pacing, 95, 96 right bundle branch block, 96, 97 ventricular pacing, 95 VA block carvedilol, ECG, 523, 524 differential diagnosis, 523
His spikes, 523, 524 junctional tachycardia, 523, 525 NavX activation map, 29, 32 NavX fractionation map, 15, 16 Naxos disease, 561–562 Noise oversensing, RV lead arrhythmia logbook, 399, 400 bradycardia, ventricular pacing inhibition, 399 diaphragmatic myopotentials, 399 shock delivery, 401 tachycardia zone redetection, 402 Noncompetitive atrial pacing (NCAP) feature, 451, 452 Nonparoxysmal junctional tachycardia, 523 Non-ventricular tachyarrhythmias, ICD discharges, 477, 478 NSVT episode, 287, 289 O Orthodromic AV reentry tachycardia, 246 diagnosis, 140, 141 radiofrequency ablation, mitral valve annulus, 140 Wolff–Parkinson–White syndrome (see Wolff–Parkinson–White syndrome) Orthotopic heart transplantation (OHT), 63, 83 Outflow-type tachycardia, 267 P Pacemaker artifacts, 317–318 pacemaker syndrome baseline rhythm strip, 329 pathophysiology, 327 prevalence, 327 single-chamber ventricular pacemaker, 327, 328 vague symptoms, 327 settings, 317 Pacemaker and Beta-Blocker Therapy after Myocardial Infarction (PACE-MI) Trial, 327–328 Pacemaker atrial refractory period (PVARP) atrial fibrillation suppression, 375 atrial sensed signals, 348 AV block, 342 bradycardia pacing, 451 premature atrial contraction, 371 tachycardia, 375, 376 ventricular pacing, 345–346 Pacemaker-repetitive nonreentrant ventriculoatrial synchronous rhythm, 375, 377 Palpitation, 41, 57, 65, 71 Para-Hisian pacing, 20–21, 95, 96, 250–251 Paroxysmal atrial fibrillation (PAF), 39 ablation dissociation, venous potential, 9, 10 earliest venous potential, 9 left inferior pulmonary vein, 8–9 in right veins, 9 amiodarone, 608 arrhythmia recurrence, 8 cordarone, 200 mg, 608 disopyramide, 608 drug combination therapy, 609–610 ECG during AF with RBBB, 608 during sinus rhythm, 607 endocardial tracing, LSPV, 8 events and therapies, 609 flecainide, 609
662 monomorphic atrial ectopy, 7 omega-3 fish oil, 608 propafenone, 608–609 reverse activation sequence, 8 rheumatic mitral insufficiency, 608 Paroxysmal supraventricular tachycardia (PSVT) adenosine after administration, 509–510 atrial fibrillation induction, 509, 510 effect, EP study, 510 initial presentation, emergency room, 509 recurrent, 201–202 Permanent form of reciprocating tachycardia (PJRT), 552 12-lead ECG, 133 long RP tachycardia, 144–145 posterior septum, accessory pathway, 133–134 retrograde activation, lateral mitral annulus, 134 Persistent left superior vena cava, 157–158 Polymorphic ventricular tachycardia (PMVT), 279, 280, 287 end-stage renal disease baseline ECG, 649, 650 cardioversion, 649, 650 causes of, 649 ECG post cardioversion, 649, 651 first-degree AV block, 649 nonsustained and sustained, 280 sinus bradycardia, 635, 636 torsades de pointes, 567 ventricular fibrillation, 287 Polyurethane insulation, ICD, 416 Post-pacing interval (PPI), 312 Post-pacing T-wave oversensing, 429 Post-ventricular atrial blanking period (PVAB), 348 Post-ventricular atrial refractory period (PVARP), 345–346, 348, 375–377, 451, 452 Preexcited tachycardia accessory pathway, 253 atrial fibrillation, 256 atrial stimulation, 255 baseline ECG, 254 baseline interval, 254 differential diagnosis, 253 post-ablation ECG, 261 post-ablation intervals, 260 RF energy, 259 shortest preexcited RR interval, 256 VA block, 257 ventricular pacing, 257 Premature atrial capture, 140, 141 Premature atrial complex, 81 Premature ventricular capture, 111, 113, 140, 141, 375–377 Premature ventricular contractions (PVC) ablation, premature Purkinje potentials, 287, 289 ablation site, 271, 273 AIV pacemaps, 267–269 cardiomyopathy ablation, 284 aortic cusp, pacemapping, 284, 285 intracardiac echo, aortic valve, 284 spontaneous VPC vs. pacemap, 285 VPC, 12-lead ECG, 283, 284 left bundle, 280 left coronary cusp, 267, 268 LV activation mapping, 287, 288 morphology, 287 nonsustained and sustained polymorphic VT, 279, 280
Index NSVT episode, 287, 289 pre-QRS activation time, 267, 268 Purkinje system, 279, 281 qrS pattern, 271 right bundle, 281 RV outflow tract pacemaps, 267, 268 trigeminal pattern, 287, 288 ventricular bigeminy, 12-lead ECG, 271, 272 ventricular outflow tract, 271 ventricular unipolar and bipolar activation mapping, 271, 272 Programmed atrial stimulation atrial ERP, 71, 74 coupling intervals, 71, 73 isoproterenol IV infusion AVNRT induction, 71, 75 slow anterograde conduction, 71, 78 retrograde conduction, 71, 74 Programmed ventricular stimulation atrial activation, 640 AVNRT decremental VA conduction, 91 SVT induction, 89 inducible sustained VT, 617 response of, 263 tachycardia, 57, 59 wide QRS tachycardia, 621, 625 Proton pump inhibitor (PPI), 29 Provocative maneuvers, 443, 445 Pseudo–pseudo-fusion complex, 331, 332 Pulmonary toxicity. See Amiodarone pulmonary toxicity Pulmonary vein isolation (PVI), 35 atrial fibrillation, 23, 24, 29, 35 pulmonary vein stenosis/occlusion, 135 symptomatic persistent atrial fibrillation, 157 Pulmonary vein (PV) stenosis/occlusion asymptomatic patient, 136 balloon angioplasty and stenting, 135–136 CT scan/MRI, 135, 136 cumulative stenosis index, 136 dyspnea, 135 incidence, 135–136 misdiagnosis, 136 pathogenesis, 136 pulmonary artery dilatation, 136 pulmonary hypertension, 135, 136 PV isolation, 135 ventilation/perfusion scan, 136 P wave oversensing accelerated junctional rhythm, 337 RV lead positioning, 337 Q QRS complex, 21 Quadripolar catheter, 23 R Radiofrequency ablation catheter (RFAC), 30 Radiofrequency distal (RFD), 47 Radiofrequency (RF) energy application anteroseptal pathway, 21 arrhythmia, 25, 36 intracardiac recording, ablation site, 45 TV-IVC area, 5 Recurrent PSVT, slow pathway ablation procedure, 233 Recurrent tachycardia, 517–518
Index Refractory heart failure ablation, left atrial appendage, 552–553 ACE inhibitor, diuretic, and beta blocker, 551, 552 cardiomegaly and pulmonary congestion, 551 dyspnea, 551 physical examination, initial presentation, 551 resolution of tachycardia induced, 552 sinus tachycardia, initial diagnosis of, 551, 552 tachycardia during sleep, 552 Resting bradycardia, 647 Retrograde atrial activation sequence induced SVT, 20 loss of His bundle capture, 21 Para-Hisian pacing, 20–21 RF application, 20 second SVT, 20 sinus rhythm, ventricular pre-excitation, 19 ventricular pacing, 19, 20 Retrograde conduction, ventricular programmed stimulation atrioventricular node, first atrial beat, 264 ECG, 263 electrical activation, 264 fourth ventricular complex, 264–265 HV duration, 265 left free-wall accessory pathway, 264 sinus node high right atrium, 263, 264 sixth ventricular complex, 265 Right anterior oblique (RAO), 83 Right bundle branch block (RBBB), 57 Right superior pulmonary vein (RSPV) intracardiac activation, 108, 110 isolation and cavotricuspid isthmus ablation, 108 Right-to left atrial conduction block, 51, 52 RV lead, nonphysiologic noise diaphragmatic myopotentials, oversensing, 419 first episode initiation, 420 ICD helix stabilizing post, 421 pin/device header connection, 421 VT zone episode detection, 419, 421 shock electrograms, 419, 420 RV outflow tract (RVOT) pacemaps, 267, 268 S Safety Pacing™, 393 Sarcoidosis granuloma, 497–498 Scar-related VT bipolar voltage map, 305 common isthmus, 303 first VT, 303, 304 second VT, 303, 304 septal scar, 303, 305 VT morphologies, 303 Sensor-driven tachycardia, 363 Septal atrial tachycardia, 546, 549 Shock atrial and ventricular channels, 389 EGM characteristics, 389 electromagnetic interference, 389–390 episode electrogram, 387 lead fracture impedance rise, 387 lead extraction, 388 lead performance report, 388
663 Sinus bradycardia ambulatory ECG monitoring, 648 AV Wenckebach, 647 symptomatic AV block, 647 Sinus tachycardia, 442 Automatic Sensitivity ControlT, 441 morphology mismatch, 441 template verification, 441 Situs inversus, 123–124 Slow pathway ablation procedure inferior vena cava, 233–234 recurrent PSVT, 233 vascular access, 233 Sotalol, 39 Spontaneous rhythm T-wave oversensing, 429–430 Steinert’s disease. See Myotonic muscular dystrophy Stimulus-QRS (S-QRS) interval, 313 Sudden cardiac death, 502, 503 Supraventricular tachycardia (SVT). See also Atrioventricular nodal reentrant tachycardia (AVNRT) ablation, left-sided AP, 65, 67–68 adenosine response, 140 antitachycardic pacing, intracardiac electrogram, 383, 385 atrial activation sequence, 65 atrial sensed events, 406 AV association, 383 A-waves, 81 device parameters, 384 duodecapolar catheter, 83, 84 echocardiography evaluation, 41 episode list, 384 four surface ECG leads, 41, 43, 65, 66 induced, 20 intracardiac recording, 40, 43 sinus rhythm restoration, 65, 67 tachycardia termination, 65, 67 ventricular stimulation, 65, 68 12-lead ECG atrial tachycardia, 41, 42 heart rate, 71, 72 sinus rhythm, 71 long RP tachycardia AVRT, 144–145 intracardiac recordings, 143, 144 12-lead ECG, 143 VPC, 143–144 mental retardation, 53 narrow complex tachycardia, 41 normal sinus rhythm, 41 orthotopic heart transplantation activation map, 102 intracardiac recording, 101 premature atrial complex, 81 recurrent intracardiac tracing during ablation, 86 left bundle branch block, 85 orthodromic reentrant tachycardia, 85 recurrent paroxysmal differential diagnosis, 201 intracardiac tracing, 202 termination of tachycardia with spontaneous AV block, 201–202 severe rheumatoid arthritis ablation, coronary sinus os, 543–544 adenosine, 539 after orthopedic surgery, 539
664 atrial fibrillation, 539, 541 baseline recordings, 539–540 fast pathway ablation, 541 flecainide, 539 hyperthyroidism, 539 left sided AV nodal fibers, 543 pseudo VAAV response, 542 slow vs. fast pathway ablation, 541 tachycardia initiation, 540–541 ventricular pacing, 540, 542 ventricular premature stimulus, 540, 543 verapamil, 539, 540, 542 slow pathway ablation mitral annulus, 221 RF current application, junctional rhythm acceleration, 221, 222 site identification, 221, 222 symptomatic, 199–200 1:1 tachycardia beta-blocker therapy, 409 1:1 conduction, 407 reconfirmation algorithm, 409 twelve-lead ECG, 65, 66 ventricular extrastimuli, 61 VPD activation sequence, 195, 197 initiation, 195, 196 ventricular pacing, 195, 198 VT zone and shock, 383 Symptomatic atrial arrhythmia, 357 Syncope. See also Vasovagal syncope carotid sinus hypersensitivity carotid sinus massage response, 594 dual chamber pacemaker, 594 ECG, 593 elderly patient, 594 hypertension and hyperlipidemia, 593 device interval plot, 395 device reprogramming, 397 double counting, 395, 397 stored electrograms, 395, 396 T Tachyarrhythmia adenosine infusion, AV block, 107, 108 cavotricuspid isthmus ablation, 108 ECG intracardiac activation, 108–110 isoproterenol infusion, 108, 109 falling, VT (See VT) recurrent palpitations, 107 right superior pulmonary vein right superior pulmonary vein isolation, 108 Tachycardia. See also Atrial tachycardia; Ventricular tachycardia ablation site, 41, 45 activation sequence, 61 anti-tachycardia pacing (ATP) termination, 481, 482 AP-mediated, 61 atrial extrastimuli, 371, 372 atrial programmed stimulation, 57, 59 atrial S2, 259 AVNRT mechanism SVT, 223, 224 tachycardia initiation, 225–226 typical AVNRT, 227
Index AVRT, 235 A-waves, CS, 81 bundle branch block, 247 catheter positions, fluoroscopic left anterior oblique views, 242, 243 cavotricuspid isthmus, 230, 231, 243 constant VA interval, 53 device programmed parameter, 363, 365 differential diagnosis, 481 3D mapping system, 69 electroanatomical mapping arrhythmia origin, 41, 43 CARTO, 41, 44 electroanatomic mapping, 230 induced, 47, 48, 57, 58 induced heart failure (see Refractory heart failure) induced narrow complex, 65 induced SVT, 248 intracardiac electrogram, 481, 482 intracardiac recordings, 81, 82 intracardiac signals, 249 intracardiac tracings, 242, 243 laboratory considerations arrhythmia inducibility, sedative medications, 299 precordial morphology, 299, 300 pre-QRS activation time, 299 PVC morphology, 299, 301 right inferior axis ventricular ectopy, 300 12-lead EKG, 363, 364 macroreentrant right atrial arrhythmia, 243 minute ventilation impedance measurements, 363, 367 sensor-driven tachycardia, 363 multipolar deflectable catheter, 242, 243 narrow complex, 41, 57, 58, 71 orthodromic and antidromic ablation site, 173–174 atrial pacing, 165 baseline intracardiac recordings, 166 bypass tract, 163 12-lead resting ECG, 164 programmed atrial stimulation, 168–169, 172 programmed ventricular stimulation, 170–171, 177–178 radiofrequency delivery, 175–176 RAO and LAO views, proximal coronary sinus, 180 ventricular pacing, 167 palpitation, 57, 58 Parahisian pacing, 250–251 premature atrial beat, sinus rhythm, 53, 55 programmed ventricular stimulation, 57, 59 P wave morphology, 235, 239 recurrent, verapamil, 147 RF application, 41, 45 rhythm change, 57, 58 right bundle branch block, 81 sinus rhythm, pacemaker interrogation AV reentry tachycardia, 372–373 premature atrial contraction, 371 RP interval, 371 slow/fast AVNRT, 81 surface electrocardiogram, 241 SVT induction, single atrial extrastimulation, 53, 54 three ventricular extrastimulation, 57, 60 two ventricular extrastimulation, 57, 60 ventricular entrainment, 371, 372 ventricular overdrive pacing
665
Index atrial tachycardia, 246 AV nodal dependent tachycardia, 246 cessation, 245 supraventricular tachycardia, 246 V pacing, 235, 238–239 VPC delivery, 237 VT induction, 61 wide complex, 57, 81, 82, 258 1:1 Tachycardia episode electrograms and marker channels, 408 shock, interval plot, 407 supraventricular tachycardia (SVT) beta-blocker therapy, 409 1:1 conduction, 407 reconfirmation algorithm, 409 Telemetry findings atrial capture, 331 AV block AAI/DDD pacing mode, 335 device settings, 335 ventricular oversensing, 335, 336 AV delay, 391, 393 posteroanterior and lateral chest radiographs, 331, 332 pseudo–pseudo-fusion complex, 331, 332 QRS complex, 391 reprogramming, 332–333 safety pacing, 331–332 spontaneous junctional complex, 332 Tetralogy of Fallot (TOF) ablation activation mapping and entrainment, 623 entrainment of VT, 621–622 pace mapping, 622–623 potential isthmuses, reentry, 623 surface ECG during sinus rhythm and ventricular tachycardia, 621 tachycardia mechanism, 622–623 ventricular septal defect, 614 wide QRS tachycardia baseline ECG, 614 ICD implantation, 615 left bundle branch block, 614, 615 monomorphic VT with programmed ventricular stimulation, 614 sudden death, 614–615 Three ventricular extrastimuli, CL, 61 Trending Plot/Sensor Replay, 363, 366 Tricuspid valve-inferior vena cava (TV-IVC) isthmus, 3–4 Typical AVNRT, 223, 227 V VA conduction, atrial rhythm dependent on, 53 Vasovagal syncope clinical diagnosis, 579 cool down exercise, 579 external loop recorder, 579–560 implanted loop recorder, 561, 579 permanent pacemaker therapy, 561 resting ECG, 560, 579 venous return, 579 Ventricular capture, 321 Ventricular depolarization, 53 Ventricular dysrhythmias, 498–499 Ventricular electrogram morphology discriminator (MDT) diagnostic and therapeutic considerations, 439 ICD reprogramming, 439
stored electrogram, 439, 440 SVT, 439 VF detection, 439 VT with 1:1 VA conduction, 439 Ventricular fibrillation (VF) cardiac arrest, 489 ICD implantation chest X-ray, biventricular system, 374 defibrillation threshold testing, 373 device settings, 373 multiple maximum energy shock, 373, 374 polymorphic ventricular tachycardia, 287 ventricular electrogram morphology discriminator, 439 Ventricular Intrinsic Preference (VIP) mode, 377 Ventricular long term histogram, 328 Ventricular pacing, 19, 20, 327 atrial capture, new lead implantation bipolar and integrated lead designs, 424 chest x-ray, 424 lead characteristics, 424 pacing threshold test, 423 tachycardia event, 424, 425 atrial stimulus delivery, 361 exceeding rate limit, 473 non-tracking mode, 345 PMT termination, 346 post-ventricular atrial refractory period, 345–346 reprogramming, mitral regurgitation abnormal cardiac activation, 370 biventricular pacing, 370 echocardiogram, 369 heart failure, 369–370 retrograde atrial activation, 345 telemetry recording, 474 upper tracking rate, 345 ventricular rate regularization device reprogramming, 474 telemetry recording, 474–475 V-V cycle length variation, 473–474 Ventricular premature contraction (VPC), 319–320 first ablation, 275, 277 LVEF, cardioverter-defibrillator insertion, 275 LVOT, 275 multiple ablations, temperature and power limitation, 275, 276 pacemapping, 275 presystolic activation, 275, 276 Ventricular rate/sense lead, 381 diaphragmatic myopotential oversensing, 380 header torque wrench insertion, 380 vs. shock coil EGMs, 380 stored EGM, 379, 380 V sensed event P wave oversensing, 379–381 QRS complexes, 379 Ventricular refractory period (VRP), 321, 323 Ventricular sensed events EGM and marker channel, 339 programmed parameters, 337, 338 P wave oversensing accelerated junctional rhythm, 337 RV lead positioning, 337 ventricular histogram, 337, 338 Ventricular sensing, 319 Ventricular stimulus, 317 Ventricular tachycardia (VT), 57
666 adenosine response, 140 biventricular voltage maps, 291, 292 bundle branch reentry ablation, right/left bundle, 589, 591 aortic valve replacement, 589 electrical cardioversion, 589, 590 His-Purkinje system, 589 ICD and resynchronization therapy, 589 intracardiac electrograms, 589, 591 12-lead ECG, 589, 590 macroreentrant circuit, 589 Cardiac Compass trend, 433, 436 ECG and intracardiac recordings pace termination, 311 postpacing interval (PPI), 312 electroanatomical map entrainment response, 311, 312 isthmus site, mapping and targeting, 314 stimulus-QRS (S-QRS) interval, 313 ICD reprogramming, 433 linear lesions, 292–293 mid-diastolic potentials cycle length, 307, 309 ECG and intracardiac recordings, 307, 308 entrainment response, 307, 308 non-propagated extrastimulus, 307 misdiagnosis, sinus tachycardia 1:1 AV relationship and cycle length, 469 dual chamber onset algorithm, 469 ICD programming, 469, 470 managed ventricular pacing, 469 morphology, 291 multiple shocks electrical noise, 416 etiologies, 416 first episode, 416 ICD, polyurethane insulation, 416 rate/sense and marker channels, 415–416 second episode, 416, 417 therapy delivery, 416–417 myocardial infarction amiodarone, 585 cardioversion, 585 ECG, emergency room, 585 endocardial ablation, 585–586 epicardial VT, 586–587 substrate based ablation, 586 overdrive pacing, 311–314 vs. pacemap morphology, 292, 293 programmed settings, 434 PVCs activation and pacemapping, 295, 297 bipolar voltage map, 295, 297 monomorphic PVCs, 295, 296 VT initiation, 295, 296 QRS fusion, 312–313 shock, 395–397 antiarrhythmic medications, 403 ATP, 403 atrial flutter ablation, 403–404 conducted atrial flutter, 403 sinus rhythm QRS complex, 291 stored electrograms, 433, 435 substrate-based ablation, 291–292 syncope
Index device interval plot, 395 device reprogramming, 397 double counting, 395, 397 stored electrograms, 395, 396 tachyarrhythmia falling atrial and ventricular EGMs, 405–406 atrial sensed events, 405 device settings, 405 differential diagnosis, 406 therapeutic options, 433 ventricular vs. atrial flutter morphology, 437 ventriculoatrial, 57 Ventriculo-atrial (VA) conduction, 57, 95–96 Ventriculoatrial (VA) interval, 53 Viral cardiomyopathy, 83 VT/VF episode list, 428 T-wave oversensing Medtronic defibrillators, reprogramming sensitivity, 430 post-pacing, 429 spontaneous rhythm, 429–430 V-V intervals, 429 W Wenckebach AV block, 53 Wide complex tachycardia, 57, 81, 82, 258 adenosine, 140, 522 ambulatory event recorder, 103, 104 antiarrhythmic agent, slow ventricular response, 643 atriofascicular pathways, 522 atrioventricular conduction system, 643 AV nodal reentry tachycardia, 140, 141 at 115 beats per minute, 151 catheter ablation, left-sided accessory pathway, 644–645 ECG, emergency room, 644 ECG post-cardioversion, 644, 645 electrical cardioversion, 643–644 left atrial myxoma atriotomy scar, 152 intracardiac recordings, 152 surface electrocardiogram, 151 left bundle branch block, 521 mild hypertension AV nodal blocking agents, 512 flecainide, 511–512 initial presentation, emergency room, 511 lisonopril, 511 metoprolol, 512 paroxysmal atrial fibrillation, 511–512 proarrhythmia, 512 orthodromic AV reentry tachycardia diagnosis, 140, 141 radiofrequency ablation, mitral valve annulus, 140 preexcited atrial fibrillation degeneration, 644 premature atrial capture, 140, 141 premature ventricular capture, 140, 141 QRS morphology, 103, 105 sinus rhythm, 140 stable vitals and abnormal rhythm, 139 ventricular vs. supraventricular tachycardia, 139–140 Wide QRS complex tachycardia with left bundle branch block antidromic tachycardia with retrograde conduction, 210–211 arrhythmia circuit, 215 atrial stimulation atrial pacing, 208 ventricular preexcitation, 207
667
Index atriofascicular pathway, 210 induced tachycardia 12-lead ECG, 210 programmed atrial stimulation, 209 intracardiac recordings at baseline, 204 normal PR interval, 203 programmed atrial stimulation A-H interval and H-V interval, 205 atrioventricular node duality, 206 radiofrequency ablation, 214 tricuspid annulus mapping during sinus rhythm mechanical block, 213 M potential, 212 Wolff–Parkinson–White syndrome (WPW), 19 ablation therapy, 535 autosomal dominant inheritance, 535 cryoenergy ablation early para-Hisian activation, 153, 154 His bundle appearance after rewarming, 153, 155 inadvertent AV block, 153, 154 ventricular preexcitation, 153, 155
Ebstein’s anomaly accessory pathways, 596 baseline 12-lead ECG, 595 catheter/surgical ablation, 596 tricuspid valve incompetence, 596 ECG, 535–536 family history, 535 left ventricular hypertrophy, 535 pacemakers, 535 PRKAG2 gene defect, 535 recurrent tachycardia AV nodal reentry and accessory pathway, 519 initiation, 518 preexcitation, retrograde P wave, 517 slow pathway ablation, 519 termination, 12 lead ECG, 518 VA interval, 519 signs and symptoms, 535 symptomatic, 153